Как Ρ€Π°ΡΡΡ‡ΠΈΡ‚Π°Ρ‚ΡŒ Π΄Π»ΠΈΠ½Ρƒ Π²Π΅ΠΊΡ‚ΠΎΡ€Π°: Как Π½Π°ΠΉΡ‚ΠΈ Π΄Π»ΠΈΠ½Ρƒ Π²Π΅ΠΊΡ‚ΠΎΡ€Π°? ΠžΡ‚Π²Π΅Ρ‚ Π½Π° webmath.ru

Π‘ΠΎΠ΄Π΅Ρ€ΠΆΠ°Π½ΠΈΠ΅

ΠΏΡ€ΠΈΠΌΠ΅Ρ€Ρ‹ ΠΈ Ρ€Π΅ΡˆΠ΅Π½ΠΈΡ, Ρ„ΠΎΡ€ΠΌΡƒΠ»Ρ‹ ΠΈ Ρ‚Π΅ΠΎΡ€Π΅ΠΌΡ‹

Π”Π»ΠΈΠ½Π° Π²Π΅ΠΊΡ‚ΠΎΡ€Π° — основныС Ρ„ΠΎΡ€ΠΌΡƒΠ»Ρ‹

Π”Π»ΠΈΠ½Ρƒ Π²Π΅ΠΊΡ‚ΠΎΡ€Π° aβ†’Β Π±ΡƒΠ΄Π΅ΠΌ ΠΎΠ±ΠΎΠ·Π½Π°Ρ‡Π°Ρ‚ΡŒ aβ†’. Π”Π°Π½Π½ΠΎΠ΅ ΠΎΠ±ΠΎΠ·Π½Π°Ρ‡Π΅Π½ΠΈΠ΅ Π°Π½Π°Π»ΠΎΠ³ΠΈΡ‡Π½ΠΎ ΠΌΠΎΠ΄ΡƒΠ»ΡŽ числа, поэтому Π΄Π»ΠΈΠ½Ρƒ Π²Π΅ΠΊΡ‚ΠΎΡ€Π° Ρ‚Π°ΠΊΠΆΠ΅ Π½Π°Π·Ρ‹Π²Π°ΡŽΡ‚ ΠΌΠΎΠ΄ΡƒΠ»Π΅ΠΌ Π²Π΅ΠΊΡ‚ΠΎΡ€Π°.

Для нахоТдСния Π΄Π»ΠΈΠ½Ρ‹ Π²Π΅ΠΊΡ‚ΠΎΡ€Π° Π½Π° плоскости ΠΏΠΎ Π΅Π³ΠΎ ΠΊΠΎΠΎΡ€Π΄ΠΈΠ½Π°Ρ‚Π°ΠΌ, трСбуСтся Ρ€Π°ΡΡΠΌΠΎΡ‚Ρ€Π΅Ρ‚ΡŒ ΠΏΡ€ΡΠΌΠΎΡƒΠ³ΠΎΠ»ΡŒΠ½ΡƒΡŽ Π΄Π΅ΠΊΠ°Ρ€Ρ‚ΠΎΠ²Ρƒ систСму ΠΊΠΎΠΎΡ€Π΄ΠΈΠ½Π°Ρ‚ Oxy. ΠŸΡƒΡΡ‚ΡŒ Π² Π½Π΅ΠΉ Π·Π°Π΄Π°Π½ Π½Π΅ΠΊΠΎΡ‚ΠΎΡ€Ρ‹ΠΉ Π²Π΅ΠΊΡ‚ΠΎΡ€ a→ с ΠΊΠΎΠΎΡ€Π΄ΠΈΠ½Π°Ρ‚Π°ΠΌΠΈ ax;ay. Π’Π²Π΅Π΄Π΅ΠΌ Ρ„ΠΎΡ€ΠΌΡƒΠ»Ρƒ для нахоТдСния Π΄Π»ΠΈΠ½Ρ‹ (модуля) Π²Π΅ΠΊΡ‚ΠΎΡ€Π° aβ†’Β Ρ‡Π΅Ρ€Π΅Π· ΠΊΠΎΠΎΡ€Π΄ΠΈΠ½Π°Ρ‚Ρ‹Β ax ΠΈ ay.

ΠžΡ‚ Π½Π°Ρ‡Π°Π»Π° ΠΊΠΎΠΎΡ€Π΄ΠΈΠ½Π°Ρ‚ ΠΎΡ‚Π»ΠΎΠΆΠΈΠΌ Π²Π΅ΠΊΡ‚ΠΎΡ€ OAβ†’=aβ†’. ΠžΠΏΡ€Π΅Π΄Π΅Π»ΠΈΠΌ соотвСтствСнныС ΠΏΡ€ΠΎΠ΅ΠΊΡ†ΠΈΠΈ Ρ‚ΠΎΡ‡ΠΊΠΈ A Π½Π° ΠΊΠΎΠΎΡ€Π΄ΠΈΠ½Π°Ρ‚Π½Ρ‹Π΅ оси ΠΊΠ°ΠΊ Ax ΠΈ Ay . Π’Π΅ΠΏΠ΅Ρ€ΡŒ рассмотрим ΠΏΡ€ΡΠΌΠΎΡƒΠ³ΠΎΠ»ΡŒΠ½ΠΈΠΊ OAxAAy с диагональю OA.

Из Ρ‚Π΅ΠΎΡ€Π΅ΠΌΡ‹ ΠŸΠΈΡ„Π°Π³ΠΎΡ€Π° слСдуСт равСнство OA2=OAx2+OAy2, ΠΎΡ‚ΠΊΡƒΠ΄Π° OA=OAx2+OAy2. Из ΡƒΠΆΠ΅ извСстного опрСдСлСния ΠΊΠΎΠΎΡ€Π΄ΠΈΠ½Π°Ρ‚ Π²Π΅ΠΊΡ‚ΠΎΡ€Π° Π² ΠΏΡ€ΡΠΌΠΎΡƒΠ³ΠΎΠ»ΡŒΠ½ΠΎΠΉ Π΄Π΅ΠΊΠ°Ρ€Ρ‚ΠΎΠ²ΠΎΠΉ систСмС ΠΊΠΎΠΎΡ€Π΄ΠΈΠ½Π°Ρ‚ ΠΏΠΎΠ»ΡƒΡ‡Π°Π΅ΠΌ, Ρ‡Ρ‚ΠΎ OAx2=ax2 ΠΈ OAy2=ay2, Π° ΠΏΠΎ ΠΏΠΎΡΡ‚Ρ€ΠΎΠ΅Π½ΠΈΡŽ Π΄Π»ΠΈΠ½Π° OA Ρ€Π°Π²Π½Π° Π΄Π»ΠΈΠ½Π΅ Π²Π΅ΠΊΡ‚ΠΎΡ€Π° OAβ†’, Π·Π½Π°Ρ‡ΠΈΡ‚, OAβ†’=OAx2+OAy2.

ΠžΡ‚ΡΡŽΠ΄Π° получаСтся, Ρ‡Ρ‚ΠΎ Ρ„ΠΎΡ€ΠΌΡƒΠ»Π° для нахоТдСния Π΄Π»ΠΈΠ½Ρ‹ Π²Π΅ΠΊΡ‚ΠΎΡ€Π° aβ†’=ax;ayΒ ΠΈΠΌΠ΅Π΅Ρ‚ ΡΠΎΠΎΡ‚Π²Π΅Ρ‚ΡΡ‚Π²ΡƒΡŽΡ‰ΠΈΠΉ Π²ΠΈΠ΄: aβ†’=ax2+ay2.

Если Π²Π΅ΠΊΡ‚ΠΎΡ€ aβ†’Β Π΄Π°Π½ Π² Π²ΠΈΠ΄Π΅ разлоТСния ΠΏΠΎ ΠΊΠΎΠΎΡ€Π΄ΠΈΠ½Π°Ρ‚Π½Ρ‹ΠΌ Π²Π΅ΠΊΡ‚ΠΎΡ€Π°ΠΌ aβ†’=axΒ·iβ†’+ayΒ·jβ†’, Ρ‚ΠΎ Π²Ρ‹Ρ‡ΠΈΡΠ»ΠΈΡ‚ΡŒ Π΅Π³ΠΎ Π΄Π»ΠΈΠ½Ρƒ ΠΌΠΎΠΆΠ½ΠΎ ΠΏΠΎ Ρ‚ΠΎΠΉ ΠΆΠ΅ Ρ„ΠΎΡ€ΠΌΡƒΠ»Π΅ aβ†’=ax2+ay2, Π² Π΄Π°Π½Π½ΠΎΠΌ случаС коэффициСнты ax ΠΈ ay Π²Ρ‹ΡΡ‚ΡƒΠΏΠ°ΡŽΡ‚ Π² Ρ€ΠΎΠ»ΠΈ ΠΊΠΎΠΎΡ€Π΄ΠΈΠ½Π°Ρ‚ Π²Π΅ΠΊΡ‚ΠΎΡ€Π° aβ†’ Π² Π·Π°Π΄Π°Π½Π½ΠΎΠΉ систСмС ΠΊΠΎΠΎΡ€Π΄ΠΈΠ½Π°Ρ‚.

ΠŸΡ€ΠΈΠΌΠ΅Ρ€ 1

Π’Ρ‹Ρ‡ΠΈΡΠ»ΠΈΡ‚ΡŒ Π΄Π»ΠΈΠ½Ρƒ Π²Π΅ΠΊΡ‚ΠΎΡ€Π° aβ†’=7;e, Π·Π°Π΄Π°Π½Π½ΠΎΠ³ΠΎ Π² ΠΏΡ€ΡΠΌΠΎΡƒΠ³ΠΎΠ»ΡŒΠ½ΠΎΠΉ систСмС ΠΊΠΎΠΎΡ€Π΄ΠΈΠ½Π°Ρ‚.

РСшСниС

Π§Ρ‚ΠΎΠ±Ρ‹ Π½Π°ΠΉΡ‚ΠΈ Π΄Π»ΠΈΠ½Ρƒ Π²Π΅ΠΊΡ‚ΠΎΡ€Π°, Π±ΡƒΠ΄Π΅ΠΌ ΠΈΡΠΏΠΎΠ»ΡŒΠ·ΠΎΠ²Π°Ρ‚ΡŒ Ρ„ΠΎΡ€ΠΌΡƒΠ»Ρƒ нахоТдСния Π΄Π»ΠΈΠ½Ρ‹ Π²Π΅ΠΊΡ‚ΠΎΡ€Π° ΠΏΠΎ ΠΊΠΎΠΎΡ€Π΄ΠΈΠ½Π°Ρ‚Π°ΠΌaβ†’=ax2+ay2:Β aβ†’=72+e2=49+e

ΠžΡ‚Π²Π΅Ρ‚:Β aβ†’=49+e.

Π€ΠΎΡ€ΠΌΡƒΠ»Π° для нахоТдСния Π΄Π»ΠΈΠ½Ρ‹ Π²Π΅ΠΊΡ‚ΠΎΡ€Π° aβ†’=ax;ay;azΒ ΠΏΠΎ Π΅Π³ΠΎ ΠΊΠΎΠΎΡ€Π΄ΠΈΠ½Π°Ρ‚Π°ΠΌ Π² Π΄Π΅ΠΊΠ°Ρ€Ρ‚ΠΎΠ²ΠΎΠΉ систСмС ΠΊΠΎΠΎΡ€Π΄ΠΈΠ½Π°Ρ‚ Oxyz Π² пространствС, выводится Π°Π½Π°Π»ΠΎΠ³ΠΈΡ‡Π½ΠΎ Ρ„ΠΎΡ€ΠΌΡƒΠ»Π΅ для случая Π½Π° плоскости (см. рисунок Π½ΠΈΠΆΠ΅)

Π’ Π΄Π°Π½Π½ΠΎΠΌ случаС OA2=OAx2+OAy2+OAz2 (Ρ‚Π°ΠΊ ΠΊΠ°ΠΊ ОА – диагональ ΠΏΡ€ΡΠΌΠΎΡƒΠ³ΠΎΠ»ΡŒΠ½ΠΎΠ³ΠΎ ΠΏΠ°Ρ€Π°Π»Π»Π΅Π»Π΅ΠΏΠΈΠΏΠ΅Π΄Π°), ΠΎΡ‚ΡΡŽΠ΄Π° OA=OAx2+OAy2+OAz2. Из опрСдСлСния ΠΊΠΎΠΎΡ€Π΄ΠΈΠ½Π°Ρ‚ Π²Π΅ΠΊΡ‚ΠΎΡ€Π° ΠΌΠΎΠΆΠ΅ΠΌ Π·Π°ΠΏΠΈΡΠ°Ρ‚ΡŒ ΡΠ»Π΅Π΄ΡƒΡŽΡ‰ΠΈΠ΅ равСнства OAx=ax;Β OAy=ay;Β OAz=az;Β , Π° Π΄Π»ΠΈΠ½Π° ОА Ρ€Π°Π²Π½Π° Π΄Π»ΠΈΠ½Π΅ Π²Π΅ΠΊΡ‚ΠΎΡ€Π°, ΠΊΠΎΡ‚ΠΎΡ€ΡƒΡŽ ΠΌΡ‹ ΠΈΡ‰Π΅ΠΌ, ΡΠ»Π΅Π΄ΠΎΠ²Π°Ρ‚Π΅Π»ΡŒΠ½ΠΎ, OAβ†’=OAx2+OAy2+OAz2.

ΠžΡ‚ΡΡŽΠ΄Π° слСдуСт, Ρ‡Ρ‚ΠΎ Π΄Π»ΠΈΠ½Π° Π²Π΅ΠΊΡ‚ΠΎΡ€Π° aβ†’=ax;ay;az Ρ€Π°Π²Π½Π° aβ†’=ax2+ay2+az2.

ΠŸΡ€ΠΈΠΌΠ΅Ρ€ 2

Π’Ρ‹Ρ‡ΠΈΡΠ»ΠΈΡ‚ΡŒ Π΄Π»ΠΈΠ½Ρƒ Π²Π΅ΠΊΡ‚ΠΎΡ€Π° aβ†’=4Β·iβ†’-3Β·jβ†’+5Β·kβ†’, Π³Π΄Π΅ iβ†’,jβ†’,kβ†’ — ΠΎΡ€Ρ‚Ρ‹ ΠΏΡ€ΡΠΌΠΎΡƒΠ³ΠΎΠ»ΡŒΠ½ΠΎΠΉ систСмы ΠΊΠΎΠΎΡ€Π΄ΠΈΠ½Π°Ρ‚.

РСшСниС

Π”Π°Π½ΠΎ Ρ€Π°Π·Π»ΠΎΠΆΠ΅Π½ΠΈΠ΅ Π²Π΅ΠΊΡ‚ΠΎΡ€Π° aβ†’=4Β·iβ†’-3Β·jβ†’+5Β·kβ†’, Π΅Π³ΠΎ ΠΊΠΎΠΎΡ€Π΄ΠΈΠ½Π°Ρ‚Ρ‹ Ρ€Π°Π²Π½Ρ‹ aβ†’=4,-3,5. Π˜ΡΠΏΠΎΠ»ΡŒΠ·ΡƒΡ Π²Ρ‹ΡˆΠ΅ Π²Ρ‹Π²Π΅Π΄Π΅Π½Π½ΡƒΡŽ Ρ„ΠΎΡ€ΠΌΡƒΠ»Ρƒ ΠΏΠΎΠ»ΡƒΡ‡ΠΈΠΌ aβ†’=ax2+ay2+az2=42+(-3)2+52=52.

ΠžΡ‚Π²Π΅Ρ‚:aβ†’=52.

Π”Π»ΠΈΠ½Π° Π²Π΅ΠΊΡ‚ΠΎΡ€Π° Ρ‡Π΅Ρ€Π΅Π· ΠΊΠΎΠΎΡ€Π΄ΠΈΠ½Π°Ρ‚Ρ‹ Ρ‚ΠΎΡ‡Π΅ΠΊ Π΅Π³ΠΎ Π½Π°Ρ‡Π°Π»Π° ΠΈ ΠΊΠΎΠ½Ρ†Π°

Π’Ρ‹ΡˆΠ΅ Π±Ρ‹Π»ΠΈ Π²Ρ‹Π²Π΅Π΄Π΅Π½Ρ‹ Ρ„ΠΎΡ€ΠΌΡƒΠ»Ρ‹, ΠΏΠΎΠ·Π²ΠΎΠ»ΡΡŽΡ‰ΠΈΠ΅ Π½Π°Ρ…ΠΎΠ΄ΠΈΡ‚ΡŒ Π΄Π»ΠΈΠ½Ρ‹ Π²Π΅ΠΊΡ‚ΠΎΡ€Π° ΠΏΠΎ Π΅Π³ΠΎ ΠΊΠΎΠΎΡ€Π΄ΠΈΠ½Π°Ρ‚Π°ΠΌ. ΠœΡ‹ рассмотрСли случаи Π½Π° плоскости ΠΈ Π² Ρ‚Ρ€Π΅Ρ…ΠΌΠ΅Ρ€Π½ΠΎΠΌ пространствС. Π’ΠΎΡΠΏΠΎΠ»ΡŒΠ·ΡƒΠ΅ΠΌΡΡ ΠΈΠΌΠΈ для нахоТдСния ΠΊΠΎΠΎΡ€Π΄ΠΈΠ½Π°Ρ‚ Π²Π΅ΠΊΡ‚ΠΎΡ€Π° ΠΏΠΎ ΠΊΠΎΠΎΡ€Π΄ΠΈΠ½Π°Ρ‚Π°ΠΌ Ρ‚ΠΎΡ‡Π΅ΠΊ Π΅Π³ΠΎ Π½Π°Ρ‡Π°Π»Π° ΠΈ ΠΊΠΎΠ½Ρ†Π°.

Π˜Ρ‚Π°ΠΊ, Π΄Π°Π½Ρ‹ Ρ‚ΠΎΡ‡ΠΊΠΈ с Π·Π°Π΄Π°Π½Π½Ρ‹ΠΌΠΈ ΠΊΠΎΠΎΡ€Π΄ΠΈΠ½Π°Ρ‚Π°ΠΌΠΈ A(ax;ay)Β ΠΈ B(bx;by), ΠΎΡ‚ΡΡŽΠ΄Π° Π²Π΅ΠΊΡ‚ΠΎΡ€ ABβ†’Β ΠΈΠΌΠ΅Π΅Ρ‚ ΠΊΠΎΠΎΡ€Π΄ΠΈΠ½Π°Ρ‚Ρ‹ (bx-ax;Β by-ay)Π·Π½Π°Ρ‡ΠΈΡ‚, Π΅Π³ΠΎ Π΄Π»ΠΈΠ½Π° ΠΌΠΎΠΆΠ΅Ρ‚ Π±Ρ‹Ρ‚ΡŒ ΠΎΠΏΡ€Π΅Π΄Π΅Π»Π΅Π½Π° ΠΏΠΎ Ρ„ΠΎΡ€ΠΌΡƒΠ»Π΅: ABβ†’=(bx-ax)2+(by-ay)2

А Ссли Π΄Π°Π½Ρ‹ Ρ‚ΠΎΡ‡ΠΊΠΈ с Π·Π°Π΄Π°Π½Π½Ρ‹ΠΌΠΈ ΠΊΠΎΠΎΡ€Π΄ΠΈΠ½Π°Ρ‚Π°ΠΌΠΈ A(ax;ay;az)Β ΠΈ B(bx;by;bz) Π² Ρ‚Ρ€Π΅Ρ…ΠΌΠ΅Ρ€Π½ΠΎΠΌ пространствС, Ρ‚ΠΎ Π΄Π»ΠΈΠ½Ρƒ Π²Π΅ΠΊΡ‚ΠΎΡ€Π° ABβ†’Β ΠΌΠΎΠΆΠ½ΠΎ Π²Ρ‹Ρ‡ΠΈΡΠ»ΠΈΡ‚ΡŒ ΠΏΠΎ Ρ„ΠΎΡ€ΠΌΡƒΠ»Π΅

AB→=(bx-ax)2+(by-ay)2+(bz-az)2

ΠŸΡ€ΠΈΠΌΠ΅Ρ€ 3

Найти Π΄Π»ΠΈΠ½Ρƒ Π²Π΅ΠΊΡ‚ΠΎΡ€Π° ABβ†’, Ссли Π² ΠΏΡ€ΡΠΌΠΎΡƒΠ³ΠΎΠ»ΡŒΠ½ΠΎΠΉ систСмС ΠΊΠΎΠΎΡ€Π΄ΠΈΠ½Π°Ρ‚ A1,Β 3,Β B-3,Β 1.

РСшСниС

Π˜ΡΠΏΠΎΠ»ΡŒΠ·ΡƒΡ Ρ„ΠΎΡ€ΠΌΡƒΠ»Ρƒ нахоТдСния Π΄Π»ΠΈΠ½Ρ‹ Π²Π΅ΠΊΡ‚ΠΎΡ€Π° ΠΏΠΎ ΠΊΠΎΠΎΡ€Π΄ΠΈΠ½Π°Ρ‚Π°ΠΌ Ρ‚ΠΎΡ‡Π΅ΠΊ Π½Π°Ρ‡Π°Π»Π° ΠΈ ΠΊΠΎΠ½Ρ†Π° Π½Π° плоскости, ΠΏΠΎΠ»ΡƒΡ‡ΠΈΠΌ ABβ†’=(bx-ax)2+(by-ay)2: ABβ†’=(-3-1)2+(1-3)2=20-23.

Π’Ρ‚ΠΎΡ€ΠΎΠΉ Π²Π°Ρ€ΠΈΠ°Π½Ρ‚ Ρ€Π΅ΡˆΠ΅Π½ΠΈΡ ΠΏΠΎΠ΄Ρ€Π°Π·ΡƒΠΌΠ΅Π²Π°Π΅Ρ‚ ΠΏΠΎΠ΄ собой ΠΏΡ€ΠΈΠΌΠ΅Π½Π΅Π½ΠΈΠ΅ Π΄Π°Π½Π½Ρ‹Ρ… Ρ„ΠΎΡ€ΠΌΡƒΠ» ΠΏΠΎ ΠΎΡ‡Π΅Ρ€Π΅Π΄ΠΈ: ABβ†’=(-3-1;Β 1-3)=(-4;Β 1-3); ABβ†’=(-4)2+(1-3)2=20-23.-

ΠžΡ‚Π²Π΅Ρ‚:Β ABβ†’=20-23.

ΠŸΡ€ΠΈΠΌΠ΅Ρ€ 4

ΠžΠΏΡ€Π΅Π΄Π΅Π»ΠΈΡ‚ΡŒ, ΠΏΡ€ΠΈ ΠΊΠ°ΠΊΠΈΡ… значСниях Β Π΄Π»ΠΈΠ½Π° Π²Π΅ΠΊΡ‚ΠΎΡ€Π° ABβ†’ Ρ€Π°Π²Π½Π° 30, СслиA(0,Β 1,Β 2);Β B(5,Β 2,Β Ξ»2)Β .

РСшСниС

Для Π½Π°Ρ‡Π°Π»Π° Ρ€Π°ΡΠΏΠΈΡˆΠ΅ΠΌ Π΄Π»ΠΈΠ½Ρƒ Π²Π΅ΠΊΡ‚ΠΎΡ€Π° ABβ†’Β ΠΏΠΎ Ρ„ΠΎΡ€ΠΌΡƒΠ»Π΅: ABβ†’=(bx-ax)2+(by-ay)2+(bz-az)2=(5-0)2+(2-1)2+(Ξ»2-2)2=26+(Ξ»2-2)2

Π—Π°Ρ‚Π΅ΠΌ ΠΏΠΎΠ»ΡƒΡ‡Π΅Π½Π½ΠΎΠ΅ Π²Ρ‹Ρ€Π°ΠΆΠ΅Π½ΠΈΠ΅ приравняСм ΠΊ 30, ΠΎΡ‚ΡΡŽΠ΄Π° Π½Π°ΠΉΠ΄Π΅ΠΌ искомыС Ξ»:

Β 26+(Ξ»2-2)2=3026+(Ξ»2-2)2=30(Ξ»2-2)2=4Ξ»2-2=2Β ΠΈΠ»ΠΈΒ Ξ»2-2=-2Β Β Ξ»1=-2,Β Ξ»2=2,Β Ξ»3=0.

ΠžΡ‚Π²Π΅Ρ‚:Β Ξ»1=-2,Β Ξ»2=2,Β Ξ»3=0.

НахоТдСниС Π΄Π»ΠΈΠ½Ρ‹ Π²Π΅ΠΊΡ‚ΠΎΡ€Π° ΠΏΠΎ Ρ‚Π΅ΠΎΡ€Π΅ΠΌΠ΅ косинусов

Π£Π²Ρ‹, Π½ΠΎ Π² Π·Π°Π΄Π°Ρ‡Π°Ρ… Π½Π΅ всСгда Π±Ρ‹Π²Π°ΡŽΡ‚ извСстны ΠΊΠΎΠΎΡ€Π΄ΠΈΠ½Π°Ρ‚Ρ‹ Π²Π΅ΠΊΡ‚ΠΎΡ€Π°, поэтому рассмотрим Π΄Ρ€ΡƒΠ³ΠΈΠ΅ способы нахоТдСния Π΄Π»ΠΈΠ½Ρ‹ Π²Π΅ΠΊΡ‚ΠΎΡ€Π°.

ΠŸΡƒΡΡ‚ΡŒ Π·Π°Π΄Π°Π½Ρ‹ Π΄Π»ΠΈΠ½Ρ‹ Π΄Π²ΡƒΡ… Π²Π΅ΠΊΡ‚ΠΎΡ€ΠΎΠ² ABβ†’, ACβ†’Β ΠΈ ΡƒΠ³ΠΎΠ» ΠΌΠ΅ΠΆΠ΄Ρƒ Π½ΠΈΠΌΠΈ (ΠΈΠ»ΠΈ косинус ΡƒΠ³Π»Π°), Π° трСбуСтся Π½Π°ΠΉΡ‚ΠΈ Π΄Π»ΠΈΠ½Ρƒ Π²Π΅ΠΊΡ‚ΠΎΡ€Π° BCβ†’Β ΠΈΠ»ΠΈ CBβ†’. Π’ Ρ‚Π°ΠΊΠΎΠΌ случаС, слСдуСт Π²ΠΎΡΠΏΠΎΠ»ΡŒΠ·ΠΎΠ²Π°Ρ‚ΡŒΡΡ Ρ‚Π΅ΠΎΡ€Π΅ΠΌΠΎΠΉ косинусов Π² Ρ‚Ρ€Π΅ΡƒΠ³ΠΎΠ»ΡŒΠ½ΠΈΠΊΠ΅ β–³ABC, Π²Ρ‹Ρ‡ΠΈΡΠ»ΠΈΡ‚ΡŒ Π΄Π»ΠΈΠ½Ρƒ стороны BC, которая ΠΈ Ρ€Π°Π²Π½Π° искомой Π΄Π»ΠΈΠ½Π΅ Π²Π΅ΠΊΡ‚ΠΎΡ€Π°.

Рассмотрим Ρ‚Π°ΠΊΠΎΠΉ случай Π½Π° ΡΠ»Π΅Π΄ΡƒΡŽΡ‰Π΅ΠΌ ΠΏΡ€ΠΈΠΌΠ΅Ρ€Π΅.

ΠŸΡ€ΠΈΠΌΠ΅Ρ€ 5

Π”Π»ΠΈΠ½Ρ‹ Π²Π΅ΠΊΡ‚ΠΎΡ€ΠΎΠ² ABβ†’ ΠΈ ACβ†’ Ρ€Π°Π²Π½Ρ‹ 3 ΠΈ 7 соотвСтствСнно, Π° ΡƒΠ³ΠΎΠ» ΠΌΠ΅ΠΆΠ΄Ρƒ Π½ΠΈΠΌΠΈ Ρ€Π°Π²Π΅Π½ Ο€3. Π’Ρ‹Ρ‡ΠΈΡΠ»ΠΈΡ‚ΡŒ Π΄Π»ΠΈΠ½Ρƒ Π²Π΅ΠΊΡ‚ΠΎΡ€Π° BCβ†’.

РСшСниС

Π”Π»ΠΈΠ½Π° Π²Π΅ΠΊΡ‚ΠΎΡ€Π° BCβ†’ Π² Π΄Π°Π½Π½ΠΎΠΌ случаС Ρ€Π°Π²Π½Π° Π΄Π»ΠΈΠ½Π΅ стороны BCΒ Ρ‚Ρ€Π΅ΡƒΠ³ΠΎΠ»ΡŒΠ½ΠΈΠΊΠ° β–³ABC. Π”Π»ΠΈΠ½Ρ‹ сторон AB ΠΈΒ ACΒ Ρ‚Ρ€Π΅ΡƒΠ³ΠΎΠ»ΡŒΠ½ΠΈΠΊΠ° извСстны ΠΈΠ· условия (ΠΎΠ½ΠΈ Ρ€Π°Π²Π½Ρ‹ Π΄Π»ΠΈΠ½Π°ΠΌ ΡΠΎΠΎΡ‚Π²Π΅Ρ‚ΡΡ‚Π²ΡƒΡŽΡ‰ΠΈΡ… Π²Π΅ΠΊΡ‚ΠΎΡ€ΠΎΠ²), Ρ‚Π°ΠΊΠΆΠ΅ извСстСн ΡƒΠ³ΠΎΠ» ΠΌΠ΅ΠΆΠ΄Ρƒ Π½ΠΈΠΌΠΈ, поэтому ΠΌΡ‹ ΠΌΠΎΠΆΠ΅ΠΌ Π²ΠΎΡΠΏΠΎΠ»ΡŒΠ·ΠΎΠ²Π°Ρ‚ΡŒΡΡ Ρ‚Π΅ΠΎΡ€Π΅ΠΌΠΎΠΉ косинусов:BC2=AB2+AC2-2Β·ABΒ·ACΒ·cos∠(AB,β†’ACβ†’)=32+72-2Β·3Β·7Β·cosΟ€3=37Β β‡’BC=37 Π’Π°ΠΊΠΈΠΌ ΠΎΠ±Ρ€Π°Π·ΠΎΠΌ, BCβ†’=37.

ΠžΡ‚Π²Π΅Ρ‚:BCβ†’=37.

Π˜Ρ‚Π°ΠΊ, для нахоТдСния Π΄Π»ΠΈΠ½Ρ‹ Π²Π΅ΠΊΡ‚ΠΎΡ€Π° ΠΏΠΎ ΠΊΠΎΠΎΡ€Π΄ΠΈΠ½Π°Ρ‚Π°ΠΌ ΡΡƒΡ‰Π΅ΡΡ‚Π²ΡƒΡŽΡ‚ ΡΠ»Π΅Π΄ΡƒΡŽΡ‰ΠΈΠ΅ Ρ„ΠΎΡ€ΠΌΡƒΠ»Ρ‹ aβ†’=ax2+ay2 ΠΈΠ»ΠΈ aβ†’=ax2+ay2+az2, ΠΏΠΎ ΠΊΠΎΠΎΡ€Π΄ΠΈΠ½Π°Ρ‚Π°ΠΌ Ρ‚ΠΎΡ‡Π΅ΠΊ Π½Π°Ρ‡Π°Π»Π° ΠΈ ΠΊΠΎΠ½Ρ†Π° Π²Π΅ΠΊΡ‚ΠΎΡ€Π° ABβ†’=(bx-ax)2+(by-ay)2Β ΠΈΠ»ΠΈ ABβ†’=(bx-ax)2+(by-ay)2+(bz-az)2, Π² Π½Π΅ΠΊΠΎΡ‚ΠΎΡ€Ρ‹Ρ… случаях слСдуСт ΠΈΡΠΏΠΎΠ»ΡŒΠ·ΠΎΠ²Π°Ρ‚ΡŒ Ρ‚Π΅ΠΎΡ€Π΅ΠΌΡƒ косинусов.

РСшСниС Π·Π°Π΄Π°Ρ‡ ΠΎΡ‚Β 1 дня / ΠΎΡ‚Β 150Β Ρ€. ΠšΡƒΡ€ΡΠΎΠ²Π°Ρ Ρ€Π°Π±ΠΎΡ‚Π° ΠΎΡ‚Β 5Β Π΄Π½Π΅ΠΉ / ΠΎΡ‚Β 1800Β Ρ€. Π Π΅Ρ„Π΅Ρ€Π°Ρ‚ ΠΎΡ‚Β 1 дня / ΠΎΡ‚Β 700Β Ρ€.

Автор: Π˜Ρ€ΠΈΠ½Π° ΠœΠ°Π»ΡŒΡ†Π΅Π²ΡΠΊΠ°Ρ

ΠŸΡ€Π΅ΠΏΠΎΠ΄Π°Π²Π°Ρ‚Π΅Π»ΡŒ ΠΌΠ°Ρ‚Π΅ΠΌΠ°Ρ‚ΠΈΠΊΠΈ ΠΈ ΠΈΠ½Ρ„ΠΎΡ€ΠΌΠ°Ρ‚ΠΈΠΊΠΈ. ΠšΠ°Ρ„Π΅Π΄Ρ€Π° бизнСс-ΠΈΠ½Ρ„ΠΎΡ€ΠΌΠ°Ρ‚ΠΈΠΊΠΈ Российского унивСрситСта транспорта

ВычислСниС Π΄Π»ΠΈΠ½Ρ‹ Π²Π΅ΠΊΡ‚ΠΎΡ€Π° — ΠŸΡ€ΠΎΠ³Ρ€Π°ΠΌΠΌΠ½Ρ‹Π΅ вопросы

#1 OFFLINE Β  Onix

ΠžΡ‚ΠΏΡ€Π°Π²Π»Π΅Π½ΠΎ 21 Август 2008 — 14:01

Как ΠΌΠΎΠΆΠ½ΠΎ ΡƒΠ·Π½Π°Ρ‚ΡŒ ΠΈΠ»ΠΈ Π²Ρ‹Ρ‡ΠΈΡΠ»ΠΈΡ‚ΡŒ ΠΏΡ€ΠΈ ΠΏΠΎΠΌΠΎΡ‰ΠΈ ΠΊΠ°ΠΊΠΎΠΉ ΠΏΡ€ΠΎΠ³Ρ€Π°ΠΌΠΌΡ‹ Π΄Π»ΠΈΠ½Ρƒ Π²Π΅ΠΊΡ‚ΠΎΡ€Π°? Π’ ΠΡ€Ρ‚ΠšΠ°ΠΌΠ΅ Π΅ΡΡ‚ΡŒ, Π½ΠΎ Ρ‚Π°ΠΌ Π½ΡƒΠΆΠ½ΠΎ Π²Ρ‹Π±ΠΈΡ€Π°Ρ‚ΡŒ ΠΏΠΎ ΠΎΠ΄Π½ΠΎΠΌΡƒ Π²Π΅ΠΊΡ‚ΠΎΡ€Ρƒ
Π—Π°Ρ€Π°Π½Π΅Π΅ Π±Π»Π°Π³ΠΎΠ΄Π°Ρ€Π΅Π½!
Β 

ΠŸΠΎΡ…ΠΎΠΆΠΈΠ΅ Ρ‚Π΅ΠΌΡ‹:
Как Π² ΠΡ€Ρ‚ΠšΠ°ΠΌΠ΅ ΠΏΠΎΡΡ‡ΠΈΡ‚Π°Ρ‚ΡŒ ΠΏΠ΅Ρ€ΠΈΠΌΠ΅Ρ‚Ρ€ Π²Ρ‹Ρ€Π΅Π·Π°Π΅ΠΌΠΎΠ³ΠΎ ΠΎΠ±ΡŠΠ΅ΠΊΡ‚Π°?
ΠŸΡ€ΠΎΠ³Ρ€Π°ΠΌΠΌΠ° для вычислСниС обьСма Π΄Π΅Ρ‚Π°Π»ΠΈ


  • НавСрх

#2 OFFLINE Β  vv92

ΠžΡ‚ΠΏΡ€Π°Π²Π»Π΅Π½ΠΎ 21 Август 2008 — 14:03

plotcalc. com

Π—Π½Π°ΡŽ Ρ‚Π΅Ρ…Π½ΠΈΠΊΡƒ бСзопасности ΠΊΠ°ΠΊ свои Ρ‚Ρ€ΠΈ ΠΏΠ°Π»ΡŒΡ†Π°.ЭкспСрт — это сущСство, ΠΊΠΎΡ‚ΠΎΡ€ΠΎΠ΅ пСрСстало ΠΌΡ‹ΡΠ»ΠΈΡ‚ΡŒ, ΠΈΠ±ΠΎ ΠΎΠ½ΠΎ Π·Π½Π°Π΅Ρ‚!Π’ ΠΌΠΈΡ€Π΅ Π΅Ρ‰Π΅ ΠΌΠ½ΠΎΠ³ΠΎ Π³Ρ€Π°Π±Π»Π΅ΠΉ, Π½Π° ΠΊΠΎΡ‚ΠΎΡ€Ρ‹Π΅ Π½Π΅ ступала Π½ΠΎΠ³Π° Ρ‡Π΅Π»ΠΎΠ²Π΅ΠΊΠ°.
ΠŸΠΎΠΆΠ°Π»ΡƒΠΉΡΡ‚Π°! Π˜ΡΠΏΡ€Π°Π²Π»ΡΠΉΡ‚Π΅ ΠΌΠΎΠΈ Π³Π»ΡƒΠΏΡ‹Π΅ ошибки (Π½ΠΎ ΠΎΡΡ‚Π°Π²ΡŒΡ‚Π΅ ΠΌΠΎΠΈ ΡƒΠΌΠ½Ρ‹Π΅ ошибки)!

  • НавСрх

#3 OFFLINE Β  Sherak

ΠžΡ‚ΠΏΡ€Π°Π²Π»Π΅Π½ΠΎ 21 Август 2008 — 14:11

Π•ΡΡ‚ΡŒ ΠΎΡ‡Π΅Π½ΡŒ ΡƒΠ΄ΠΎΠ±Π½Ρ‹ΠΉ ΠΏΠ»Π°Π³ΠΈΠ½ для CorelDraw — Measure Perimeter называСтся. Β measure_perimeter_1_1.zip Β Β 9,01К Β Β 2892 скачиваний

WBR, Anatoly

  • НавСрх

#4 OFFLINE Β  Onix

ΠžΡ‚ΠΏΡ€Π°Π²Π»Π΅Π½ΠΎ 21 Август 2008 — 14:26

Бпасибо Π±ΡƒΠ΄Π΅ΠΌ ΠΏΡ€ΠΎΠ±ΠΎΠ²Π°Ρ‚ΡŒ )))!
П. Π‘. Π’Π΅ΡΡŒΠΌΠ° Π΄ΠΎΠ²ΠΎΠ»Π΅Π½ Ρ„ΠΎΡ€ΡƒΠΌΠΎΠΌ!!!

Π‘ΠΎΠΎΠ±Ρ‰Π΅Π½ΠΈΠ΅ ΠΎΡ‚Ρ€Π΅Π΄Π°ΠΊΡ‚ΠΈΡ€ΠΎΠ²Π°Π» 3D-BiG: 21 Август 2008 — 15:10

  • НавСрх

#5 OFFLINE Β  Sherak

ΠžΡ‚ΠΏΡ€Π°Π²Π»Π΅Π½ΠΎ 21 Август 2008 — 14:40

http://plotcalc.com/

Π‘Π»ΠΈΠ½, ΠΎΠ½Π° платная…

WBR, Anatoly

  • НавСрх

#6 OFFLINE Β  3D-BiG

ΠžΡ‚ΠΏΡ€Π°Π²Π»Π΅Π½ΠΎ 21 Август 2008 — 15:18

Sherak:
ВсС Π² этом ΠΌΠΈΡ€Π΅ ΠΈΠΌΠ΅Π΅Ρ‚ Π΄Π²Π΅ цСнности, хотя plotcalc написан русским программистом ΠΈ стоит чСстно говоря — ΠΊΠΎΠΏΠ΅ΠΉΠΊΠΈ, Π° Π΄Π°Π΅Ρ‚ экономию — бааальшиС Ρ€ΡƒΠ±Π»ΠΈ. .. А ряд Ρ„ΡƒΠ½ΠΊΡ†ΠΈΠΉ Ρ€Π°Π±ΠΎΡ‚Π°ΡŽΡ‚ ΠΈ Π² Π΄Π΅ΠΌΠΎ-Ρ€Π΅ΠΆΠΈΠΌΠ΅…

  • НавСрх

#7 OFFLINE Β  vv92

ΠžΡ‚ΠΏΡ€Π°Π²Π»Π΅Π½ΠΎ 21 Август 2008 — 15:53

Π˜Π·ΠΌΠ΅Ρ€Π΅Π½ΠΈΠ΅ Π΄Π»ΠΈΠ½Ρ‹ Ρ€Π°Π±ΠΎΡ‚Π°Π΅Ρ‚ Π² Π΄Π΅ΠΌΠΊΠ΅.

Π—Π½Π°ΡŽ Ρ‚Π΅Ρ…Π½ΠΈΠΊΡƒ бСзопасности ΠΊΠ°ΠΊ свои Ρ‚Ρ€ΠΈ ΠΏΠ°Π»ΡŒΡ†Π°.ЭкспСрт — это сущСство, ΠΊΠΎΡ‚ΠΎΡ€ΠΎΠ΅ пСрСстало ΠΌΡ‹ΡΠ»ΠΈΡ‚ΡŒ, ΠΈΠ±ΠΎ ΠΎΠ½ΠΎ Π·Π½Π°Π΅Ρ‚!Π’ ΠΌΠΈΡ€Π΅ Π΅Ρ‰Π΅ ΠΌΠ½ΠΎΠ³ΠΎ Π³Ρ€Π°Π±Π»Π΅ΠΉ, Π½Π° ΠΊΠΎΡ‚ΠΎΡ€Ρ‹Π΅ Π½Π΅ ступала Π½ΠΎΠ³Π° Ρ‡Π΅Π»ΠΎΠ²Π΅ΠΊΠ°.
ΠŸΠΎΠΆΠ°Π»ΡƒΠΉΡΡ‚Π°! Π˜ΡΠΏΡ€Π°Π²Π»ΡΠΉΡ‚Π΅ ΠΌΠΎΠΈ Π³Π»ΡƒΠΏΡ‹Π΅ ошибки (Π½ΠΎ ΠΎΡΡ‚Π°Π²ΡŒΡ‚Π΅ ΠΌΠΎΠΈ ΡƒΠΌΠ½Ρ‹Π΅ ошибки)!

  • НавСрх

#8 OFFLINE Β  Onix

ΠžΡ‚ΠΏΡ€Π°Π²Π»Π΅Π½ΠΎ 21 Август 2008 — 18:35

НашСл Π»ΠΎΠΌΠ°Π½ΡƒΡŽ Π²Π΅Ρ€ΡΠΈΡŽ, Ρ‚ΠΎΠ»ΡŒΠΊΠΎ ΠΏΡ€ΠΎΠ±Π»Π΅ΠΌΠ° Π² Ρ‚ΠΎΠΌ Ρ‡Ρ‚ΠΎ Ρƒ мСня ΠšΠΎΡ€Π΅Π» 14 Π° ΠΎΠ½ Ρ‚ΠΎΠ»ΡŒΠΊΠΎ для 13 написал, ΠΏΠ΅Ρ€Π΅Π²Π΅ΡˆΠΈΠ²Π°Ρ‚ΡŒ нСхочСтся, Π° Π² ΠΏΡ€ΠΈΠ½Ρ†Π΅ΠΏΠ΅ Ρ…ΠΎΡ€ΠΎΡˆΠ°Ρ софтина!
Пока measure perimeter мСня устраиваСт!

  • НавСрх

#9 OFFLINE Β  Buzl

ΠžΡ‚ΠΏΡ€Π°Π²Π»Π΅Π½ΠΎ 22 Август 2008 — 10:44

Π’ Π°Ρ€Ρ‚ΠΊΠ°ΠΌΠ΅ ΠΊΠΎΠ³Π΄Π° создаСшь УП Ρ‚Π°ΠΌ Π΅ΡΡ‚ΡŒ Π·Π½Π°Ρ‡ΠΎΠΊ Π±Π»ΠΎΠΊΠ½ΠΎΡ‚ΠΈΠΊΠ° Ρ‚Π°ΠΌ ΠΏΠΈΡˆΠ΅Ρ‚ ΠΈ Π΄Π»ΠΈΠ½Ρƒ Ρ€Π΅Π·Π° ΠΈ свободного пСрСмСщСния ΠΈ врСмя

  • НавСрх

#10 OFFLINE Β  woodman

ΠžΡ‚ΠΏΡ€Π°Π²Π»Π΅Π½ΠΎ 01 Π‘Π΅Π½Ρ‚ΡΠ±Ρ€ΡŒ 2008 — 12:41

Π² ΠΏΡ€ΠΎΠ³Ρ€Π°ΠΌΠΌΠ΅ TypeEdit (Type3) Π΅ΡΡ‚ΡŒ такая функция. Π’Ρ‹Π±ΠΈΡ€Π°Π΅ΡˆΡŒ любой Π²Π΅ΠΊΡ‚ΠΎΡ€Π½Ρ‹ΠΉ ΠΎΠ±ΡŠΠ΅ΠΊΡ‚ наТимаСшь ΡΠΎΠΎΡ‚Π²Π΅Ρ‚ΡΠ²ΡƒΡŽΡ‰ΡƒΡŽ ΠΊΠ½ΠΎΠΏΠΊΡƒ ΠΈ ΠΏΠΎΠ»ΡƒΡ‡Π°Π΅ΡˆΡŒ Ρ€Π΅Π·ΡƒΠ»ΡŒΡ‚Π°Ρ‚.

  • НавСрх

#11 OFFLINE Β  wizard

ΠžΡ‚ΠΏΡ€Π°Π²Π»Π΅Π½ΠΎ 01 Π‘Π΅Π½Ρ‚ΡΠ±Ρ€ΡŒ 2008 — 13:16

Π― Π΄Π°Π²Π½ΠΎ ΠΏΠΎΠ»ΡŒΠ·ΡƒΡŽΡΡŒ ΠΏΡ€ΠΎΡΡ‚Π΅Π½ΡŒΠΊΠΎΠΉ ΠΏΡ€ΠΎΠ³Ρ€Π°ΠΌΠΌΠΊΠΎΠΉ PLTV.EXE. Π˜Π½ΡΡ‚Π°Π»ΡΡˆΠΊΠ° вСсит 100 с ΠΌΠ΅Π»ΠΎΡ‡ΡŒΡŽ ΠšΠΈΠ»ΠΎΠ±Π°ΠΉΡ‚.. И ΠΊΠ»ΠΈΠ΅Π½Ρ‚Π°ΠΌ ΠΏΠΎΡ‡Ρ‚ΠΈ всСм Π΅Π΅ пристроил.. Π’ΠΎΠΊΠΎ сначала приходится ΡΠΎΡ…Ρ€Π°Π½ΡΡ‚ΡŒ Π² PLT- Ρ„ΠΎΡ€ΠΌΠ°Ρ‚Π΅.. Если ΠΊΠΎΠΌΡƒ Π½Π°Π΄ΠΎ- ΠΊΠΈΠ½Ρƒ..

НС Π΄Π΅Π»Π°ΠΉ Π΄ΠΎΠ±Ρ€ΠΎ..НС бросай Π΅Π³ΠΎ Π² Π²ΠΎΠ΄Ρƒ..

  • НавСрх

#12 OFFLINE Β  vv92

ΠžΡ‚ΠΏΡ€Π°Π²Π»Π΅Π½ΠΎ 01 Π‘Π΅Π½Ρ‚ΡΠ±Ρ€ΡŒ 2008 — 13:19

Plotcalc ΡƒΠ΄ΠΎΠ±Π΅Π½ Ρ‚Π΅ΠΌ, Ρ‡Ρ‚ΠΎ ΠΌΠΎΠΆΠ½ΠΎ Ρ€Π°ΡΡ‡ΠΈΡ‚Π°Ρ‚ΡŒ ΡΡ‚ΠΎΠΈΠΌΠΎΡΡ‚ΡŒ (Π΄Π»ΠΈΠ½Ρƒ Ρ€Π΅Π·Π°) прямо ΠΈΠ· присланного Ρ„Π°ΠΉΠ»Π° (Π½Π΅ Π½ΡƒΠΆΠ½ΠΎ Π΄Π΅Π»Π°Ρ‚ΡŒ массы Π»ΠΈΡˆΠ½ΠΈΡ… ΠΎΠΏΠ΅Ρ€Π°Ρ†ΠΈΠΉ ΠΈ ΡΠ΄Π΅Π»Π°Ρ‚ΡŒ это ΠΌΠΎΠΆΠ΅Ρ‚ ΠΌΠ΅Π½Π΅Π΄ΠΆΠ΅Ρ€).

Π—Π½Π°ΡŽ Ρ‚Π΅Ρ…Π½ΠΈΠΊΡƒ бСзопасности ΠΊΠ°ΠΊ свои Ρ‚Ρ€ΠΈ ΠΏΠ°Π»ΡŒΡ†Π°.ЭкспСрт — это сущСство, ΠΊΠΎΡ‚ΠΎΡ€ΠΎΠ΅ пСрСстало ΠΌΡ‹ΡΠ»ΠΈΡ‚ΡŒ, ΠΈΠ±ΠΎ ΠΎΠ½ΠΎ Π·Π½Π°Π΅Ρ‚!Π’ ΠΌΠΈΡ€Π΅ Π΅Ρ‰Π΅ ΠΌΠ½ΠΎΠ³ΠΎ Π³Ρ€Π°Π±Π»Π΅ΠΉ, Π½Π° ΠΊΠΎΡ‚ΠΎΡ€Ρ‹Π΅ Π½Π΅ ступала Π½ΠΎΠ³Π° Ρ‡Π΅Π»ΠΎΠ²Π΅ΠΊΠ°.
ΠŸΠΎΠΆΠ°Π»ΡƒΠΉΡΡ‚Π°! Π˜ΡΠΏΡ€Π°Π²Π»ΡΠΉΡ‚Π΅ ΠΌΠΎΠΈ Π³Π»ΡƒΠΏΡ‹Π΅ ошибки (Π½ΠΎ ΠΎΡΡ‚Π°Π²ΡŒΡ‚Π΅ ΠΌΠΎΠΈ ΡƒΠΌΠ½Ρ‹Π΅ ошибки)!

  • НавСрх

#13 OFFLINE Β  Sherak

ΠžΡ‚ΠΏΡ€Π°Π²Π»Π΅Π½ΠΎ 01 Π‘Π΅Π½Ρ‚ΡΠ±Ρ€ΡŒ 2008 — 13:30

Если ΠΌΠ΅Π½Π΅Π΄ΠΆΠ΅Ρ€ ΡƒΠΌΠ΅Π΅Ρ‚ ΠΏΠΎΠ»ΡŒΠ·ΠΎΠ²Π°Ρ‚ΡŒΡΡ ΠšΠΎΡ€Π΅Π»ΠΎΠΌ, ΠΊΠ°ΠΊ Π² этом случаС, Ρ‚ΠΎ ΠΏΡ€ΠΎΡ‰Π΅ Ρ‚Π°ΠΊΠΈ Π²Ρ‹Π΄Π΅Π»ΠΈΡ‚ΡŒ всС, Ρ‡Ρ‚ΠΎ Π½ΡƒΠΆΠ½ΠΎ для Ρ€Π΅Π·ΠΊΠΈ ΠΈ ΠΆΠ°ΠΌΠΊΠ½ΡƒΡ‚ΡŒ ΠΊΠ½ΠΎΠΏΠΊΡƒ мСасура ΠΏΠ΅Ρ€ΠΈΠΌΠ΅Ρ‚Ρ€Π°. А ΠΏΠΎΠ»ΡƒΡ‡ΠΈΠ²ΡˆΡƒΡŽΡΡ Π΄Π»ΠΈΠ½Ρƒ ΡƒΠΌΠ½ΠΎΠΆΠΈΡ‚ΡŒ Π½Π° ΡΡ‚ΠΎΠΈΠΌΠΎΡΡ‚ΡŒ Π·Π° ΠΌΠ΅Ρ‚Ρ€ ΡƒΠΆΠ΅ Π½Π° ΠΊΠ°Π»ΡŒΠΊΡƒΠ»ΡΡ‚ΠΎΡ€Π΅.

WBR, Anatoly

  • НавСрх

#14 OFFLINE Β  rexti64

ΠžΡ‚ΠΏΡ€Π°Π²Π»Π΅Π½ΠΎ 01 Π‘Π΅Π½Ρ‚ΡΠ±Ρ€ΡŒ 2008 — 14:58

Если ΠΏΡ€ΠΎ Π΄Π»ΠΈΠ½Ρƒ Π²Π΅ΠΊΡ‚ΠΎΡ€ΠΎΠ², Ρ‚ΠΎ Π² Π°Ρ€ΠΊΠ°ΠΌΠ΅ Π²Ρ‹Π΄Π΅Π»ΡΠ΅ΡˆΡŒ Π²Π΅ΠΊΡ‚ΠΎΡ€ Ρ‰Π΅Π»ΠΊΠ°Π΅ΡˆΡŒ ΠΏΡ€Π°Π²ΠΎΠΉ ΠΊΠ½ΠΎΠΏΠΊΠΎΠΉ Π²Ρ‹Π±ΠΈΡ€Π°Π΅ΡˆΡŒ свойства ΠΈ Ρ‚Π°ΠΌ Ρ‚Π΅Π±Π΅ ΠΏΡ€ΠΎΠΏΠΈΡˆΠ΅Ρ‚ΡΡ всС ΠΈ Π΄Π»ΠΈΠ½Π° Π²Π΅ΠΊΡ‚ΠΎΡ€Π° ΠΈ ΠΏΠΎΡ‰Π°Π΄ΡŒ ΠΈ Ρ‚ΠΏ (Ρ‚ΠΎΠ»ΡŒΠΊΠΎ с Π΅Π΄ΠΈΠ½ΠΈΡ‡Π½Ρ‹ΠΌ Π²Π΅ΠΊΡ‚ΠΎΡ€ΠΎΠΌ). А Ссли ΠΏΡ€ΠΎ Π΄Π»ΠΈΠ½Ρƒ всСх Π²Ρ‹Π±Ρ€Π°Π½Π½Ρ‹Ρ… Π²Π΅Ρ‚ΠΎΡ€ΠΎΠ² — сам писал ΠΏΡ€ΠΎΠ³Ρ€Π°ΠΌΠΊΡƒ Π½Π° Π΄Π΅Π»ΡŒΡ„ΠΈ Ссли Π½Π°Π΄ΠΎ ΠΌΠΎΠ³Ρƒ ΡΠΊΠΈΠ½ΡƒΡ‚ΡŒ. Π’Ρ‹Π±ΠΈΡ€Π°Π΅ΡˆΡŒ всС Π²Π΅ΠΊΡ‚ΠΎΡ€Π° дСлаСшь УП ΠžΠ±Ρ€Π°Π±ΠΎΡ‚ΠΊΠ° вдоль Π²Π΅ΠΊΡ‚ΠΎΡ€ΠΎΠ² Π³Π»ΡƒΠ±ΠΈΠ½Π° любая Ρ„Ρ€Π΅Π·Π° любая ΠΈ ΡΠΎΡ…Ρ€Π°Π½ΡΠ΅ΡˆΡŒ Π΅Π΅ Π² Ρ„ΠΎΡ€ΠΌΠ°Ρ‚Π΅ Π²Ρ‹Ρ…ΠΎΠ΄Π½ΠΎΠ³ΠΎ Ρ„Π°ΠΉΠ»Π° Cipher (*.plt) (с этим Ρ„ΠΎΡ€ΠΌΠ°Ρ‚ΠΎΠΌ Ρ€Π°Π±ΠΎΡ‚Π°Π΅Ρ‚ моя ΠΏΡ€ΠΎΠ³Ρ€Π°ΠΌΠΌΠ°).

  • НавСрх

#15 OFFLINE Β  rexti64

ΠžΡ‚ΠΏΡ€Π°Π²Π»Π΅Π½ΠΎ 02 Π‘Π΅Π½Ρ‚ΡΠ±Ρ€ΡŒ 2008 — 08:41

Извиняюсь насчСт Π³Π»ΡƒΠ±ΠΈΠ½Ρ‹. Π“Π»ΡƒΠ±ΠΈΠ½Π° Π΄ΠΎΠ»ΠΆΠ½Π° Π±Ρ‹Ρ‚ΡŒ для ΠΏΡ€ΠΎΡ…ΠΎΠ΄Π° ΠΎΠ΄ΠΈΠ½ Ρ€Π°Π· Ρ„Ρ€Π΅Π·ΠΎΠΉ (я ΡΡ‚Π°Π²Π»ΡŽ 0).

  • НавСрх

#16 OFFLINE Β  Sherak

ΠžΡ‚ΠΏΡ€Π°Π²Π»Π΅Π½ΠΎ 17 ΠΠΎΡΠ±Ρ€ΡŒ 2008 — 08:26

На счСт ΠΏΠ»ΠΎΡ‚ΠΊΠ°Π»ΠΊΠ°. 2 мСсяца Ρ€Π°Π±ΠΎΡ‚Ρ‹ с Π½ΠΈΠΌ мСня ΠΏΠΎΠ»Π½ΠΎΡΡ‚ΡŒΡŽ ΡƒΠ±Π΅Π΄ΠΈΠ»ΠΈ, Ρ‡Ρ‚ΠΎ Ρ€Π°Π±ΠΎΡ‚Π°Ρ‚ΡŒ с Π½ΠΈΠΌ Π½Π΅Π²ΠΎΠ·ΠΌΠΎΠΆΠ½ΠΎ. Π­Ρ‚ΠΎ ΠΎΠΊΠ½ΠΎ постоянно ΠΌΠ΅ΡˆΠ°Π΅Ρ‚ΡΡ Π²ΠΎ всСх ΠΏΡ€ΠΎΠ³Π°Ρ… Ρ‡Ρ‚ΠΎ Π½Π΅ΠΈΠ·Π±Π΅ΠΆΠ½ΠΎ ΠΏΡ€ΠΈΠ²ΠΎΠ΄ΠΈΡ‚ ΠΊ Π΅Π³ΠΎ Π·Π°ΠΊΡ€Ρ‹Ρ‚ΠΈΡŽ, послС Ρ‡Π΅Π³ΠΎ ΠΏΠΎ Π·Π°ΠΊΠΎΠ½Ρƒ подлости Π²ΠΎΠ·Π½ΠΈΠΊΠ°Π΅Ρ‚ острая Π½Π΅ΠΎΠ±Ρ…ΠΎΠ΄ΠΈΠΌΠΎΡΡ‚ΡŒ Ρ‡Ρ‚ΠΎ-Π½ΠΈΠ±ΡƒΠ΄ΡŒ ΠΈΠ·ΠΌΠ΅Ρ€ΠΈΡ‚ΡŒ. РасскидываСт ΠΎΠ±ΡŠΠ΅ΠΊΡ‚Ρ‹ коряво Π½Π° ΠΌΠΎΠΉ взгляд — Π²Ρ€ΡƒΡ‡Π½ΡƒΡŽ ΠΏΡ€ΠΎΡ‰Π΅ ΠΈ экономичнСС. ΠœΠ΅Ρ‚ΠΎΠ΄ упрощСния чистки ΠΏΠ»Π΅Π½ΠΊΠΈ Ρƒ нас ΠΊΠ°ΠΊ-Ρ‚ΠΎ Π½Π΅ приТился. ΠœΠ΅Π°ΡΡƒΡ€Π΅ ΠΈ ΠΏΡ€ΠΎΡ‰Π΅ ΠΈ ΡƒΠ΄ΠΎΠ±Π½Π΅Π΅. ВсС ИМΠ₯О

WBR, Anatoly

  • НавСрх

#17 OFFLINE Β  rexti64

ΠžΡ‚ΠΏΡ€Π°Π²Π»Π΅Π½ΠΎ 17 ΠΠΎΡΠ±Ρ€ΡŒ 2008 — 14:08

ΠŸΡ€ΠΎΠ³Ρ€Π°ΠΌΠΌΠΊΠ°: Β V_Length.rar Β Β 300,28К Β Β 1476 скачиваний

  • НавСрх

#18 OFFLINE Β  gost2

ΠžΡ‚ΠΏΡ€Π°Π²Π»Π΅Π½ΠΎ 22 ΠœΠ°Ρ€Ρ‚ 2009 — 23:49

8.2008, 14:01″>

Как ΠΌΠΎΠΆΠ½ΠΎ ΡƒΠ·Π½Π°Ρ‚ΡŒ ΠΈΠ»ΠΈ Π²Ρ‹Ρ‡ΠΈΡΠ»ΠΈΡ‚ΡŒ ΠΏΡ€ΠΈ ΠΏΠΎΠΌΠΎΡ‰ΠΈ ΠΊΠ°ΠΊΠΎΠΉ ΠΏΡ€ΠΎΠ³Ρ€Π°ΠΌΠΌΡ‹ Π΄Π»ΠΈΠ½Ρƒ Π²Π΅ΠΊΡ‚ΠΎΡ€Π°? Π’ ΠΡ€Ρ‚ΠšΠ°ΠΌΠ΅ Π΅ΡΡ‚ΡŒ, Π½ΠΎ Ρ‚Π°ΠΌ Π½ΡƒΠΆΠ½ΠΎ Π²Ρ‹Π±ΠΈΡ€Π°Ρ‚ΡŒ ΠΏΠΎ ΠΎΠ΄Π½ΠΎΠΌΡƒ Π²Π΅ΠΊΡ‚ΠΎΡ€Ρƒ
Π—Π°Ρ€Π°Π½Π΅Π΅ Π±Π»Π°Π³ΠΎΠ΄Π°Ρ€Π΅Π½!

Π•ΡΡ‚ΡŒ Ρ‚Π°ΠΊΠΆΠ΅ макросы для Corel: Perimeter Β Perimeter.rar Β Β 6,37К Β Β 1073 скачиваний, GetArea Text (считаСт Ρ‚Π°ΠΊΠΆΠ΅ ΠΏΠ»ΠΎΡ‰Π°Π΄ΡŒ) Β GetAreaText.rar Β Β 16,44К Β Β 924 скачиваний

  • НавСрх

#19 OFFLINE Β  kontrolnaya

ΠžΡ‚ΠΏΡ€Π°Π²Π»Π΅Π½ΠΎ 05 Π‘Π΅Π½Ρ‚ΡΠ±Ρ€ΡŒ 2009 — 21:51

Как ΠΌΠΎΠΆΠ½ΠΎ ΡƒΠ·Π½Π°Ρ‚ΡŒ ΠΈΠ»ΠΈ Π²Ρ‹Ρ‡ΠΈΡΠ»ΠΈΡ‚ΡŒ ΠΏΡ€ΠΈ ΠΏΠΎΠΌΠΎΡ‰ΠΈ ΠΊΠ°ΠΊΠΎΠΉ ΠΏΡ€ΠΎΠ³Ρ€Π°ΠΌΠΌΡ‹ Π΄Π»ΠΈΠ½Ρƒ Π²Π΅ΠΊΡ‚ΠΎΡ€Π°? Π’ ΠΡ€Ρ‚ΠšΠ°ΠΌΠ΅ Π΅ΡΡ‚ΡŒ, Π½ΠΎ Ρ‚Π°ΠΌ Π½ΡƒΠΆΠ½ΠΎ Π²Ρ‹Π±ΠΈΡ€Π°Ρ‚ΡŒ ΠΏΠΎ ΠΎΠ΄Π½ΠΎΠΌΡƒ Π²Π΅ΠΊΡ‚ΠΎΡ€Ρƒ
Π—Π°Ρ€Π°Π½Π΅Π΅ Π±Π»Π°Π³ΠΎΠ΄Π°Ρ€Π΅Π½!

Π”Π»ΠΈΠ½Π° Π²Π΅ΠΊΡ‚ΠΎΡ€Π° ΠΎΠ½Π»Π°ΠΉΠ½

  • НавСрх

#20 OFFLINE Β  3D-BiG

ΠžΡ‚ΠΏΡ€Π°Π²Π»Π΅Π½ΠΎ 06 Π‘Π΅Π½Ρ‚ΡΠ±Ρ€ΡŒ 2009 — 00:22

kontrolnaya, здСсь Π² этом Ρ€Π°Π·Π΄Π΅Π»Π΅ ΠΏΠΎΠ΄ Π΄Π»ΠΈΠ½ΠΎΠΉ Π²Π΅ΠΊΡ‚ΠΎΡ€Π° ΠΆΠ°Ρ€Π³ΠΎΠ½Π½ΠΎ подразумСваСтся ΠΏΠ΅Ρ€ΠΈΠΌΠ΅Ρ‚Ρ€ Π»ΠΈΠ½ΠΈΠΈ Π·Π°Π΄Π°Π½Π½ΠΎΠΉ ΠΊΡ€ΠΈΠ²Ρ‹ΠΌΠΈ Π‘Π΅Π»ΡŒΠ·ΡŠΠ΅ ΠΈ ΠΎΡ‚Ρ€Π΅Π·ΠΊΠ°ΠΌΠΈ — базис Corel. ..

  • НавСрх

Π΄Π»ΠΈΠ½Π° суммы Π²Π΅ΠΊΡ‚ΠΎΡ€ΠΎΠ² ΠΈ Ρ‚Π΅ΠΎΡ€Π΅ΠΌΠ° косинусов

  • Π‘Π»ΠΎΠΆΠ΅Π½ΠΈΠ΅ Π²Π΅ΠΊΡ‚ΠΎΡ€ΠΎΠ²: ΠΊΠ°ΠΊ Π΄Π΅ΠΉΡΡ‚Π²ΠΎΠ²Π°Ρ‚ΡŒ
  • Π‘Π»ΠΎΠΆΠ΅Π½ΠΈΠ΅ Π²Π΅ΠΊΡ‚ΠΎΡ€ΠΎΠ²: Ρ€Π΅ΡˆΠ΅Π½ΠΈΠ΅ ΠΏΡ€ΠΈΠΌΠ΅Ρ€ΠΎΠ²
  • Π‘Π»ΠΎΠΆΠ΅Π½ΠΈΠ΅ Π²Π΅ΠΊΡ‚ΠΎΡ€ΠΎΠ² — ΠΎΠ½Π»Π°ΠΉΠ½ ΠΊΠ°Π»ΡŒΠΊΡƒΠ»ΡΡ‚ΠΎΡ€

Π‘Π»ΠΎΠΆΠ΅Π½ΠΈΠ΅ Π²Π΅ΠΊΡ‚ΠΎΡ€ΠΎΠ² ΠΏΠΎ ΠΏΡ€Π°Π²ΠΈΠ»Ρƒ Ρ‚Ρ€Π΅ΡƒΠ³ΠΎΠ»ΡŒΠ½ΠΈΠΊΠ° (суммой Π²Π΅ΠΊΡ‚ΠΎΡ€ΠΎΠ² ΠΈ называСтся Π²Π΅ΠΊΡ‚ΠΎΡ€ , Π½Π°Ρ‡Π°Π»ΠΎ ΠΊΠΎΡ‚ΠΎΡ€ΠΎΠ³ΠΎ совпадаСт с Π½Π°Ρ‡Π°Π»ΠΎΠΌ Π²Π΅ΠΊΡ‚ΠΎΡ€Π° , Π° ΠΊΠΎΠ½Π΅Ρ† — с ΠΊΠΎΠ½Ρ†ΠΎΠΌ Π²Π΅ΠΊΡ‚ΠΎΡ€Π° , ΠΏΡ€ΠΈ условии, Ρ‡Ρ‚ΠΎ Π½Π°Ρ‡Π°Π»ΠΎ Π²Π΅ΠΊΡ‚ΠΎΡ€Π° ΠΏΡ€ΠΈΠ»ΠΎΠΆΠ΅Π½ΠΎ ΠΊ ΠΊΠΎΠ½Ρ†Ρƒ Π²Π΅ΠΊΡ‚ΠΎΡ€Π° ) Π΄Π°Ρ‘Ρ‚ Π²ΠΎΠ·ΠΌΠΎΠΆΠ½ΠΎΡΡ‚ΡŒ ΡƒΠΏΡ€ΠΎΡ‰Π°Ρ‚ΡŒ Π²Ρ‹Ρ€Π°ΠΆΠ΅Π½ΠΈΠ΅ ΠΏΠ΅Ρ€Π΅Π΄ вычислСниСм ΠΏΡ€ΠΎΠΈΠ·Π²Π΅Π΄Π΅Π½ΠΈΠΉ Π²Π΅ΠΊΡ‚ΠΎΡ€ΠΎΠ².

Π‘Π»ΠΎΠΆΠ΅Π½ΠΈΠ΅ Π²Π΅ΠΊΡ‚ΠΎΡ€ΠΎΠ², Π·Π°Π΄Π°Π½Π½Ρ‹Ρ… ΠΊΠΎΠΎΡ€Π΄ΠΈΠ½Π°Ρ‚Π°ΠΌΠΈ (ΠΏΡ€ΠΈ слоТСнии ΠΎΠ΄Π½ΠΎΠΈΠΌΡ‘Π½Π½Ρ‹Π΅ ΠΊΠΎΠΎΡ€Π΄ΠΈΠ½Π°Ρ‚Ρ‹ ΡΠΊΠ»Π°Π΄Ρ‹Π²Π°ΡŽΡ‚ΡΡ) Π΄Π°Ρ‘Ρ‚ Π²ΠΎΠ·ΠΌΠΎΠΆΠ½ΠΎΡΡ‚ΡŒ ΡƒΠ·Π½Π°Ρ‚ΡŒ, ΠΊΠ°ΠΊ располоТСн ΠΎΡ‚Π½ΠΎΡΠΈΡ‚Π΅Π»ΡŒΠ½ΠΎ Π½Π°Ρ‡Π°Π»Π° ΠΊΠΎΠΎΡ€Π΄ΠΈΠ½Π°Ρ‚ Π²Π΅ΠΊΡ‚ΠΎΡ€, ΡΠ²Π»ΡΡŽΡ‰ΠΈΠΉΡΡ суммой слагаСмых Π²Π΅ΠΊΡ‚ΠΎΡ€ΠΎΠ². ΠŸΠΎΠ΄Ρ€ΠΎΠ±Π½ΠΎ эти Π΄Π²Π΅ ΠΎΠΏΠ΅Ρ€Π°Ρ†ΠΈΠΈ Ρ€Π°Π·Π±ΠΈΡ€Π°Π»ΠΈΡΡŒ Π½Π° ΡƒΡ€ΠΎΠΊΠ΅ «Π’Π΅ΠΊΡ‚ΠΎΡ€Ρ‹ ΠΈ ΠΎΠΏΠ΅Ρ€Π°Ρ†ΠΈΠΈ Π½Π°Π΄ Π²Π΅ΠΊΡ‚ΠΎΡ€Π°ΠΌΠΈ».

Π’Π΅ΠΏΠ΅Ρ€ΡŒ ΠΆΠ΅ Π½Π°ΠΌ прСдстоит ΡƒΠ·Π½Π°Ρ‚ΡŒ, ΠΊΠ°ΠΊ Π½Π°ΠΉΡ‚ΠΈ Π΄Π»ΠΈΠ½Ρƒ Π²Π΅ΠΊΡ‚ΠΎΡ€Π°, ΡΠ²Π»ΡΡŽΡ‰Π΅Π³ΠΎΡΡ Ρ€Π΅Π·ΡƒΠ»ΡŒΡ‚Π°Ρ‚ΠΎΠΌ слоТСния Π²Π΅ΠΊΡ‚ΠΎΡ€ΠΎΠ². Для этого потрСбуСтся ΠΈΡΠΏΠΎΠ»ΡŒΠ·ΠΎΠ²Π°Ρ‚ΡŒ Ρ‚Π΅ΠΎΡ€Π΅ΠΌΡƒ косинусов. Π’Π°ΠΊΡƒΡŽ Π·Π°Π΄Π°Ρ‡Ρƒ приходится Ρ€Π΅ΡˆΠ°Ρ‚ΡŒ, Π½Π°ΠΏΡ€ΠΈΠΌΠ΅Ρ€, ΠΊΠΎΠ³Π΄Π° Π΄ΠΎΡ€ΠΎΠ³Π° ΠΈΠ· ΠΏΡƒΠ½ΠΊΡ‚Π° A Π² ΠΏΡƒΠ½ΠΊΡ‚ Π‘ — Π½Π΅ прямая, Π° отклоняСтся ΠΎΡ‚ прямой, Ρ‡Ρ‚ΠΎΠ±Ρ‹ ΠΏΡ€ΠΎΠΉΡ‚ΠΈ Π΅Ρ‰Ρ‘ Ρ‡Π΅Ρ€Π΅Π· ΠΊΠ°ΠΊΠΎΠΉ-Ρ‚ΠΎ ΠΏΡƒΠ½ΠΊΡ‚ B, Π° Π½ΡƒΠΆΠ½ΠΎ ΡƒΠ·Π½Π°Ρ‚ΡŒ Π΄Π»ΠΈΠ½Ρƒ ΠΏΡ€Π΅Π΄ΠΏΠΎΠ»Π°Π³Π°Π΅ΠΌΠΎΠΉ прямой Π΄ΠΎΡ€ΠΎΠ³ΠΈ. ΠšΡΡ‚Π°Ρ‚ΠΈ, гСодСзия — ΠΎΠ΄Π½Π° ΠΈΠ· Ρ‚Π΅Ρ… сфСр Π΄Π΅ΡΡ‚Π΅Π»ΡŒΠ½ΠΎΡΡ‚ΠΈ, Π³Π΄Π΅ тригономСтричСскиС Ρ„ΡƒΠ½ΠΊΡ†ΠΈΠΈ ΠΏΡ€ΠΈΠΌΠ΅Π½ΡΡŽΡ‚ΡΡ Π²ΠΎ всСх ΠΈΡ… ΠΏΠΎΠ»Π½ΠΎΡ‚Π΅.

ΠŸΡ€ΠΈ слоТСнии Π²Π΅ΠΊΡ‚ΠΎΡ€ΠΎΠ² для нахоТдСния Π΄Π»ΠΈΠ½Ρ‹ суммы Π²Π΅ΠΊΡ‚ΠΎΡ€ΠΎΠ² ΠΈΡΠΏΠΎΠ»ΡŒΠ·ΡƒΠ΅Ρ‚ΡΡ Ρ‚Π΅ΠΎΡ€Π΅ΠΌΠ° косинусов. ΠŸΡƒΡΡ‚ΡŒ ΠΈ — Π²Π΅ΠΊΡ‚ΠΎΡ€Ρ‹, — ΡƒΠ³ΠΎΠ» ΠΌΠ΅ΠΆΠ΄Ρƒ Π½ΠΈΠΌΠΈ, Π° — сумма Π²Π΅ΠΊΡ‚ΠΎΡ€ΠΎΠ² ΠΊΠ°ΠΊ Ρ€Π΅Π·ΡƒΠ»ΡŒΡ‚Π°Ρ‚ слоТСния Π²Π΅ΠΊΡ‚ΠΎΡ€ΠΎΠ² ΠΏΠΎ ΠΏΡ€Π°Π²ΠΈΠ»Ρƒ Ρ‚Ρ€Π΅ΡƒΠ³ΠΎΠ»ΡŒΠ½ΠΈΠΊΠ°. Π’ΠΎΠ³Π΄Π° Π²Π΅Ρ€Π½ΠΎ ΡΠ»Π΅Π΄ΡƒΡŽΡ‰Π΅Π΅ ΡΠΎΠΎΡ‚Π½ΠΎΡˆΠ΅Π½ΠΈΠ΅:

,

Π³Π΄Π΅ — ΡƒΠ³ΠΎΠ», смСТный с ΡƒΠ³Π»ΠΎΠΌ . Π£ смСТных ΡƒΠ³Π»ΠΎΠ² ΠΎΠ΄Π½Π° сторона общая, Π° Π΄Ρ€ΡƒΠ³ΠΈΠ΅ стороны Π»Π΅ΠΆΠ°Ρ‚ Π½Π° ΠΎΠ΄Π½ΠΎΠΉ прямой (см. рисунок Π²Ρ‹ΡˆΠ΅).

ΠŸΠΎΡΡ‚ΠΎΠΌΡƒ для слоТСния Π²Π΅ΠΊΡ‚ΠΎΡ€ΠΎΠ² ΠΈ опрСдСлСния Π΄Π»ΠΈΠ½Ρ‹ суммы Π²Π΅ΠΊΡ‚ΠΎΡ€ΠΎΠ² Π½ΡƒΠΆΠ½ΠΎ ΠΈΠ·Π²Π»Π΅Ρ‡ΡŒ ΠΊΠ²Π°Π΄Ρ€Π°Ρ‚Π½Ρ‹ΠΉ ΠΊΠΎΡ€Π΅Π½ΡŒ ΠΈΠ· ΠΊΠ°ΠΆΠ΄ΠΎΠΉ части равСнства, Ρ‚ΠΎΠ³Π΄Π° получится Ρ„ΠΎΡ€ΠΌΡƒΠ»Π° Π΄Π»ΠΈΠ½Ρ‹:

.

Π’ случаС вычитания Π²Π΅ΠΊΡ‚ΠΎΡ€ΠΎΠ² () происходит слоТСниС Π²Π΅ΠΊΡ‚ΠΎΡ€Π° с Π²Π΅ΠΊΡ‚ΠΎΡ€ΠΎΠΌ , ΠΏΡ€ΠΎΡ‚ΠΈΠ²ΠΎΠΏΠΎΠ»ΠΎΠΆΠ½Ρ‹ΠΌ Π²Π΅ΠΊΡ‚ΠΎΡ€Ρƒ , Ρ‚ΠΎ Π΅ΡΡ‚ΡŒ ΠΈΠΌΠ΅ΡŽΡ‰ΠΈΠΌ Ρ‚Ρƒ ΠΆΠ΅ Π΄Π»ΠΈΠ½Ρƒ, Π½ΠΎ ΠΏΡ€ΠΎΡ‚ΠΈΠ²ΠΎΠΏΠΎΠ»ΠΎΠΆΠ½Ρ‹ΠΌ ΠΏΠΎ Π½Π°ΠΏΡ€Π°Π²Π»Π΅Π½ΠΈΡŽ. Π£Π³Π»Ρ‹ ΠΌΠ΅ΠΆΠ΄Ρƒ ΠΈ ΠΈ ΠΈ ΠΌΠ΅ΠΆΠ΄Ρƒ ΠΈ ΡΠ²Π»ΡΡŽΡ‚ΡΡ смСТными ΡƒΠ³Π»Π°ΠΌΠΈ, Ρƒ Π½ΠΈΡ…, ΠΊΠ°ΠΊ ΡƒΠΆΠ΅ Π±Ρ‹Π»ΠΎ ΠΎΡ‚ΠΌΠ΅Ρ‡Π΅Π½ΠΎ, ΠΎΠ΄Π½Π° сторона общая, Π° Π΄Ρ€ΡƒΠ³ΠΈΠ΅ стороны Π»Π΅ΠΆΠ°Ρ‚ Π½Π° ΠΎΠ΄Π½ΠΎΠΉ прямой. Π’ случаС вычитания Π²Π΅ΠΊΡ‚ΠΎΡ€ΠΎΠ² для нахоТдСния Π΄Π»ΠΈΠ½Ρ‹ разности Π²Π΅ΠΊΡ‚ΠΎΡ€ΠΎΠ² Π½ΡƒΠΆΠ½ΠΎ Π·Π½Π°Ρ‚ΡŒ ΡΠ»Π΅Π΄ΡƒΡŽΡ‰Π΅Π΅ свойство косинусов смСТных ΡƒΠ³Π»ΠΎΠ²:

косинусы смСТных ΡƒΠ³Π»ΠΎΠ² Ρ€Π°Π²Π½Ρ‹ ΠΏΠΎ Π°Π±ΡΠΎΠ»ΡŽΡ‚Π½ΠΎΠΉ Π²Π΅Π»ΠΈΡ‡ΠΈΠ½Π΅ (Π²Π΅Π»ΠΈΡ‡ΠΈΠ½Π΅ ΠΏΠΎ ΠΌΠΎΠ΄ΡƒΠ»ΡŽ), Π½ΠΎ ΠΈΠΌΠ΅ΡŽΡ‚ ΠΏΡ€ΠΎΡ‚ΠΈΠ²ΠΎΠΏΠΎΠ»ΠΎΠΆΠ½Ρ‹Π΅ Π·Π½Π°ΠΊΠΈ.

ΠŸΠ΅Ρ€Π΅ΠΉΠ΄Ρ‘ΠΌ ΠΊ ΠΏΡ€ΠΈΠΌΠ΅Ρ€Π°ΠΌ.

  • ΠŸΡ€ΠΈΠ³ΠΎΠ΄ΠΈΡ‚ΡΡ: тригономСтричСская Ρ‚Π°Π±Π»ΠΈΡ†Π° (синусы, косинусы, тангСнсы ΠΈ котангСнсы распространСнных ΡƒΠ³Π»ΠΎΠ²)

ΠŸΡ€ΠΈΠΌΠ΅Ρ€ 1. Π’Π΅ΠΊΡ‚ΠΎΡ€Ρ‹ ΠΈ ΠΎΠ±Ρ€Π°Π·ΡƒΡŽΡ‚ ΡƒΠ³ΠΎΠ» . Π˜Ρ… Π΄Π»ΠΈΠ½Ρ‹: ΠΈ . Π’Ρ‹ΠΏΠΎΠ»Π½ΠΈΡ‚ΡŒ слоТСниС Π²Π΅ΠΊΡ‚ΠΎΡ€ΠΎΠ² ΠΈ Π½Π°ΠΉΡ‚ΠΈ ΠΈΡ… сумму . Π’Ρ‹ΠΏΠΎΠ»Π½ΠΈΡ‚ΡŒ Π²Ρ‹Ρ‡ΠΈΡ‚Π°Π½ΠΈΠ΅ Π²Π΅ΠΊΡ‚ΠΎΡ€ΠΎΠ² ΠΈ Π½Π°ΠΉΡ‚ΠΈ ΠΈΡ… Ρ€Π°Π·Π½ΠΎΡΡ‚ΡŒ .

РСшСниС. Из элСмСнтарной Ρ‚Ρ€ΠΈΠ³ΠΎΠ½ΠΎΠΌΠ΅Ρ‚Ρ€ΠΈΠΈ извСстно, Ρ‡Ρ‚ΠΎ .

Π¨Π°Π³ 1. ВыполняСм слоТСниС Π²Π΅ΠΊΡ‚ΠΎΡ€ΠΎΠ². Находим Π΄Π»ΠΈΠ½Ρƒ суммы Π²Π΅ΠΊΡ‚ΠΎΡ€ΠΎΠ², поставляя Π² Ρ„ΠΎΡ€ΠΌΡƒΠ»Ρƒ Π΄Π»ΠΈΠ½Ρ‹ косинус ΡƒΠ³Π»Π°, смСТного с ΡƒΠ³Π»ΠΎΠΌ ΠΌΠ΅ΠΆΠ΄Ρƒ Π²Π΅ΠΊΡ‚ΠΎΡ€Π°ΠΌΠΈ:

Π¨Π°Π³ 2. ВыполняСм Π²Ρ‹Ρ‡ΠΈΡ‚Π°Π½ΠΈΠ΅ Π²Π΅ΠΊΡ‚ΠΎΡ€ΠΎΠ². Находим Π΄Π»ΠΈΠ½Ρƒ разности Π²Π΅ΠΊΡ‚ΠΎΡ€ΠΎΠ², подставляя Π² Ρ„ΠΎΡ€ΠΌΡƒΠ»Ρƒ косинус «ΠΈΠ·Π½Π°Ρ‡Π°Π»ΡŒΠ½ΠΎΠ³ΠΎ» ΡƒΠ³Π»Π°:

ΠŸΡ€ΠΎΠ²Π΅Ρ€ΠΈΡ‚ΡŒ Ρ€Π΅ΡˆΠ΅Π½ΠΈΠ΅ ΠΌΠΎΠΆΠ½ΠΎ Π½Π° ΠšΠ°Π»ΡŒΠΊΡƒΠ»ΡΡ‚ΠΎΡ€Π΅ ΠΎΠ½Π»Π°ΠΉΠ½.

Π’Ρ‹ΠΏΠΎΠ»Π½ΠΈΡ‚ΡŒ слоТСниС ΠΈ Π²Ρ‹Ρ‡ΠΈΡ‚Π°Π½ΠΈΠ΅ Π²Π΅ΠΊΡ‚ΠΎΡ€ΠΎΠ² ΡΠ°ΠΌΠΎΡΡ‚ΠΎΡΡ‚Π΅Π»ΡŒΠ½ΠΎ, Π° Π·Π°Ρ‚Π΅ΠΌ ΠΏΠΎΡΠΌΠΎΡ‚Ρ€Π΅Ρ‚ΡŒ Ρ€Π΅ΡˆΠ΅Π½ΠΈΠ΅

ΠŸΡ€ΠΈΠΌΠ΅Ρ€ 2. Π’Π΅ΠΊΡ‚ΠΎΡ€Ρ‹ ΠΈ ΠΎΠ±Ρ€Π°Π·ΡƒΡŽΡ‚ ΡƒΠ³ΠΎΠ» . Π˜Ρ… Π΄Π»ΠΈΠ½Ρ‹: ΠΈ . Π’Ρ‹ΠΏΠΎΠ»Π½ΠΈΡ‚ΡŒ слоТСниС Π²Π΅ΠΊΡ‚ΠΎΡ€ΠΎΠ² ΠΈ Π½Π°ΠΉΡ‚ΠΈ ΠΈΡ… сумму . Π’Ρ‹ΠΏΠΎΠ»Π½ΠΈΡ‚ΡŒ Π²Ρ‹Ρ‡ΠΈΡ‚Π°Π½ΠΈΠ΅ Π²Π΅ΠΊΡ‚ΠΎΡ€ΠΎΠ² ΠΈ Π½Π°ΠΉΡ‚ΠΈ ΠΈΡ… Ρ€Π°Π·Π½ΠΎΡΡ‚ΡŒ .

ΠŸΡ€Π°Π²ΠΈΠ»ΡŒΠ½ΠΎΠ΅ Ρ€Π΅ΡˆΠ΅Π½ΠΈΠ΅ ΠΈ ΠΎΡ‚Π²Π΅Ρ‚.


ΠŸΡ€ΠΈΠΌΠ΅Ρ€ 3. Π”Π°Π½Ρ‹ Π΄Π»ΠΈΠ½Ρ‹ Π²Π΅ΠΊΡ‚ΠΎΡ€ΠΎΠ² ΠΈ Π΄Π»ΠΈΠ½Π° ΠΈΡ… суммы . Найти Π΄Π»ΠΈΠ½Ρƒ ΠΈΡ… разности .

РСшСниС.

Π¨Π°Π³ 1. По Ρ‚Π΅ΠΎΡ€Π΅ΠΌΠ΅ косинусов составляСм ΡƒΡ€Π°Π²Π½Π΅Π½ΠΈΠ΅, Ρ‡Ρ‚ΠΎΠ±Ρ‹ Π½Π°ΠΉΡ‚ΠΈ косинус ΡƒΠ³Π»Π°, смСТного с ΡƒΠ³Π»ΠΎΠΌ ΠΌΠ΅ΠΆΠ΄Ρƒ Π²Π΅ΠΊΡ‚ΠΎΡ€Π°ΠΌΠΈ ΠΈ Π½Π°Ρ…ΠΎΠ΄ΠΈΠΌ Π΅Π³ΠΎ:

НС Π·Π°Π±Ρ‹Π²Π°Π΅ΠΌ, Ρ‡Ρ‚ΠΎ косинус смСТного ΡƒΠ³Π»Π° получился со Π·Π½Π°ΠΊΠΎΠΌ минус. Π­Ρ‚ΠΎ Π·Π½Π°Ρ‡ΠΈΡ‚, Ρ‡Ρ‚ΠΎ косинус «ΠΈΠ·Π½Π°Ρ‡Π°Π»ΡŒΠ½ΠΎΠ³ΠΎ» ΡƒΠ³Π»Π° Π±ΡƒΠ΄Π΅Ρ‚ со Π·Π½Π°ΠΊΠΎΠΌ плюс.

Π¨Π°Π³ 2. ВыполняСм Π²Ρ‹Ρ‡ΠΈΡ‚Π°Π½ΠΈΠ΅ Π²Π΅ΠΊΡ‚ΠΎΡ€ΠΎΠ². Находим Π΄Π»ΠΈΠ½Ρƒ разности Π²Π΅ΠΊΡ‚ΠΎΡ€ΠΎΠ², подставляя Π² Ρ„ΠΎΡ€ΠΌΡƒΠ»Ρƒ косинус «ΠΈΠ·Π½Π°Ρ‡Π°Π»ΡŒΠ½ΠΎΠ³ΠΎ» ΡƒΠ³Π»Π°:

ΠŸΡ€ΠΎΠ²Π΅Ρ€ΠΈΡ‚ΡŒ Ρ€Π΅ΡˆΠ΅Π½ΠΈΠ΅ ΠΌΠΎΠΆΠ½ΠΎ Π½Π° ΠšΠ°Π»ΡŒΠΊΡƒΠ»ΡΡ‚ΠΎΡ€Π΅ ΠΎΠ½Π»Π°ΠΉΠ½.

ΠŸΡ€ΠΈΠΌΠ΅Ρ€ 4. Π”Π°Π½Ρ‹ Π΄Π»ΠΈΠ½Ρ‹ Π²Π΅ΠΊΡ‚ΠΎΡ€ΠΎΠ² ΠΈ Π΄Π»ΠΈΠ½Π° ΠΈΡ… разности . Найти Π΄Π»ΠΈΠ½Ρƒ ΠΈΡ… суммы .

РСшСниС.

Π¨Π°Π³ 1. По Ρ‚Π΅ΠΎΡ€Π΅ΠΌΠ΅ косинусов составляСм ΡƒΡ€Π°Π²Π½Π΅Π½ΠΈΠ΅, Ρ‡Ρ‚ΠΎΠ±Ρ‹ Π½Π°ΠΉΡ‚ΠΈ косинус «ΠΈΠ·Π½Π°Ρ‡Π°Π»ΡŒΠ½ΠΎΠ³ΠΎ» ΡƒΠ³Π»Π° (Π·Π°Π΄Π°Ρ‡Π° обратная ΠΏΠΎ ΠΎΡ‚Π½ΠΎΡˆΠ΅Π½ΠΈΡŽ ΠΊ ΠΏΡ€ΠΈΠΌΠ΅Ρ€Ρƒ 1) ΠΈ Π½Π°Ρ…ΠΎΠ΄ΠΈΠΌ Π΅Π³ΠΎ:

Π¨Π°Π³ 2. МСняСм Π·Π½Π°ΠΊ косинуса ΠΈ ΠΏΠΎΠ»ΡƒΡ‡Π°Π΅ΠΌ косинус смСТного ΡƒΠ³Π»Π° ΠΌΠ΅ΠΆΠ΄Ρƒ ΠΈ :

Π¨Π°Π³ 3. ВыполняСм слоТСниС Π²Π΅ΠΊΡ‚ΠΎΡ€ΠΎΠ². Находим Π΄Π»ΠΈΠ½Ρƒ суммы Π²Π΅ΠΊΡ‚ΠΎΡ€ΠΎΠ², подставляя Π² Ρ„ΠΎΡ€ΠΌΡƒΠ»Ρƒ косинус смСТного ΡƒΠ³Π»Π°:

ΠŸΡ€ΠΎΠ²Π΅Ρ€ΠΈΡ‚ΡŒ Ρ€Π΅ΡˆΠ΅Π½ΠΈΠ΅ ΠΌΠΎΠΆΠ½ΠΎ Π½Π° ΠšΠ°Π»ΡŒΠΊΡƒΠ»ΡΡ‚ΠΎΡ€Π΅ ΠΎΠ½Π»Π°ΠΉΠ½.

  • ΠŸΡ€ΠΈΠ³ΠΎΠ΄ΠΈΡ‚ΡΡ: тригономСтричСская Ρ‚Π°Π±Π»ΠΈΡ†Π° (синусы, косинусы, тангСнсы ΠΈ котангСнсы распространСнных ΡƒΠ³Π»ΠΎΠ²)

ΠŸΡ€ΠΈΠΌΠ΅Ρ€ 5. Π’Π΅ΠΊΡ‚ΠΎΡ€Ρ‹ ΠΈ Π²Π·Π°ΠΈΠΌΠ½ΠΎ пСрпСндикулярны, Π° ΠΈΡ… Π΄Π»ΠΈΠ½Ρ‹ . Найти Π΄Π»ΠΈΠ½Ρƒ ΠΈΡ… суммы ΠΈ ΠΈ Π΄Π»ΠΈΠ½Ρƒ ΠΈΡ… разности .

РСшСниС.

Π”Π²Π° смСТных ΡƒΠ³Π»Π°, ΠΊΠ°ΠΊ Π½Π΅Ρ‚Ρ€ΡƒΠ΄Π½ΠΎ Π΄ΠΎΠ³Π°Π΄Π°Ρ‚ΡŒΡΡ ΠΈΠ· ΠΏΡ€ΠΈΠ²Π΅Π΄Ρ‘Π½Π½ΠΎΠ³ΠΎ Π² Π½Π°Ρ‡Π°Π»Π΅ ΡƒΡ€ΠΎΠΊΠ° опрСдСлСния, Π² суммС ΡΠΎΡΡ‚Π°Π²Π»ΡΡŽΡ‚ 180 градусов. Π‘Π»Π΅Π΄ΠΎΠ²Π°Ρ‚Π΅Π»ΡŒΠ½ΠΎ, смСТный с прямым ΡƒΠ³Π»ΠΎΠΌ (90 градусов) ΡƒΠ³ΠΎΠ» — Ρ‚ΠΎΠΆΠ΅ прямой (Ρ‚ΠΎΠΆΠ΅ 90 градусов). ΠšΠΎΡΠΈΠ½ΡƒΡ Ρ‚Π°ΠΊΠΎΠ³ΠΎ ΡƒΠ³Π»Π° Ρ€Π°Π²Π΅Π½ Π½ΡƒΠ»ΡŽ, Ρ‚ΠΎ ΠΆΠ΅ самоС относится ΠΈ ΠΊ косинусу смСТного ΡƒΠ³Π»Π°. ΠŸΠΎΡΡ‚ΠΎΠΌΡƒ, подставляя это Π·Π½Π°Ρ‡Π΅Π½ΠΈΠ΅ Π² выраТСния ΠΏΠΎΠ΄ ΠΊΠΎΡ€Π½Π΅ΠΌ Π² Ρ„ΠΎΡ€ΠΌΡƒΠ»Π΅ Π΄Π»ΠΈΠ½Ρ‹ суммы ΠΈ разности Π²Π΅ΠΊΡ‚ΠΎΡ€ΠΎΠ², ΠΏΠΎΠ»ΡƒΡ‡Π°Π΅ΠΌ Π½ΡƒΠ»ΠΈ ΠΊΠ°ΠΊ послСдниС выраТСния — произвСдСния ΠΏΠΎΠ΄ Π·Π½Π°ΠΊΠΎΠΌ корня. Π’ΠΎ Π΅ΡΡ‚ΡŒ Π΄Π»ΠΈΠ½Ρ‹ суммы ΠΈ разности Π΄Π°Π½Π½Ρ‹Ρ… Π²Π΅ΠΊΡ‚ΠΎΡ€ΠΎΠ² Ρ€Π°Π²Π½Ρ‹, вычисляСм ΠΈΡ…:

ΠŸΡ€ΠΈΠΌΠ΅Ρ€ 6. ΠšΠ°ΠΊΠΎΠΌΡƒ ΡƒΡΠ»ΠΎΠ²ΠΈΡŽ Π΄ΠΎΠ»ΠΆΠ½Ρ‹ ΡƒΠ΄ΠΎΠ²Π»Π΅Ρ‚Π²ΠΎΡ€ΡΡ‚ΡŒ Π²Π΅ΠΊΡ‚ΠΎΡ€Ρ‹ ΠΈ , Ρ‡Ρ‚ΠΎΠ±Ρ‹ ΠΈΠΌΠ΅Π»ΠΈ мСсто ΡΠ»Π΅Π»ΡƒΡŽΡ‰ΠΈΠ΅ ΡΠΎΠΎΡ‚Π½ΠΎΡˆΠ΅Π½ΠΈΡ:

1) Π΄Π»ΠΈΠ½Π° суммы Π²Π΅ΠΊΡ‚ΠΎΡ€ΠΎΠ² Ρ€Π°Π²Π½Π° Π΄Π»ΠΈΠ½Π΅ разности Π²Π΅ΠΊΡ‚ΠΎΡ€ΠΎΠ², Ρ‚. Π΅. ,

2) Π΄Π»ΠΈΠ½Π° суммы Π²Π΅ΠΊΡ‚ΠΎΡ€ΠΎΠ² большС Π΄Π»ΠΈΠ½Ρ‹ разности Π²Π΅ΠΊΡ‚ΠΎΡ€ΠΎΠ², Ρ‚. Π΅. ,

3) Π΄Π»ΠΈΠ½Π° суммы Π²Π΅ΠΊΡ‚ΠΎΡ€ΠΎΠ² мСньшС Π΄Π»ΠΈΠ½Ρ‹ разности Π²Π΅ΠΊΡ‚ΠΎΡ€ΠΎΠ², Ρ‚. Π΅. ?

РСшСниС.

Находим условиС для ΠΏΠ΅Ρ€Π²ΠΎΠ³ΠΎ ΡΠΎΠΎΡ‚Π½ΠΎΡˆΠ΅Π½ΠΈΡ. Для этого Ρ€Π΅ΡˆΠ°Π΅ΠΌ ΡΠ»Π΅Π΄ΡƒΡŽΡ‰Π΅Π΅ ΡƒΡ€Π°Π²Π½Π΅Π½ΠΈΠ΅:

Π’ΠΎ Π΅ΡΡ‚ΡŒ, для Ρ‚ΠΎΠ³ΠΎ, Ρ‡Ρ‚ΠΎΠ±Ρ‹ Π΄Π»ΠΈΠ½Π° суммы Π²Π΅ΠΊΡ‚ΠΎΡ€ΠΎΠ² Π±Ρ‹Π»Π° Ρ€Π°Π²Π½Π° Π΄Π»ΠΈΠ½Π΅ ΠΈΡ… разности, Π½Π΅ΠΎΠ±Ρ…ΠΎΠ΄ΠΈΠΌΡ‹, Ρ‡Ρ‚ΠΎΠ±Ρ‹ косинус ΡƒΠ³Π»Π° ΠΌΠ΅ΠΆΠ΄Ρƒ Π½ΠΈΠΌΠΈ ΠΈ косинус смСТного Π΅ΠΌΡƒ ΡƒΠ³Π»Π° Π±Ρ‹Π»ΠΈ Ρ€Π°Π²Π½Ρ‹. Π­Ρ‚ΠΎ условиС выполняСтся, ΠΊΠΎΠ³Π΄Π° ΡƒΠ³Π»Ρ‹ ΠΎΠ±Ρ€Π°Π·ΡƒΡŽΡ‚ прямой ΡƒΠ³ΠΎΠ».

Находим условиС для Π²Ρ‚ΠΎΡ€ΠΎΠ³ΠΎ ΡΠΎΠΎΡ‚Π½ΠΎΡˆΠ΅Π½ΠΈΡ. РСшаСм ΡƒΡ€Π°Π²Π½Π΅Π½ΠΈΠ΅:

НайдСнноС условиС выполняСтся, ΠΊΠΎΠ³Π΄Π° косинус ΡƒΠ³Π»Π° ΠΌΠ΅ΠΆΠ΄Ρƒ Π²Π΅ΠΊΡ‚ΠΎΡ€Π°ΠΌΠΈ мСньшС косинуса смСТных ΡƒΠ³Π»ΠΎΠ². Π’ΠΎ Π΅ΡΡ‚ΡŒ, Ρ‡Ρ‚ΠΎΠ±Ρ‹ Π΄Π»ΠΈΠ½Π° суммы Π²Π΅ΠΊΡ‚ΠΎΡ€ΠΎΠ² Π±Ρ‹Π»Π° большС Π΄Π»ΠΈΠ½Ρ‹ разности Π²Π΅ΠΊΡ‚ΠΎΡ€ΠΎΠ², Π½Π΅ΠΎΠ±Ρ…ΠΎΠ΄ΠΈΠΌΠΎ, Ρ‡Ρ‚ΠΎΠ±Ρ‹ ΡƒΠ³Π»Ρ‹ ΠΎΠ±Ρ€Π°Π·ΠΎΠ²Π°Π»ΠΈ острый ΡƒΠ³ΠΎΠ» (ΠΏΡ€ΠΈΠΌΠ΅Ρ€ 1).

Находим условиС для Ρ‚Ρ€Π΅Ρ‚ΡŒΠ΅Π³ΠΎ ΡΠΎΠΎΡ‚Π½ΠΎΡˆΠ΅Π½ΠΈΡ. РСшаСм ΡƒΡ€Π°Π²Π½Π΅Π½ΠΈΠ΅:

НайдСнноС условиС выполняСтся, ΠΊΠΎΠ³Π΄Π° косинус ΡƒΠ³Π»Π° ΠΌΠ΅ΠΆΠ΄Ρƒ Π²Π΅ΠΊΡ‚ΠΎΡ€Π°ΠΌΠΈ большС косинуса смСТных ΡƒΠ³Π»ΠΎΠ². Π’ΠΎ Π΅ΡΡ‚ΡŒ, Ρ‡Ρ‚ΠΎΠ±Ρ‹ Π΄Π»ΠΈΠ½Π° суммы Π²Π΅ΠΊΡ‚ΠΎΡ€ΠΎΠ² Π±Ρ‹Π»Π° мСньшС Π΄Π»ΠΈΠ½Ρ‹ разности Π²Π΅ΠΊΡ‚ΠΎΡ€ΠΎΠ², Π½Π΅ΠΎΠ±Ρ…ΠΎΠ΄ΠΈΠΌΠΎ, Ρ‡Ρ‚ΠΎΠ±Ρ‹ ΡƒΠ³Π»Ρ‹ ΠΎΠ±Ρ€Π°Π·ΠΎΠ²Π°Π»ΠΈ Ρ‚ΡƒΠΏΠΎΠΉ ΡƒΠ³ΠΎΠ».

  • ΠŸΡ€ΠΈΠ³ΠΎΠ΄ΠΈΡ‚ΡΡ: тригономСтричСская Ρ‚Π°Π±Π»ΠΈΡ†Π° (синусы, косинусы, тангСнсы ΠΈ котангСнсы распространСнных ΡƒΠ³Π»ΠΎΠ²)

ΠŸΡ€ΠΎΠ²Π΅Ρ€ΠΈΡ‚ΡŒ Ρ€Π΅ΡˆΠ΅Π½ΠΈΠ΅ ΠΌΠΎΠΆΠ½ΠΎ Π½Π° ΠšΠ°Π»ΡŒΠΊΡƒΠ»ΡΡ‚ΠΎΡ€Π΅ ΠΎΠ½Π»Π°ΠΉΠ½.

ΠΠ°Π·Π°Π΄Π›ΠΈΡΡ‚Π°Ρ‚ΡŒΠ’ΠΏΠ΅Ρ€Ρ‘Π΄>>>

НСт Π²Ρ€Π΅ΠΌΠ΅Π½ΠΈ Π²Π½ΠΈΠΊΠ°Ρ‚ΡŒ Π² Ρ€Π΅ΡˆΠ΅Π½ΠΈΠ΅? МоТно Π·Π°ΠΊΠ°Π·Π°Ρ‚ΡŒ Ρ€Π°Π±ΠΎΡ‚Ρƒ!

К Π½Π°Ρ‡Π°Π»Ρƒ страницы

ΠŸΡ€ΠΎΠΉΡ‚ΠΈ тСст ΠΏΠΎ Ρ‚Π΅ΠΌΠ΅ Π’Π΅ΠΊΡ‚ΠΎΡ€Ρ‹

ΠŸΠΎΠ΄Π΅Π»ΠΈΡ‚ΡŒΡΡ с Π΄Ρ€ΡƒΠ·ΡŒΡΠΌΠΈ

Начало Ρ‚Π΅ΠΌΡ‹ «Π’Π΅ΠΊΡ‚ΠΎΡ€Ρ‹»

Π’Π΅ΠΊΡ‚ΠΎΡ€Ρ‹: опрСдСлСния ΠΈ дСйствия Π½Π°Π΄ Π²Π΅ΠΊΡ‚ΠΎΡ€Π°ΠΌΠΈ

ΠŸΡ€ΠΎΠ΄ΠΎΠ»ΠΆΠ΅Π½ΠΈΠ΅ Ρ‚Π΅ΠΌΡ‹ «Π’Π΅ΠΊΡ‚ΠΎΡ€Ρ‹»

ЛинСйная Π·Π°Π²ΠΈΡΠΈΠΌΠΎΡΡ‚ΡŒ Π²Π΅ΠΊΡ‚ΠΎΡ€ΠΎΠ²

Базис систСмы Π²Π΅ΠΊΡ‚ΠΎΡ€ΠΎΠ². АффинныС ΠΊΠΎΠΎΡ€Π΄ΠΈΠ½Π°Ρ‚Ρ‹

Π’Π΅ΠΊΡ‚ΠΎΡ€Π½ΠΎΠ΅ ΠΈ смСшанноС ΠΏΡ€ΠΎΠΈΠ·Π²Π΅Π΄Π΅Π½ΠΈΠ΅ Π²Π΅ΠΊΡ‚ΠΎΡ€ΠΎΠ²

БообщСство ЭкспонСнта

  • ΠŸΡƒΠ±Π»ΠΈΠΊΠ°Ρ†ΠΈΡ
  • 15.09.2022

БистСмы управлСния, Π”Ρ€ΡƒΠ³ΠΎΠ΅

Π’ΠΈΠ΄Π΅Π» видос Π½Π° ΠΊΠ°Π½Π°Π»Π΅ экспонСнты ΠΏΠΎ созданию Ρ‚ΠΎΠΏΠ»ΠΈΠ²Π½ΠΎΠΉ систСмы. Вопрос Π·Π°ΠΊΠ»ΡŽΡ‡Π°Π΅Ρ‚ΡΡ Π² Π½Π°Π»ΠΈΡ‡ΠΈΠΈ Π±ΠΎΠ»Π΅Π΅ ΠΏΠΎΠ»Π½ΠΎΠ³ΠΎ описания Π³ΠΎΡ‚ΠΎΠ²ΠΎΠ³ΠΎ ΠΏΡ€ΠΈΠΌΠ΅Ρ€Π° ΠΈΠ»ΠΈ ΡΠΎΠΎΡ‚Π²Π΅Ρ‚ΡΠ²ΡƒΡŽΡ‰Π΅Π΅ Π΄ΠΎΠΊΡƒΠΌΠ΅Π½Ρ‚Π°Ρ†ΠΈΠΈ. Π― Π½ΠΎΠ²ΠΈΡ‡ΠΎΠΊ Π² симулинкС ΠΈ Π΅Ρ‰Ρ‘ ΠΌΠ½ΠΎΠ³ΠΎΠ³ΠΎ Π½Π΅ знаю. АдСкватных ΠΈ раскрытых пособий ΠΏΠΎ созданию Π³ΠΈΠ΄Ρ€Π°Π²…

ΠœΠΎΠ΄Π΅Π»ΠΈΡ€ΠΎΠ²Π°Π½ΠΈΠ΅ гидравличСских систСм Π² simulink

  • ΠŸΡƒΠ±Π»ΠΈΠΊΠ°Ρ†ΠΈΡ
  • 10.09.2022

БистСмы управлСния, Π­Π»Π΅ΠΊΡ‚Ρ€ΠΎΠΏΡ€ΠΈΠ²ΠΎΠ΄ ΠΈ силовая элСктроника, Π”Ρ€ΡƒΠ³ΠΎΠ΅

ΠŸΠ»Π°Π½ΠΈΡ€ΡƒΡŽ Π½Π°ΠΏΠΈΡΠ°Ρ‚ΡŒ ΠΊΠ½ΠΈΠ³Ρƒ ΠΏΡ€ΠΎ модСльно-ΠΎΡ€ΠΈΠ΅Π½Ρ‚ΠΈΡ€ΠΎΠ²Π°Π½Π½ΠΎΠ΅ ΠΏΡ€ΠΎΠ³Ρ€Π°ΠΌΠΌΠΈΡ€ΠΎΠ²Π°Π½ΠΈΠ΅ с автоматичСским Π³Π΅Π½Π΅Ρ€ΠΈΡ€ΠΎΠ²Π°Π½ΠΈΠ΅ΠΌ ΠΊΠΎΠ΄Π° ΠΏΡ€ΠΈΠΌΠ΅Π½ΠΈΡ‚Π΅Π»ΡŒΠ½ΠΎ ΠΊ Ρ€Π°Π·Ρ€Π°Π±ΠΎΡ‚ΠΊΠ΅ Ρ€Π°Π·Π½ΠΎΠΎΠ±Ρ€Π°Π·Π½Ρ‹Ρ… микропроцСссорных систСм управлСния элСктроприводов. Π’ этой ΠΊΠ½ΠΈΠ³Π΅ Π² Π½Π°ΡƒΡ‡Π½ΠΎ-практичСско-мСтодичСской Ρ„ΠΎΡ€ΠΌΠ΅ я ΠΏΠ»Π°Π½…

ΠŸΠ»Π°Π½ΠΈΡ€ΡƒΡŽ Π½Π°ΠΏΠΈΡΠ°Ρ‚ΡŒ ΠΊΠ½ΠΈΠ³Ρƒ ΠΏΡ€ΠΎ модСльно-ΠΎΡ€ΠΈΠ΅Π½Ρ‚ΠΈΡ€ΠΎΠ²Π°Π½Π½ΠΎΠ΅ ΠΏΡ€ΠΎΠ³Ρ€Π°ΠΌΠΌΠΈΡ€ΠΎΠ²Π°Π½ΠΈΠ΅ с автоматичСским Π³Π΅Π½Π΅Ρ€ΠΈΡ€ΠΎΠ²Π°Π½ΠΈΠ΅ΠΌ ΠΊΠΎΠ΄Π° ΠΏΡ€ΠΈΠΌΠ΅Π½ΠΈΡ‚Π΅Π»ΡŒΠ½ΠΎ ΠΊ Ρ€Π°Π·Ρ€Π°Π±ΠΎΡ‚ΠΊΠ΅ Ρ€Π°Π·Π½ΠΎΠΎΠ±Ρ€Π°Π·Π½Ρ‹Ρ… микропроцСссорных систСм управлСния элСктроприводов.

  • ΠŸΡƒΠ±Π»ΠΈΠΊΠ°Ρ†ΠΈΡ
  • 24.08.2022

Цифровая ΠΎΠ±Ρ€Π°Π±ΠΎΡ‚ΠΊΠ° сигналов, БистСмы связи, ΠœΠ°Ρ‚Π΅ΠΌΠ°Ρ‚ΠΈΠΊΠ° ΠΈ статистика

Β  Β  Β  Β  Β  Β  Β  Β  Β  Β  Β  Β  Β  Β  Β  Β  Β  Β  Β  Β  Β  Β  Β  Β  Β  Β  Β  Β  Β  Β  Β  Β  Β  Β  Β  Β  Β  &…

Π—Π΄Π΅ΡΡŒ собрана Π»ΠΈΡ‚Π΅Ρ€Π°Ρ‚ΡƒΡ€Π° ΠΏΠΎ ΠΊΠΎΠΌΠ±ΠΈΠ½ΠΈΡ€ΠΎΠ²Π°Π½Π½Ρ‹ΠΌ ΠΌΠ΅Ρ‚ΠΎΠ΄Π°ΠΌ мноТСствСнного доступа, Π² ΠΊΠΎΡ‚ΠΎΡ€Ρ‹Ρ… ΠΈΡΠΏΠΎΠ»ΡŒΠ·ΡƒΠ΅Ρ‚ΡΡ Ρ€Π°Π·Π΄Π΅Π»Π΅Π½ΠΈΠ΅ ΠΏΠΎΠ»ΡŒΠ·ΠΎΠ²Π°Ρ‚Π΅Π»Π΅ΠΉ Π² Π½Π΅ΡΠΊΠΎΠ»ΡŒΠΊΠΈΡ… рСсурсных пространствах.

  • вопрос
  • 23.08.2022

ΠœΠ°Ρ‚Π΅ΠΌΠ°Ρ‚ΠΈΠΊΠ° ΠΈ статистика, Радиолокация, Цифровая ΠΎΠ±Ρ€Π°Π±ΠΎΡ‚ΠΊΠ° сигналов

Π•ΡΡ‚ΡŒ записанный сигнал с Π΄Π°Ρ‚Ρ‡ΠΈΠΊΠ° (синус с ΡˆΡƒΠΌΠΎΠΌ). Как ΠΎΠΏΡ€Π΅Π΄Π΅Π»ΠΈΡ‚ΡŒ ΡΠΎΠΎΡ‚Π½ΠΎΡˆΠ΅Π½ΠΈΠ΅ сигнал/ΡˆΡƒΠΌ?

Π•ΡΡ‚ΡŒ записанный сигнал с Π΄Π°Ρ‚Ρ‡ΠΈΠΊΠ° (синус с ΡˆΡƒΠΌΠΎΠΌ). Как ΠΎΠΏΡ€Π΅Π΄Π΅Π»ΠΈΡ‚ΡŒ ΡΠΎΠΎΡ‚Π½ΠΎΡˆΠ΅Π½ΠΈΠ΅ сигнал/ΡˆΡƒΠΌ?

4 ΠžΡ‚Π²Π΅Ρ‚Π°

  • ЦОБ
  • цифровая ΠΎΠ±Ρ€Π°Π±ΠΎΡ‚ΠΊΠ° сигналов

23.08.2022

  • ΠŸΡƒΠ±Π»ΠΈΠΊΠ°Ρ†ΠΈΡ
  • 23.08.2022

Цифровая ΠΎΠ±Ρ€Π°Π±ΠΎΡ‚ΠΊΠ° сигналов, БистСмы связи, ΠœΠ°Ρ‚Π΅ΠΌΠ°Ρ‚ΠΈΠΊΠ° ΠΈ статистика

Β  Β  Β  Β  Β  Β  Β  Β  Β  Β  Β  Β  Β  Β  Β  Β  Β  Β  Β  Β  Β  Β  Β  Β  Β  Β  Β  Β  Β  Β  Β  Β  Β  Β  Β  Β  Β  &. ..

Π—Π΄Π΅ΡΡŒ соборана Π»ΠΈΡ‚Π΅Ρ€Π°Ρ‚ΡƒΡ€Π° ΠΏΠΎ ΠΌΠ΅Ρ‚ΠΎΠ΄Π°ΠΌ мноТСствСнного доступа с поляризационным Ρ€Π°Π·Π΄Π΅Π»Π΅Π½ΠΈΠ΅ΠΌ ΠΈ Ρ€Π°Π·Π΄Π΅Π»Π΅Π½ΠΈΠ΅ΠΌ ΠΏΠΎ ΠΎΡ€Π±ΠΈΡ‚Π°Π»ΡŒΠ½ΠΎΠΌ ΡƒΠ³Π»ΠΎΠ²ΠΎΠΌΡƒ ΠΌΠΎΠΌΠ΅Π½Ρ‚Ρƒ.

  • ΠŸΡƒΠ±Π»ΠΈΠΊΠ°Ρ†ΠΈΡ
  • 16.08.2022

Цифровая ΠΎΠ±Ρ€Π°Π±ΠΎΡ‚ΠΊΠ° сигналов, БистСмы связи, ΠœΠ°Ρ‚Π΅ΠΌΠ°Ρ‚ΠΈΠΊΠ° ΠΈ статистика

Β  Β  Β  Β  Β  Β  Β  Β  Β  Β  Β  Β  Β  Β  Β  Β  Β  Β  Β Β 

Π—Π΄Π΅ΡΡŒ собрана Π»ΠΈΡ‚Π΅Ρ€Π°Ρ‚ΡƒΡ€Π° ΠΏΠΎ ΠΌΠ΅Ρ‚ΠΎΠ΄Π°ΠΌ мноТСствСнного доступа с пространствСнным Ρ€Π°Π·Π΄Π΅Π»Π΅Π½ΠΈΠ΅ΠΌ.

  • вопрос
  • 22.07.2022

Π˜Π·ΠΎΠ±Ρ€Π°ΠΆΠ΅Π½ΠΈΡ ΠΈ Π²ΠΈΠ΄Π΅ΠΎ, Цифровая ΠΎΠ±Ρ€Π°Π±ΠΎΡ‚ΠΊΠ° сигналов, ΠœΠ°Ρ‚Π΅ΠΌΠ°Ρ‚ΠΈΠΊΠ° ΠΈ статистика, Биология, ВстраиваСмыС систСмы, Π“Π»ΡƒΠ±ΠΎΠΊΠΎΠ΅ ΠΈ машинноС ΠΎΠ±ΡƒΡ‡Π΅Π½ΠΈΠ΅(ИИ), Автоматизация испытаний, ΠŸΠ›Π˜Π‘ ΠΈ БнК, БистСмы управлСния, Π”Ρ€ΡƒΠ³ΠΎΠ΅

ЗдравствуйтС. МнС Π½ΡƒΠΆΠ½ΠΎ ΠΎΠ±Ρ€Π°Π±ΠΎΡ‚Π°Ρ‚ΡŒ большоС количСство Ρ„Π°ΠΉΠ»ΠΎΠ² с ΠΏΠΎΡ…ΠΎΠΆΠΈΠΌΠΈ названиями, ΠΊΠ°ΠΆΠ΄Ρ‹ΠΉ Π±Π»ΠΎΠΊ Ρ„Π°ΠΉΠ»ΠΎΠ² относится ΠΊ ΠΎΡ‚Π΄Π΅Π»ΡŒΠ½ΠΎΠΌΡƒ ΠΎΠ±ΡŠΠ΅ΠΊΡ‚Ρƒ, Π½Π°ΠΏΡ€ΠΈΠΌΠ΅Ρ€: file_1_1.txt file_1_2.txt file_1_3.txt file_1_4.txt fil…

ЗдравствуйтС. МнС Π½ΡƒΠΆΠ½ΠΎ ΠΎΠ±Ρ€Π°Π±ΠΎΡ‚Π°Ρ‚ΡŒ большоС количСство Ρ„Π°ΠΉΠ»ΠΎΠ² с ΠΏΠΎΡ…ΠΎΠΆΠΈΠΌΠΈ названиями, ΠΊΠ°ΠΆΠ΄Ρ‹ΠΉ Π±Π»ΠΎΠΊ Ρ„Π°ΠΉΠ»ΠΎΠ² относится ΠΊ ΠΎΡ‚Π΄Π΅Π»ΡŒΠ½ΠΎΠΌΡƒ ΠΎΠ±ΡŠΠ΅ΠΊΡ‚Ρƒ, Π½Π°ΠΏΡ€ΠΈΠΌΠ΅Ρ€: file_1_1.txt file_1_2.txt file_1_3.txt file_1_4.txt fil…

2 ΠžΡ‚Π²Π΅Ρ‚Π°

  • Ρ‡Ρ‚Π΅Π½ΠΈΠ΅

22.07.2022

  • вопрос
  • 17.07.2022

ΠœΠ°Ρ‚Π΅ΠΌΠ°Ρ‚ΠΈΠΊΠ° ΠΈ статистика, Цифровая ΠΎΠ±Ρ€Π°Π±ΠΎΡ‚ΠΊΠ° сигналов

Π£Π²Π°ΠΆΠ°Π΅ΠΌΡ‹Π΅ ΠΊΠΎΠ»Π»Π΅Π³ΠΈ, Π΄ΠΎΠ±Ρ€Ρ‹ΠΉ Π²Π΅Ρ‡Π΅Ρ€! Π’ ΠΎΠ±Ρ‰Π΅ΠΌ, Π²ΠΎΠ·Π½ΠΈΠΊΠ»Π° ΠΏΡ€ΠΎΠ±Π»Π΅ΠΌΠ° ΡΠ»Π΅Π΄ΡƒΡŽΡ‰Π΅Π³ΠΎ Ρ…Π°Ρ€Π°ΠΊΡ‚Π΅Ρ€Π°. Π˜ΠΌΠ΅Π΅Ρ‚ΡΡ сигнал, достаточно большой объСм Ρ‚ΠΎΡ‡Π΅ΠΊ, Π΄Π»ΠΈΡ‚Π΅Π»ΡŒΠ½ΠΎΡΡ‚ΡŒ порядка 35-40 сСкунд. Он прСдставлят собой ΠΏΠΎΡΠ»Π΅Π΄ΠΎΠ²Π°Ρ‚Π΅Π»ΡŒΠ½…

Π£Π²Π°ΠΆΠ°Π΅ΠΌΡ‹Π΅ ΠΊΠΎΠ»Π»Π΅Π³ΠΈ, Π΄ΠΎΠ±Ρ€Ρ‹ΠΉ Π²Π΅Ρ‡Π΅Ρ€! Π’ ΠΎΠ±Ρ‰Π΅ΠΌ, Π²ΠΎΠ·Π½ΠΈΠΊΠ»Π° ΠΏΡ€ΠΎΠ±Π»Π΅ΠΌΠ° ΡΠ»Π΅Π΄ΡƒΡŽΡ‰Π΅Π³ΠΎ Ρ…Π°Ρ€Π°ΠΊΡ‚Π΅Ρ€Π°. Π˜ΠΌΠ΅Π΅Ρ‚ΡΡ сигнал, достаточно большой объСм Ρ‚ΠΎΡ‡Π΅ΠΊ, Π΄Π»ΠΈΡ‚Π΅Π»ΡŒΠ½ΠΎΡΡ‚ΡŒ порядка 35-40 сСкунд. Он прСдставлят собой ΠΏΠΎΡΠ»Π΅Π΄ΠΎΠ²Π°Ρ‚Π΅Π»ΡŒΠ½…

  • MATLAB
  • Signal Processing

17.07.2022

  • вопрос
  • 15.07.2022

БистСмы связи, Цифровая ΠΎΠ±Ρ€Π°Π±ΠΎΡ‚ΠΊΠ° сигналов

ЗдравствуйтС! Π‘Π΄Π΅Π»Π°Π»Π° Π² симулинкС модСль сигнала с модуляциСй QPSK. На Π²Ρ…ΠΎΠ΄Π΅ сигнала ΠΏΠΎΠ΄Π°Π»Π° ΠΏΠΎΡΠ»Π΅Π΄ΠΎΠ²Π°Ρ‚Π»ΡŒΠ½ΠΎΠΌΡ‚ΡŒ с Π“Π΅Π½Π΅Ρ€Π°Ρ‚ΠΎΡ€Π° Π‘Π΅Ρ€Π½ΡƒΠ»ΠΈ Π±ΠΈΠ½Π°Ρ€Π½ΠΎΠ³ΠΎ. Sample time: 1/8000. ПРи Π²Ρ‹Π²ΠΎΠ΄Π΅ сигнала Π½Π° Π°Π½Π°Π»ΠΈΠ·Π°Ρ‚ΠΎΡ€ спСк…

ЗдравствуйтС! Π‘Π΄Π΅Π»Π°Π»Π° Π² симулинкС модСль сигнала с модуляциСй QPSK. На Π²Ρ…ΠΎΠ΄Π΅ сигнала ΠΏΠΎΠ΄Π°Π»Π° ΠΏΠΎΡΠ»Π΅Π΄ΠΎΠ²Π°Ρ‚Π»ΡŒΠ½ΠΎΠΌΡ‚ΡŒ с Π“Π΅Π½Π΅Ρ€Π°Ρ‚ΠΎΡ€Π° Π‘Π΅Ρ€Π½ΡƒΠ»ΠΈ Π±ΠΈΠ½Π°Ρ€Π½ΠΎΠ³ΠΎ. Sample time: 1/8000. ПРи Π²Ρ‹Π²ΠΎΠ΄Π΅ сигнала Π½Π° Π°Π½Π°Π»ΠΈΠ·Π°Ρ‚ΠΎΡ€ спСк…

  • сигнал
  • модуляция
  • qpsk
  • ΡΠΊΠΎΡ€ΠΎΡΡ‚ΡŒ Π±ΠΈΡ‚Π°
  • Π±ΠΈΡ‚Ρ€Π΅ΠΉΡ‚
  • символьная ΡΠΊΠΎΡ€ΠΎΡΡ‚ΡŒ
  • ΡΠΊΠΎΡ€ΠΎΡΡ‚ΡŒ ΠΏΠ΅Ρ€Π΅Π΄Π°Ρ‡ΠΈ ΠΈΠ½Ρ„ΠΎΡ€ΠΌΠ°Ρ†ΠΈΠΈ
  • цифровая манипуляция

15.07.2022

  • ΠŸΡƒΠ±Π»ΠΈΠΊΠ°Ρ†ΠΈΡ
  • 13.07.2022

Цифровая ΠΎΠ±Ρ€Π°Π±ΠΎΡ‚ΠΊΠ° сигналов, БистСмы связи, ΠœΠ°Ρ‚Π΅ΠΌΠ°Ρ‚ΠΈΠΊΠ° ΠΈ статистика

Β  Β  Β  Β  Β  Β  Β  Β  Β  Β  Β  Β  Β  Β  Β  Β  Β  Β  Β  Β  Β  Β  Β  Β  Β  Β  Β  Β  Β  Β  Β  Β  Β  Β  Β  Β  Β  &…

Π—Π΄Π΅ΡΡŒ собрана Π»ΠΈΡ‚Π΅Ρ€Π°Ρ‚ΡƒΡ€Π° ΠΏΠΎ ΠΌΠ΅Ρ‚ΠΎΠ΄Π°ΠΌ мноТСствСнного доступа с ΠΊΠΎΠ΄ΠΎΠ²Ρ‹ΠΌ Ρ€Π°Π·Π΄Π΅Π»Π΅Π½ΠΈΠ΅ΠΌ

Π Π΅Π·ΡƒΠ»ΡŒΡ‚Π°Ρ‚Ρ‹ поиска

НСт Ρ€Π΅Π·ΡƒΠ»ΡŒΡ‚Π°Ρ‚ΠΎΠ² поиска, ΠΏΠΎΠΏΡ€ΠΎΠ±ΡƒΠΉΡ‚Π΅ Π·Π°Π΄Π°Ρ‚ΡŒ Π΄Ρ€ΡƒΠ³ΠΈΠ΅ ΠΏΠ°Ρ€Π°ΠΌΠ΅Ρ‚Ρ€Ρ‹.

НСТноС Π²Π²Π΅Π΄Π΅Π½ΠΈΠ΅ Π² Π²Π΅ΠΊΡ‚ΠΎΡ€Π½Ρ‹Π΅ Π½ΠΎΡ€ΠΌΡ‹ Π² машинном ΠΎΠ±ΡƒΡ‡Π΅Π½ΠΈΠΈ

Π”Π°Ρ‚Π° ΠΏΡƒΠ±Π»ΠΈΠΊΠ°Ρ†ΠΈΠΈ 2018-02-05

ВычислСниС Π΄Π»ΠΈΠ½Ρ‹ ΠΈΠ»ΠΈ Π²Π΅Π»ΠΈΡ‡ΠΈΠ½Ρ‹ Π²Π΅ΠΊΡ‚ΠΎΡ€ΠΎΠ² часто трСбуСтся Π»ΠΈΠ±ΠΎ нСпосрСдствСнно ΠΊΠ°ΠΊ ΠΌΠ΅Ρ‚ΠΎΠ΄ рСгуляризации Π² машинном ΠΎΠ±ΡƒΡ‡Π΅Π½ΠΈΠΈ, Π»ΠΈΠ±ΠΎ ΠΊΠ°ΠΊ Ρ‡Π°ΡΡ‚ΡŒ Π±ΠΎΠ»Π΅Π΅ ΡˆΠΈΡ€ΠΎΠΊΠΈΡ… Π²Π΅ΠΊΡ‚ΠΎΡ€Π½Ρ‹Ρ… ΠΈΠ»ΠΈ ΠΌΠ°Ρ‚Ρ€ΠΈΡ‡Π½Ρ‹Ρ… ΠΎΠΏΠ΅Ρ€Π°Ρ†ΠΈΠΉ.

Π’ этом ΡƒΡ€ΠΎΠΊΠ΅ Π²Ρ‹ ΡƒΠ·Π½Π°Π΅Ρ‚Π΅, ΠΊΠ°ΠΊ Ρ€Π°ΡΡΡ‡ΠΈΡ‚Π°Ρ‚ΡŒ Π΄Π»ΠΈΠ½Ρƒ ΠΈΠ»ΠΈ Π²Π΅Π»ΠΈΡ‡ΠΈΠ½Ρƒ Π²Π΅ΠΊΡ‚ΠΎΡ€Π°, Π½Π°Π·Ρ‹Π²Π°Π΅ΠΌΡƒΡŽ Π²Π΅ΠΊΡ‚ΠΎΡ€Π½ΠΎΠΉ Π½ΠΎΡ€ΠΌΠΎΠΉ.

ПослС Π·Π°Π²Π΅Ρ€ΡˆΠ΅Π½ΠΈΡ этого ΡƒΡ€ΠΎΠΊΠ° Π²Ρ‹ ΡƒΠ·Π½Π°Π΅Ρ‚Π΅:

  • Норма L1, которая рассчитываСтся ΠΊΠ°ΠΊ сумма Π°Π±ΡΠΎΠ»ΡŽΡ‚Π½Ρ‹Ρ… Π·Π½Π°Ρ‡Π΅Π½ΠΈΠΉ Π²Π΅ΠΊΡ‚ΠΎΡ€Π°.
  • Норма L2, которая рассчитываСтся ΠΊΠ°ΠΊ ΠΊΠ²Π°Π΄Ρ€Π°Ρ‚Π½Ρ‹ΠΉ ΠΊΠΎΡ€Π΅Π½ΡŒ ΠΈΠ· суммы ΠΊΠ²Π°Π΄Ρ€Π°Ρ‚ΠΎΠ² Π²Π΅ΠΊΡ‚ΠΎΡ€Π½Ρ‹Ρ… Π·Π½Π°Ρ‡Π΅Π½ΠΈΠΉ.
  • Максимальная Π½ΠΎΡ€ΠΌΠ°, которая рассчитываСтся ΠΊΠ°ΠΊ ΠΌΠ°ΠΊΡΠΈΠΌΠ°Π»ΡŒΠ½Ρ‹Π΅ значСния Π²Π΅ΠΊΡ‚ΠΎΡ€Π°.

Π”Π°Π²Π°ΠΉΡ‚Π΅ Π½Π°Ρ‡Π½Π΅ΠΌ.

  • ОбновлСниС ΠΌΠ°Ρ€Ρ‚ / 2018: Π˜ΡΠΏΡ€Π°Π²Π»Π΅Π½Π° ​​опСчатка Π² ΡƒΡ€Π°Π²Π½Π΅Π½ΠΈΠΈ максимальной Π½ΠΎΡ€ΠΌΡ‹.
  • ОбновлСниС ΡΠ΅Π½Ρ‚ΡΠ±Ρ€ΡŒ / 2018: Π˜ΡΠΏΡ€Π°Π²Π»Π΅Π½Π° ​​опСчатка, связанная с Ρ€Π°Π·ΠΌΠ΅Ρ€ΠΎΠΌ Π·Π°Π΄Π°Π½Π½Ρ‹Ρ… Π²Π΅ΠΊΡ‚ΠΎΡ€ΠΎΠ².

ΠžΠ±Π·ΠΎΡ€ ΡƒΡ‡Π΅Π±Π½ΠΈΠΊΠ°

Π­Ρ‚ΠΎΡ‚ ΡƒΡ€ΠΎΠΊ Ρ€Π°Π·Π΄Π΅Π»Π΅Π½ Π½Π° 4 части; ΠΎΠ½ΠΈ Π΅ΡΡ‚ΡŒ:

  1. Π’Π΅ΠΊΡ‚ΠΎΡ€ Норма
  2. Π’Π΅ΠΊΡ‚ΠΎΡ€ L1 Норма
  3. Π’Π΅ΠΊΡ‚ΠΎΡ€ L2 Норма
  4. Π’Π΅ΠΊΡ‚ΠΎΡ€ Макс Норм

Π’Π΅ΠΊΡ‚ΠΎΡ€ Норма

ВычислСниС Ρ€Π°Π·ΠΌΠ΅Ρ€Π° ΠΈΠ»ΠΈ Π΄Π»ΠΈΠ½Ρ‹ Π²Π΅ΠΊΡ‚ΠΎΡ€Π° часто трСбуСтся Π»ΠΈΠ±ΠΎ нСпосрСдствСнно, Π»ΠΈΠ±ΠΎ ΠΊΠ°ΠΊ Ρ‡Π°ΡΡ‚ΡŒ Π±ΠΎΠ»Π΅Π΅ ΡˆΠΈΡ€ΠΎΠΊΠΎΠΉ ΠΎΠΏΠ΅Ρ€Π°Ρ†ΠΈΠΈ Π½Π°Π΄ Π²Π΅ΠΊΡ‚ΠΎΡ€ΠΎΠΌ ΠΈΠ»ΠΈ Π²Π΅ΠΊΡ‚ΠΎΡ€Π½ΠΎΠΉ ΠΌΠ°Ρ‚Ρ€ΠΈΡ†Π΅ΠΉ.

Π”Π»ΠΈΠ½Π° Π²Π΅ΠΊΡ‚ΠΎΡ€Π° называСтся Π²Π΅ΠΊΡ‚ΠΎΡ€Π½ΠΎΠΉ Π½ΠΎΡ€ΠΌΠΎΠΉ ΠΈΠ»ΠΈ Π²Π΅Π»ΠΈΡ‡ΠΈΠ½ΠΎΠΉ Π²Π΅ΠΊΡ‚ΠΎΡ€Π°.

Π”Π»ΠΈΠ½Π° Π²Π΅ΠΊΡ‚ΠΎΡ€Π° прСдставляСт собой Π½Π΅ΠΎΡ‚Ρ€ΠΈΡ†Π°Ρ‚Π΅Π»ΡŒΠ½ΠΎΠ΅ число, ΠΊΠΎΡ‚ΠΎΡ€ΠΎΠ΅ описываСт экстСнт Π²Π΅ΠΊΡ‚ΠΎΡ€Π° Π² пространствС, ΠΈ ΠΈΠ½ΠΎΠ³Π΄Π° Π΅Π³ΠΎ Π½Π°Π·Ρ‹Π²Π°ΡŽΡ‚ Π²Π΅Π»ΠΈΡ‡ΠΈΠ½ΠΎΠΉ ΠΈΠ»ΠΈ Π½ΠΎΡ€ΠΌΠΎΠΉ Π²Π΅ΠΊΡ‚ΠΎΡ€Π°.

— Π‘Ρ‚Ρ€Π°Π½ΠΈΡ†Π° 112,Руководство ΠΏΠΎ Π»ΠΈΠ½Π΅ΠΉΠ½ΠΎΠΉ Π°Π»Π³Π΅Π±Ρ€Π΅, 2017

Π”Π»ΠΈΠ½Π° Π²Π΅ΠΊΡ‚ΠΎΡ€Π° всСгда являСтся ΠΏΠΎΠ»ΠΎΠΆΠΈΡ‚Π΅Π»ΡŒΠ½Ρ‹ΠΌ числом, Π·Π° ΠΈΡΠΊΠ»ΡŽΡ‡Π΅Π½ΠΈΠ΅ΠΌ Π²Π΅ΠΊΡ‚ΠΎΡ€Π° со всСми Π½ΡƒΠ»Π΅Π²Ρ‹ΠΌΠΈ значСниями. Он рассчитываСтся с использованиСм Π½Π΅ΠΊΠΎΡ‚ΠΎΡ€ΠΎΠΉ ΠΌΠ΅Ρ€Ρ‹, которая суммируСт расстояниС Π²Π΅ΠΊΡ‚ΠΎΡ€Π° ΠΎΡ‚ Π½Π°Ρ‡Π°Π»Π° Π²Π΅ΠΊΡ‚ΠΎΡ€Π½ΠΎΠ³ΠΎ пространства. НапримСр, источником Π²Π΅ΠΊΡ‚ΠΎΡ€Π½ΠΎΠ³ΠΎ пространства для Π²Π΅ΠΊΡ‚ΠΎΡ€Π° с 3 элСмСнтами являСтся (0, 0, 0). 1.

ΠžΠ±ΠΎΠ·Π½Π°Ρ‡Π΅Π½ΠΈΡ для Π½ΠΎΡ€ΠΌΡ‹ L1 Π²Π΅ΠΊΡ‚ΠΎΡ€Π°: || v || 1, Π³Π΄Π΅ 1 — индСкс. Π’Π°ΠΊΠΈΠΌ ΠΎΠ±Ρ€Π°Π·ΠΎΠΌ, эту Π΄Π»ΠΈΠ½Ρƒ ΠΈΠ½ΠΎΠ³Π΄Π° Π½Π°Π·Ρ‹Π²Π°ΡŽΡ‚ Π½ΠΎΡ€ΠΌΠΎΠΉ такси ΠΈΠ»ΠΈ Π½ΠΎΡ€ΠΌΠΎΠΉ ΠœΠ°Π½Ρ…ΡΡ‚Ρ‚Π΅Π½Π°.

l1(v) = ||v||1

Норма L1 рассчитываСтся ΠΊΠ°ΠΊ сумма Π°Π±ΡΠΎΠ»ΡŽΡ‚Π½Ρ‹Ρ… Π·Π½Π°Ρ‡Π΅Π½ΠΈΠΉ Π²Π΅ΠΊΡ‚ΠΎΡ€Π°, Π³Π΄Π΅ Π°Π±ΡΠΎΠ»ΡŽΡ‚Π½ΠΎΠ΅ Π·Π½Π°Ρ‡Π΅Π½ΠΈΠ΅ скаляра ΠΈΡΠΏΠΎΠ»ΡŒΠ·ΡƒΠ΅Ρ‚ ΠΎΠ±ΠΎΠ·Π½Π°Ρ‡Π΅Π½ΠΈΠ΅ | a1 |. По сути, Π½ΠΎΡ€ΠΌΠ° — это вычислСниС манхэттСнского расстояния ΠΎΡ‚ Π½Π°Ρ‡Π°Π»Π° Π²Π΅ΠΊΡ‚ΠΎΡ€Π½ΠΎΠ³ΠΎ пространства.

||v||1 = |a1| + |a2| + |a3|

Норма L1 Π²Π΅ΠΊΡ‚ΠΎΡ€Π° ΠΌΠΎΠΆΠ΅Ρ‚ Π±Ρ‹Ρ‚ΡŒ вычислСна Π² NumPy с ΠΏΠΎΠΌΠΎΡ‰ΡŒΡŽ Ρ„ΡƒΠ½ΠΊΡ†ΠΈΠΈ norm () с ΠΏΠ°Ρ€Π°ΠΌΠ΅Ρ‚Ρ€ΠΎΠΌ для указания порядка Π½ΠΎΡ€ΠΌΡ‹, Π² Π΄Π°Π½Π½ΠΎΠΌ случаС 1.

# l1 norm of a vector
from numpy import array
from numpy.linalg import norm
a = array([1, 2, 3])
print(a)
l1 = norm(a, 1)
print(l1)

Π‘Π½Π°Ρ‡Π°Π»Π° опрСдСляСтся Π²Π΅ΠΊΡ‚ΠΎΡ€ 1 Γ— 3, Π·Π°Ρ‚Π΅ΠΌ вычисляСтся Π½ΠΎΡ€ΠΌΠ° Π²Π΅ΠΊΡ‚ΠΎΡ€Π° L1.

ΠŸΡ€ΠΈ Π²Ρ‹ΠΏΠΎΠ»Π½Π΅Π½ΠΈΠΈ ΠΏΡ€ΠΈΠΌΠ΅Ρ€Π° сначала пСчатаСтся ΠΎΠΏΡ€Π΅Π΄Π΅Π»Π΅Π½Π½Ρ‹ΠΉ Π²Π΅ΠΊΡ‚ΠΎΡ€, Π° Π·Π°Ρ‚Π΅ΠΌ Π½ΠΎΡ€ΠΌΠ° L1 Π²Π΅ΠΊΡ‚ΠΎΡ€Π°.

[1 2 3]

6.0

Норма L1 часто ΠΈΡΠΏΠΎΠ»ΡŒΠ·ΡƒΠ΅Ρ‚ΡΡ ΠΏΡ€ΠΈ ΠΏΠΎΠ΄Π±ΠΎΡ€Π΅ Π°Π»Π³ΠΎΡ€ΠΈΡ‚ΠΌΠΎΠ² машинного обучСния Π² качСствС ΠΌΠ΅Ρ‚ΠΎΠ΄Π° рСгуляризации, Π½Π°ΠΏΡ€ΠΈΠΌΠ΅Ρ€ ΠΌΠ΅Ρ‚ΠΎΠ΄, ΠΏΠΎΠ·Π²ΠΎΠ»ΡΡŽΡ‰ΠΈΠΉ ΡΠΎΡ…Ρ€Π°Π½ΡΡ‚ΡŒ коэффициСнты ΠΌΠΎΠ΄Π΅Π»ΠΈ ΠΌΠ°Π»Ρ‹ΠΌΠΈ, ΠΈ, Π² свою ΠΎΡ‡Π΅Ρ€Π΅Π΄ΡŒ, модСль ΠΌΠ΅Π½Π΅Π΅ слоТной. 2)

Норму L2 Π²Π΅ΠΊΡ‚ΠΎΡ€Π° ΠΌΠΎΠΆΠ½ΠΎ Ρ€Π°ΡΡΡ‡ΠΈΡ‚Π°Ρ‚ΡŒ Π² NumPy с ΠΏΠΎΠΌΠΎΡ‰ΡŒΡŽ Ρ„ΡƒΠ½ΠΊΡ†ΠΈΠΈ norm () с ΠΏΠ°Ρ€Π°ΠΌΠ΅Ρ‚Ρ€Π°ΠΌΠΈ ΠΏΠΎ ΡƒΠΌΠΎΠ»Ρ‡Π°Π½ΠΈΡŽ.

# l2 norm of a vector
from numpy import array
from numpy.linalg import norm
a = array([1, 2, 3])
print(a)
l2 = norm(a)
print(l2)

Π‘Π½Π°Ρ‡Π°Π»Π° опрСдСляСтся Π²Π΅ΠΊΡ‚ΠΎΡ€ 1 Γ— 3, Π·Π°Ρ‚Π΅ΠΌ вычисляСтся Π½ΠΎΡ€ΠΌΠ° Π²Π΅ΠΊΡ‚ΠΎΡ€Π° L2.

ΠŸΡ€ΠΈ Π²Ρ‹ΠΏΠΎΠ»Π½Π΅Π½ΠΈΠΈ ΠΏΡ€ΠΈΠΌΠ΅Ρ€Π° сначала пСчатаСтся ΠΎΠΏΡ€Π΅Π΄Π΅Π»Π΅Π½Π½Ρ‹ΠΉ Π²Π΅ΠΊΡ‚ΠΎΡ€, Π° Π·Π°Ρ‚Π΅ΠΌ Π½ΠΎΡ€ΠΌΠ° L2 Π²Π΅ΠΊΡ‚ΠΎΡ€Π°.

[1 2 3]

3.74165738677

Как ΠΈ Π½ΠΎΡ€ΠΌΠ° L1, Π½ΠΎΡ€ΠΌΠ° L2 часто ΠΈΡΠΏΠΎΠ»ΡŒΠ·ΡƒΠ΅Ρ‚ΡΡ ΠΏΡ€ΠΈ ΠΏΠΎΠ΄Π±ΠΎΡ€Π΅ Π°Π»Π³ΠΎΡ€ΠΈΡ‚ΠΌΠΎΠ² машинного обучСния Π² качСствС ΠΌΠ΅Ρ‚ΠΎΠ΄Π° рСгуляризации, Π½Π°ΠΏΡ€ΠΈΠΌΠ΅Ρ€ ΠΌΠ΅Ρ‚ΠΎΠ΄, ΠΏΠΎΠ·Π²ΠΎΠ»ΡΡŽΡ‰ΠΈΠΉ ΡΠΎΡ…Ρ€Π°Π½ΡΡ‚ΡŒ коэффициСнты ΠΌΠΎΠ΄Π΅Π»ΠΈ ΠΌΠ°Π»Ρ‹ΠΌΠΈ ΠΈ, Π² свою ΠΎΡ‡Π΅Ρ€Π΅Π΄ΡŒ, модСль ΠΌΠ΅Π½Π΅Π΅ слоТной.

БСзусловно, Π½ΠΎΡ€ΠΌΠ° L2 Ρ‡Π°Ρ‰Π΅ ΠΈΡΠΏΠΎΠ»ΡŒΠ·ΡƒΠ΅Ρ‚ΡΡ, Ρ‡Π΅ΠΌ Π΄Ρ€ΡƒΠ³ΠΈΠ΅ Π²Π΅ΠΊΡ‚ΠΎΡ€Π½Ρ‹Π΅ Π½ΠΎΡ€ΠΌΡ‹ Π² машинном ΠΎΠ±ΡƒΡ‡Π΅Π½ΠΈΠΈ.

Π’Π΅ΠΊΡ‚ΠΎΡ€ Макс Норм

Π”Π»ΠΈΠ½Π° Π²Π΅ΠΊΡ‚ΠΎΡ€Π° ΠΌΠΎΠΆΠ΅Ρ‚ Π±Ρ‹Ρ‚ΡŒ рассчитана с использованиСм максимальной Π½ΠΎΡ€ΠΌΡ‹, Ρ‚Π°ΠΊΠΆΠ΅ Π½Π°Π·Ρ‹Π²Π°Π΅ΠΌΠΎΠΉ максимальной Π½ΠΎΡ€ΠΌΠΎΠΉ.

Максимальная Π½ΠΎΡ€ΠΌΠ° Π²Π΅ΠΊΡ‚ΠΎΡ€Π° называСтся L ^ inf, Π³Π΄Π΅ inf — Π²Π΅Ρ€Ρ…Π½ΠΈΠΉ индСкс ΠΈ ΠΌΠΎΠΆΠ΅Ρ‚ Π±Ρ‹Ρ‚ΡŒ прСдставлСн символом бСсконСчности. ΠžΠ±ΠΎΠ·Π½Π°Ρ‡Π΅Π½ΠΈΡ для максимальной Π½ΠΎΡ€ΠΌΡ‹: || x || inf, Π³Π΄Π΅ inf — индСкс.

maxnorm(v) = ||v||inf

Максимальная Π½ΠΎΡ€ΠΌΠ° вычисляСтся ΠΊΠ°ΠΊ Π²ΠΎΠ·Π²Ρ€Π°Ρ‰Π°ΡŽΡ‰Π°Ρ максимальноС Π·Π½Π°Ρ‡Π΅Π½ΠΈΠ΅ Π²Π΅ΠΊΡ‚ΠΎΡ€Π°, ΠΎΡ‚ΡΡŽΠ΄Π° ΠΈ Π½Π°Π·Π²Π°Π½ΠΈΠ΅.

||v||inf = max(|a1|, |a2|, |a3|)

Максимальная Π½ΠΎΡ€ΠΌΠ° Π²Π΅ΠΊΡ‚ΠΎΡ€Π° ΠΌΠΎΠΆΠ΅Ρ‚ Π±Ρ‹Ρ‚ΡŒ вычислСна Π² NumPy с ΠΏΠΎΠΌΠΎΡ‰ΡŒΡŽ Ρ„ΡƒΠ½ΠΊΡ†ΠΈΠΈ norm () с ΠΏΠ°Ρ€Π°ΠΌΠ΅Ρ‚Ρ€ΠΎΠΌ порядка, установлСнным Π² inf.

# max norm of a vector
from numpy import inf
from numpy import array
from numpy.linalg import norm
a = array([1, 2, 3])
print(a)
maxnorm = norm(a, inf)
print(maxnorm)

Π‘Π½Π°Ρ‡Π°Π»Π° опрСдСляСтся Π²Π΅ΠΊΡ‚ΠΎΡ€ 1 Γ— 3, Π·Π°Ρ‚Π΅ΠΌ вычисляСтся максимальная Π½ΠΎΡ€ΠΌΠ° Π²Π΅ΠΊΡ‚ΠΎΡ€Π°.

ΠŸΡ€ΠΈ запускС ΠΏΡ€ΠΈΠΌΠ΅Ρ€Π° сначала пСчатаСтся ΠΎΠΏΡ€Π΅Π΄Π΅Π»Π΅Π½Π½Ρ‹ΠΉ Π²Π΅ΠΊΡ‚ΠΎΡ€, Π° Π·Π°Ρ‚Π΅ΠΌ максимальная Π½ΠΎΡ€ΠΌΠ° Π²Π΅ΠΊΡ‚ΠΎΡ€Π°

[1 2 3]

3.0

Максимальная Π½ΠΎΡ€ΠΌΠ° Ρ‚Π°ΠΊΠΆΠ΅ ΠΈΡΠΏΠΎΠ»ΡŒΠ·ΡƒΠ΅Ρ‚ΡΡ Π² качСствС рСгуляризации Π² машинном ΠΎΠ±ΡƒΡ‡Π΅Π½ΠΈΠΈ, Π½Π°ΠΏΡ€ΠΈΠΌΠ΅Ρ€, Π² вСсах Π½Π΅ΠΉΡ€ΠΎΠ½Π½Ρ‹Ρ… сСтСй, Π½Π°Π·Ρ‹Π²Π°Π΅ΠΌΠΎΠΉ максимальной Π½ΠΎΡ€ΠΌΠ°Π»ΠΈΠ·Π°Ρ†ΠΈΠ΅ΠΉ.

Ρ€Π°ΡΡˆΠΈΡ€Π΅Π½ΠΈΡ

Π’ этом Ρ€Π°Π·Π΄Π΅Π»Π΅ пСрСчислСны Π½Π΅ΠΊΠΎΡ‚ΠΎΡ€Ρ‹Π΅ ΠΈΠ΄Π΅ΠΈ ΠΏΠΎ Ρ€Π°ΡΡˆΠΈΡ€Π΅Π½ΠΈΡŽ ΡƒΡ‡Π΅Π±Π½ΠΈΠΊΠ°, ΠΊΠΎΡ‚ΠΎΡ€Ρ‹Π΅ Π²Ρ‹, Π²ΠΎΠ·ΠΌΠΎΠΆΠ½ΠΎ, Π·Π°Ρ…ΠΎΡ‚ΠΈΡ‚Π΅ ΠΈΠ·ΡƒΡ‡ΠΈΡ‚ΡŒ.

  • Π‘ΠΎΠ·Π΄Π°ΠΉΡ‚Π΅ 5 ΠΏΡ€ΠΈΠΌΠ΅Ρ€ΠΎΠ², ΠΈΡΠΏΠΎΠ»ΡŒΠ·ΡƒΡ ΠΊΠ°ΠΆΠ΄ΡƒΡŽ ΠΎΠΏΠ΅Ρ€Π°Ρ†ΠΈΡŽ, ΠΈΡΠΏΠΎΠ»ΡŒΠ·ΡƒΡ ваши собствСнныС Π΄Π°Π½Π½Ρ‹Π΅.
  • Π Π΅Π°Π»ΠΈΠ·ΡƒΠΉΡ‚Π΅ ΠΊΠ°ΠΆΠ΄ΡƒΡŽ ΠΌΠ°Ρ‚Ρ€ΠΈΡ‡Π½ΡƒΡŽ ΠΎΠΏΠ΅Ρ€Π°Ρ†ΠΈΡŽ Π²Ρ€ΡƒΡ‡Π½ΡƒΡŽ для ΠΌΠ°Ρ‚Ρ€ΠΈΡ†, ΠΎΠΏΡ€Π΅Π΄Π΅Π»Π΅Π½Π½Ρ‹Ρ… ΠΊΠ°ΠΊ списки списков.
  • НайдитС Π΄ΠΎΠΊΡƒΠΌΠ΅Π½Ρ‚Ρ‹ ΠΏΠΎ ΠΌΠ°ΡˆΠΈΠ½Π½ΠΎΠΌΡƒ ΠΎΠ±ΡƒΡ‡Π΅Π½ΠΈΡŽ ΠΈ Π½Π°ΠΉΠ΄ΠΈΡ‚Π΅ 1 ΠΏΡ€ΠΈΠΌΠ΅Ρ€ ΠΊΠ°ΠΆΠ΄ΠΎΠΉ ΠΈΡΠΏΠΎΠ»ΡŒΠ·ΡƒΠ΅ΠΌΠΎΠΉ ΠΎΠΏΠ΅Ρ€Π°Ρ†ΠΈΠΈ.

Если Π²Ρ‹ исслСдуСтС ΠΊΠ°ΠΊΠΎΠ΅-Π»ΠΈΠ±ΠΎ ΠΈΠ· этих Ρ€Π°ΡΡˆΠΈΡ€Π΅Π½ΠΈΠΉ, я Ρ…ΠΎΡ‚Π΅Π» Π±Ρ‹ Π·Π½Π°Ρ‚ΡŒ.

Π”Π°Π»ΡŒΠ½Π΅ΠΉΡˆΠ΅Π΅ Ρ‡Ρ‚Π΅Π½ΠΈΠ΅

Π­Ρ‚ΠΎΡ‚ Ρ€Π°Π·Π΄Π΅Π» прСдоставляСт большС рСсурсов ΠΏΠΎ Ρ‚Π΅ΠΌΠ΅, Ссли Π²Ρ‹ Ρ…ΠΎΡ‚ΠΈΡ‚Π΅ ΡƒΠ³Π»ΡƒΠ±ΠΈΡ‚ΡŒΡΡ.

ΠΊΠ½ΠΈΠ³ΠΈ

  • Π’Π²Π΅Π΄Π΅Π½ΠΈΠ΅ Π² Π»ΠΈΠ½Π΅ΠΉΠ½ΡƒΡŽ Π°Π»Π³Π΅Π±Ρ€Ρƒ, 2016
  • Π“Π»Π°Π²Π° 2, ЛинСйная Π°Π»Π³Π΅Π±Ρ€Π°,Π“Π»ΡƒΠ±ΠΎΠΊΠΎΠ΅ ΠΎΠ±ΡƒΡ‡Π΅Π½ΠΈΠ΅, 2016

API

  • API numpy.linalg.norm ()

ΡΡ‚Π°Ρ‚ΡŒΠΈ

  • Норма (ΠΌΠ°Ρ‚Π΅ΠΌΠ°Ρ‚ΠΈΠΊΠ°) Π² Π’ΠΈΠΊΠΈΠΏΠ΅Π΄ΠΈΠΈ

РСзюмС

Π’ этом ΡƒΡ€ΠΎΠΊΠ΅ Π²Ρ‹ ΠΎΠ±Π½Π°Ρ€ΡƒΠΆΠΈΠ»ΠΈ Ρ€Π°Π·Π»ΠΈΡ‡Π½Ρ‹Π΅ способы вычислСния Π΄Π»ΠΈΠ½Ρ‹ ΠΈΠ»ΠΈ Π²Π΅Π»ΠΈΡ‡ΠΈΠ½Ρ‹ Π²Π΅ΠΊΡ‚ΠΎΡ€Π°, Π½Π°Π·Ρ‹Π²Π°Π΅ΠΌΡ‹Π΅ Π²Π΅ΠΊΡ‚ΠΎΡ€Π½ΠΎΠΉ Π½ΠΎΡ€ΠΌΠΎΠΉ.

Π’ частности, Π²Ρ‹ ΡƒΠ·Π½Π°Π»ΠΈ:

  • Норма L1, которая рассчитываСтся ΠΊΠ°ΠΊ сумма Π°Π±ΡΠΎΠ»ΡŽΡ‚Π½Ρ‹Ρ… Π·Π½Π°Ρ‡Π΅Π½ΠΈΠΉ Π²Π΅ΠΊΡ‚ΠΎΡ€Π°.
  • Норма L2, которая рассчитываСтся ΠΊΠ°ΠΊ ΠΊΠ²Π°Π΄Ρ€Π°Ρ‚Π½Ρ‹ΠΉ ΠΊΠΎΡ€Π΅Π½ΡŒ ΠΈΠ· суммы ΠΊΠ²Π°Π΄Ρ€Π°Ρ‚ΠΎΠ² Π²Π΅ΠΊΡ‚ΠΎΡ€Π½Ρ‹Ρ… Π·Π½Π°Ρ‡Π΅Π½ΠΈΠΉ.
  • Максимальная Π½ΠΎΡ€ΠΌΠ°, которая рассчитываСтся ΠΊΠ°ΠΊ ΠΌΠ°ΠΊΡΠΈΠΌΠ°Π»ΡŒΠ½Ρ‹Π΅ значСния Π²Π΅ΠΊΡ‚ΠΎΡ€Π°.

Π£ вас Π΅ΡΡ‚ΡŒ вопросы?
Π—Π°Π΄Π°ΠΉΡ‚Π΅ свои вопросы Π² коммСнтариях Π½ΠΈΠΆΠ΅, ΠΈ я сдСлаю всС Π²ΠΎΠ·ΠΌΠΎΠΆΠ½ΠΎΠ΅, Ρ‡Ρ‚ΠΎΠ±Ρ‹ ΠΎΡ‚Π²Π΅Ρ‚ΠΈΡ‚ΡŒ.

ΠžΡ€ΠΈΠ³ΠΈΠ½Π°Π»ΡŒΠ½Π°Ρ ΡΡ‚Π°Ρ‚ΡŒΡ

Π’Π΅ΠΊΡ‚ΠΎΡ€Ρ‹. ДСйствия с Π²Π΅ΠΊΡ‚ΠΎΡ€Π°ΠΌΠΈ

Π’Π΅ΠΊΡ‚ΠΎΡ€Ρ‹. ДСйствия с Π²Π΅ΠΊΡ‚ΠΎΡ€Π°ΠΌΠΈ.Β Π’ этой ΡΡ‚Π°Ρ‚ΡŒΠ΅ ΠΌΡ‹ ΠΏΠΎΠ³ΠΎΠ²ΠΎΡ€ΠΈΠΌ ΠΎ Ρ‚ΠΎΠΌ, Ρ‡Ρ‚ΠΎ Ρ‚Π°ΠΊΠΎΠ΅ Π²Π΅ΠΊΡ‚ΠΎΡ€, Β ΠΊΠ°ΠΊ Π½Π°Ρ…ΠΎΠ΄ΠΈΡ‚ΡŒ Π΅Π³ΠΎ Π΄Π»ΠΈΠ½Ρƒ, ΠΈ ΠΊΠ°ΠΊ ΡƒΠΌΠ½ΠΎΠΆΠ°Ρ‚ΡŒ Π²Π΅ΠΊΡ‚ΠΎΡ€ Π½Π° число, Β Π° Ρ‚Π°ΠΊΠΆΠ΅ ΠΊΠ°ΠΊ Π½Π°Ρ…ΠΎΠ΄ΠΈΡ‚ΡŒ сумму, Ρ€Π°Π·Π½ΠΎΡΡ‚ΡŒ ΠΈ скалярноС ΠΏΡ€ΠΎΠΈΠ·Π²Π΅Π΄Π΅Π½ΠΈΠ΅ Π΄Π²ΡƒΡ… Π²Π΅ΠΊΡ‚ΠΎΡ€ΠΎΠ².

Как ΠΎΠ±Ρ‹Ρ‡Π½ΠΎ, Π½Π΅ΠΌΠ½ΠΎΠ³ΠΎ самой Π½Π΅ΠΎΠ±Ρ…ΠΎΠ΄ΠΈΠΌΠΎΠΉ Ρ‚Π΅ΠΎΡ€ΠΈΠΈ.

Π’Π΅ΠΊΡ‚ΠΎΡ€ — это Π½Π°ΠΏΡ€Π°Π²Π»Π΅Π½Π½Ρ‹ΠΉ ΠΎΡ‚Ρ€Π΅Π·ΠΎΠΊ, Ρ‚ΠΎ Π΅ΡΡ‚ΡŒ Ρ‚Π°ΠΊΠΎΠΉ ΠΎΡ‚Ρ€Π΅Π·ΠΎΠΊ, Ρƒ ΠΊΠΎΡ‚ΠΎΡ€ΠΎΠ³ΠΎ Π΅ΡΡ‚ΡŒ Π½Π°Ρ‡Π°Π»ΠΎ ΠΈ ΠΊΠΎΠ½Π΅Ρ†:

Π—Π΄Π΅ΡΡŒ Ρ‚ΠΎΡ‡ΠΊΠ° А — Π½Π°Ρ‡Π°Π»ΠΎ Π²Π΅ΠΊΡ‚ΠΎΡ€Π°, Π° Ρ‚ΠΎΡ‡ΠΊΠ° Π’ — Π΅Π³ΠΎ ΠΊΠΎΠ½Π΅Ρ†.

Π£ Π²Π΅ΠΊΡ‚ΠΎΡ€Π° Π΅ΡΡ‚ΡŒ Π΄Π²Π° ΠΏΠ°Ρ€Π°ΠΌΠ΅Ρ‚Ρ€Π°: Π΅Π³ΠΎ Π΄Π»ΠΈΠ½Π° ΠΈ Π½Π°ΠΏΡ€Π°Π²Π»Π΅Π½ΠΈΠ΅.

Π”Π»ΠΈΠ½Π° Π²Π΅ΠΊΡ‚ΠΎΡ€Π° — это Π΄Π»ΠΈΠ½Π° ΠΎΡ‚Ρ€Π΅Π·ΠΊΠ°, ΡΠΎΠ΅Π΄ΠΈΠ½ΡΡŽΡ‰Π΅Π³ΠΎ Π½Π°Ρ‡Π°Π»ΠΎ ΠΈ ΠΊΠΎΠ½Π΅Ρ† Π²Π΅ΠΊΡ‚ΠΎΡ€Π°. Π”Π»ΠΈΠ½Π° Π²Π΅ΠΊΡ‚ΠΎΡ€Π°Β  обозначаСтся 

Π”Π²Π° Π²Π΅ΠΊΡ‚ΠΎΡ€Π° Π½Π°Π·Ρ‹Π²Π°ΡŽΡ‚ΡΡ Ρ€Π°Π²Π½Ρ‹ΠΌΠΈ, Ссли ΠΎΠ½ΠΈ ΠΈΠΌΠ΅ΡŽΡ‚ ΠΎΠ΄ΠΈΠ½Π°ΠΊΠΎΠ²ΡƒΡŽ Π΄Π»ΠΈΠ½Ρƒ ΠΈ сонаправлСны.

Π”Π²Π° Π²Π΅ΠΊΡ‚ΠΎΡ€Π° Π½Π°Π·Ρ‹Π²Π°ΡŽΡ‚ΡΡ сонаправлСнными, Ссли ΠΎΠ½ΠΈ Π»Π΅ΠΆΠ°Ρ‚ Π½Π° ΠΏΠ°Ρ€Π°Π»Π»Π΅Π»ΡŒΠ½Ρ‹Ρ… прямых ΠΈ Π½Π°ΠΏΡ€Π°Π²Π»Π΅Π½Ρ‹ Π² ΠΎΠ΄Π½Ρƒ сторону: Π²Π΅ΠΊΡ‚ΠΎΡ€Π° ΠΈ Β  сонаправлСны:

Π”Π²Π° Π²Π΅ΠΊΡ‚ΠΎΡ€Π° Π½Π°Π·Ρ‹Π²Π°ΡŽΡ‚ΡΡ ΠΏΡ€ΠΎΡ‚ΠΈΠ²ΠΎΠΏΠΎΠ»ΠΎΠΆΠ½ΠΎ Π½Π°ΠΏΡ€Π°Π²Π»Π΅Π½Π½Ρ‹ΠΌΠΈ, Ссли ΠΎΠ½ΠΈ Π»Π΅ΠΆΠ°Ρ‚ Π½Π° ΠΏΠ°Ρ€Π°Π»Π»Π΅Π»ΡŒΠ½Ρ‹Ρ… прямых ΠΈ Π½Π°ΠΏΡ€Π°Π²Π»Π΅Π½Ρ‹ Π² ΠΏΡ€ΠΎΡ‚ΠΈΠ²ΠΎΠΏΠΎΠ»ΠΎΠΆΠ½Ρ‹Π΅ стороны: Π²Π΅ΠΊΡ‚ΠΎΡ€Π° Β ΠΈ , Π° Ρ‚Π°ΠΊΠΆΠ΅Β  ΠΈ Β Π½Π°ΠΏΡ€Π°Π²Π»Π΅Π½Ρ‹ Π² ΠΏΡ€ΠΎΡ‚ΠΈΠ²ΠΎΠΏΠΎΠ»ΠΎΠΆΠ½Ρ‹Π΅ стороны:

Π’Π΅ΠΊΡ‚ΠΎΡ€Π°, Π»Π΅ΠΆΠ°Ρ‰ΠΈΠ΅ Π½Π° ΠΏΠ°Ρ€Π°Π»Π»Π΅Π»ΡŒΠ½Ρ‹Ρ… прямых Π½Π°Π·Ρ‹Π²Π°ΡŽΡ‚ΡΡ ΠΊΠΎΠ»Π»ΠΈΠ½Π΅Π°Ρ€Π½Ρ‹ΠΌΠΈ: Π²Π΅ΠΊΡ‚ΠΎΡ€Π°Β ,Β  ΠΈΒ  — ΠΊΠΎΠ»Π»ΠΈΠ½Π΅Π°Ρ€Π½Ρ‹.

ΠŸΡ€ΠΎΠΈΠ·Π²Π΅Π΄Π΅Π½ΠΈΠ΅ΠΌ Π²Π΅ΠΊΡ‚ΠΎΡ€Π°Β  Π½Π° число  называСтся Π²Π΅ΠΊΡ‚ΠΎΡ€, сонаправлСнный Π²Π΅ΠΊΡ‚ΠΎΡ€ΡƒΒ , Ссли Β , ΠΈ Π½Π°ΠΏΡ€Π°Π²Π»Π΅Π½Π½Ρ‹ΠΉ Π² ΠΏΡ€ΠΎΡ‚ΠΈΠ²ΠΎΠΏΠΎΠ»ΠΎΠΆΠ½ΡƒΡŽ сторону, Ссли  , ΠΈ Π΄Π»ΠΈΠ½Π° ΠΊΠΎΡ‚ΠΎΡ€ΠΎΠ³ΠΎ Ρ€Π°Π²Π½Π° Π΄Π»ΠΈΠ½Π΅ Π²Π΅ΠΊΡ‚ΠΎΡ€Π°Β Β , ΡƒΠΌΠ½ΠΎΠΆΠ΅Π½Π½ΠΎΠΉ Π½Π°Β :

=k:

Π§Ρ‚ΠΎΠ±Ρ‹ ΡΠ»ΠΎΠΆΠΈΡ‚ΡŒ Β Π΄Π²Π° Π²Π΅ΠΊΡ‚ΠΎΡ€Π°Β  ΠΈ , Π½ΡƒΠΆΠ½ΠΎ Π½Π°Ρ‡Π°Π»ΠΎ Π²Π΅ΠΊΡ‚ΠΎΡ€Π° Β  ΡΠΎΠ΅Π΄ΠΈΠ½ΠΈΡ‚ΡŒ с ΠΊΠΎΠ½Ρ†ΠΎΠΌ Π²Π΅ΠΊΡ‚ΠΎΡ€Π°Β . Π’Π΅ΠΊΡ‚ΠΎΡ€ суммы соСдиняСт Π½Π°Ρ‡Π°Π»ΠΎ Π²Π΅ΠΊΡ‚ΠΎΡ€Π°Β  с ΠΊΠΎΠ½Ρ†ΠΎΠΌ Π²Π΅ΠΊΡ‚ΠΎΡ€Π°Β :

Π­Ρ‚ΠΎ ΠΏΡ€Π°Π²ΠΈΠ»ΠΎ слоТСния Π²Π΅ΠΊΡ‚ΠΎΡ€ΠΎΠ² называСтся ΠΏΡ€Π°Π²ΠΈΠ»ΠΎΠΌ Ρ‚Ρ€Π΅ΡƒΠ³ΠΎΠ»ΡŒΠ½ΠΈΠΊΠ°.

Π§Ρ‚ΠΎΠ±Ρ‹ ΡΠ»ΠΎΠΆΠΈΡ‚ΡŒ Π΄Π²Π° Π²Π΅ΠΊΡ‚ΠΎΡ€Π° ΠΏΠΎ ΠΏΡ€Π°Π²ΠΈΠ»Ρƒ ΠΏΠ°Ρ€Π°Π»Π»Π΅Π»ΠΎΠ³Ρ€Π°ΠΌΠΌΠ°, Π½ΡƒΠΆΠ½ΠΎ ΠΎΡ‚Π»ΠΎΠΆΠΈΡ‚ΡŒ Π²Π΅ΠΊΡ‚ΠΎΡ€Π° ΠΎΡ‚ ΠΎΠ΄Π½ΠΎΠΉ Ρ‚ΠΎΡ‡ΠΊΠΈ ΠΈ Π΄ΠΎΡΡ‚Ρ€ΠΎΠΈΡ‚ΡŒ Π΄ΠΎ ΠΏΠ°Ρ€Π°Π»Π»Π΅Π»ΠΎΠ³Ρ€Π°ΠΌΠΌΠ°. Π’Π΅ΠΊΡ‚ΠΎΡ€ суммы соСдиняСт Ρ‚ΠΎΡ‡ΠΊΡƒ Β Π½Π°Ρ‡Π°Π»Π° Π²Π΅ΠΊΡ‚ΠΎΡ€ΠΎΠ² с ΠΏΡ€ΠΎΡ‚ΠΈΠ²ΠΎΠΏΠΎΠ»ΠΎΠΆΠ½Ρ‹ΠΌ ΡƒΠ³Π»ΠΎΠΌ ΠΏΠ°Ρ€Π°Π»Π»Π΅Π»ΠΎΠ³Ρ€Π°ΠΌΠΌΠ°:

Π Π°Π·Π½ΠΎΡΡ‚ΡŒ Π΄Π²ΡƒΡ… Π²Π΅ΠΊΡ‚ΠΎΡ€ΠΎΠ² опрСдСляСтся Ρ‡Π΅Ρ€Π΅Π· сумму: Ρ€Π°Π·Π½ΠΎΡΡ‚ΡŒΡŽ Π²Π΅ΠΊΡ‚ΠΎΡ€ΠΎΠ²Β  ΠΈ называСтся Ρ‚Π°ΠΊΠΎΠΉ Π²Π΅ΠΊΡ‚ΠΎΡ€Β , ΠΊΠΎΡ‚ΠΎΡ€Ρ‹ΠΉ Π² суммС с Π²Π΅ΠΊΡ‚ΠΎΡ€ΠΎΠΌΒ  даст Π²Π΅ΠΊΡ‚ΠΎΡ€Β :

: Β  Β  Β  Β 

ΠžΡ‚ΡΡŽΠ΄Π° Π²Ρ‹Ρ‚Π΅ΠΊΠ°Π΅Ρ‚ ΠΏΡ€Π°Π²ΠΈΠ»ΠΎ нахоТдСния разности Π΄Π²ΡƒΡ… Π²Π΅ΠΊΡ‚ΠΎΡ€ΠΎΠ²: Ρ‡Ρ‚ΠΎΠ±Ρ‹ ΠΈΠ· Π²Π΅ΠΊΡ‚ΠΎΡ€Π°Β  Β Π²Ρ‹Ρ‡Π΅ΡΡ‚ΡŒ Π²Π΅ΠΊΡ‚ΠΎΡ€Β , Π½ΡƒΠΆΠ½ΠΎ ΠΎΡ‚Π»ΠΎΠΆΠΈΡ‚ΡŒ эти Π²Π΅ΠΊΡ‚ΠΎΡ€Π° ΠΎΡ‚ ΠΎΠ΄Π½ΠΎΠΉ Ρ‚ΠΎΡ‡ΠΊΠΈ. Π’Π΅ΠΊΡ‚ΠΎΡ€ разности соСдиняСт ΠΊΠΎΠ½Π΅Ρ† Π²Π΅ΠΊΡ‚ΠΎΡ€Π°Β  с ΠΊΠΎΠ½Ρ†ΠΎΠΌ Π²Π΅ΠΊΡ‚ΠΎΡ€Π°Β ( Ρ‚ΠΎ Π΅ΡΡ‚ΡŒ ΠΊΠΎΠ½Π΅Ρ† Π²Ρ‹Ρ‡ΠΈΡ‚Π°Π΅ΠΌΠΎΠ³ΠΎ с ΠΊΠΎΠ½Ρ†ΠΎΠΌ ΡƒΠΌΠ΅Π½ΡŒΡˆΠ°Π΅ΠΌΠΎΠ³ΠΎ):

Π§Ρ‚ΠΎΠ±Ρ‹ Π½Π°ΠΉΡ‚ΠΈ ΡƒΠ³ΠΎΠ» ΠΌΠ΅ΠΆΠ΄Ρƒ Π²Π΅ΠΊΡ‚ΠΎΡ€ΠΎΠΌΒ  ΠΈ Π²Π΅ΠΊΡ‚ΠΎΡ€ΠΎΠΌΒ , Π½ΡƒΠΆΠ½ΠΎ ΠΎΡ‚Π»ΠΎΠΆΠΈΡ‚ΡŒ эти Π²Π΅ΠΊΡ‚ΠΎΡ€Π° ΠΎΡ‚ ΠΎΠ΄Π½ΠΎΠΉ Ρ‚ΠΎΡ‡ΠΊΠΈ. Π£Π³ΠΎΠ», ΠΎΠ±Ρ€Π°Π·ΠΎΠ²Π°Π½Π½Ρ‹ΠΉ Π»ΡƒΡ‡Π°ΠΌΠΈ, Π½Π° ΠΊΠΎΡ‚ΠΎΡ€Ρ‹Ρ… Π»Π΅ΠΆΠ°Ρ‚ Π²Π΅ΠΊΡ‚ΠΎΡ€Π°, называСтся ΡƒΠ³Π»ΠΎΠΌ ΠΌΠ΅ΠΆΠ΄Ρƒ Π²Π΅ΠΊΡ‚ΠΎΡ€Π°ΠΌΠΈ:Β 

Бкалярным ΠΏΡ€ΠΎΠΈΠ·Π²Π΅Π΄Π΅Π½ΠΈΠ΅ΠΌ Π΄Π²ΡƒΡ… Π²Π΅ΠΊΡ‚ΠΎΡ€ΠΎΠ² называСтся число, Ρ€Π°Π²Π½ΠΎΠ΅ ΠΏΡ€ΠΎΠΈΠ·Π²Π΅Π΄Π΅Π½ΠΈΡŽ Π΄Π»ΠΈΠ½ этих Π²Π΅ΠΊΡ‚ΠΎΡ€ΠΎΠ² Π½Π° косинус ΡƒΠ³Π»Π° ΠΌΠ΅ΠΆΠ΄Ρƒ Π½ΠΈΠΌΠΈ:

ΠŸΡ€Π΅Π΄Π»Π°Π³Π°ΡŽ Π²Π°ΠΌ Ρ€Π΅ΡˆΠΈΡ‚ΡŒ Π·Π°Π΄Π°Ρ‡ΠΈ Β ΠΈΠ· ΠžΡ‚ΠΊΡ€Ρ‹Ρ‚ΠΎΠ³ΠΎ Π±Π°Π½ΠΊΠ° Π·Π°Π΄Π°Π½ΠΈΠΉ для  подготовки ΠΊ Π•Π“Π­ Β ΠΏΠΎ ΠΌΠ°Ρ‚Π΅ΠΌΠ°Ρ‚ΠΈΠΊΠ΅, Π° Π·Π°Ρ‚Π΅ΠΌ ΡΠ²Π΅Ρ€ΠΈΡ‚ΡŒ свС Ρ€Π΅ΡˆΠ΅Π½ΠΈΠ΅ с Π’Π˜Π”Π•ΠžΠ£Π ΠžΠšΠΠœΠ˜:

1. Π—Π°Π΄Π°Π½ΠΈΠ΅ 4 (β„– 27709)

Π”Π²Π΅ стороны ΠΏΡ€ΡΠΌΠΎΡƒΠ³ΠΎΠ»ΡŒΠ½ΠΈΠΊΠ°Β ABCDΒ Ρ€Π°Π²Π½Ρ‹ 6 ΠΈ 8. НайдитС Π΄Π»ΠΈΠ½Ρƒ разности Π²Π΅ΠΊΡ‚ΠΎΡ€ΠΎΠ²Β Β  ΠΈ .

2. Π—Π°Π΄Π°Π½ΠΈΠ΅ 4 (β„– 27710)

Π”Π²Π΅ стороны ΠΏΡ€ΡΠΌΠΎΡƒΠ³ΠΎΠ»ΡŒΠ½ΠΈΠΊΠ°Β ABCDΒ Ρ€Π°Π²Π½Ρ‹ 6 ΠΈ 8. НайдитС скалярноС ΠΏΡ€ΠΎΠΈΠ·Π²Π΅Π΄Π΅Π½ΠΈΠ΅ Π²Π΅ΠΊΡ‚ΠΎΡ€ΠΎΠ²Β Β  ΠΈ .Β Β (Ρ‡Π΅Ρ€Ρ‚Π΅ΠΆ ΠΈΠ· ΠΏΡ€Π΅Π΄Ρ‹Π΄ΡƒΡ‰Π΅ΠΉ Π·Π°Π΄Π°Ρ‡ΠΈ).

Β 

3.Β Π—Π°Π΄Π°Π½ΠΈΠ΅ 4 (β„– 27711)

Π”Π²Π΅ стороны ΠΏΡ€ΡΠΌΠΎΡƒΠ³ΠΎΠ»ΡŒΠ½ΠΈΠΊΠ°Β ABCDΒ Ρ€Π°Π²Π½Ρ‹ 6 ΠΈ 8. Π”ΠΈΠ°Π³ΠΎΠ½Π°Π»ΠΈ ΠΏΠ΅Ρ€Π΅ΡΠ΅ΠΊΠ°ΡŽΡ‚ΡΡ Π² Ρ‚ΠΎΡ‡ΠΊΠ΅Β O. НайдитС Π΄Π»ΠΈΠ½Ρƒ суммы Π²Π΅ΠΊΡ‚ΠΎΡ€ΠΎΠ²Β Β  ΠΈ .

4.Β Π—Π°Π΄Π°Π½ΠΈΠ΅ 4 (β„– 27712)

Π”Π²Π΅ стороны ΠΏΡ€ΡΠΌΠΎΡƒΠ³ΠΎΠ»ΡŒΠ½ΠΈΠΊΠ°Β ABCDΒ Ρ€Π°Π²Π½Ρ‹ 6 ΠΈ 8. Π”ΠΈΠ°Π³ΠΎΠ½Π°Π»ΠΈ ΠΏΠ΅Ρ€Π΅ΡΠ΅ΠΊΠ°ΡŽΡ‚ΡΡ Π² Ρ‚ΠΎΡ‡ΠΊΠ΅Β O. НайдитС Π΄Π»ΠΈΠ½Ρƒ разности Π²Π΅ΠΊΡ‚ΠΎΡ€ΠΎΠ²Β Β  ΠΈ . Β (Ρ‡Π΅Ρ€Ρ‚Π΅ΠΆ ΠΈΠ· ΠΏΡ€Π΅Π΄Ρ‹Π΄ΡƒΡ‰Π΅ΠΉ Π·Π°Π΄Π°Ρ‡ΠΈ).

Β 

Β 

5.Β Π—Π°Π΄Π°Π½ΠΈΠ΅ 4 (β„– 27713)

Π”ΠΈΠ°Π³ΠΎΠ½Π°Π»ΠΈ Ρ€ΠΎΠΌΠ±Π°Β ABCDΒ Ρ€Π°Π²Π½Ρ‹Β 12 ΠΈ 16. НайдитС Π΄Π»ΠΈΠ½Ρƒ Π²Π΅ΠΊΡ‚ΠΎΡ€Π°Β .

6.Β Π—Π°Π΄Π°Π½ΠΈΠ΅ 4 (β„– 27714)

Π”ΠΈΠ°Π³ΠΎΠ½Π°Π»ΠΈ Ρ€ΠΎΠΌΠ±Π°Β ABCDΒ Ρ€Π°Π²Π½Ρ‹Β 12 ΠΈ 16. НайдитС Π΄Π»ΠΈΠ½Ρƒ Π²Π΅ΠΊΡ‚ΠΎΡ€Π°Β Β +Β .

Β 

7.Π—Π°Π΄Π°Π½ΠΈΠ΅ 4 (β„– 27715)

Π”ΠΈΠ°Π³ΠΎΠ½Π°Π»ΠΈ Ρ€ΠΎΠΌΠ±Π°Β ABCDΒ Ρ€Π°Π²Π½Ρ‹Β 12 ΠΈ 16. НайдитС Π΄Π»ΠΈΠ½Ρƒ Π²Π΅ΠΊΡ‚ΠΎΡ€Π°Β —Β .(Ρ‡Π΅Ρ€Ρ‚Π΅ΠΆ ΠΈΠ· ΠΏΡ€Π΅Π΄Ρ‹Π΄ΡƒΡ‰Π΅ΠΉ Π·Π°Π΄Π°Ρ‡ΠΈ).

Β 

8.Π—Π°Π΄Π°Π½ΠΈΠ΅ 4 (β„– 27716)

Π”ΠΈΠ°Π³ΠΎΠ½Π°Π»ΠΈ Ρ€ΠΎΠΌΠ±Π°Β ABCDΒ Ρ€Π°Π²Π½Ρ‹Β 12 ΠΈ 16. НайдитС Π΄Π»ΠΈΠ½Ρƒ Π²Π΅ΠΊΡ‚ΠΎΡ€Π°Β —Β .

9.Β Π—Π°Π΄Π°Π½ΠΈΠ΅ 4 (β„– 27717)

Π”ΠΈΠ°Π³ΠΎΠ½Π°Π»ΠΈ Ρ€ΠΎΠΌΠ±Π°Β ABCDΒ ΠΏΠ΅Ρ€Π΅ΡΠ΅ΠΊΠ°ΡŽΡ‚ΡΡ Π² Ρ‚ΠΎΡ‡ΠΊΠ΅Β OΒ ΠΈ Ρ€Π°Π²Π½Ρ‹Β 12 ΠΈ 16. НайдитС Π΄Π»ΠΈΠ½Ρƒ Π²Π΅ΠΊΡ‚ΠΎΡ€Π°Β Β +Β .

10.Β Π—Π°Π΄Π°Π½ΠΈΠ΅ 4 (β„– 27718)

Π”ΠΈΠ°Π³ΠΎΠ½Π°Π»ΠΈ Ρ€ΠΎΠΌΠ±Π°Β ABCDΒ ΠΏΠ΅Ρ€Π΅ΡΠ΅ΠΊΠ°ΡŽΡ‚ΡΡ Π² Ρ‚ΠΎΡ‡ΠΊΠ΅Β OΒ ΠΈ Ρ€Π°Π²Π½Ρ‹Β 12 ΠΈ 16. НайдитС Π΄Π»ΠΈΠ½Ρƒ Π²Π΅ΠΊΡ‚ΠΎΡ€Π°Β  — .(Ρ‡Π΅Ρ€Ρ‚Π΅ΠΆ ΠΈΠ· ΠΏΡ€Π΅Π΄Ρ‹Π΄ΡƒΡ‰Π΅ΠΉ Π·Π°Π΄Π°Ρ‡ΠΈ).

Β 

11.Π—Π°Π΄Π°Π½ΠΈΠ΅ 4 (β„– 27719)

Π”ΠΈΠ°Π³ΠΎΠ½Π°Π»ΠΈ Ρ€ΠΎΠΌΠ±Π°Β ABCDΒ ΠΏΠ΅Ρ€Π΅ΡΠ΅ΠΊΠ°ΡŽΡ‚ΡΡ Π² Ρ‚ΠΎΡ‡ΠΊΠ΅Β OΒ ΠΈ Ρ€Π°Π²Π½Ρ‹Β 12 ΠΈ 16. НайдитС скалярноС ΠΏΡ€ΠΎΠΈΠ·Π²Π΅Π΄Π΅Π½ΠΈΠ΅ Π²Π΅ΠΊΡ‚ΠΎΡ€ΠΎΠ²Β Β  ΠΈ .(Ρ‡Π΅Ρ€Ρ‚Π΅ΠΆ ΠΈΠ· ΠΏΡ€Π΅Π΄Ρ‹Π΄ΡƒΡ‰Π΅ΠΉ Π·Π°Π΄Π°Ρ‡ΠΈ).

Β 

Β 

12.Β Π—Π°Π΄Π°Π½ΠΈΠ΅ 4 (β„– 27720)

Π‘Ρ‚ΠΎΡ€ΠΎΠ½Ρ‹ ΠΏΡ€Π°Π²ΠΈΠ»ΡŒΠ½ΠΎΠ³ΠΎ Ρ‚Ρ€Π΅ΡƒΠ³ΠΎΠ»ΡŒΠ½ΠΈΠΊΠ°Β ABCΒ Ρ€Π°Π²Π½Ρ‹Β   НайдитС Π΄Π»ΠΈΠ½Ρƒ Π²Π΅ΠΊΡ‚ΠΎΡ€Π°Β Β  +.

13.Β Π—Π°Π΄Π°Π½ΠΈΠ΅ 4 (β„– 27721)

Π‘Ρ‚ΠΎΡ€ΠΎΠ½Ρ‹ ΠΏΡ€Π°Π²ΠΈΠ»ΡŒΠ½ΠΎΠ³ΠΎ Ρ‚Ρ€Π΅ΡƒΠ³ΠΎΠ»ΡŒΠ½ΠΈΠΊΠ°Β ABCΒ Ρ€Π°Π²Π½Ρ‹ 3. НайдитС Π΄Π»ΠΈΠ½Ρƒ Π²Π΅ΠΊΡ‚ΠΎΡ€Π°Β  -.(Ρ‡Π΅Ρ€Ρ‚Π΅ΠΆ ΠΈΠ· ΠΏΡ€Π΅Π΄Ρ‹Π΄ΡƒΡ‰Π΅ΠΉ Π·Π°Π΄Π°Ρ‡ΠΈ).

Β 

14.Β Π—Π°Π΄Π°Π½ΠΈΠ΅ 4 (β„– 27722)

Π‘Ρ‚ΠΎΡ€ΠΎΠ½Ρ‹ ΠΏΡ€Π°Π²ΠΈΠ»ΡŒΠ½ΠΎΠ³ΠΎ Ρ‚Ρ€Π΅ΡƒΠ³ΠΎΠ»ΡŒΠ½ΠΈΠΊΠ°Β ABCΒ Ρ€Π°Π²Π½Ρ‹ 3. НайдитС скалярноС ΠΏΡ€ΠΎΠΈΠ·Π²Π΅Π΄Π΅Π½ΠΈΠ΅ Π²Π΅ΠΊΡ‚ΠΎΡ€ΠΎΠ²Β  ΠΈ .Β (Ρ‡Π΅Ρ€Ρ‚Π΅ΠΆ ΠΈΠ· ΠΏΡ€Π΅Π΄Ρ‹Π΄ΡƒΡ‰Π΅ΠΉ Π·Π°Π΄Π°Ρ‡ΠΈ).

Β 

Β 

ВСроятно, Π’Π°Ρˆ Π±Ρ€Π°ΡƒΠ·Π΅Ρ€ Π½Π΅ поддСрТиваСтся. Π§Ρ‚ΠΎΠ±Ρ‹ ΠΈΡΠΏΠΎΠ»ΡŒΠ·ΠΎΠ²Π°Ρ‚ΡŒ Ρ‚Ρ€Π΅Π½Π°ΠΆΡ‘Ρ€ «Π§Π°Ρ Π•Π“Π­», ΠΏΠΎΠΏΡ€ΠΎΠ±ΡƒΠΉΡ‚Π΅ ΡΠΊΠ°Ρ‡Π°Ρ‚ΡŒ
Firefox

Β 

И.Π’. ЀСльдман, Ρ€Π΅ΠΏΠ΅Ρ‚ΠΈΡ‚ΠΎΡ€ ΠΏΠΎ ΠΌΠ°Ρ‚Π΅ΠΌΠ°Ρ‚ΠΈΠΊΠ΅.

Β 

Π”Π»ΠΈΠ½Π° Π²Π΅ΠΊΡ‚ΠΎΡ€Π° – ΠΎΠΏΡ€Π΅Π΄Π΅Π»Π΅Π½ΠΈΠ΅, Ρ„ΠΎΡ€ΠΌΡƒΠ»Ρ‹ ΠΈ ΠΏΡ€ΠΈΠΌΠ΅Ρ€Ρ‹

Π”Π»ΠΈΠ½Π° Π²Π΅ΠΊΡ‚ΠΎΡ€Π° позволяСт Π½Π°ΠΌ ΠΏΠΎΠ½ΡΡ‚ΡŒ, насколько Π²Π΅Π»ΠΈΠΊ Π²Π΅ΠΊΡ‚ΠΎΡ€ с Ρ‚ΠΎΡ‡ΠΊΠΈ зрСния Ρ€Π°Π·ΠΌΠ΅Ρ€ΠΎΠ². Π­Ρ‚ΠΎ Ρ‚Π°ΠΊΠΆΠ΅ ΠΏΠΎΠΌΠΎΠ³Π°Π΅Ρ‚ Π½Π°ΠΌ ΠΏΠΎΠ½ΠΈΠΌΠ°Ρ‚ΡŒ Π²Π΅ΠΊΡ‚ΠΎΡ€Π½Ρ‹Π΅ Π²Π΅Π»ΠΈΡ‡ΠΈΠ½Ρ‹, Ρ‚Π°ΠΊΠΈΠ΅ ΠΊΠ°ΠΊ ΠΏΠ΅Ρ€Π΅ΠΌΠ΅Ρ‰Π΅Π½ΠΈΠ΅, ΡΠΊΠΎΡ€ΠΎΡΡ‚ΡŒ, сила ΠΈ Ρ‚. Π΄. ПониманиС Ρ„ΠΎΡ€ΠΌΡƒΠ»Ρ‹ для вычислСния Π΄Π»ΠΈΠ½Ρ‹ Π²Π΅ΠΊΡ‚ΠΎΡ€Π° ΠΏΠΎΠΌΠΎΠΆΠ΅Ρ‚ Π½Π°ΠΌ ΡƒΡΡ‚Π°Π½ΠΎΠ²ΠΈΡ‚ΡŒ Ρ„ΠΎΡ€ΠΌΡƒΠ»Ρƒ для Π΄Π»ΠΈΠ½Ρ‹ Π΄ΡƒΠ³ΠΈ Π²Π΅ΠΊΡ‚ΠΎΡ€Π½ΠΎΠΉ Ρ„ΡƒΠ½ΠΊΡ†ΠΈΠΈ.

Π”Π»ΠΈΠ½Π° Π²Π΅ΠΊΡ‚ΠΎΡ€Π° (извСстная ΠΊΠ°ΠΊ Π²Π΅Π»ΠΈΡ‡ΠΈΠ½Π°) позволяСт Π½Π°ΠΌ количСствСнно ΠΎΡ†Π΅Π½ΠΈΡ‚ΡŒ свойство Π΄Π°Π½Π½ΠΎΠ³ΠΎ Π²Π΅ΠΊΡ‚ΠΎΡ€Π°. Π§Ρ‚ΠΎΠ±Ρ‹ Π½Π°ΠΉΡ‚ΠΈ Π΄Π»ΠΈΠ½Ρƒ Π²Π΅ΠΊΡ‚ΠΎΡ€Π°, просто Π΄ΠΎΠ±Π°Π²ΡŒΡ‚Π΅ ΠΊΠ²Π°Π΄Ρ€Π°Ρ‚ Π΅Π³ΠΎ ΠΊΠΎΠΌΠΏΠΎΠ½Π΅Π½Ρ‚ΠΎΠ², Π·Π°Ρ‚Π΅ΠΌ ΠΈΠ·Π²Π»Π΅ΠΊΠΈΡ‚Π΅ ΠΊΠ²Π°Π΄Ρ€Π°Ρ‚Π½Ρ‹ΠΉ ΠΊΠΎΡ€Π΅Π½ΡŒ ΠΈΠ· Ρ€Π΅Π·ΡƒΠ»ΡŒΡ‚Π°Ρ‚Π° .

Π’ этой ΡΡ‚Π°Ρ‚ΡŒΠ΅ ΠΌΡ‹ Ρ€Π°ΡΡˆΠΈΡ€ΠΈΠΌ нашС ΠΏΠΎΠ½ΠΈΠΌΠ°Π½ΠΈΠ΅ Π²Π΅Π»ΠΈΡ‡ΠΈΠ½ Π΄ΠΎ Ρ‚Ρ€Π΅Ρ…ΠΌΠ΅Ρ€Π½Ρ‹Ρ… Π²Π΅ΠΊΡ‚ΠΎΡ€ΠΎΠ². ΠœΡ‹ Ρ‚Π°ΠΊΠΆΠ΅ рассмотрим Ρ„ΠΎΡ€ΠΌΡƒΠ»Ρƒ Π΄Π»ΠΈΠ½Ρ‹ Π΄ΡƒΠ³ΠΈ Π²Π΅ΠΊΡ‚ΠΎΡ€Π½ΠΎΠΉ Ρ„ΡƒΠ½ΠΊΡ†ΠΈΠΈ. К ΠΊΠΎΠ½Ρ†Ρƒ нашСго обсуТдСния наша Ρ†Π΅Π»ΡŒ состоит Π² Ρ‚ΠΎΠΌ, Ρ‡Ρ‚ΠΎΠ±Ρ‹ Π²Ρ‹ ΡƒΠ²Π΅Ρ€Π΅Π½Π½ΠΎ Ρ€Π°Π±ΠΎΡ‚Π°Π»ΠΈ Π½Π°Π΄ Ρ€Π°Π·Π»ΠΈΡ‡Π½Ρ‹ΠΌΠΈ Π·Π°Π΄Π°Ρ‡Π°ΠΌΠΈ, связанными с Π²Π΅ΠΊΡ‚ΠΎΡ€Π°ΠΌΠΈ ΠΈ Π΄Π»ΠΈΠ½Π°ΠΌΠΈ Π²Π΅ΠΊΡ‚ΠΎΡ€Π½Ρ‹Ρ… Ρ„ΡƒΠ½ΠΊΡ†ΠΈΠΉ.

Какова Π΄Π»ΠΈΠ½Π° Π²Π΅ΠΊΡ‚ΠΎΡ€Π°?

Π”Π»ΠΈΠ½Π° Π²Π΅ΠΊΡ‚ΠΎΡ€Π° прСдставляСт собой расстояниС Π²Π΅ΠΊΡ‚ΠΎΡ€Π° Π² стандартной ΠΏΠΎΠ·ΠΈΡ†ΠΈΠΈ ΠΎΡ‚ Π½Π°Ρ‡Π°Π»Π° ΠΊΠΎΠΎΡ€Π΄ΠΈΠ½Π°Ρ‚. Π’ нашСм ΠΏΡ€Π΅Π΄Ρ‹Π΄ΡƒΡ‰Π΅ΠΌ обсуТдСнии Π²Π΅ΠΊΡ‚ΠΎΡ€Π½Ρ‹Ρ… свойств ΠΌΡ‹ ΡƒΠ·Π½Π°Π»ΠΈ, Ρ‡Ρ‚ΠΎ Π΄Π»ΠΈΠ½Π° Π²Π΅ΠΊΡ‚ΠΎΡ€Π° Ρ‚Π°ΠΊΠΆΠ΅ извСстна ΠΊΠ°ΠΊ 9-кратная Π΄Π»ΠΈΠ½Π° Π²Π΅ΠΊΡ‚ΠΎΡ€Π°. 2}\ΠΊΠΎΠ½Π΅Ρ†{Π²Ρ‹Ρ€ΠΎΠ²Π½Π΅Π½ΠΎ}

ЀактичСски, ΠΌΡ‹ ΠΌΠΎΠΆΠ΅ΠΌ Ρ€Π°ΡΡˆΠΈΡ€ΠΈΡ‚ΡŒ нашС ΠΏΠΎΠ½ΠΈΠΌΠ°Π½ΠΈΠ΅ Ρ‚Ρ€Π΅Ρ…ΠΊΠΎΠΎΡ€Π΄ΠΈΠ½Π°Ρ‚Π½Ρ‹Ρ… систСм ΠΈ Π²Π΅ΠΊΡ‚ΠΎΡ€ΠΎΠ², Ρ‡Ρ‚ΠΎΠ±Ρ‹ Π΄ΠΎΠΊΠ°Π·Π°Ρ‚ΡŒ Ρ„ΠΎΡ€ΠΌΡƒΠ»Ρƒ для Π΄Π»ΠΈΠ½Ρ‹ Π²Π΅ΠΊΡ‚ΠΎΡ€Π° Π² пространствС.

Π”ΠΎΠΊΠ°Π·Π°Ρ‚Π΅Π»ΡŒΡΡ‚Π²ΠΎ Ρ„ΠΎΡ€ΠΌΡƒΠ»Ρ‹ Π΄Π»ΠΈΠ½Ρ‹ Π²Π΅ΠΊΡ‚ΠΎΡ€Π° Π² 3D

ΠŸΡ€Π΅Π΄ΠΏΠΎΠ»ΠΎΠΆΠΈΠΌ, Ρ‡Ρ‚ΠΎ Ρƒ нас Π΅ΡΡ‚ΡŒ Π²Π΅ΠΊΡ‚ΠΎΡ€, $\textbf{u} = x_o \textbf{i} + y_o \textbf{j} +z_o \textbf{k}$ , ΠΌΡ‹ ΠΌΠΎΠΆΠ΅ΠΌ ΠΏΠ΅Ρ€Π΅ΠΏΠΈΡΠ°Ρ‚ΡŒ Π²Π΅ΠΊΡ‚ΠΎΡ€ ΠΊΠ°ΠΊ сумму Π΄Π²ΡƒΡ… Π²Π΅ΠΊΡ‚ΠΎΡ€ΠΎΠ². Π‘Π»Π΅Π΄ΠΎΠ²Π°Ρ‚Π΅Π»ΡŒΠ½ΠΎ, ΠΌΡ‹ ΠΈΠΌΠ΅Π΅ΠΌ ΡΠ»Π΅Π΄ΡƒΡŽΡ‰Π΅Π΅:

\begin{aligned}\textbf{v}_1 &= \\ \textbf{v}_2 &= <0 , 0, z_o>\\\textbf{u} &= \\&= +<0 ,0, z_o>\\&=\textbf{v}_1+ \textbf{v}_2\end{Π²Ρ‹Ρ€ΠΎΠ²Π½Π΅Π½ΠΎ} 92}\end{aligned}

Π­Ρ‚ΠΎ ΠΎΠ·Π½Π°Ρ‡Π°Π΅Ρ‚, Ρ‡Ρ‚ΠΎ для вычислСния Π΄Π»ΠΈΠ½Ρ‹ Π²Π΅ΠΊΡ‚ΠΎΡ€Π° Π² Ρ‚Ρ€Π΅Ρ… измСрСниях всС, Ρ‡Ρ‚ΠΎ Π½Π°ΠΌ Π½ΡƒΠΆΠ½ΠΎ ΡΠ΄Π΅Π»Π°Ρ‚ΡŒ, это ΡΠ»ΠΎΠΆΠΈΡ‚ΡŒ ΠΊΠ²Π°Π΄Ρ€Π°Ρ‚Ρ‹ Π΅Π³ΠΎ ΠΊΠΎΠΌΠΏΠΎΠ½Π΅Π½Ρ‚ΠΎΠ², Π° Π·Π°Ρ‚Π΅ΠΌ ΠΈΠ·Π²Π»Π΅Ρ‡ΡŒ ΠΊΠ²Π°Π΄Ρ€Π°Ρ‚Π½Ρ‹ΠΉ ΠΊΠΎΡ€Π΅Π½ΡŒ ΠΈΠ· Ρ€Π΅Π·ΡƒΠ»ΡŒΡ‚Π°Ρ‚Π°.

Π”Π»ΠΈΠ½Π° Π΄ΡƒΠ³ΠΈ Π²Π΅ΠΊΡ‚ΠΎΡ€Π½ΠΎΠΉ Ρ„ΡƒΠ½ΠΊΡ†ΠΈΠΈ

ΠœΡ‹ ΠΌΠΎΠΆΠ΅ΠΌ Ρ€Π°ΡΠΏΡ€ΠΎΡΡ‚Ρ€Π°Π½ΠΈΡ‚ΡŒ это понятиС Π΄Π»ΠΈΠ½Ρ‹ Π½Π° Π²Π΅ΠΊΡ‚ΠΎΡ€Π½Ρ‹Π΅ Ρ„ΡƒΠ½ΠΊΡ†ΠΈΠΈ β€” Π½Π° этот Ρ€Π°Π· ΠΌΡ‹ аппроксимируСм расстояниС Π²Π΅ΠΊΡ‚ΠΎΡ€Π½ΠΎΠΉ Ρ„ΡƒΠ½ΠΊΡ†ΠΈΠΈ Π½Π° ΠΈΠ½Ρ‚Π΅Ρ€Π²Π°Π»Π΅ $t$. Π”Π»ΠΈΠ½Ρƒ Π²Π΅ΠΊΡ‚ΠΎΡ€-Ρ„ΡƒΠ½ΠΊΡ†ΠΈΠΈ $\textbf{r}(t)$ Π² ΠΈΠ½Ρ‚Π΅Ρ€Π²Π°Π»Π΅ $[a, b]$ ΠΌΠΎΠΆΠ½ΠΎ Ρ€Π°ΡΡΡ‡ΠΈΡ‚Π°Ρ‚ΡŒ ΠΏΠΎ ΠΏΡ€ΠΈΠ²Π΅Π΄Π΅Π½Π½ΠΎΠΉ Π½ΠΈΠΆΠ΅ Ρ„ΠΎΡ€ΠΌΡƒΠ»Π΅. 9{b} |\textbf{r}\prime(t)| \phantom{x} dt\end{aligned}

ΠœΡ‹ рассмотрСли всС основныС опрСдСлСния Π΄Π»ΠΈΠ½ Π²Π΅ΠΊΡ‚ΠΎΡ€ΠΎΠ² ΠΈ Π΄Π»ΠΈΠ½ Π²Π΅ΠΊΡ‚ΠΎΡ€Π½Ρ‹Ρ… Ρ„ΡƒΠ½ΠΊΡ†ΠΈΠΉ, Ρ‚Π΅ΠΏΠ΅Ρ€ΡŒ ΠΏΡ€ΠΈΡˆΠ»ΠΎ врСмя ΠΏΡ€ΠΈΠΌΠ΅Π½ΠΈΡ‚ΡŒ ΠΈΡ… для вычислСния ΠΈΡ… Π·Π½Π°Ρ‡Π΅Π½ΠΈΠΉ.

Как Π²Ρ‹Ρ‡ΠΈΡΠ»ΠΈΡ‚ΡŒ Π΄Π»ΠΈΠ½Ρƒ Π²Π΅ΠΊΡ‚ΠΎΡ€Π° ΠΈ Π²Π΅ΠΊΡ‚ΠΎΡ€Π½ΡƒΡŽ Ρ„ΡƒΠ½ΠΊΡ†ΠΈΡŽ?

ΠœΡ‹ ΠΌΠΎΠΆΠ΅ΠΌ Π²Ρ‹Ρ‡ΠΈΡΠ»ΠΈΡ‚ΡŒ Π΄Π»ΠΈΠ½Ρƒ Π²Π΅ΠΊΡ‚ΠΎΡ€Π°, ΠΏΡ€ΠΈΠΌΠ΅Π½ΠΈΠ² Ρ„ΠΎΡ€ΠΌΡƒΠ»Ρƒ для Π²Π΅Π»ΠΈΡ‡ΠΈΠ½Ρ‹ . Π’ΠΎΡ‚ Ρ€Π°Π·Π±ΠΈΠ²ΠΊΠ° шагов для вычислСния Π΄Π»ΠΈΠ½Ρ‹ Π²Π΅ΠΊΡ‚ΠΎΡ€Π°:

    92}\\&=\sqrt{4 + 16 + 1}\\&= \sqrt{21}\end{aligned}

    Π‘Π»Π΅Π΄ΠΎΠ²Π°Ρ‚Π΅Π»ΡŒΠ½ΠΎ, Π΄Π»ΠΈΠ½Π° Π²Π΅ΠΊΡ‚ΠΎΡ€Π° $\textbf{u}$ Ρ€Π°Π²Π½Π° $\sqrt{21}$ Π΅Π΄ΠΈΠ½ΠΈΡ† ΠΈΠ»ΠΈ ΠΏΡ€ΠΈΠΌΠ΅Ρ€Π½ΠΎ Ρ€Π°Π²Π½ΠΎ $4,58$ Π΅Π΄ΠΈΠ½ΠΈΡ†.

    Как ΠΌΡ‹ ΠΏΠΎΠΊΠ°Π·Π°Π»ΠΈ Π² ΠΏΡ€Π΅Π΄Ρ‹Π΄ΡƒΡ‰Π΅ΠΌ обсуТдСнии, Π΄Π»ΠΈΠ½Π° Π΄ΡƒΠ³ΠΈ Π²Π΅ΠΊΡ‚ΠΎΡ€Π½ΠΎΠΉ Ρ„ΡƒΠ½ΠΊΡ†ΠΈΠΈ зависит ΠΎΡ‚ ΠΊΠ°ΡΠ°Ρ‚Π΅Π»ΡŒΠ½ΠΎΠ³ΠΎ Π²Π΅ΠΊΡ‚ΠΎΡ€Π° . Π’ΠΎΡ‚ подсказка, которая ΠΏΠΎΠΌΠΎΠΆΠ΅Ρ‚ Π²Π°ΠΌ Π²Ρ‹Ρ‡ΠΈΡΠ»ΠΈΡ‚ΡŒ Π΄Π»ΠΈΠ½Ρƒ Π΄ΡƒΠ³ΠΈ Π²Π΅ΠΊΡ‚ΠΎΡ€Π½ΠΎΠΉ Ρ„ΡƒΠ½ΠΊΡ†ΠΈΠΈ:

    • ΠŸΠ΅Ρ€Π΅Ρ‡ΠΈΡΠ»ΠΈΡ‚Π΅ ΠΊΠΎΠΌΠΏΠΎΠ½Π΅Π½Ρ‚Ρ‹ Π²Π΅ΠΊΡ‚ΠΎΡ€Π° ΠΈ Π²ΠΎΠ·Π²Π΅Π΄ΠΈΡ‚Π΅ ΠΈΡ… ΠΊΠ²Π°Π΄Ρ€Π°Ρ‚Ρ‹. 9{b} |\textbf{r}\prime(t)| \phantom{x} dt$, Π³Π΄Π΅ $\textbf{r}\prime(t)$ прСдставляСт собой ΠΊΠ°ΡΠ°Ρ‚Π΅Π»ΡŒΠ½Ρ‹ΠΉ Π²Π΅ΠΊΡ‚ΠΎΡ€. 2} \\&= \sqrt{ 20}\end{Π²Ρ‹Ρ€ΠΎΠ²Π½Π΅Π½ΠΎ} 94\\&= 2\sqrt{5}( 4 -0)\\&= 8\sqrt{5}\end{aligned}

      Π­Ρ‚ΠΎ ΠΎΠ·Π½Π°Ρ‡Π°Π΅Ρ‚, Ρ‡Ρ‚ΠΎ Π΄Π»ΠΈΠ½Π° Π΄ΡƒΠ³ΠΈ $\textbf{r}(t)$ ΠΎΡ‚ $t=0$ Π΄ΠΎ $t=4$ Ρ€Π°Π²Π½ΠΎ $8\sqrt{5}$ Π΅Π΄ΠΈΠ½ΠΈΡ† ΠΈΠ»ΠΈ ΠΏΡ€ΠΈΠ±Π»ΠΈΠ·ΠΈΡ‚Π΅Π»ΡŒΠ½ΠΎ $17,89$ Π΅Π΄ΠΈΠ½ΠΈΡ†.

      Π­Ρ‚ΠΎ Π΄Π²Π° Π·Π°ΠΌΠ΅Ρ‡Π°Ρ‚Π΅Π»ΡŒΠ½Ρ‹Ρ… ΠΏΡ€ΠΈΠΌΠ΅Ρ€Π° Ρ‚ΠΎΠ³ΠΎ, ΠΊΠ°ΠΊ ΠΌΡ‹ ΠΌΠΎΠΆΠ΅ΠΌ ΠΏΡ€ΠΈΠΌΠ΅Π½ΡΡ‚ΡŒ Ρ„ΠΎΡ€ΠΌΡƒΠ»Ρ‹ для Π΄Π»ΠΈΠ½ Π²Π΅ΠΊΡ‚ΠΎΡ€ΠΎΠ² ΠΈ Π²Π΅ΠΊΡ‚ΠΎΡ€Π½Ρ‹Ρ… Ρ„ΡƒΠ½ΠΊΡ†ΠΈΠΉ. ΠœΡ‹ ΠΏΡ€ΠΈΠ³ΠΎΡ‚ΠΎΠ²ΠΈΠ»ΠΈ для вас Π΅Ρ‰Π΅ нСсколько Π·Π°Π΄Π°Ρ‡, Ρ‚Π°ΠΊ Ρ‡Ρ‚ΠΎ ΠΏΠ΅Ρ€Π΅Ρ…ΠΎΠ΄ΠΈΡ‚Π΅ ΠΊ ΡΠ»Π΅Π΄ΡƒΡŽΡ‰Π΅ΠΌΡƒ Ρ€Π°Π·Π΄Π΅Π»Ρƒ, ΠΊΠΎΠ³Π΄Π° Π±ΡƒΠ΄Π΅Ρ‚Π΅ Π³ΠΎΡ‚ΠΎΠ²Ρ‹!

      ΠŸΡ€ΠΈΠΌΠ΅Ρ€ 1

      Π’Π΅ΠΊΡ‚ΠΎΡ€ $\textbf{u}$ ΠΈΠΌΠ΅Π΅Ρ‚ Π½Π°Ρ‡Π°Π»ΡŒΠ½ΡƒΡŽ Ρ‚ΠΎΡ‡ΠΊΡƒ Π² $P(-2, 0, 1 )$ ΠΈ ΠΊΠΎΠ½Π΅Ρ‡Π½ΡƒΡŽ Ρ‚ΠΎΡ‡ΠΊΡƒ Π² $Q(4, -2, 3)$ . Какова Π΄Π»ΠΈΠ½Π° Π²Π΅ΠΊΡ‚ΠΎΡ€Π°? 92}\\&= \sqrt{36+ 4+ 4}\\&= \sqrt{44}\\&= 2\sqrt{11}\\&\ΠΏΡ€ΠΈΠ±Π»ΠΈΠ·ΠΈΡ‚Π΅Π»ΡŒΠ½ΠΎ 6,63 \end{aligned}

      Π­Ρ‚ΠΎ ΠΎΠ·Π½Π°Ρ‡Π°Π΅Ρ‚, Ρ‡Ρ‚ΠΎ Π²Π΅ΠΊΡ‚ΠΎΡ€ $\textbf{u}$ ΠΈΠΌΠ΅Π΅Ρ‚ Π΄Π»ΠΈΠ½Ρƒ $2\sqrt{11}$ Π΅Π΄ΠΈΠ½ΠΈΡ† ΠΈΠ»ΠΈ ΠΏΡ€ΠΈΠΌΠ΅Ρ€Π½ΠΎ $6,33$ Π΅Π΄ΠΈΠ½ΠΈΡ†.

      ΠŸΡ€ΠΈΠΌΠ΅Ρ€ 2

      Π’Ρ‹Ρ‡ΠΈΡΠ»ΠΈΡ‚ΡŒ Π΄Π»ΠΈΠ½Ρƒ Π΄ΡƒΠ³ΠΈ Π²Π΅ΠΊΡ‚ΠΎΡ€-Ρ„ΡƒΠ½ΠΊΡ†ΠΈΠΈ, $\textbf{r}(t) = \left<2\cos t, 2\sin t, 4t\right>$, Ссли $t$ находится Π²Π½ΡƒΡ‚Ρ€ΠΈ ΠΈΠ½Ρ‚Π΅Ρ€Π²Π°Π»Π°, $t \in [0, 2\pi]$.

      РСшСниС

      Π’Π΅ΠΏΠ΅Ρ€ΡŒ ΠΌΡ‹ ΠΈΡ‰Π΅ΠΌ Π΄Π»ΠΈΠ½Ρƒ Π΄ΡƒΠ³ΠΈ Π²Π΅ΠΊΡ‚ΠΎΡ€Π½ΠΎΠΉ Ρ„ΡƒΠ½ΠΊΡ†ΠΈΠΈ, поэтому Π²ΠΎΡΠΏΠΎΠ»ΡŒΠ·ΡƒΠ΅ΠΌΡΡ Ρ„ΠΎΡ€ΠΌΡƒΠ»ΠΎΠΉ, ΠΏΠΎΠΊΠ°Π·Π°Π½Π½ΠΎΠΉ Π½ΠΈΠΆΠ΅. 9{b} |\textbf{r}\prime(t)| \phantom{x}dt\end{aligned}

      Π‘Π½Π°Ρ‡Π°Π»Π° возьмСм ΠΏΡ€ΠΎΠΈΠ·Π²ΠΎΠ΄Π½ΡƒΡŽ ΠΊΠ°ΠΆΠ΄ΠΎΠΉ ΠΊΠΎΠΌΠΏΠΎΠ½Π΅Π½Ρ‚Ρ‹, Ρ‡Ρ‚ΠΎΠ±Ρ‹ Π½Π°ΠΉΡ‚ΠΈ $\textbf{r}\prime(t)$.

      \begin{Π²Ρ‹Ρ€ΠΎΠ²Π½Π΅Π½ΠΎ}x\prime(t)\end{Π²Ρ‹Ρ€ΠΎΠ²Π½Π΅Π½ΠΎ}

      \begin{align}x\prime(t) &= \dfrac{d}{dt}(2 \cos t)\\&= 2(-\sin t)\\&= -2\sin t \end {Π²Ρ‹Ρ€ΠΎΠ²Π½Π΅Π½ΠΎ}

      \begin{Π²Ρ‹Ρ€ΠΎΠ²Π½Π΅Π½ΠΎ}y \prime(t)\end{Π²Ρ‹Ρ€ΠΎΠ²Π½Π΅Π½ΠΎ}

      \begin{Π²Ρ‹Ρ€ΠΎΠ²Π½Π΅Π½ΠΎ}y\prime(t) &= \dfrac{d}{dt}(2 \sin t)\\&= 2(\cos t)\\&= 2\cos t\end{Π²Ρ‹Ρ€ΠΎΠ²Π½Π΅Π½ΠΎ }

      \begin{Π²Ρ‹Ρ€ΠΎΠ²Π½Π΅Π½ΠΎ}z\prime(t)\end{Π²Ρ‹Ρ€ΠΎΠ²Π½Π΅Π½ΠΎ}

      \begin{Π²Ρ‹Ρ€ΠΎΠ²Π½Π΅Π½ΠΎ}y\prime(t) &= \dfrac{d}{dt}(2 4t)\\&= 4(1)\\&= 4\end{Π²Ρ‹Ρ€ΠΎΠ²Π½Π΅Π½ΠΎ}

      \begin{Π²Ρ‹Ρ€ΠΎΠ²Π½Π΅Π½ΠΎ}\textbf{r}\prime(t) &= \left\\&= \left <-2\sin t, 2\cos t, 4\right>\end{Π²Ρ‹Ρ€ΠΎΠ²Π½Π΅Π½ΠΎ}

      Π’Π΅ΠΏΠ΅Ρ€ΡŒ Π½Π°ΠΉΠ΄Π΅ΠΌ Π²Π΅Π»ΠΈΡ‡ΠΈΠ½Ρƒ $\textbf{r}\prime(t)$, Π΄ΠΎΠ±Π°Π²ΠΈΠ² ΠΊΠ²Π°Π΄Ρ€Π°Ρ‚Ρ‹ ΠΊΠΎΠΌΠΏΠΎΠ½Π΅Π½Ρ‚ΠΎΠ² ΠΊΠ°ΡΠ°Ρ‚Π΅Π»ΡŒΠ½ΠΎΠ³ΠΎ Π²Π΅ΠΊΡ‚ΠΎΡ€Π°. Π—Π°ΠΏΠΈΡˆΠΈΡ‚Π΅ ΠΊΠ²Π°Π΄Ρ€Π°Ρ‚Π½Ρ‹ΠΉ ΠΊΠΎΡ€Π΅Π½ΡŒ ΠΈΠ· суммы, Ρ‡Ρ‚ΠΎΠ±Ρ‹ Π²Ρ‹Ρ€Π°Π·ΠΈΡ‚ΡŒ Π²Π΅Π»ΠΈΡ‡ΠΈΠ½Ρƒ Ρ‡Π΅Ρ€Π΅Π· $t$. 9{2\pi} \phantom{x}dt\\&= 2\sqrt{5}(2\pi – 0)\\&= 4\sqrt{5}\pi\\&\ΠΏΡ€ΠΈΠΌΠ΅Ρ€Π½ΠΎ 28,10\end{Π²Ρ‹Ρ€ΠΎΠ²Π½Π΅Π½ΠΎ }

      Π­Ρ‚ΠΎ ΠΎΠ·Π½Π°Ρ‡Π°Π΅Ρ‚, Ρ‡Ρ‚ΠΎ Π΄Π»ΠΈΠ½Π° Π΄ΡƒΠ³ΠΈ Π²Π΅ΠΊΡ‚ΠΎΡ€Π½ΠΎΠΉ Ρ„ΡƒΠ½ΠΊΡ†ΠΈΠΈ составляСт $4\sqrt{5}\pi$ ΠΈΠ»ΠΈ ΠΏΡ€ΠΈΠ±Π»ΠΈΠ·ΠΈΡ‚Π΅Π»ΡŒΠ½ΠΎ $28,10$ Π΅Π΄ΠΈΠ½ΠΈΡ†.

      ΠŸΡ€Π°ΠΊΡ‚ΠΈΡ‡Π΅ΡΠΊΠΈΠ΅ вопросы

      1. Π’Π΅ΠΊΡ‚ΠΎΡ€ $\textbf{u}$ ΠΈΠΌΠ΅Π΅Ρ‚ Π½Π°Ρ‡Π°Π»ΡŒΠ½ΡƒΡŽ Ρ‚ΠΎΡ‡ΠΊΡƒ Π² Ρ‚ΠΎΡ‡ΠΊΠ΅ $P(-4, 2, -2 )$ ΠΈ ΠΊΠΎΠ½Π΅Ρ† Π² Ρ‚ΠΎΡ‡ΠΊΠ΅ $Q(-1, 3, 1)$. Какова Π΄Π»ΠΈΠ½Π° Π²Π΅ΠΊΡ‚ΠΎΡ€Π°?

      2. Π’Ρ‹Ρ‡ΠΈΡΠ»ΠΈΡ‚ΡŒ Π΄Π»ΠΈΠ½Ρƒ Π΄ΡƒΠ³ΠΈ Π²Π΅ΠΊΡ‚ΠΎΡ€-Ρ„ΡƒΠ½ΠΊΡ†ΠΈΠΈ $\textbf{r}(t) = \left$, Ссли $t$ находится Π² ΠΏΡ€Π΅Π΄Π΅Π»Π°Ρ… ΠΈΠ½Ρ‚Π΅Ρ€Π²Π°Π», $t \in [0, 2\pi]$.

      ΠšΠ»ΡŽΡ‡ ΠΎΡ‚Π²Π΅Ρ‚Π°

      1. Π’Π΅ΠΊΡ‚ΠΎΡ€ ΠΈΠΌΠ΅Π΅Ρ‚ Π΄Π»ΠΈΠ½Ρƒ $\sqrt{19}$ Π΅Π΄ΠΈΠ½ΠΈΡ† ΠΈΠ»ΠΈ ΠΏΡ€ΠΈΠΌΠ΅Ρ€Π½ΠΎ $4,36$ Π΅Π΄ΠΈΠ½ΠΈΡ†.
      2. Π”Π»ΠΈΠ½Π° Π΄ΡƒΠ³ΠΈ ΠΏΡ€ΠΈΠΌΠ΅Ρ€Π½ΠΎ Ρ€Π°Π²Π½Π° $25,343$ Π΅Π΄ΠΈΠ½ΠΈΡ†.

      Π’Ρ€Π΅Ρ…ΠΌΠ΅Ρ€Π½Ρ‹Π΅ изобраТСния/матСматичСскиС Ρ‡Π΅Ρ€Ρ‚Π΅ΠΆΠΈ ΡΠΎΠ·Π΄Π°ΡŽΡ‚ΡΡ с ΠΏΠΎΠΌΠΎΡ‰ΡŒΡŽ GeoGebra.

      Π’Π΅ΠΊΡ‚ΠΎΡ€Ρ‹ Π² Π΄Π²ΡƒΡ…- ΠΈ Ρ‚Ρ€Π΅Ρ…ΠΌΠ΅Ρ€Π½Ρ‹Ρ… Π΄Π΅ΠΊΠ°Ρ€Ρ‚ΠΎΠ²Ρ‹Ρ… ΠΊΠΎΠΎΡ€Π΄ΠΈΠ½Π°Ρ‚Π°Ρ…

      Π’ΠΎ Π²Π²Π΅Π΄Π΅Π½ΠΈΠΈ ΠΊ Π²Π΅ΠΊΡ‚ΠΎΡ€Π°ΠΌ ΠΌΡ‹ обсуТдали Π²Π΅ΠΊΡ‚ΠΎΡ€Ρ‹ Π±Π΅Π· привязки ΠΊ ΠΊΠ°ΠΊΠΎΠΉ-Π»ΠΈΠ±ΠΎ систСмС ΠΊΠΎΠΎΡ€Π΄ΠΈΠ½Π°Ρ‚. Работая Ρ‚ΠΎΠ»ΡŒΠΊΠΎ с гСомСтричСским ΠΎΠΏΡ€Π΅Π΄Π΅Π»Π΅Π½ΠΈΠ΅ΠΌ Π²Π΅Π»ΠΈΡ‡ΠΈΠ½Ρ‹ ΠΈ направлСния Π²Π΅ΠΊΡ‚ΠΎΡ€ΠΎΠ², ΠΌΡ‹ смогли ΠΎΠΏΡ€Π΅Π΄Π΅Π»ΠΈΡ‚ΡŒ Ρ‚Π°ΠΊΠΈΠ΅ ΠΎΠΏΠ΅Ρ€Π°Ρ†ΠΈΠΈ, ΠΊΠ°ΠΊ слоТСниС, Π²Ρ‹Ρ‡ΠΈΡ‚Π°Π½ΠΈΠ΅, ΠΈ ΡƒΠΌΠ½ΠΎΠΆΠ΅Π½ΠΈΠ΅ Π½Π° скаляры. ΠœΡ‹ Ρ‚Π°ΠΊΠΆΠ΅ обсудили свойства этих ΠΎΠΏΠ΅Ρ€Π°Ρ†ΠΈΠΉ.

      Часто систСма ΠΊΠΎΠΎΡ€Π΄ΠΈΠ½Π°Ρ‚ оказываСтся ΠΏΠΎΠ»Π΅Π·Π½ΠΎΠΉ, ΠΏΠΎΡ‚ΠΎΠΌΡƒ Ρ‡Ρ‚ΠΎ ΠΏΡ€ΠΎΡ‰Π΅ ΡƒΠΏΡ€Π°Π²Π»ΡΡ‚ΡŒ ΠΊΠΎΠΎΡ€Π΄ΠΈΠ½Π°Ρ‚Π°ΠΌΠΈ Π²Π΅ΠΊΡ‚ΠΎΡ€Π°, Ρ‡Π΅ΠΌ нСпосрСдствСнно Π΅Π³ΠΎ Π²Π΅Π»ΠΈΡ‡ΠΈΠ½ΠΎΠΉ ΠΈ Π½Π°ΠΏΡ€Π°Π²Π»Π΅Π½ΠΈΠ΅ΠΌ. Когда ΠΌΡ‹ Π²Ρ‹Ρ€Π°ΠΆΠ°Π΅ΠΌ Π²Π΅ΠΊΡ‚ΠΎΡ€ Π² систСмС ΠΊΠΎΠΎΡ€Π΄ΠΈΠ½Π°Ρ‚, ΠΌΡ‹ ΠΈΠ΄Π΅Π½Ρ‚ΠΈΡ„ΠΈΡ†ΠΈΡ€ΡƒΠ΅ΠΌ Π²Π΅ΠΊΡ‚ΠΎΡ€ с ΠΏΠΎΠΌΠΎΡ‰ΡŒΡŽ списка чисСл, Π½Π°Π·Ρ‹Π²Π°Π΅ΠΌΡ‹Ρ… ΠΊΠΎΠΎΡ€Π΄ΠΈΠ½Π°Ρ‚Π°ΠΌΠΈ ΠΈΠ»ΠΈ ΠΊΠΎΠΌΠΏΠΎΠ½Π΅Π½Ρ‚Π°ΠΌΠΈ, ΠΊΠΎΡ‚ΠΎΡ€Ρ‹Π΅ ΠΎΠΏΡ€Π΅Π΄Π΅Π»ΡΡŽΡ‚ Π³Π΅ΠΎΠΌΠ΅Ρ‚Ρ€ΠΈΡŽ Π²Π΅ΠΊΡ‚ΠΎΡ€Π° Π² Ρ‚Π΅Ρ€ΠΌΠΈΠ½Π°Ρ… систСмы ΠΊΠΎΠΎΡ€Π΄ΠΈΠ½Π°Ρ‚. Π—Π΄Π΅ΡΡŒ ΠΌΡ‹ обсудим стандартныС Π΄Π΅ΠΊΠ°Ρ€Ρ‚ΠΎΠ²Ρ‹ систСмы ΠΊΠΎΠΎΡ€Π΄ΠΈΠ½Π°Ρ‚ Π½Π° плоскости ΠΈ Π² Ρ‚Ρ€Π΅Ρ…ΠΌΠ΅Ρ€Π½ΠΎΠΌ пространствС.

      Π’Π΅ΠΊΡ‚ΠΎΡ€Ρ‹ Π½Π° плоскости

      ΠœΡ‹ ΠΏΡ€Π΅Π΄ΠΏΠΎΠ»Π°Π³Π°Π΅ΠΌ, Ρ‡Ρ‚ΠΎ Π²Ρ‹ Π·Π½Π°ΠΊΠΎΠΌΡ‹ со стандартной Π΄Π΅ΠΊΠ°Ρ€Ρ‚ΠΎΠ²ΠΎΠΉ систСмой ΠΊΠΎΠΎΡ€Π΄ΠΈΠ½Π°Ρ‚ $(x,y)$ Π½Π° плоскости. КаТдая Ρ‚ΠΎΡ‡ΠΊΠ° $\vc{p}$ Π½Π° плоскости отоТдСствляСтся со своими ΠΊΠΎΠΌΠΏΠΎΠ½Π΅Π½Ρ‚Π°ΠΌΠΈ $x$ ΠΈ $y$: $\vc{p} = (p_1,p_2)$.

      Π§Ρ‚ΠΎΠ±Ρ‹ ΠΎΠΏΡ€Π΅Π΄Π΅Π»ΠΈΡ‚ΡŒ ΠΊΠΎΠΎΡ€Π΄ΠΈΠ½Π°Ρ‚Ρ‹ Π²Π΅ΠΊΡ‚ΠΎΡ€Π° $\vc{a}$ Π½Π° плоскости, ΠΏΠ΅Ρ€Π²Ρ‹ΠΉ шаг β€” пСрСвСсти Π²Π΅ΠΊΡ‚ΠΎΡ€ Ρ‚Π°ΠΊ, Ρ‡Ρ‚ΠΎΠ±Ρ‹ Π΅Π³ΠΎ хвост находился Π² Π½Π°Ρ‡Π°Π»Π΅ ΠΊΠΎΠΎΡ€Π΄ΠΈΠ½Π°Ρ‚ систСмы ΠΊΠΎΠΎΡ€Π΄ΠΈΠ½Π°Ρ‚. 2$, Ρ‡Ρ‚ΠΎΠ±Ρ‹ ΠΎΠ±ΠΎΠ·Π½Π°Ρ‡ΠΈΡ‚ΡŒ, Ρ‡Ρ‚ΠΎ Π΅Π³ΠΎ ΠΌΠΎΠΆΠ½ΠΎ ΠΎΠΏΠΈΡΠ°Ρ‚ΡŒ двумя Π΄Π΅ΠΉΡΡ‚Π²ΠΈΡ‚Π΅Π»ΡŒΠ½Ρ‹ΠΌΠΈ ΠΊΠΎΠΎΡ€Π΄ΠΈΠ½Π°Ρ‚Π°ΠΌΠΈ. 92} = 5$.

      ΠŸΡ€ΠΈΠ²Π΅Π΄Π΅Π½Π½Ρ‹ΠΉ Π½ΠΈΠΆΠ΅ Π°ΠΏΠΏΠ»Π΅Ρ‚, ΠΏΠΎΠ²Ρ‚ΠΎΡ€ΡΡŽΡ‰ΠΈΠΉΡΡ ΠΈΠ· ввСдСния Π²Π΅ΠΊΡ‚ΠΎΡ€Π°, позволяСт Π²Π°ΠΌ ΠΈΡΡΠ»Π΅Π΄ΠΎΠ²Π°Ρ‚ΡŒ взаимосвязь ΠΌΠ΅ΠΆΠ΄Ρƒ ΠΊΠΎΠΌΠΏΠΎΠ½Π΅Π½Ρ‚Π°ΠΌΠΈ Π²Π΅ΠΊΡ‚ΠΎΡ€Π° ΠΈ Π΅Π³ΠΎ Π²Π΅Π»ΠΈΡ‡ΠΈΠ½ΠΎΠΉ.

      Π’Π΅Π»ΠΈΡ‡ΠΈΠ½Π° ΠΈ Π½Π°ΠΏΡ€Π°Π²Π»Π΅Π½ΠΈΠ΅ Π²Π΅ΠΊΡ‚ΠΎΡ€Π°. Биняя стрСлка ΠΎΠ±ΠΎΠ·Π½Π°Ρ‡Π°Π΅Ρ‚ Π²Π΅ΠΊΡ‚ΠΎΡ€ $\vc{a}$. Π”Π²Π° ΠΎΠΏΡ€Π΅Π΄Π΅Π»ΡΡŽΡ‰ΠΈΡ… свойства Π²Π΅ΠΊΡ‚ΠΎΡ€Π°, Π²Π΅Π»ΠΈΡ‡ΠΈΠ½Π° ΠΈ Π½Π°ΠΏΡ€Π°Π²Π»Π΅Π½ΠΈΠ΅, ΠΏΠΎΠΊΠ°Π·Π°Π½Ρ‹ красной полосой ΠΈ Π·Π΅Π»Π΅Π½ΠΎΠΉ стрСлкой соотвСтствСнно. Π”Π»ΠΈΠ½Π° красной полосы β€” это Π²Π΅Π»ΠΈΡ‡ΠΈΠ½Π° $\|\vc{a}\|$ Π²Π΅ΠΊΡ‚ΠΎΡ€Π° $\vc{a}$. ЗСлСная стрСлка всСгда ΠΈΠΌΠ΅Π΅Ρ‚ Π΄Π»ΠΈΠ½Ρƒ Π΅Π΄ΠΈΠ½ΠΈΡ†Ρƒ, Π½ΠΎ Π΅Π΅ Π½Π°ΠΏΡ€Π°Π²Π»Π΅Π½ΠΈΠ΅ совпадаСт с Π½Π°ΠΏΡ€Π°Π²Π»Π΅Π½ΠΈΠ΅ΠΌ Π²Π΅ΠΊΡ‚ΠΎΡ€Π° $\vc{a}$. ЕдинствСнным ΠΈΡΠΊΠ»ΡŽΡ‡Π΅Π½ΠΈΠ΅ΠΌ являСтся случай, ΠΊΠΎΠ³Π΄Π° $\vc{a}$ являСтся Π½ΡƒΠ»Π΅Π²Ρ‹ΠΌ Π²Π΅ΠΊΡ‚ΠΎΡ€ΠΎΠΌ (СдинствСнным Π²Π΅ΠΊΡ‚ΠΎΡ€ΠΎΠΌ с Π½ΡƒΠ»Π΅Π²ΠΎΠΉ Π²Π΅Π»ΠΈΡ‡ΠΈΠ½ΠΎΠΉ), для ΠΊΠΎΡ‚ΠΎΡ€ΠΎΠ³ΠΎ Π½Π°ΠΏΡ€Π°Π²Π»Π΅Π½ΠΈΠ΅ Π½Π΅ ΠΎΠΏΡ€Π΅Π΄Π΅Π»Π΅Π½ΠΎ. Π’Ρ‹ ΠΌΠΎΠΆΠ΅Ρ‚Π΅ ΠΈΠ·ΠΌΠ΅Π½ΠΈΡ‚ΡŒ любой ΠΊΠΎΠ½Π΅Ρ† $\vc{a}$, ΠΏΠ΅Ρ€Π΅Ρ‚Π°Ρ‰ΠΈΠ² Π΅Π³ΠΎ ΠΌΡ‹ΡˆΡŒΡŽ. Π’Ρ‹ Ρ‚Π°ΠΊΠΆΠ΅ ΠΌΠΎΠΆΠ΅Ρ‚Π΅ ΠΏΠ΅Ρ€Π΅ΠΌΠ΅ΡΡ‚ΠΈΡ‚ΡŒ $\vc{a}$, ΠΏΠ΅Ρ€Π΅Ρ‚Π°Ρ‰ΠΈΠ² сСрСдину Π²Π΅ΠΊΡ‚ΠΎΡ€Π°; ΠΎΠ΄Π½Π°ΠΊΠΎ ΠΈΠ·ΠΌΠ΅Π½Π΅Π½ΠΈΠ΅ полоТСния $\vc{a}$ Ρ‚Π°ΠΊΠΈΠΌ ΠΎΠ±Ρ€Π°Π·ΠΎΠΌ Π½Π΅ мСняСт Π²Π΅ΠΊΡ‚ΠΎΡ€, Ρ‚Π°ΠΊ ΠΊΠ°ΠΊ Π΅Π³ΠΎ Π²Π΅Π»ΠΈΡ‡ΠΈΠ½Π° ΠΈ Π½Π°ΠΏΡ€Π°Π²Π»Π΅Π½ΠΈΠ΅ ΠΎΡΡ‚Π°ΡŽΡ‚ΡΡ Π½Π΅ΠΈΠ·ΠΌΠ΅Π½Π½Ρ‹ΠΌΠΈ.

      Π”ΠΎΠΏΠΎΠ»Π½ΠΈΡ‚Π΅Π»ΡŒΠ½Π°Ρ информация ΠΎΠ± Π°ΠΏΠΏΠ»Π΅Ρ‚Π΅.

      Π’Π΅ΠΊΡ‚ΠΎΡ€Π½Ρ‹Π΅ ΠΎΠΏΠ΅Ρ€Π°Ρ†ΠΈΠΈ, ΠΊΠΎΡ‚ΠΎΡ€Ρ‹Π΅ ΠΌΡ‹ ΠΎΠΏΡ€Π΅Π΄Π΅Π»ΠΈΠ»ΠΈ Π²ΠΎ Π²Π²Π΅Π΄Π΅Π½ΠΈΠΈ ΠΊ Π²Π΅ΠΊΡ‚ΠΎΡ€Π°ΠΌ, Π»Π΅Π³ΠΊΠΎ Π²Ρ‹Ρ€Π°Π·ΠΈΡ‚ΡŒ Π² Ρ‚Π΅Ρ€ΠΌΠΈΠ½Π°Ρ… этих ΠΊΠΎΠΎΡ€Π΄ΠΈΠ½Π°Ρ‚. Если $\vc{a}=(a_1,a_2)$ ΠΈ $\vc{b}=(b_1,b_2)$, ΠΈΡ… сумма просто $\vc{a}+\vc{b}=(a_1+b_1,a_2+b_2)$, ΠΊΠ°ΠΊ ΠΏΠΎΠΊΠ°Π·Π°Π½ΠΎ Π½Π° рисункС Π½ΠΈΠΆΠ΅. Π’Π°ΠΊΠΆΠ΅ Π»Π΅Π³ΠΊΠΎ Π²ΠΈΠ΄Π΅Ρ‚ΡŒ, Ρ‡Ρ‚ΠΎ $\vc{b}-\vc{a} = (b_1-a_1,b_2-a_2)$ ΠΈ $\lambda \vc{a} = (\lambda a_1, \lambda a_2)$ для любого скаляра $\lambda$.

      ΠŸΡ€ΠΈΠ²Π΅Π΄Π΅Π½Π½Ρ‹ΠΉ Π½ΠΈΠΆΠ΅ Π°ΠΏΠΏΠ»Π΅Ρ‚, Ρ‚Π°ΠΊΠΆΠ΅ ΠΏΠΎΠ²Ρ‚ΠΎΡ€ΡΡŽΡ‰ΠΈΠΉΡΡ ΠΈΠ· ввСдСния Π²Π΅ΠΊΡ‚ΠΎΡ€Π°, позволяСт Π²Π°ΠΌ ΠΈΡΡΠ»Π΅Π΄ΠΎΠ²Π°Ρ‚ΡŒ взаимосвязь ΠΌΠ΅ΠΆΠ΄Ρƒ гСомСтричСским ΠΎΠΏΡ€Π΅Π΄Π΅Π»Π΅Π½ΠΈΠ΅ΠΌ слоТСния Π²Π΅ΠΊΡ‚ΠΎΡ€ΠΎΠ² ΠΈ суммированиСм ΠΊΠΎΠΌΠΏΠΎΠ½Π΅Π½Ρ‚ΠΎΠ² Π²Π΅ΠΊΡ‚ΠΎΡ€Π°.

      Π‘ΡƒΠΌΠΌΠ° Π΄Π²ΡƒΡ… Π²Π΅ΠΊΡ‚ΠΎΡ€ΠΎΠ². Π‘ΡƒΠΌΠΌΠ° $\vc{a}+\vc{b}$ Π²Π΅ΠΊΡ‚ΠΎΡ€Π° $\vc{a}$ (синяя стрСлка) ΠΈ Π²Π΅ΠΊΡ‚ΠΎΡ€Π° $\vc{b}$ (красная стрСлка) ΠΏΠΎΠΊΠ°Π·Π°Π½Π° Π·Π΅Π»Π΅Π½ΠΎΠΉ стрСлкой . ΠŸΠΎΡΠΊΠΎΠ»ΡŒΠΊΡƒ Π²Π΅ΠΊΡ‚ΠΎΡ€Ρ‹ Π½Π΅ зависят ΠΎΡ‚ ΠΈΡ… Π½Π°Ρ‡Π°Π»ΡŒΠ½ΠΎΠ³ΠΎ полоТСния, ΠΎΠ±Π΅ синиС стрСлки ΠΏΡ€Π΅Π΄ΡΡ‚Π°Π²Π»ΡΡŽΡ‚ ΠΎΠ΄ΠΈΠ½ ΠΈ Ρ‚ΠΎΡ‚ ΠΆΠ΅ Π²Π΅ΠΊΡ‚ΠΎΡ€ $\vc{a}$, Π° ΠΎΠ±Π΅ красныС стрСлки ΠΏΡ€Π΅Π΄ΡΡ‚Π°Π²Π»ΡΡŽΡ‚ ΠΎΠ΄ΠΈΠ½ ΠΈ Ρ‚ΠΎΡ‚ ΠΆΠ΅ Π²Π΅ΠΊΡ‚ΠΎΡ€ $\vc{b}$. Π‘ΡƒΠΌΠΌΡƒ $\vc{a}+\vc{b}$ ΠΌΠΎΠΆΠ½ΠΎ ΡΠΎΡΡ‚Π°Π²ΠΈΡ‚ΡŒ, помСстив хвост Π²Π΅ΠΊΡ‚ΠΎΡ€Π° $\vc{b}$ Π² Π½Π°Ρ‡Π°Π»ΠΎ Π²Π΅ΠΊΡ‚ΠΎΡ€Π° $\vc{a}$. Π’ΠΎ ΠΆΠ΅ самоС ΠΌΠΎΠΆΠ½ΠΎ ΡΠ΄Π΅Π»Π°Ρ‚ΡŒ, помСстив хвост Π²Π΅ΠΊΡ‚ΠΎΡ€Π° $\vc{a}$ Π² Π½Π°Ρ‡Π°Π»ΠΎ Π²Π΅ΠΊΡ‚ΠΎΡ€Π° $\vc{b}$. ОбС конструкции вмСстС ΠΎΠ±Ρ€Π°Π·ΡƒΡŽΡ‚ ΠΏΠ°Ρ€Π°Π»Π»Π΅Π»ΠΎΠ³Ρ€Π°ΠΌΠΌ, сумма $\vc{a}+\vc{b}$ ΠΊΠΎΡ‚ΠΎΡ€ΠΎΠ³ΠΎ являСтся диагональю. (По этой ΠΏΡ€ΠΈΡ‡ΠΈΠ½Π΅ Π·Π°ΠΊΠΎΠ½ пСрСстановки $\vc{a}+\vc{b}=\vc{b}+\vc{a}$ ΠΈΠ½ΠΎΠ³Π΄Π° Π½Π°Π·Ρ‹Π²Π°ΡŽΡ‚ Π·Π°ΠΊΠΎΠ½ΠΎΠΌ ΠΏΠ°Ρ€Π°Π»Π»Π΅Π»ΠΎΠ³Ρ€Π°ΠΌΠΌΠ°.) Π’Ρ‹ ΠΌΠΎΠΆΠ΅Ρ‚Π΅ ΠΈΠ·ΠΌΠ΅Π½ΠΈΡ‚ΡŒ $\vc{a} $ ΠΈ $\vc{b}$, пСрСтаскивая ΠΆΠ΅Π»Ρ‚Ρ‹Π΅ Ρ‚ΠΎΡ‡ΠΊΠΈ.

      Π”ΠΎΠΏΠΎΠ»Π½ΠΈΡ‚Π΅Π»ΡŒΠ½Π°Ρ информация ΠΎΠ± Π°ΠΏΠΏΠ»Π΅Ρ‚Π΅.

      Π’Ρ‹ ΠΌΠΎΠ³Π»ΠΈ Π·Π°ΠΌΠ΅Ρ‚ΠΈΡ‚ΡŒ, Ρ‡Ρ‚ΠΎ ΠΌΡ‹ ΠΈΡΠΏΠΎΠ»ΡŒΠ·ΡƒΠ΅ΠΌ ΠΎΠ΄Π½ΠΈ ΠΈ Ρ‚Π΅ ΠΆΠ΅ обозначСния для обозначСния Ρ‚ΠΎΡ‡ΠΊΠΈ ΠΈ Π²Π΅ΠΊΡ‚ΠΎΡ€Π°. ΠœΡ‹ Π½Π΅ склонны ΠΏΠΎΠ΄Ρ‡Π΅Ρ€ΠΊΠΈΠ²Π°Ρ‚ΡŒ ΠΊΠ°ΠΊΠΎΠ΅-Π»ΠΈΠ±ΠΎ Ρ€Π°Π·Π»ΠΈΡ‡ΠΈΠ΅ ΠΌΠ΅ΠΆΠ΄Ρƒ Ρ‚ΠΎΡ‡ΠΊΠΎΠΉ ΠΈ Π²Π΅ΠΊΡ‚ΠΎΡ€ΠΎΠΌ. Π’Ρ‹ ΠΌΠΎΠΆΠ΅Ρ‚Π΅ Π΄ΡƒΠΌΠ°Ρ‚ΡŒ ΠΎ Ρ‚ΠΎΡ‡ΠΊΠ΅ ΠΊΠ°ΠΊ ΠΎ прСдставлСнии Π²Π΅ΠΊΡ‚ΠΎΡ€Π°, хвост ΠΊΠΎΡ‚ΠΎΡ€ΠΎΠ³ΠΎ зафиксирован Π² Π½Π°Ρ‡Π°Π»Π΅ ΠΊΠΎΠΎΡ€Π΄ΠΈΠ½Π°Ρ‚. Π’Π°ΠΌ придСтся Π²Ρ‹ΡΡΠ½ΠΈΡ‚ΡŒ ΠΏΠΎ контСксту, Π΄ΡƒΠΌΠ°Π΅ΠΌ Π»ΠΈ ΠΌΡ‹ ΠΎ Π²Π΅ΠΊΡ‚ΠΎΡ€Π΅ ΠΈΠ»ΠΈ Π½Π΅Ρ‚. ΠΊΠ°ΠΊ с фиксированным хвостом Π² Π½Π°Ρ‡Π°Π»Π΅ ΠΊΠΎΠΎΡ€Π΄ΠΈΠ½Π°Ρ‚.

      Π”Ρ€ΡƒΠ³ΠΎΠΉ способ обозначСния Π²Π΅ΠΊΡ‚ΠΎΡ€ΠΎΠ² β€” стандартныС Π΅Π΄ΠΈΠ½ΠΈΡ‡Π½Ρ‹Π΅ Π²Π΅ΠΊΡ‚ΠΎΡ€Ρ‹ ΠΎΠ±ΠΎΠ·Π½Π°Ρ‡Π°ΡŽΡ‚ΡΡ $\vc{i}$ ΠΈ $\vc{j}$. Π•Π΄ΠΈΠ½ΠΈΡ‡Π½Ρ‹ΠΉ Π²Π΅ΠΊΡ‚ΠΎΡ€ β€” это Π²Π΅ΠΊΡ‚ΠΎΡ€, Π΄Π»ΠΈΠ½Π° ΠΊΠΎΡ‚ΠΎΡ€ΠΎΠ³ΠΎ Ρ€Π°Π²Π½Π° Π΅Π΄ΠΈΠ½ΠΈΡ†Π΅. Π’Π΅ΠΊΡ‚ΠΎΡ€ $\vc{i}$ являСтся Π΅Π΄ΠΈΠ½ΠΈΡ‡Π½Ρ‹ΠΌ Π²Π΅ΠΊΡ‚ΠΎΡ€ΠΎΠΌ Π² Π½Π°ΠΏΡ€Π°Π²Π»Π΅Π½ΠΈΠΈ ΠΏΠΎΠ»ΠΎΠΆΠΈΡ‚Π΅Π»ΡŒΠ½ΠΎΠΉ оси $x$. Π’ ΠΊΠΎΠΎΡ€Π΄ΠΈΠ½Π°Ρ‚Π°Ρ… ΠΌΡ‹ ΠΌΠΎΠΆΠ΅ΠΌ Π½Π°ΠΏΠΈΡΠ°Ρ‚ΡŒ $\vc{i}=(1,0)$. Π’ΠΎΡ‡Π½ΠΎ Ρ‚Π°ΠΊ ΠΆΠ΅ Π²Π΅ΠΊΡ‚ΠΎΡ€ $\vc{j}$ являСтся Π΅Π΄ΠΈΠ½ΠΈΡ‡Π½Ρ‹ΠΌ Π²Π΅ΠΊΡ‚ΠΎΡ€ΠΎΠΌ Π² Π½Π°ΠΏΡ€Π°Π²Π»Π΅Π½ΠΈΠΈ ΠΏΠΎΠ»ΠΎΠΆΠΈΡ‚Π΅Π»ΡŒΠ½ΠΎΠΉ оси $y$: $\vc{j}=(0,1)$. ΠœΡ‹ ΠΌΠΎΠΆΠ΅ΠΌ Π·Π°ΠΏΠΈΡΠ°Ρ‚ΡŒ любой Π΄Π²ΡƒΠΌΠ΅Ρ€Π½Ρ‹ΠΉ Π²Π΅ΠΊΡ‚ΠΎΡ€ Π² Ρ‚Π΅Ρ€ΠΌΠΈΠ½Π°Ρ… этих Π΅Π΄ΠΈΠ½ΠΈΡ‡Π½Ρ‹Ρ… Π²Π΅ΠΊΡ‚ΠΎΡ€ΠΎΠ² ΠΊΠ°ΠΊ $\vc{a}=(a_1,a_2) = a_1\vc{i}+a_2\vc{j}$.

      Π’Π΅ΠΊΡ‚ΠΎΡ€Ρ‹ Π² Ρ‚Ρ€Π΅Ρ…ΠΌΠ΅Ρ€Π½ΠΎΠΌ пространствС

      Π’ Ρ‚Ρ€Π΅Ρ…ΠΌΠ΅Ρ€Π½ΠΎΠΌ пространствС сущСствуСт стандартная Π΄Π΅ΠΊΠ°Ρ€Ρ‚ΠΎΠ²Π° систСма ΠΊΠΎΠΎΡ€Π΄ΠΈΠ½Π°Ρ‚ $(x,y,z)$. Начиная с Ρ‚ΠΎΡ‡ΠΊΠΈ, ΠΊΠΎΡ‚ΠΎΡ€ΡƒΡŽ ΠΌΡ‹ Π½Π°Π·Ρ‹Π²Π°Π΅ΠΌ Π½Π°Ρ‡Π°Π»ΠΎΠΌ ΠΊΠΎΠΎΡ€Π΄ΠΈΠ½Π°Ρ‚, построим Ρ‚Ρ€ΠΈ Π²Π·Π°ΠΈΠΌΠ½ΠΎ пСрпСндикулярныС оси, ΠΊΠΎΡ‚ΠΎΡ€Ρ‹Π΅ ΠΌΡ‹ Π½Π°Π·Ρ‹Π²Π°Π΅ΠΌ осью $x$, осью $y$ ΠΈ осью $z$. Π’ΠΎΡ‚ ΠΎΠ΄ΠΈΠ½ ΠΈΠ· способов ΠΈΠ·ΠΎΠ±Ρ€Π°Π·ΠΈΡ‚ΡŒ эти оси. Π’ΡΡ‚Π°Π½ΡŒΡ‚Π΅ Π² ΡƒΠ³Π»Ρƒ ΠΊΠΎΠΌΠ½Π°Ρ‚Ρ‹ ΠΈ посмотритС Π²Π½ΠΈΠ·, Π² Ρ‚ΠΎΡ‡ΠΊΡƒ, Π³Π΄Π΅ стСны ΡΠΎΠΏΡ€ΠΈΠΊΠ°ΡΠ°ΡŽΡ‚ΡΡ с ΠΏΠΎΠ»ΠΎΠΌ. Π—Π°Ρ‚Π΅ΠΌ ΠΏΠΎΠ» ΠΈ стСна слСва ΠΎΡ‚ вас ΠΏΠ΅Ρ€Π΅ΡΠ΅ΠΊΠ°ΡŽΡ‚ΡΡ ΠΏΠΎ Π»ΠΈΠ½ΠΈΠΈ, ΡΠ²Π»ΡΡŽΡ‰Π΅ΠΉΡΡ ΠΏΠΎΠ»ΠΎΠΆΠΈΡ‚Π΅Π»ΡŒΠ½ΠΎΠΉ осью $x$. Пол ΠΈ стСна справа ΠΎΡ‚ вас ΠΏΠ΅Ρ€Π΅ΡΠ΅ΠΊΠ°ΡŽΡ‚ΡΡ ΠΏΠΎ Π»ΠΈΠ½ΠΈΠΈ, ΡΠ²Π»ΡΡŽΡ‰Π΅ΠΉΡΡ ΠΏΠΎΠ»ΠΎΠΆΠΈΡ‚Π΅Π»ΡŒΠ½ΠΎΠΉ осью $y$. Π‘Ρ‚Π΅Π½Ρ‹ ΠΏΠ΅Ρ€Π΅ΡΠ΅ΠΊΠ°ΡŽΡ‚ΡΡ ΠΏΠΎ Π²Π΅Ρ€Ρ‚ΠΈΠΊΠ°Π»ΡŒΠ½ΠΎΠΉ Π»ΠΈΠ½ΠΈΠΈ, ΡΠ²Π»ΡΡŽΡ‰Π΅ΠΉΡΡ ΠΏΠΎΠ»ΠΎΠΆΠΈΡ‚Π΅Π»ΡŒΠ½ΠΎΠΉ осью $z$. Π­Ρ‚ΠΈ ΠΏΠΎΠ»ΠΎΠΆΠΈΡ‚Π΅Π»ΡŒΠ½Ρ‹Π΅ оси ΠΈΠ·ΠΎΠ±Ρ€Π°ΠΆΠ΅Π½Ρ‹ Π² ΠΏΡ€ΠΈΠ²Π΅Π΄Π΅Π½Π½ΠΎΠΌ Π½ΠΈΠΆΠ΅ Π°ΠΏΠΏΠ»Π΅Ρ‚Π΅ ΠΈ ΠΏΠΎΠΌΠ΅Ρ‡Π΅Π½Ρ‹ ΠΊΠ°ΠΊ $x$, $y$ ΠΈ $z$. ΠžΡ‚Ρ€ΠΈΡ†Π°Ρ‚Π΅Π»ΡŒΠ½Π°Ρ Ρ‡Π°ΡΡ‚ΡŒ ΠΊΠ°ΠΆΠ΄ΠΎΠΉ оси находится Π½Π° ΠΏΡ€ΠΎΡ‚ΠΈΠ²ΠΎΠΏΠΎΠ»ΠΎΠΆΠ½ΠΎΠΉ сторонС Π½Π°Ρ‡Π°Π»Π° ΠΊΠΎΠΎΡ€Π΄ΠΈΠ½Π°Ρ‚, Π³Π΄Π΅ оси ΠΏΠ΅Ρ€Π΅ΡΠ΅ΠΊΠ°ΡŽΡ‚ΡΡ.

      Π—Π°Π³Ρ€ΡƒΠ·ΠΊΠ° Π°ΠΏΠΏΠ»Π΅Ρ‚Π°

      Π’Ρ€Π΅Ρ…ΠΌΠ΅Ρ€Π½Ρ‹Π΅ Π΄Π΅ΠΊΠ°Ρ€Ρ‚ΠΎΠ²Ρ‹ оси ΠΊΠΎΠΎΡ€Π΄ΠΈΠ½Π°Ρ‚. ΠŸΡ€Π΅Π΄ΡΡ‚Π°Π²Π»Π΅Π½ΠΈΠ΅ Ρ‚Ρ€Π΅Ρ… осСй Ρ‚Ρ€Π΅Ρ…ΠΌΠ΅Ρ€Π½ΠΎΠΉ Π΄Π΅ΠΊΠ°Ρ€Ρ‚ΠΎΠ²ΠΎΠΉ систСмы ΠΊΠΎΠΎΡ€Π΄ΠΈΠ½Π°Ρ‚. ΠŸΠΎΠ»ΠΎΠΆΠΈΡ‚Π΅Π»ΡŒΠ½Π°Ρ ось $x$, ΠΏΠΎΠ»ΠΎΠΆΠΈΡ‚Π΅Π»ΡŒΠ½Π°Ρ ось $y$ ΠΈ ΠΏΠΎΠ»ΠΎΠΆΠΈΡ‚Π΅Π»ΡŒΠ½Π°Ρ ось $z$ β€” это стороны, ΠΏΠΎΠΌΠ΅Ρ‡Π΅Π½Π½Ρ‹Π΅ $x$, $y$ ΠΈ $z$. Начало β€” это пСрСсСчСниС всСх осСй. Π’Π΅Ρ‚Π²ΡŒ ΠΊΠ°ΠΆΠ΄ΠΎΠΉ оси Π½Π° ΠΏΡ€ΠΎΡ‚ΠΈΠ²ΠΎΠΏΠΎΠ»ΠΎΠΆΠ½ΠΎΠΉ сторонС ΠΎΡ‚ Π½Π°Ρ‡Π°Π»Π° ΠΊΠΎΠΎΡ€Π΄ΠΈΠ½Π°Ρ‚ (нСмаркированная сторона) являСтся ΠΎΡ‚Ρ€ΠΈΡ†Π°Ρ‚Π΅Π»ΡŒΠ½ΠΎΠΉ Ρ‡Π°ΡΡ‚ΡŒΡŽ. Π’Ρ‹ ΠΌΠΎΠΆΠ΅Ρ‚Π΅ ΠΏΠ΅Ρ€Π΅Ρ‚Π°Ρ‰ΠΈΡ‚ΡŒ Ρ„ΠΈΠ³ΡƒΡ€Ρƒ с ΠΏΠΎΠΌΠΎΡ‰ΡŒΡŽ ΠΌΡ‹ΡˆΠΈ, Ρ‡Ρ‚ΠΎΠ±Ρ‹ ΠΏΠΎΠ²Π΅Ρ€Π½ΡƒΡ‚ΡŒ Π΅Π΅.

      Π”ΠΎΠΏΠΎΠ»Π½ΠΈΡ‚Π΅Π»ΡŒΠ½Π°Ρ информация ΠΎΠ± Π°ΠΏΠΏΠ»Π΅Ρ‚Π΅.

      ΠœΡ‹ установили ΠΎΡ‚Π½ΠΎΡΠΈΡ‚Π΅Π»ΡŒΠ½ΠΎΠ΅ располоТСниС ΠΏΠΎΠ»ΠΎΠΆΠΈΡ‚Π΅Π»ΡŒΠ½Ρ‹Ρ… осСй $x$, $y$ ΠΈ $z$ Ρ‡Ρ‚ΠΎΠ±Ρ‹ ΡΠ΄Π΅Π»Π°Ρ‚ΡŒ систСму ΠΊΠΎΠΎΡ€Π΄ΠΈΠ½Π°Ρ‚ ΠΏΡ€Π°Π²ΠΎΠΉ систСмой ΠΊΠΎΠΎΡ€Π΄ΠΈΠ½Π°Ρ‚ . ΠžΠ±Ρ€Π°Ρ‚ΠΈΡ‚Π΅ Π²Π½ΠΈΠΌΠ°Π½ΠΈΠ΅, Ρ‡Ρ‚ΠΎ Ссли ΡΠΎΠ³Π½ΡƒΡ‚ΡŒ ΠΏΠ°Π»ΡŒΡ†Ρ‹ ΠΏΡ€Π°Π²ΠΎΠΉ Ρ€ΡƒΠΊΠΈ ΠΎΡ‚ ΠΏΠΎΠ»ΠΎΠΆΠΈΡ‚Π΅Π»ΡŒΠ½ΠΎΠΉ оси $x$ ΠΊ ΠΏΠΎΠ»ΠΎΠΆΠΈΡ‚Π΅Π»ΡŒΠ½ΠΎΠΉ оси $y$, большой ΠΏΠ°Π»Π΅Ρ† ΠΏΡ€Π°Π²ΠΎΠΉ Ρ€ΡƒΠΊΠΈ Π±ΡƒΠ΄Π΅Ρ‚ ΡƒΠΊΠ°Π·Ρ‹Π²Π°Ρ‚ΡŒ Π² Π½Π°ΠΏΡ€Π°Π²Π»Π΅Π½ΠΈΠΈ ΠΏΠΎΠ»ΠΎΠΆΠΈΡ‚Π΅Π»ΡŒΠ½ΠΎΠΉ оси $z$.

      Если Π²Ρ‹ помСняли мСстами ΠΏΠΎΠ»ΠΎΠΆΠΈΡ‚Π΅Π»ΡŒΠ½ΡƒΡŽ ось $x$ ΠΈ ΠΏΠΎΠ»ΠΎΠΆΠΈΡ‚Π΅Π»ΡŒΠ½ΡƒΡŽ ось $y$, Ρ‚ΠΎΠ³Π΄Π° Ρƒ вас Π±ΡƒΠ΄Π΅Ρ‚ лСвосторонняя систСма ΠΊΠΎΠΎΡ€Π΄ΠΈΠ½Π°Ρ‚. Если Π²Ρ‹ сдСлаСтС это, Π²Ρ‹ Π±ΡƒΠ΄Π΅Ρ‚Π΅ ΠΆΠΈΡ‚ΡŒ Π² матСматичСской всСлСнной, Π² ΠΊΠΎΡ‚ΠΎΡ€ΠΎΠΉ Π½Π΅ΠΊΠΎΡ‚ΠΎΡ€Ρ‹Π΅ Ρ„ΠΎΡ€ΠΌΡƒΠ»Ρ‹ Π±ΡƒΠ΄ΡƒΡ‚ ΠΎΡ‚Π»ΠΈΡ‡Π°Ρ‚ΡŒΡΡ Π½Π° Π·Π½Π°ΠΊ минус ΠΎΡ‚ Ρ„ΠΎΡ€ΠΌΡƒΠ»Ρ‹ Π²ΠΎ всСлСнной, ΠΊΠΎΡ‚ΠΎΡ€ΡƒΡŽ ΠΌΡ‹ здСсь ΠΈΡΠΏΠΎΠ»ΡŒΠ·ΡƒΠ΅ΠΌ. Π’Π°ΡˆΠ° всСлСнная Π±ΡƒΠ΄Π΅Ρ‚ Ρ‚Π°ΠΊΠΎΠΉ ΠΆΠ΅ достовСрной, ΠΊΠ°ΠΊ ΠΈ наша, Π½ΠΎ Π±ΡƒΠ΄Π΅Ρ‚ ΠΌΠ½ΠΎΠ³ΠΎ ΠΏΡƒΡ‚Π°Π½ΠΈΡ†Ρ‹. ΠœΡ‹ ΠΏΡ€Π΅Π΄Π»Π°Π³Π°Π΅ΠΌ Π²Π°ΠΌ ΠΆΠΈΡ‚ΡŒ Π² нашСй всСлСнной, изучая эти страницы.

      Π‘ ΠΏΠΎΠΌΠΎΡ‰ΡŒΡŽ этих осСй любой Ρ‚ΠΎΡ‡ΠΊΠ΅ $\vc{p}$ Π² пространствС ΠΌΠΎΠΆΠ½ΠΎ ΠΏΡ€ΠΈΡΠ²ΠΎΠΈΡ‚ΡŒ Ρ‚Ρ€ΠΈ ΠΊΠΎΠΎΡ€Π΄ΠΈΠ½Π°Ρ‚Ρ‹ $\vc{p}=(p_1,p_2,p_3)$. НапримСр, учитывая ΠΏΡ€ΠΈΠ²Π΅Π΄Π΅Π½Π½ΡƒΡŽ Π²Ρ‹ΡˆΠ΅ аналогию с ΡƒΠ³Π»ΠΎΠΌ ΠΊΠΎΠΌΠ½Π°Ρ‚Ρ‹, ΠΏΡ€Π΅Π΄ΠΏΠΎΠ»ΠΎΠΆΠΈΠΌ, Π²Ρ‹ Π½Π°Ρ‡ΠΈΠ½Π°Π΅Ρ‚Π΅ с ΡƒΠ³Π»Π° ΠΊΠΎΠΌΠ½Π°Ρ‚Ρ‹ ΠΈ ΠΏΡ€ΠΎΡ…ΠΎΠ΄ΠΈΡ‚Π΅ Ρ‡Π΅Ρ‚Ρ‹Ρ€Π΅ ΠΌΠ΅Ρ‚Ρ€Π° ΠΏΠΎ оси $x$, Π·Π°Ρ‚Π΅ΠΌ ΠΏΠΎΠ²ΠΎΡ€Π°Ρ‡ΠΈΠ²Π°Π΅Ρ‚Π΅ Π½Π°Π»Π΅Π²ΠΎ ΠΈ ΠΏΡ€ΠΎΡ…ΠΎΠ΄ΠΈΡ‚Π΅ Ρ‚Ρ€ΠΈ ΠΌΠ΅Ρ‚Ρ€Π° Π²Π³Π»ΡƒΠ±ΡŒ ΠΊΠΎΠΌΠ½Π°Ρ‚Ρ‹. Если ваш рост Π΄Π²Π° ΠΌΠ΅Ρ‚Ρ€Π°, Ρ‚ΠΎ ваша ΠΌΠ°ΠΊΡƒΡˆΠΊΠ° находится Π² Ρ‚ΠΎΡ‡ΠΊΠ΅ $(4,3,2)$. 93$ для обозначСния Ρ‚ΠΎΠ³ΠΎ, Ρ‡Ρ‚ΠΎ Π΅Π³ΠΎ ΠΌΠΎΠΆΠ½ΠΎ ΠΎΠΏΠΈΡΠ°Ρ‚ΡŒ трСмя Π΄Π΅ΠΉΡΡ‚Π²ΠΈΡ‚Π΅Π»ΡŒΠ½Ρ‹ΠΌΠΈ ΠΊΠΎΠΎΡ€Π΄ΠΈΠ½Π°Ρ‚Π°ΠΌΠΈ. Π‘ΡƒΠΌΠΌΡ‹, разности ΠΈ скалярныС умноТСния Ρ‚Ρ€Π΅Ρ…ΠΌΠ΅Ρ€Π½Ρ‹Ρ… Π²Π΅ΠΊΡ‚ΠΎΡ€ΠΎΠ² Π²Ρ‹ΠΏΠΎΠ»Π½ΡΡŽΡ‚ΡΡ для ΠΊΠ°ΠΆΠ΄ΠΎΠ³ΠΎ ΠΊΠΎΠΌΠΏΠΎΠ½Π΅Π½Ρ‚Π°. Если $\vc{a}=(a_1,a_2,a_3)$ ΠΈ $\vc{b}=(b_1,b_2,b_3)$, Ρ‚ΠΎ $\vc{a}+\vc{b}=(a_1+ b_1,a_2+b_2,a_3+b_3)$, $\vc{b}-\vc{a}=(b_1-a_1,b_2-a_2,b_3-a_3)$ ΠΈ $\lambda\vc{a}= (\лямбда Π°_1, \лямбда Π°_2, \лямбда Π°_3)$.

      Π—Π°Π³Ρ€ΡƒΠ·ΠΊΠ° Π°ΠΏΠΏΠ»Π΅Ρ‚Π°

      Π’Π΅ΠΊΡ‚ΠΎΡ€ Π² Ρ‚Ρ€Π΅Ρ…ΠΌΠ΅Ρ€Π½ΠΎΠΌ пространствС. ΠŸΡ€Π΅Π΄ΡΡ‚Π°Π²Π»Π΅Π½ΠΈΠ΅ Π²Π΅ΠΊΡ‚ΠΎΡ€Π° $\vc{a}=(a_1,a_2,a_3)$ Π² Ρ‚Ρ€Π΅Ρ…ΠΌΠ΅Ρ€Π½ΠΎΠΉ Π΄Π΅ΠΊΠ°Ρ€Ρ‚ΠΎΠ²ΠΎΠΉ систСмС ΠΊΠΎΠΎΡ€Π΄ΠΈΠ½Π°Ρ‚. Π’Π΅ΠΊΡ‚ΠΎΡ€ $\vc{a}$ изобраТаСтся Π² Π²ΠΈΠ΄Π΅ Π·Π΅Π»Π΅Π½ΠΎΠΉ стрСлки с хвостом, Π·Π°ΠΊΡ€Π΅ΠΏΠ»Π΅Π½Π½Ρ‹ΠΌ Π² Π½Π°Ρ‡Π°Π»Π΅ ΠΊΠΎΠΎΡ€Π΄ΠΈΠ½Π°Ρ‚. Π’Ρ‹ ΠΌΠΎΠΆΠ΅Ρ‚Π΅ ΠΏΠ΅Ρ€Π΅Ρ‚Π°Ρ‰ΠΈΡ‚ΡŒ ΠΌΡ‹ΡˆΠΊΠΎΠΉ ΠΊΠΎΠ½Ρ‡ΠΈΠΊ Π·Π΅Π»Π΅Π½ΠΎΠΉ стрСлки, Ρ‡Ρ‚ΠΎΠ±Ρ‹ ΠΈΠ·ΠΌΠ΅Π½ΠΈΡ‚ΡŒ Π²Π΅ΠΊΡ‚ΠΎΡ€. Π§Ρ‚ΠΎΠ±Ρ‹ ΠΏΠΎΠΊΠ°Π·Π°Ρ‚ΡŒ Ρ‚Ρ€Π΅Ρ…ΠΌΠ΅Ρ€Π½ΡƒΡŽ пСрспСктиву, Ρ€ΠΎΠ·ΠΎΠ²Ρ‹ΠΉ Ρ‚Ρ€Π΅ΡƒΠ³ΠΎΠ»ΡŒΠ½ΠΈΠΊ соСдиняСт Π²Π΅ΠΊΡ‚ΠΎΡ€ с Π΅Π³ΠΎ ΠΏΡ€ΠΎΠ΅ΠΊΡ†ΠΈΠ΅ΠΉ $(a_1,a_2,0)$ Π½Π° $xy$-ΠΏΠ»ΠΎΡΠΊΠΎΡΡ‚ΡŒ (сСрая стрСлка). Π€ΠΈΠΎΠ»Π΅Ρ‚ΠΎΠ²Ρ‹Π΅ Π²Π΅ΠΊΡ‚ΠΎΡ€Ρ‹ ΠΏΠΎΠΊΠ°Π·Ρ‹Π²Π°ΡŽΡ‚ ΠΏΡ€ΠΎΠ΅ΠΊΡ†ΠΈΠΈ $\vc{a}$ Π½Π° ΠΊΠ°ΠΆΠ΄ΡƒΡŽ ось ΠΈ ΠΏΡ€Π΅Π΄ΡΡ‚Π°Π²Π»ΡΡŽΡ‚ ΠΊΠΎΠΎΡ€Π΄ΠΈΠ½Π°Ρ‚Ρ‹ $a_1$, $a_2$ ΠΈ $a_3$. Π’Ρ‹ Ρ‚Π°ΠΊΠΆΠ΅ ΠΌΠΎΠΆΠ΅Ρ‚Π΅ ΠΏΠ΅Ρ€Π΅Ρ‚Π°ΡΠΊΠΈΠ²Π°Ρ‚ΡŒ Π³ΠΎΠ»ΠΎΠ²ΠΊΠΈ Ρ„ΠΈΠΎΠ»Π΅Ρ‚ΠΎΠ²Ρ‹Ρ… Π²Π΅ΠΊΡ‚ΠΎΡ€ΠΎΠ², Ρ‡Ρ‚ΠΎΠ±Ρ‹ ΠΈΠ·ΠΌΠ΅Π½ΠΈΡ‚ΡŒ Ρ‚ΠΎΠ»ΡŒΠΊΠΎ ΠΎΠ΄Π½Ρƒ ΠΈΠ· ΠΊΠΎΠΎΡ€Π΄ΠΈΠ½Π°Ρ‚ Π²Π΅ΠΊΡ‚ΠΎΡ€Π°. Или ΠΏΠ΅Ρ€Π΅Ρ‚Π°Ρ‰ΠΈΡ‚Π΅ Π²Π΅Ρ€ΡˆΠΈΠ½Ρƒ сСрого Π²Π΅ΠΊΡ‚ΠΎΡ€Π° Π² плоскости $xy$, Ρ‡Ρ‚ΠΎΠ±Ρ‹ ΠΈΠ·ΠΌΠ΅Π½ΠΈΡ‚ΡŒ Ρ‚ΠΎΠ»ΡŒΠΊΠΎ ΠΊΠΎΠΎΡ€Π΄ΠΈΠ½Π°Ρ‚Ρ‹ $x$ ΠΈ $y$.

      Π”ΠΎΠΏΠΎΠ»Π½ΠΈΡ‚Π΅Π»ΡŒΠ½Π°Ρ информация ΠΎΠ± Π°ΠΏΠΏΠ»Π΅Ρ‚Π΅.

      Π’Π°ΠΊ ΠΆΠ΅, ΠΊΠ°ΠΊ ΠΈ Π² Π΄Π²ΡƒΡ… измСрСниях, ΠΌΡ‹ Ρ‚Π°ΠΊΠΆΠ΅ ΠΌΠΎΠΆΠ΅ΠΌ ΠΎΠ±ΠΎΠ·Π½Π°Ρ‡Π°Ρ‚ΡŒ Ρ‚Ρ€Π΅Ρ…ΠΌΠ΅Ρ€Π½Ρ‹Π΅ Π²Π΅ΠΊΡ‚ΠΎΡ€Ρ‹ is Π² Ρ‚Π΅Ρ€ΠΌΠΈΠ½Π°Ρ… стандартных Π΅Π΄ΠΈΠ½ΠΈΡ‡Π½Ρ‹Ρ… Π²Π΅ΠΊΡ‚ΠΎΡ€ΠΎΠ² $\vc{i}$, $\vc{j}$ ΠΈ $\vc{k}$. Π­Ρ‚ΠΈ Π²Π΅ΠΊΡ‚ΠΎΡ€Ρ‹ ΡΠ²Π»ΡΡŽΡ‚ΡΡ Π΅Π΄ΠΈΠ½ΠΈΡ‡Π½Ρ‹ΠΌΠΈ Π²Π΅ΠΊΡ‚ΠΎΡ€Π°ΠΌΠΈ Π² ΠΏΠΎΠ»ΠΎΠΆΠΈΡ‚Π΅Π»ΡŒΠ½Ρ‹Ρ… направлСниях $x$, $y$ ΠΈ $z$ соотвСтствСнно. Π’ Ρ‚Π΅Ρ€ΠΌΠΈΠ½Π°Ρ… ΠΊΠΎΠΎΡ€Π΄ΠΈΠ½Π°Ρ‚ ΠΌΡ‹ ΠΌΠΎΠΆΠ΅ΠΌ Π·Π°ΠΏΠΈΡΠ°Ρ‚ΡŒ ΠΈΡ… ΠΊΠ°ΠΊ $\vc{i}=(1,0,0)$, $\vc{j}=(0,1,0)$ ΠΈ $\vc{k}= (0,0,1)$. ΠœΡ‹ ΠΌΠΎΠΆΠ΅ΠΌ Π²Ρ‹Ρ€Π°Π·ΠΈΡ‚ΡŒ любой Ρ‚Ρ€Π΅Ρ…ΠΌΠ΅Ρ€Π½Ρ‹ΠΉ Π²Π΅ΠΊΡ‚ΠΎΡ€ ΠΊΠ°ΠΊ сумму скалярных ΠΊΡ€Π°Ρ‚Π½Ρ‹Ρ… этих Π΅Π΄ΠΈΠ½ΠΈΡ‡Π½Ρ‹Ρ… Π²Π΅ΠΊΡ‚ΠΎΡ€ΠΎΠ² Π² Ρ„ΠΎΡ€ΠΌΠ΅ $\vc{a}=(a_1,a_2,a_3) = a_1\vc{i}+a_2\vc{j}+a_3\vc{k}$.

      Π—Π°Π³Ρ€ΡƒΠ·ΠΊΠ° Π°ΠΏΠΏΠ»Π΅Ρ‚Π°

      Π‘Ρ‚Π°Π½Π΄Π°Ρ€Ρ‚Π½Ρ‹Π΅ Π΅Π΄ΠΈΠ½ΠΈΡ‡Π½Ρ‹Π΅ Π²Π΅ΠΊΡ‚ΠΎΡ€Ρ‹ Π² Ρ‚Ρ€Π΅Ρ… измСрСниях. Π‘Ρ‚Π°Π½Π΄Π°Ρ€Ρ‚Π½Ρ‹Π΅ Π΅Π΄ΠΈΠ½ΠΈΡ‡Π½Ρ‹Π΅ Π²Π΅ΠΊΡ‚ΠΎΡ€Ρ‹ Π² Ρ‚Ρ€Π΅Ρ… измСрСниях, $\vc{i}$ (Π·Π΅Π»Π΅Π½Ρ‹ΠΉ), $\vc{j}$ (синий) ΠΈ $\vc{k}$ (красный), ΠΏΡ€Π΅Π΄ΡΡ‚Π°Π²Π»ΡΡŽΡ‚ собой Π²Π΅ΠΊΡ‚ΠΎΡ€Ρ‹ Π΄Π»ΠΈΠ½Ρ‹ ΠΎΠ΄ΠΈΠ½, ΠΊΠΎΡ‚ΠΎΡ€Ρ‹Π΅ ΡƒΠΊΠ°Π·Ρ‹Π²Π°ΡŽΡ‚ ΠΏΠ°Ρ€Π°Π»Π»Π΅Π»ΡŒΠ½ΠΎ ось $x$, ось $y$ ΠΈ ось $z$ соотвСтствСнно. ΠŸΠ΅Ρ€Π΅ΠΌΠ΅Ρ‰Π΅Π½ΠΈΠ΅ ΠΈΡ… с ΠΏΠΎΠΌΠΎΡ‰ΡŒΡŽ ΠΌΡ‹ΡˆΠΈ Π½Π΅ мСняСт Π²Π΅ΠΊΡ‚ΠΎΡ€Π°, ΠΏΠΎΡΠΊΠΎΠ»ΡŒΠΊΡƒ ΠΎΠ½ΠΈ всСгда ΡƒΠΊΠ°Π·Ρ‹Π²Π°ΡŽΡ‚ Π² ΠΏΠΎΠ»ΠΎΠΆΠΈΡ‚Π΅Π»ΡŒΠ½ΠΎΠΌ Π½Π°ΠΏΡ€Π°Π²Π»Π΅Π½ΠΈΠΈ ΡΠΎΠΎΡ‚Π²Π΅Ρ‚ΡΡ‚Π²ΡƒΡŽΡ‰Π΅ΠΉ оси.

      Π”ΠΎΠΏΠΎΠ»Π½ΠΈΡ‚Π΅Π»ΡŒΠ½Π°Ρ информация ΠΎΠ± Π°ΠΏΠΏΠ»Π΅Ρ‚Π΅.

      Какова Π΄Π»ΠΈΠ½Π° Π²Π΅ΠΊΡ‚ΠΎΡ€Π° $\vc{a}=(a_1,a_2,a_3)$? ΠœΡ‹ ΠΌΠΎΠΆΠ΅ΠΌ Ρ€Π°Π·Π»ΠΎΠΆΠΈΡ‚ΡŒ Π²Π΅ΠΊΡ‚ΠΎΡ€ Π½Π° $(a_1,a_2,a_3) = (a_1,a_2,0)+(0,0,a_3)$, Π³Π΄Π΅ Π΄Π²Π° Π²Π΅ΠΊΡ‚ΠΎΡ€Π° справа ΡΠΎΠΎΡ‚Π²Π΅Ρ‚ΡΡ‚Π²ΡƒΡŽΡ‚ Π΄Π²ΡƒΠΌ Π·Π΅Π»Π΅Π½Ρ‹ΠΌ сСгмСнтам Π»ΠΈΠ½ΠΈΠΈ Π² Π²Ρ‹ΡˆΠ΅ΡƒΠΊΠ°Π·Π°Π½Π½ΠΎΠΌ Π°ΠΏΠΏΠ»Π΅Ρ‚Π΅. 2}.$$ 9n$, Π³Π΄Π΅ $n$ β€” Π½Π΅ΠΊΠΎΡ‚ΠΎΡ€ΠΎΠ΅ Π½Π°Ρ‚ΡƒΡ€Π°Π»ΡŒΠ½ΠΎΠ΅ число.

      ΠŸΠ΅Ρ€Π΅ΠΉΡ‚ΠΈ Π² Π±ΠΎΠ»Π΅Π΅ высокоС ΠΈΠ·ΠΌΠ΅Ρ€Π΅Π½ΠΈΠ΅ Π»Π΅Π³ΠΊΠΎ со списками чисСл, хотя, ΠΊΠΎΠ½Π΅Ρ‡Π½ΠΎ ΠΌΠ½ΠΎΠ³ΠΎΠΌΠ΅Ρ€Π½Ρ‹Π΅ Π²Π΅ΠΊΡ‚ΠΎΡ€Ρ‹ Π½Π΅Π»Π΅Π³ΠΊΠΎ (Π½Π΅Π²ΠΎΠ·ΠΌΠΎΠΆΠ½ΠΎ?) Π²ΠΈΠ·ΡƒΠ°Π»ΠΈΠ·ΠΈΡ€ΠΎΠ²Π°Ρ‚ΡŒ. Π’Ρ‹ ΠΌΠΎΠΆΠ΅Ρ‚Π΅ ΠΏΡ€ΠΎΡ‡ΠΈΡ‚Π°Ρ‚ΡŒ большС ΠΎ ΠΌΠ½ΠΎΠ³ΠΎΠΌΠ΅Ρ€Π½Ρ‹Ρ… Π²Π΅ΠΊΡ‚ΠΎΡ€Π°Ρ… ΠΈΠ»ΠΈ ΠΏΡ€ΠΎΠ²Π΅Ρ€ΠΈΡ‚ΡŒ ΠΏΡ€ΠΈΠΌΠ΅Ρ€Ρ‹ $n$-ΠΌΠ΅Ρ€Π½Ρ‹Ρ… Π²Π΅ΠΊΡ‚ΠΎΡ€Ρ‹, ΠΊΠΎΡ‚ΠΎΡ€Ρ‹Π΅ ΠΈΠ»Π»ΡŽΡΡ‚Ρ€ΠΈΡ€ΡƒΡŽΡ‚, ΠΊΠ°ΠΊ ΡΠΎΠ±ΠΈΡ€Π°ΡŽΡ‚ΡΡ Ρ€Π°Π·ΠΌΠ΅Ρ€Ρ‹ число Π²Ρ‹ΡˆΠ΅ Ρ‚Ρ€Π΅Ρ… ΠΌΠΎΠΆΠ΅Ρ‚ Π±Ρ‹Ρ‚ΡŒ ΠΏΠΎΠ»Π΅Π·Π½Ρ‹ΠΌ Π²ΠΎ ΠΌΠ½ΠΎΠ³ΠΈΡ… ситуациях.

      `

      13.3: Π”Π»ΠΈΠ½Π° Π΄ΡƒΠ³ΠΈ ΠΈ ΠΊΡ€ΠΈΠ²ΠΈΠ·Π½Π°

      1. ПослСднСС обновлСниС
      2. Π‘ΠΎΡ…Ρ€Π°Π½ΠΈΡ‚ΡŒ ΠΊΠ°ΠΊ PDF
    • Π˜Π΄Π΅Π½Ρ‚ΠΈΡ„ΠΈΠΊΠ°Ρ‚ΠΎΡ€ страницы
      2596
      • Π“ΠΈΠ»Π±Π΅Ρ€Ρ‚ Бтрэнг ΠΈ Π­Π΄Π²ΠΈΠ½ Β«Π”ΠΆΠ΅Π΄Β» Π“Π΅Ρ€ΠΌΠ°Π½
      • OpenStax
      Π¦Π΅Π»ΠΈ обучСния
      • ΠžΠΏΡ€Π΅Π΄Π΅Π»ΠΈΡ‚Π΅ Π΄Π»ΠΈΠ½Ρƒ ΠΏΡƒΡ‚ΠΈ частицы Π² пространствС с ΠΏΠΎΠΌΠΎΡ‰ΡŒΡŽ Ρ„ΡƒΠ½ΠΊΡ†ΠΈΠΈ Π΄Π»ΠΈΠ½Ρ‹ Π΄ΡƒΠ³ΠΈ.
      • ΠžΠ±ΡŠΡΡΠ½ΠΈΡ‚Π΅ Π·Π½Π°Ρ‡Π΅Π½ΠΈΠ΅ ΠΊΡ€ΠΈΠ²ΠΈΠ·Π½Ρ‹ ΠΊΡ€ΠΈΠ²ΠΎΠΉ Π² пространствС ΠΈ Π½Π°Π·ΠΎΠ²ΠΈΡ‚Π΅ Π΅Π΅ Ρ„ΠΎΡ€ΠΌΡƒΠ»Ρƒ.
      • ΠžΠ±ΡŠΡΡΠ½ΠΈΡ‚Π΅, Ρ‡Ρ‚ΠΎ ΠΎΠ·Π½Π°Ρ‡Π°ΡŽΡ‚ Π²Π΅ΠΊΡ‚ΠΎΡ€Ρ‹ Π½ΠΎΡ€ΠΌΠ°Π»ΠΈ ΠΈ Π±ΠΈΠ½ΠΎΡ€ΠΌΠ°Π»ΠΈ ΠΊΡ€ΠΈΠ²ΠΎΠΉ Π² пространствС.

      Π’ этом Ρ€Π°Π·Π΄Π΅Π»Π΅ ΠΌΡ‹ ΠΈΠ·ΡƒΡ‡Π°Π΅ΠΌ Ρ„ΠΎΡ€ΠΌΡƒΠ»Ρ‹, относящиСся ΠΊ ΠΊΡ€ΠΈΠ²Ρ‹ΠΌ ΠΊΠ°ΠΊ Π² Π΄Π²ΡƒΡ…, Ρ‚Π°ΠΊ ΠΈ Π² Ρ‚Ρ€Π΅Ρ… измСрСниях, ΠΈ Π²ΠΈΠ΄ΠΈΠΌ, ΠΊΠ°ΠΊ ΠΎΠ½ΠΈ связаны с Ρ€Π°Π·Π»ΠΈΡ‡Π½Ρ‹ΠΌΠΈ свойствами ΠΎΠ΄Π½ΠΎΠΉ ΠΈ Ρ‚ΠΎΠΉ ΠΆΠ΅ ΠΊΡ€ΠΈΠ²ΠΎΠΉ. НапримСр, ΠΏΡ€Π΅Π΄ΠΏΠΎΠ»ΠΎΠΆΠΈΠΌ, Ρ‡Ρ‚ΠΎ Π²Π΅ΠΊΡ‚ΠΎΡ€-функция описываСт Π΄Π²ΠΈΠΆΠ΅Π½ΠΈΠ΅ частицы Π² пространствС. ΠœΡ‹ Ρ…ΠΎΡ‚Π΅Π»ΠΈ Π±Ρ‹ ΠΎΠΏΡ€Π΅Π΄Π΅Π»ΠΈΡ‚ΡŒ, ΠΊΠ°ΠΊΠΎΠ΅ расстояниС ΠΏΡ€ΠΎΡˆΠ»Π° частица Π·Π° Π·Π°Π΄Π°Π½Π½Ρ‹ΠΉ ΠΈΠ½Ρ‚Π΅Ρ€Π²Π°Π» Π²Ρ€Π΅ΠΌΠ΅Π½ΠΈ, ΠΊΠΎΡ‚ΠΎΡ€Ρ‹ΠΉ ΠΌΠΎΠΆΠ½ΠΎ ΠΎΠΏΠΈΡΠ°Ρ‚ΡŒ Π΄Π»ΠΈΠ½ΠΎΠΉ Π΄ΡƒΠ³ΠΈ ΠΏΡƒΡ‚ΠΈ, ΠΏΠΎ ΠΊΠΎΡ‚ΠΎΡ€ΠΎΠΌΡƒ ΠΎΠ½Π° слСдуСт. Или ΠΏΡ€Π΅Π΄ΠΏΠΎΠ»ΠΎΠΆΠΈΠΌ, Ρ‡Ρ‚ΠΎ Π²Π΅ΠΊΡ‚ΠΎΡ€-функция описываСт Π΄ΠΎΡ€ΠΎΠ³Ρƒ, ΠΊΠΎΡ‚ΠΎΡ€ΡƒΡŽ ΠΌΡ‹ строим, ΠΈ ΠΌΡ‹ Ρ…ΠΎΡ‚ΠΈΠΌ ΠΎΠΏΡ€Π΅Π΄Π΅Π»ΠΈΡ‚ΡŒ, насколько ΠΊΡ€ΡƒΡ‚ΠΎ изгибаСтся Π΄ΠΎΡ€ΠΎΠ³Π° Π² Π΄Π°Π½Π½ΠΎΠΉ Ρ‚ΠΎΡ‡ΠΊΠ΅. Π­Ρ‚ΠΎ описываСтся ΠΊΡ€ΠΈΠ²ΠΈΠ·Π½ΠΎΠΉ Ρ„ΡƒΠ½ΠΊΡ†ΠΈΠΈ Π² этой Ρ‚ΠΎΡ‡ΠΊΠ΅. Π’ этом Ρ€Π°Π·Π΄Π΅Π»Π΅ ΠΌΡ‹ исслСдуСм ΠΊΠ°ΠΆΠ΄ΡƒΡŽ ΠΈΠ· этих ΠΊΠΎΠ½Ρ†Π΅ΠΏΡ†ΠΈΠΉ. 9{b}_{a} \|\vecs rβ€²(t)\|dt . \label{Arc3D} \end{align} \]

      Π­Ρ‚ΠΈ Π΄Π²Π΅ Ρ„ΠΎΡ€ΠΌΡƒΠ»Ρ‹ ΠΎΡ‡Π΅Π½ΡŒ ΠΏΠΎΡ…ΠΎΠΆΠΈ; ΠΎΠ½ΠΈ ΠΎΡ‚Π»ΠΈΡ‡Π°ΡŽΡ‚ΡΡ Ρ‚ΠΎΠ»ΡŒΠΊΠΎ Ρ‚Π΅ΠΌ, Ρ‡Ρ‚ΠΎ пространствСнная кривая ΠΈΠΌΠ΅Π΅Ρ‚ Ρ‚Ρ€ΠΈ ΡΠΎΡΡ‚Π°Π²Π»ΡΡŽΡ‰ΠΈΠ΅ Ρ„ΡƒΠ½ΠΊΡ†ΠΈΠΈ вмСсто Π΄Π²ΡƒΡ…. ΠžΠ±Ρ€Π°Ρ‚ΠΈΡ‚Π΅ Π²Π½ΠΈΠΌΠ°Π½ΠΈΠ΅, Ρ‡Ρ‚ΠΎ Ρ„ΠΎΡ€ΠΌΡƒΠ»Ρ‹ ΠΎΠΏΡ€Π΅Π΄Π΅Π»Π΅Π½Ρ‹ для Π³Π»Π°Π΄ΠΊΠΈΡ… ΠΊΡ€ΠΈΠ²Ρ‹Ρ…: ΠΊΡ€ΠΈΠ²Ρ‹Ρ…, Π³Π΄Π΅ Π²Π΅ΠΊΡ‚ΠΎΡ€-функция \(\vecs r(t)\) Π΄ΠΈΡ„Ρ„Π΅Ρ€Π΅Π½Ρ†ΠΈΡ€ΡƒΠ΅ΠΌΠ° с Π½Π΅Π½ΡƒΠ»Π΅Π²ΠΎΠΉ ΠΏΡ€ΠΎΠΈΠ·Π²ΠΎΠ΄Π½ΠΎΠΉ. УсловиС гладкости Π³Π°Ρ€Π°Π½Ρ‚ΠΈΡ€ΡƒΠ΅Ρ‚, Ρ‡Ρ‚ΠΎ кривая Π½Π΅ ΠΈΠΌΠ΅Π΅Ρ‚ Ρ‚ΠΎΡ‡Π΅ΠΊ Π²ΠΎΠ·Π²Ρ€Π°Ρ‚Π° (ΠΈΠ»ΠΈ ΡƒΠ³Π»ΠΎΠ²), ΠΊΠΎΡ‚ΠΎΡ€Ρ‹Π΅ ΠΌΠΎΠ³Π»ΠΈ Π±Ρ‹ ΡΠ΄Π΅Π»Π°Ρ‚ΡŒ Ρ„ΠΎΡ€ΠΌΡƒΠ»Ρƒ ΠΏΡ€ΠΎΠ±Π»Π΅ΠΌΠ°Ρ‚ΠΈΡ‡Π½ΠΎΠΉ.

      ΠŸΡ€ΠΈΠΌΠ΅Ρ€ \(\PageIndex{1}\): ΠΎΠΏΡ€Π΅Π΄Π΅Π»Π΅Π½ΠΈΠ΅ Π΄Π»ΠΈΠ½Ρ‹ Π΄ΡƒΠ³ΠΈ 9{3/2})β‰ˆ37,785\) Π΅Π΄ΠΈΠ½ΠΈΡ†

      Π’Π΅ΠΏΠ΅Ρ€ΡŒ вСрнСмся ΠΊ спирали, прСдставлСнной Ρ€Π°Π½Π΅Π΅ Π² этой Π³Π»Π°Π²Π΅. Π’Π΅ΠΊΡ‚ΠΎΡ€-Ρ„ΡƒΠ½ΠΊΡ†ΠΈΡŽ, ΠΎΠΏΠΈΡΡ‹Π²Π°ΡŽΡ‰ΡƒΡŽ ΡΠΏΠΈΡ€Π°Π»ΡŒ, ΠΌΠΎΠΆΠ½ΠΎ Π·Π°ΠΏΠΈΡΠ°Ρ‚ΡŒ Π² Π²ΠΈΠ΄Π΅

      \[\vecs r(t)=R \cos \left(\dfrac{2Ο€Nt}{h}\right) \,\hat{\mathbf {i}} +R \sin\left(\dfrac{2Ο€Nt}{h}\right) \,\hat{\mathbf{j}}+t \,\hat{\mathbf{k}},0≀t ≀h, \nonumber \]

      Π³Π΄Π΅ \(R\) прСдставляСт радиус спирали, \(h\) прСдставляСт высоту (расстояниС ΠΌΠ΅ΠΆΠ΄Ρƒ двумя ΠΏΠΎΡΠ»Π΅Π΄ΠΎΠ²Π°Ρ‚Π΅Π»ΡŒΠ½Ρ‹ΠΌΠΈ Π²ΠΈΡ‚ΠΊΠ°ΠΌΠΈ), Π° ΡΠΏΠΈΡ€Π°Π»ΡŒ Π·Π°Π²Π΅Ρ€ΡˆΠ°Π΅Ρ‚ \(N\) Π²ΠΈΡ‚ΠΊΠΎΠ². Π”Π°Π²Π°ΠΉΡ‚Π΅ Π²Ρ‹Π²Π΅Π΄Π΅ΠΌ Ρ„ΠΎΡ€ΠΌΡƒΠ»Ρƒ для Π΄Π»ΠΈΠ½Ρ‹ Π΄ΡƒΠ³ΠΈ этой спирали, ΠΈΡΠΏΠΎΠ»ΡŒΠ·ΡƒΡ ΡƒΡ€Π°Π²Π½Π΅Π½ΠΈΠ΅ \ref{Arc3D}. Π’ΠΎ-ΠΏΠ΅Ρ€Π²Ρ‹Ρ…, 92}.\end{align*}\]

      Π­Ρ‚ΠΎ Π΄Π°Π΅Ρ‚ Ρ„ΠΎΡ€ΠΌΡƒΠ»Ρƒ для Π΄Π»ΠΈΠ½Ρ‹ ΠΏΡ€ΠΎΠ²ΠΎΠ΄Π°, Π½Π΅ΠΎΠ±Ρ…ΠΎΠ΄ΠΈΠΌΠΎΠΉ для формирования спирали с \(N\) Π²ΠΈΡ‚ΠΊΠ°ΠΌΠΈ, которая ΠΈΠΌΠ΅Π΅Ρ‚ радиус \(R\) ΠΈ высоту \(h\) .

      ΠŸΠ°Ρ€Π°ΠΌΠ΅Ρ‚Ρ€ΠΈΠ·Π°Ρ†ΠΈΡ Π΄Π»ΠΈΠ½Ρ‹ Π΄ΡƒΠ³ΠΈ

      Π’Π΅ΠΏΠ΅Ρ€ΡŒ Ρƒ нас Π΅ΡΡ‚ΡŒ Ρ„ΠΎΡ€ΠΌΡƒΠ»Π° для Π΄Π»ΠΈΠ½Ρ‹ Π΄ΡƒΠ³ΠΈ ΠΊΡ€ΠΈΠ²ΠΎΠΉ, опрСдСляСмой Π²Π΅ΠΊΡ‚ΠΎΡ€Π½ΠΎΠ·Π½Π°Ρ‡Π½ΠΎΠΉ Ρ„ΡƒΠ½ΠΊΡ†ΠΈΠ΅ΠΉ. Π”Π°Π²Π°ΠΉΡ‚Π΅ сдСлаСм Π΅Ρ‰Π΅ ΠΎΠ΄ΠΈΠ½ шаг Π²ΠΏΠ΅Ρ€Π΅Π΄ ΠΈ рассмотрим, Ρ‡Ρ‚ΠΎ Ρ‚Π°ΠΊΠΎΠ΅ функция Π΄Π»ΠΈΠ½Ρ‹ Π΄ΡƒΠ³ΠΈ .

      Если вСкторнозначная функция прСдставляСт ΠΏΠΎΠ»ΠΎΠΆΠ΅Π½ΠΈΠ΅ частицы Π² пространствС ΠΊΠ°ΠΊ Ρ„ΡƒΠ½ΠΊΡ†ΠΈΡŽ Π²Ρ€Π΅ΠΌΠ΅Π½ΠΈ, Ρ‚ΠΎ функция Π΄Π»ΠΈΠ½Ρ‹ Π΄ΡƒΠ³ΠΈ измСряСт, ΠΊΠ°ΠΊ Π΄Π°Π»Π΅ΠΊΠΎ эта частица ΠΏΡ€ΠΎΡ…ΠΎΠ΄ΠΈΡ‚ ΠΊΠ°ΠΊ Ρ„ΡƒΠ½ΠΊΡ†ΠΈΡŽ Π²Ρ€Π΅ΠΌΠ΅Π½ΠΈ. Π€ΠΎΡ€ΠΌΡƒΠ»Π° для Ρ„ΡƒΠ½ΠΊΡ†ΠΈΠΈ Π΄Π»ΠΈΠ½Ρ‹ Π΄ΡƒΠ³ΠΈ нСпосрСдствСнно слСдуСт ΠΈΠ· Ρ„ΠΎΡ€ΠΌΡƒΠ»Ρ‹ для Π΄Π»ΠΈΠ½Ρ‹ Π΄ΡƒΠ³ΠΈ: 92} дю. \label{arclength3} \]

      Если кривая двухмСрная, Ρ‚ΠΎ ΠΏΠΎΠ΄ ΠΊΠ²Π°Π΄Ρ€Π°Ρ‚Π½Ρ‹ΠΌ ΠΊΠΎΡ€Π½Π΅ΠΌ Π²Π½ΡƒΡ‚Ρ€ΠΈ ΠΈΠ½Ρ‚Π΅Π³Ρ€Π°Π»Π° ΠΏΠΎΡΠ²Π»ΡΡŽΡ‚ΡΡ Ρ‚ΠΎΠ»ΡŒΠΊΠΎ Π΄Π²Π° Ρ‡Π»Π΅Π½Π°. ΠŸΡ€ΠΈΡ‡ΠΈΠ½Π° использования нСзависимой ΠΏΠ΅Ρ€Π΅ΠΌΠ΅Π½Π½ΠΎΠΉ u состоит Π² Ρ‚ΠΎΠΌ, Ρ‡Ρ‚ΠΎΠ±Ρ‹ Ρ€Π°Π·Π»ΠΈΡ‡Π°Ρ‚ΡŒ врСмя ΠΈ ΠΏΠ΅Ρ€Π΅ΠΌΠ΅Π½Π½ΡƒΡŽ интСгрирования. ΠŸΠΎΡΠΊΠΎΠ»ΡŒΠΊΡƒ \(s(t)\) измСряСт ΠΏΡ€ΠΎΠΉΠ΄Π΅Π½Π½ΠΎΠ΅ расстояниС ΠΊΠ°ΠΊ Ρ„ΡƒΠ½ΠΊΡ†ΠΈΡŽ Π²Ρ€Π΅ΠΌΠ΅Π½ΠΈ, \(sβ€²(t)\) измСряСт ΡΠΊΠΎΡ€ΠΎΡΡ‚ΡŒ частицы Π² любой ΠΌΠΎΠΌΠ΅Π½Ρ‚ Π²Ρ€Π΅ΠΌΠ΅Π½ΠΈ. ΠŸΠΎΡΠΊΠΎΠ»ΡŒΠΊΡƒ Ρƒ нас Π΅ΡΡ‚ΡŒ Ρ„ΠΎΡ€ΠΌΡƒΠ»Π° для \(s(t)\) Π² ΡƒΡ€Π°Π²Π½Π΅Π½ΠΈΠΈ \ref{arclength3}, ΠΌΡ‹ ΠΌΠΎΠΆΠ΅ΠΌ Π΄ΠΈΡ„Ρ„Π΅Ρ€Π΅Π½Ρ†ΠΈΡ€ΠΎΠ²Π°Ρ‚ΡŒ ΠΎΠ±Π΅ части уравнСния: 9{t}_{a} β€–\vecs rβ€²(u)β€–\,du \nonumber \]

      ΠšΡ€ΠΎΠΌΠ΅ Ρ‚ΠΎΠ³ΠΎ,

      \[\dfrac{ds}{dt}=β€–\vecs rβ€²(t)β€–> 0. \nonumber \]

      Если \(β€–\vecs rβ€²(t)β€–=1\) для всСх \(tβ‰₯a\), Ρ‚ΠΎ ΠΏΠ°Ρ€Π°ΠΌΠ΅Ρ‚Ρ€ \(t\) прСдставляСт Π΄Π»ΠΈΠ½Ρƒ Π΄ΡƒΠ³ΠΈ ΠΎΡ‚ Π½Π°Ρ‡Π°Π»ΡŒΠ½ΠΎΠΉ Ρ‚ΠΎΡ‡ΠΊΠΈ Π² \ (Ρ‚=Π°\).

      ΠŸΠΎΠ»Π΅Π·Π½Ρ‹ΠΌ ΠΏΡ€ΠΈΠΌΠ΅Π½Π΅Π½ΠΈΠ΅ΠΌ этой Ρ‚Π΅ΠΎΡ€Π΅ΠΌΡ‹ являСтся Π½Π°Ρ…ΠΎΠΆΠ΄Π΅Π½ΠΈΠ΅ Π°Π»ΡŒΡ‚Π΅Ρ€Π½Π°Ρ‚ΠΈΠ²Π½ΠΎΠΉ ΠΏΠ°Ρ€Π°ΠΌΠ΅Ρ‚Ρ€ΠΈΠ·Π°Ρ†ΠΈΠΈ Π΄Π°Π½Π½ΠΎΠΉ ΠΊΡ€ΠΈΠ²ΠΎΠΉ, Π½Π°Π·Ρ‹Π²Π°Π΅ΠΌΠΎΠΉ ΠΏΠ°Ρ€Π°ΠΌΠ΅Ρ‚Ρ€ΠΈΠ·Π°Ρ†ΠΈΠ΅ΠΉ Π΄Π»ΠΈΠ½Ρ‹ Π΄ΡƒΠ³ΠΈ . Напомним, Ρ‡Ρ‚ΠΎ Π»ΡŽΠ±ΡƒΡŽ Π²Π΅ΠΊΡ‚ΠΎΡ€-Ρ„ΡƒΠ½ΠΊΡ†ΠΈΡŽ ΠΌΠΎΠΆΠ½ΠΎ ΠΏΠ΅Ρ€Π΅ΠΏΠ°Ρ€Π°ΠΌΠ΅Ρ‚Ρ€ΠΈΡ€ΠΎΠ²Π°Ρ‚ΡŒ Π·Π°ΠΌΠ΅Π½ΠΎΠΉ ΠΏΠ΅Ρ€Π΅ΠΌΠ΅Π½Π½Ρ‹Ρ…. НапримСр, Ссли Ρƒ нас Π΅ΡΡ‚ΡŒ функция \(\vecs r(t)=⟨3 \cos t,3 \sin t⟩,0≀t≀2Ο€\), которая ΠΏΠ°Ρ€Π°ΠΌΠ΅Ρ‚Ρ€ΠΈΠ·ΡƒΠ΅Ρ‚ ΠΎΠΊΡ€ΡƒΠΆΠ½ΠΎΡΡ‚ΡŒ радиуса 3, ΠΌΡ‹ ΠΌΠΎΠΆΠ΅ΠΌ ΠΈΠ·ΠΌΠ΅Π½ΠΈΡ‚ΡŒ ΠΏΠ°Ρ€Π°ΠΌΠ΅Ρ‚Ρ€ с \(t\) Π² \(4t\), ΠΏΠΎΠ»ΡƒΡ‡ΠΈΠ² Π½ΠΎΠ²ΡƒΡŽ ΠΏΠ°Ρ€Π°ΠΌΠ΅Ρ‚Ρ€ΠΈΠ·Π°Ρ†ΠΈΡŽ \(\vecs r(t)=⟨3 \cos 4t,3 \sin 4t⟩\). Новая парамСтризация ΠΏΠΎ-ΠΏΡ€Π΅ΠΆΠ½Π΅ΠΌΡƒ опрСдСляСт ΠΎΠΊΡ€ΡƒΠΆΠ½ΠΎΡΡ‚ΡŒ радиуса 3, Π½ΠΎ Ρ‚Π΅ΠΏΠ΅Ρ€ΡŒ Π½Π°ΠΌ Π½ΡƒΠΆΠ½ΠΎ ΠΈΡΠΏΠΎΠ»ΡŒΠ·ΠΎΠ²Π°Ρ‚ΡŒ Ρ‚ΠΎΠ»ΡŒΠΊΠΎ значСния \(0≀t≀π/2\), Ρ‡Ρ‚ΠΎΠ±Ρ‹ ΠΏΡ€ΠΎΠΉΡ‚ΠΈ ΠΊΡ€ΡƒΠ³ ΠΎΠ΄ΠΈΠ½ Ρ€Π°Π·.

      ΠŸΡ€Π΅Π΄ΠΏΠΎΠ»ΠΎΠΆΠΈΠΌ, Ρ‡Ρ‚ΠΎ ΠΌΡ‹ нашли Ρ„ΡƒΠ½ΠΊΡ†ΠΈΡŽ Π΄Π»ΠΈΠ½Ρ‹ Π΄ΡƒΠ³ΠΈ \(s(t)\) ΠΈ ΠΌΠΎΠΆΠ΅ΠΌ Ρ€Π΅ΡˆΠΈΡ‚ΡŒ эту Ρ„ΡƒΠ½ΠΊΡ†ΠΈΡŽ для \(t\) ΠΊΠ°ΠΊ Ρ„ΡƒΠ½ΠΊΡ†ΠΈΡŽ ΠΎΡ‚ \(s\) . Π—Π°Ρ‚Π΅ΠΌ ΠΌΡ‹ ΠΌΠΎΠΆΠ΅ΠΌ ΠΏΠ΅Ρ€Π΅ΠΏΠ°Ρ€Π°ΠΌΠ΅Ρ‚Ρ€ΠΈΡ€ΠΎΠ²Π°Ρ‚ΡŒ ΠΈΡΡ…ΠΎΠ΄Π½ΡƒΡŽ Ρ„ΡƒΠ½ΠΊΡ†ΠΈΡŽ \(\vecs r(t)\), подставив Π²Ρ‹Ρ€Π°ΠΆΠ΅Π½ΠΈΠ΅ для \(t\) ΠΎΠ±Ρ€Π°Ρ‚Π½ΠΎ Π² \(\vecs r(t)\). Π’Π΅ΠΏΠ΅Ρ€ΡŒ Π²Π΅ΠΊΡ‚ΠΎΡ€-функция записываСтся Π² Ρ‚Π΅Ρ€ΠΌΠΈΠ½Π°Ρ… ΠΏΠ°Ρ€Π°ΠΌΠ΅Ρ‚Ρ€Π° \(s\) . ΠŸΠΎΡΠΊΠΎΠ»ΡŒΠΊΡƒ пСрСмСнная \(s\) прСдставляСт Π΄Π»ΠΈΠ½Ρƒ Π΄ΡƒΠ³ΠΈ, ΠΌΡ‹ Π½Π°Π·Ρ‹Π²Π°Π΅ΠΌ это ΠΏΠ°Ρ€Π°ΠΌΠ΅Ρ‚Ρ€ΠΈΠ·Π°Ρ†ΠΈΠ΅ΠΉ Π΄Π»ΠΈΠ½Ρ‹ Π΄ΡƒΠ³ΠΈ исходной Ρ„ΡƒΠ½ΠΊΡ†ΠΈΠΈ \(\vecs r(t)\). Одним ΠΈΠ· прСимущСств нахоТдСния ΠΏΠ°Ρ€Π°ΠΌΠ΅Ρ‚Ρ€ΠΈΠ·Π°Ρ†ΠΈΠΈ Π΄Π»ΠΈΠ½Ρ‹ Π΄ΡƒΠ³ΠΈ являСтся Ρ‚ΠΎ, Ρ‡Ρ‚ΠΎ расстояниС, ΠΏΡ€ΠΎΠΉΠ΄Π΅Π½Π½ΠΎΠ΅ вдоль ΠΊΡ€ΠΈΠ²ΠΎΠΉ, начиная с \(s=0\), Ρ‚Π΅ΠΏΠ΅Ρ€ΡŒ Ρ€Π°Π²Π½ΠΎ ΠΏΠ°Ρ€Π°ΠΌΠ΅Ρ‚Ρ€Ρƒ \(s\). ΠŸΠ°Ρ€Π°ΠΌΠ΅Ρ‚Ρ€ΠΈΠ·Π°Ρ†ΠΈΡ Π΄Π»ΠΈΠ½Ρ‹ Π΄ΡƒΠ³ΠΈ Ρ‚Π°ΠΊΠΆΠ΅ появляСтся Π² контСкстС ΠΊΡ€ΠΈΠ²ΠΈΠ·Π½Ρ‹ (ΠΊΠΎΡ‚ΠΎΡ€ΡƒΡŽ ΠΌΡ‹ рассмотрим ΠΏΠΎΠ·ΠΆΠ΅ Π² этом Ρ€Π°Π·Π΄Π΅Π»Π΅) ΠΈ Π»ΠΈΠ½Π΅ΠΉΠ½Ρ‹Ρ… ΠΈΠ½Ρ‚Π΅Π³Ρ€Π°Π»ΠΎΠ².

      ΠŸΡ€ΠΈΠΌΠ΅Ρ€ \(\PageIndex{2}\): поиск ΠΏΠ°Ρ€Π°ΠΌΠ΅Ρ‚Ρ€ΠΈΠ·Π°Ρ†ΠΈΠΈ Π΄Π»ΠΈΠ½Ρ‹ Π΄ΡƒΠ³ΠΈ

      НайдитС ΠΏΠ°Ρ€Π°ΠΌΠ΅Ρ‚Ρ€ΠΈΠ·Π°Ρ†ΠΈΡŽ Π΄Π»ΠΈΠ½Ρ‹ Π΄ΡƒΠ³ΠΈ для ΠΊΠ°ΠΆΠ΄ΠΎΠΉ ΠΈΠ· ΡΠ»Π΅Π΄ΡƒΡŽΡ‰ΠΈΡ… ΠΊΡ€ΠΈΠ²Ρ‹Ρ…:

      1. \(\vecs r(t)=4 \cos t \,\hat{\mathbf{i}}+ 4 \sin t \,\hat{\mathbf{j}},\quad tβ‰₯0\)
      2. \(\vecs r(t)=⟨t+3,2tβˆ’4,2t⟩,\quad tβ‰₯3\)

      РСшСниС

      1. Π‘Π½Π°Ρ‡Π°Π»Π° Π½Π°ΠΉΠ΄Π΅ΠΌ Ρ„ΡƒΠ½ΠΊΡ†ΠΈΡŽ Π΄Π»ΠΈΠ½Ρ‹ Π΄ΡƒΠ³ΠΈ, ΠΈΡΠΏΠΎΠ»ΡŒΠ·ΡƒΡ ΡƒΡ€Π°Π²Π½Π΅Π½ΠΈΠ΅ \ref{arclength3}:

        \[\begin{align*} s(t) &= \int_a^t β€–\vecs rβ€²(u)β€– \,du \\[4pt] &= \int_0^t β€–βŸ¨βˆ’4 \sin u, 4 \cos uβŸ©β€– \,du \\[4pt] &= \int_0^t \sqrt{(βˆ’4 \sin u)^2+(4 \cos u)^2} \,du \\[4pt] &= \int_0^t \sqrt{16 \sin ^2 u+16 \cos ^2 u} \,du \\[4pt] &= \int_0^t 4\,du = 4t, \end{align*} \]

      2. , ΠΊΠΎΡ‚ΠΎΡ€Ρ‹ΠΉ Π΄Π°Π΅Ρ‚ связь ΠΌΠ΅ΠΆΠ΄Ρƒ Π΄Π»ΠΈΠ½ΠΎΠΉ Π΄ΡƒΠ³ΠΈ \(s\) ΠΈ ΠΏΠ°Ρ€Π°ΠΌΠ΅Ρ‚Ρ€ΠΎΠΌ \(t\) ΠΊΠ°ΠΊ \(s=4t;\), Ρ‚ΠΎ Π΅ΡΡ‚ΡŒ \(t=s/4\). Π”Π°Π»Π΅Π΅ замСняСм ΠΏΠ΅Ρ€Π΅ΠΌΠ΅Π½Π½ΡƒΡŽ \(t\) Π² исходной Ρ„ΡƒΠ½ΠΊΡ†ΠΈΠΈ \(\vecs r(t)=4 \cos t \,\hat{\mathbf{i}}+4 \sin t \,\hat{\mathbf {j}}\) с Π²Ρ‹Ρ€Π°ΠΆΠ΅Π½ΠΈΠ΅ΠΌ \(s/4\), Ρ‡Ρ‚ΠΎΠ±Ρ‹ ΠΏΠΎΠ»ΡƒΡ‡ΠΈΡ‚ΡŒ

        \[\vecs r(s)=4 \cos\left(\frac{s}{4}\right) \,\hat{\mathbf{i}} + 4 \sin\left(\frac{s} {4}\справа) \,\шляпа{\mathbf{j}}. \Π½ΠΎΠΌΠ΅Ρ€\]

        Π­Ρ‚ΠΎ парамСтризация Π΄Π»ΠΈΠ½Ρ‹ Π΄ΡƒΠ³ΠΈ \(\vecs r(t)\). ΠŸΠΎΡΠΊΠΎΠ»ΡŒΠΊΡƒ исходноС ΠΎΠ³Ρ€Π°Π½ΠΈΡ‡Π΅Π½ΠΈΠ΅ Π½Π° \(t\) Π±Ρ‹Π»ΠΎ Π·Π°Π΄Π°Π½ΠΎ Π²Ρ‹Ρ€Π°ΠΆΠ΅Π½ΠΈΠ΅ΠΌ \(tβ‰₯0\), ΠΎΠ³Ρ€Π°Π½ΠΈΡ‡Π΅Π½ΠΈΠ΅ Π½Π° 9t 3 \,du \\[4pt] &= 3t — 9. \end{align*}\]

        Π‘Π»Π΅Π΄ΠΎΠ²Π°Ρ‚Π΅Π»ΡŒΠ½ΠΎ, связь ΠΌΠ΅ΠΆΠ΄Ρƒ Π΄Π»ΠΈΠ½ΠΎΠΉ Π΄ΡƒΠ³ΠΈ \(s\) ΠΈ ΠΏΠ°Ρ€Π°ΠΌΠ΅Ρ‚Ρ€ΠΎΠΌ \(t\) Ρ€Π°Π²Π½Π° \(s=3tβˆ’9\), поэтому \(t= \frac{s}{3}+3\). ΠŸΠΎΠ΄ΡΡ‚Π°Π½ΠΎΠ²ΠΊΠ° этого Π² ΠΈΡΡ…ΠΎΠ΄Π½ΡƒΡŽ Ρ„ΡƒΠ½ΠΊΡ†ΠΈΡŽ \(\vecs r(t)=⟨t+3,2tβˆ’4,2t⟩ \) Π΄Π°Π΅Ρ‚

        \[\vecs r(s)=⟨\left(\frac{s}{3}+3\right)+3,\,2\left(\frac{s}{3}+3\right)βˆ’ 4,\,2\Π²Π»Π΅Π²ΠΎ(\frac{s}{3}+3\Π²ΠΏΡ€Π°Π²ΠΎ)⟩=⟨\frac{s}{3}+6, \frac{2s}{3}+2,\frac{2s {3}+6⟩.\Π½ΠΎΠΌΠ΅Ρ€ \]

        Π­Ρ‚ΠΎ парамСтризация Π΄Π»ΠΈΠ½Ρ‹ Π΄ΡƒΠ³ΠΈ \(\vecs r(t)\). Π˜ΡΡ…ΠΎΠ΄Π½ΠΎΠ΅ ΠΎΠ³Ρ€Π°Π½ΠΈΡ‡Π΅Π½ΠΈΠ΅ Π½Π° ΠΏΠ°Ρ€Π°ΠΌΠ΅Ρ‚Ρ€ \(t\) Π±Ρ‹Π»ΠΎ \(tβ‰₯3\), поэтому ΠΎΠ³Ρ€Π°Π½ΠΈΡ‡Π΅Π½ΠΈΠ΅ Π½Π° \(s\) Ρ€Π°Π²Π½ΠΎ \((s/3)+3β‰₯3\) ΠΈΠ»ΠΈ \(sβ‰₯0 \).
      Π£ΠΏΡ€Π°ΠΆΠ½Π΅Π½ΠΈΠ΅ \(\PageIndex{2}\)

      НайдитС Ρ„ΡƒΠ½ΠΊΡ†ΠΈΡŽ Π΄Π»ΠΈΠ½Ρ‹ Π΄ΡƒΠ³ΠΈ для спирали

      \[\vecs r(t)=⟨3 \cos t, 3 \sin t,4t⟩,\quad Ρ‚β‰₯0. \nonumber \]

      Π—Π°Ρ‚Π΅ΠΌ ΠΈΡΠΏΠΎΠ»ΡŒΠ·ΡƒΠΉΡ‚Π΅ связь ΠΌΠ΅ΠΆΠ΄Ρƒ Π΄Π»ΠΈΠ½ΠΎΠΉ Π΄ΡƒΠ³ΠΈ ΠΈ ΠΏΠ°Ρ€Π°ΠΌΠ΅Ρ‚Ρ€ΠΎΠΌ \(t\), Ρ‡Ρ‚ΠΎΠ±Ρ‹ Π½Π°ΠΉΡ‚ΠΈ ΠΏΠ°Ρ€Π°ΠΌΠ΅Ρ‚Ρ€ΠΈΠ·Π°Ρ†ΠΈΡŽ Π΄Π»ΠΈΠ½Ρ‹ Π΄ΡƒΠ³ΠΈ для \(\vecs r(t)\).

      Подсказка

      НачнитС с нахоТдСния Ρ„ΡƒΠ½ΠΊΡ†ΠΈΠΈ Π΄Π»ΠΈΠ½Ρ‹ Π΄ΡƒΠ³ΠΈ.

      ΠžΡ‚Π²Π΅Ρ‚ΠΈΡ‚ΡŒ

      \(s=5t\) ΠΈΠ»ΠΈ \(t=s/5\). ΠŸΠΎΠ΄ΡΡ‚Π°Π½ΠΎΠ²ΠΊΠ° этого Π² \(\vecs r(t)=⟨3 \cos t,3 \sin t,4t⟩\) Π΄Π°Π΅Ρ‚

      \[\vecs r(s)=⟨3 \cos\left(\frac{s}{5}\right),3 \sin\left(\frac{s}{5}\right),\frac{ 4s}{5}⟩,\quad sβ‰₯0 \nonumber \]

        ΠšΡ€ΠΈΠ²ΠΈΠ·Π½Π°

        ΠšΡ€ΠΈΠ²ΠΈΠ·Π½Π° являСтся Π²Π°ΠΆΠ½ΠΎΠΉ Ρ‚Π΅ΠΌΠΎΠΉ, связанной с Π΄Π»ΠΈΠ½ΠΎΠΉ Π΄ΡƒΠ³ΠΈ. ΠŸΠΎΠ½ΡΡ‚ΠΈΠ΅ ΠΊΡ€ΠΈΠ²ΠΈΠ·Π½Ρ‹ позволяСт ΠΈΠ·ΠΌΠ΅Ρ€ΠΈΡ‚ΡŒ, насколько Ρ€Π΅Π·ΠΊΠΎ ΠΏΠΎΠ²ΠΎΡ€Π°Ρ‡ΠΈΠ²Π°Π΅Ρ‚ гладкая кривая. ΠžΠΊΡ€ΡƒΠΆΠ½ΠΎΡΡ‚ΡŒ ΠΈΠΌΠ΅Π΅Ρ‚ ΠΏΠΎΡΡ‚ΠΎΡΠ½Π½ΡƒΡŽ ΠΊΡ€ΠΈΠ²ΠΈΠ·Π½Ρƒ. Π§Π΅ΠΌ мСньшС радиус окруТности, Ρ‚Π΅ΠΌ большС ΠΊΡ€ΠΈΠ²ΠΈΠ·Π½Π°.

        ΠŸΡ€Π΅Π΄ΡΡ‚Π°Π²ΡŒΡ‚Π΅, Ρ‡Ρ‚ΠΎ Π²Ρ‹ Π΅Π΄Π΅Ρ‚Π΅ ΠΏΠΎ Π΄ΠΎΡ€ΠΎΠ³Π΅. ΠŸΡ€Π΅Π΄ΠΏΠΎΠ»ΠΎΠΆΠΈΠΌ, Ρ‡Ρ‚ΠΎ Π΄ΠΎΡ€ΠΎΠ³Π° Π»Π΅ΠΆΠΈΡ‚ Π½Π° Π΄ΡƒΠ³Π΅ большого ΠΊΡ€ΡƒΠ³Π°. Π’ этом случаС Π²Π°ΠΌ Π΅Π΄Π²Π° Π»ΠΈ придСтся ΠΊΡ€ΡƒΡ‚ΠΈΡ‚ΡŒ Ρ€ΡƒΠ»ΡŒ, Ρ‡Ρ‚ΠΎΠ±Ρ‹ ΠΎΡΡ‚Π°Π²Π°Ρ‚ΡŒΡΡ Π½Π° Π΄ΠΎΡ€ΠΎΠ³Π΅. Π’Π΅ΠΏΠ΅Ρ€ΡŒ ΠΏΡ€Π΅Π΄ΠΏΠΎΠ»ΠΎΠΆΠΈΠΌ, Ρ‡Ρ‚ΠΎ радиус мСньшС. Π’ этом случаС Π²Π°ΠΌ Π½ΡƒΠΆΠ½ΠΎ Π±ΡƒΠ΄Π΅Ρ‚ ΠΏΠΎΠ²ΠΎΡ€Π°Ρ‡ΠΈΠ²Π°Ρ‚ΡŒ Π±ΠΎΠ»Π΅Π΅ Ρ€Π΅Π·ΠΊΠΎ, Ρ‡Ρ‚ΠΎΠ±Ρ‹ ΠΎΡΡ‚Π°Ρ‚ΡŒΡΡ Π½Π° Π΄ΠΎΡ€ΠΎΠ³Π΅. Π’ случаС ΠΊΡ€ΠΈΠ²ΠΎΠΉ, ΠΎΡ‚Π»ΠΈΡ‡Π½ΠΎΠΉ ΠΎΡ‚ окруТности, часто ΠΏΠΎΠ»Π΅Π·Π½ΠΎ сначала Π²ΠΏΠΈΡΠ°Ρ‚ΡŒ ΠΎΠΊΡ€ΡƒΠΆΠ½ΠΎΡΡ‚ΡŒ Π² ΠΊΡ€ΠΈΠ²ΡƒΡŽ Π² Π΄Π°Π½Π½ΠΎΠΉ Ρ‚ΠΎΡ‡ΠΊΠ΅ Ρ‚Π°ΠΊ, Ρ‡Ρ‚ΠΎΠ±Ρ‹ ΠΎΠ½Π° касалась ΠΊΡ€ΠΈΠ²ΠΎΠΉ Π² этой Ρ‚ΠΎΡ‡ΠΊΠ΅ ΠΈ Β«ΠΎΠ±Π½ΠΈΠΌΠ°Π»Π°Β» ΠΊΡ€ΠΈΠ²ΡƒΡŽ ΠΊΠ°ΠΊ ΠΌΠΎΠΆΠ½ΠΎ тСснСС Π² Π·Π°Π΄Π°Π½Π½ΠΎΠΉ Ρ‚ΠΎΡ‡ΠΊΠ΅. окрСстности Ρ‚ΠΎΡ‡ΠΊΠΈ (рис. \(\PageIndex{1}\)). Π—Π°Ρ‚Π΅ΠΌ ΠΊΡ€ΠΈΠ²ΠΈΠ·Π½Π° Π³Ρ€Π°Ρ„ΠΈΠΊΠ° Π² этой Ρ‚ΠΎΡ‡ΠΊΠ΅ опрСдСляСтся ΠΊΠ°ΠΊ такая ΠΆΠ΅, ΠΊΠ°ΠΊ ΠΊΡ€ΠΈΠ²ΠΈΠ·Π½Π° вписанной окруТности.

        Рисунок \(\PageIndex{1}\): Π“Ρ€Π°Ρ„ΠΈΠΊ прСдставляСт ΠΊΡ€ΠΈΠ²ΠΈΠ·Π½Ρƒ Ρ„ΡƒΠ½ΠΊΡ†ΠΈΠΈ \(y=f(x).\) Π§Π΅ΠΌ ΠΊΡ€ΡƒΡ‡Π΅ ΠΏΠΎΠ²ΠΎΡ€ΠΎΡ‚ Π½Π° Π³Ρ€Π°Ρ„ΠΈΠΊΠ΅, Ρ‚Π΅ΠΌ большС ΠΊΡ€ΠΈΠ²ΠΈΠ·Π½Π° ΠΈ мСньшС радиус вписанный ΠΊΡ€ΡƒΠ³.

        ΠžΠΏΡ€Π΅Π΄Π΅Π»Π΅Π½ΠΈΠ΅: ΠΊΡ€ΠΈΠ²ΠΈΠ·Π½Π°

        ΠŸΡƒΡΡ‚ΡŒ \(C\) β€” гладкая кривая Π½Π° плоскости ΠΈΠ»ΠΈ Π² пространствС, заданная Ρ„ΠΎΡ€ΠΌΡƒΠ»ΠΎΠΉ \(\vecs r(s)\), Π³Π΄Π΅ \(s\) β€” ΠΏΠ°Ρ€Π°ΠΌΠ΅Ρ‚Ρ€ Π΄Π»ΠΈΠ½Ρ‹ Π΄ΡƒΠ³ΠΈ. ΠšΡ€ΠΈΠ²ΠΈΠ·Π½Π° \(ΞΊ\) Π² Ρ‚ΠΎΡ‡ΠΊΠ΅ \(s\) Ρ€Π°Π²Π½Π°

        \[ΞΊ =\bigg{\|}\dfrac{d\vecs{T}}{ds}\bigg{\|}=β€–\vecs T Β«(с)Β». \nonumber \]

        ΠŸΠΎΡΠΌΠΎΡ‚Ρ€ΠΈΡ‚Π΅ это Π²ΠΈΠ΄Π΅ΠΎ, Ρ‡Ρ‚ΠΎΠ±Ρ‹ ΡƒΠ·Π½Π°Ρ‚ΡŒ большС ΠΎ ΠΊΡ€ΠΈΠ²ΠΈΠ·Π½Π΅ пространствСнной ΠΊΡ€ΠΈΠ²ΠΎΠΉ.

        Π€ΠΎΡ€ΠΌΡƒΠ»Π° опрСдСлСния ΠΊΡ€ΠΈΠ²ΠΈΠ·Π½Ρ‹ Π½Π΅ ΠΎΡ‡Π΅Π½ΡŒ ΠΏΠΎΠ»Π΅Π·Π½Π° с Ρ‚ΠΎΡ‡ΠΊΠΈ зрСния вычислСний. Π’ частности, Π½Π°ΠΏΠΎΠΌΠ½ΠΈΠΌ, Ρ‡Ρ‚ΠΎ \(\vecs T(t)\) прСдставляСт собой Π΅Π΄ΠΈΠ½ΠΈΡ‡Π½Ρ‹ΠΉ ΠΊΠ°ΡΠ°Ρ‚Π΅Π»ΡŒΠ½Ρ‹ΠΉ Π²Π΅ΠΊΡ‚ΠΎΡ€ ΠΊ Π·Π°Π΄Π°Π½Π½ΠΎΠΉ Π²Π΅ΠΊΡ‚ΠΎΡ€-Ρ„ΡƒΠ½ΠΊΡ†ΠΈΠΈ \(\vecs r(t)\), Π° Ρ„ΠΎΡ€ΠΌΡƒΠ»Π° для \(\vecs T(t) \) Ρ€Π°Π²Π½ΠΎ

        \[\vecs T(t)=\frac{\vecs rβ€²(t)}{βˆ₯\vecs rβ€²(t)βˆ₯}. \nonumber \]

        Π§Ρ‚ΠΎΠ±Ρ‹ ΠΈΡΠΏΠΎΠ»ΡŒΠ·ΠΎΠ²Π°Ρ‚ΡŒ Ρ„ΠΎΡ€ΠΌΡƒΠ»Ρƒ для ΠΊΡ€ΠΈΠ²ΠΈΠ·Π½Ρ‹, сначала Π½Π΅ΠΎΠ±Ρ…ΠΎΠ΄ΠΈΠΌΠΎ Π²Ρ‹Ρ€Π°Π·ΠΈΡ‚ΡŒ \(\vecs r(t)\) Ρ‡Π΅Ρ€Π΅Π· ΠΏΠ°Ρ€Π°ΠΌΠ΅Ρ‚Ρ€ Π΄Π»ΠΈΠ½Ρ‹ Π΄ΡƒΠ³ΠΈ \(s\), Π·Π°Ρ‚Π΅ΠΌ Π½Π°ΠΉΡ‚ΠΈ Π΅Π΄ΠΈΠ½ΠΈΡ‡Π½Ρ‹ΠΉ ΠΊΠ°ΡΠ°Ρ‚Π΅Π»ΡŒΠ½Ρ‹ΠΉ Π²Π΅ΠΊΡ‚ΠΎΡ€ \ (\vecs T(s)\) для Ρ„ΡƒΠ½ΠΊΡ†ΠΈΠΈ \(\vecs r(s)\), Ρ‚ΠΎ возьмСм ΠΏΡ€ΠΎΠΈΠ·Π²ΠΎΠ΄Π½ΡƒΡŽ ΠΎΡ‚ \(\vecs T(s)\) ΠΏΠΎ \(s\). Π­Ρ‚ΠΎ ΡƒΡ‚ΠΎΠΌΠΈΡ‚Π΅Π»ΡŒΠ½Ρ‹ΠΉ процСсс. К ΡΡ‡Π°ΡΡ‚ΡŒΡŽ, ΡΡƒΡ‰Π΅ΡΡ‚Π²ΡƒΡŽΡ‚ эквивалСнтныС Ρ„ΠΎΡ€ΠΌΡƒΠ»Ρ‹ для ΠΊΡ€ΠΈΠ²ΠΈΠ·Π½Ρ‹. 9{3/2}}.\label{EqK4} \]

        Π”ΠΎΠΊΠ°Π·Π°Ρ‚Π΅Π»ΡŒΡΡ‚Π²ΠΎ

        ΠŸΠ΅Ρ€Π²Π°Ρ Ρ„ΠΎΡ€ΠΌΡƒΠ»Π° нСпосрСдствСнно слСдуСт ΠΈΠ· Ρ†Π΅ΠΏΠ½ΠΎΠ³ΠΎ ΠΏΡ€Π°Π²ΠΈΠ»Π°:

        \[\dfrac{d\vecs{T}}{dt} = \dfrac{ d\vecs{T}}{ds} \dfrac{ds}{dt}, \nonumber \]

        , Π³Π΄Π΅ \(s\) — Π΄Π»ΠΈΠ½Π° Π΄ΡƒΠ³ΠΈ вдоль ΠΊΡ€ΠΈΠ²ΠΎΠΉ \(C\). Π Π°Π·Π΄Π΅Π»ΠΈΠ² ΠΎΠ±Π΅ части Π½Π° \(ds/dt\) ΠΈ взяв Π²Π΅Π»ΠΈΡ‡ΠΈΠ½Ρƒ ΠΎΠ±Π΅ΠΈΡ… сторон, ΠΌΡ‹ ΠΏΠΎΠ»ΡƒΡ‡ΠΈΠΌ

        \[\bigg{\|}\dfrac{d\vecs{T}}{ds}\bigg{\|}= \left\lVert\frac{\vecs T'(t)}{\dfrac{ds}{dt}}\right\rVert.\nonumber \]

        Π’Π°ΠΊ ΠΊΠ°ΠΊ \(ds/dt=β€–\vecs r'(t )β€–\), это Π΄Π°Π΅Ρ‚ Ρ„ΠΎΡ€ΠΌΡƒΠ»Ρƒ ΠΊΡ€ΠΈΠ²ΠΈΠ·Π½Ρ‹ \(ΞΊ\) ΠΊΡ€ΠΈΠ²ΠΎΠΉ \(C\) Π² Ρ‚Π΅Ρ€ΠΌΠΈΠ½Π°Ρ… любой ΠΏΠ°Ρ€Π°ΠΌΠ΅Ρ‚Ρ€ΠΈΠ·Π°Ρ†ΠΈΠΈ \(C\): 93}.\nonumber \]

        Π­Ρ‚ΠΎ Π΄ΠΎΠΊΠ°Π·Ρ‹Π²Π°Π΅Ρ‚ \(\ref{EqK3}\). Π§Ρ‚ΠΎΠ±Ρ‹ Π΄ΠΎΠΊΠ°Π·Π°Ρ‚ΡŒ \(\ref{EqK4}\), ΠΌΡ‹ Π½Π°Ρ‡Π½Π΅ΠΌ с прСдполоТСния, Ρ‡Ρ‚ΠΎ кривая \(C\) опрСдСляСтся Ρ„ΡƒΠ½ΠΊΡ†ΠΈΠ΅ΠΉ \(y=f(x)\). Π—Π°Ρ‚Π΅ΠΌ ΠΌΡ‹ ΠΌΠΎΠΆΠ΅ΠΌ ΠΎΠΏΡ€Π΅Π΄Π΅Π»ΠΈΡ‚ΡŒ \(\vecs r(t)=x \,\hat{\mathbf{i}}+f(x) \,\hat{\mathbf{j}}+0 \,\hat{\ mathbf{k}}\). Π˜ΡΠΏΠΎΠ»ΡŒΠ·ΡƒΡ ΠΏΡ€Π΅Π΄Ρ‹Π΄ΡƒΡ‰ΡƒΡŽ Ρ„ΠΎΡ€ΠΌΡƒΠ»Ρƒ для ΠΊΡ€ΠΈΠ²ΠΈΠ·Π½Ρ‹:

        \[\begin{align*} \vecs r'(t) &=\,\hat{\mathbf{i}}+f'(x)\,\hat{\mathbf {j}} \\[4pt] \vecs rβ€³(t) &=fβ€³(x)\,\hat{\mathbf{j}} \\[4pt] \vecs rβ€²(t)Γ—\vecs r β€³(t) &= \begin{vmatrix} \hat{\mathbf{i}} & \hat{\mathbf{j}} & \hat{\mathbf{k}} \\ 1 & f'(x) & 0 \\ 0 & f»(x) & 0 \end{vmatrix} =f»(x)\,\hat{\mathbf{k}}. \end{Π²Ρ‹Ρ€Π°Π²Π½ΠΈΠ²Π°Π½ΠΈΠ΅*}\] 9{3/2}}β‰ˆ0,0059\)

          ΠΠΎΡ€ΠΌΠ°Π»ΡŒΠ½Ρ‹ΠΉ ΠΈ Π±ΠΈΠ½ΠΎΡ€ΠΌΠ°Π»ΡŒΠ½Ρ‹ΠΉ Π²Π΅ΠΊΡ‚ΠΎΡ€Ρ‹

          ΠœΡ‹ Π²ΠΈΠ΄Π΅Π»ΠΈ, Ρ‡Ρ‚ΠΎ производная \(\vecs rβ€²(t)\) Π²Π΅ΠΊΡ‚ΠΎΡ€-Ρ„ΡƒΠ½ΠΊΡ†ΠΈΠΈ являСтся ΠΊΠ°ΡΠ°Ρ‚Π΅Π»ΡŒΠ½Ρ‹ΠΌ Π²Π΅ΠΊΡ‚ΠΎΡ€ΠΎΠΌ ΠΊ ΠΊΡ€ΠΈΠ²ΠΎΠΉ, Π·Π°Π΄Π°Π½Π½ΠΎΠΉ \(\vecs r(t) \), Π° Π΅Π΄ΠΈΠ½ΠΈΡ‡Π½Ρ‹ΠΉ ΠΊΠ°ΡΠ°Ρ‚Π΅Π»ΡŒΠ½Ρ‹ΠΉ Π²Π΅ΠΊΡ‚ΠΎΡ€ \(\vecs T(t)\) ΠΌΠΎΠΆΠ½ΠΎ Ρ€Π°ΡΡΡ‡ΠΈΡ‚Π°Ρ‚ΡŒ, Ρ€Π°Π·Π΄Π΅Π»ΠΈΠ² \(\vecs rβ€²(t)\) Π½Π° Π΅Π³ΠΎ Π²Π΅Π»ΠΈΡ‡ΠΈΠ½Ρƒ. ΠŸΡ€ΠΈ ΠΈΠ·ΡƒΡ‡Π΅Π½ΠΈΠΈ двиТСния Π² Ρ‚Ρ€Π΅Ρ… измСрСниях для описания двиТСния частицы ΠΏΠΎ Ρ‚Ρ€Π°Π΅ΠΊΡ‚ΠΎΡ€ΠΈΠΈ Π² пространствС ΠΏΠΎΠ»Π΅Π·Π½Ρ‹ Π΄Π²Π° Π΄Ρ€ΡƒΠ³ΠΈΡ… Π²Π΅ΠΊΡ‚ΠΎΡ€Π°: Π³Π»Π°Π²Π½Ρ‹ΠΉ Π΅Π΄ΠΈΠ½ΠΈΡ‡Π½Ρ‹ΠΉ Π²Π΅ΠΊΡ‚ΠΎΡ€ Π½ΠΎΡ€ΠΌΠ°Π»ΠΈ ΠΈ Π²Π΅ΠΊΡ‚ΠΎΡ€ Π±ΠΈΠ½ΠΎΡ€ΠΌΠ°Π»ΡŒΠ½Ρ‹ΠΉ Π²Π΅ΠΊΡ‚ΠΎΡ€ .

          ΠžΠΏΡ€Π΅Π΄Π΅Π»Π΅Π½ΠΈΠ΅: Π±ΠΈΠ½ΠΎΡ€ΠΌΠ°Π»ΡŒΠ½Ρ‹Π΅ Π²Π΅ΠΊΡ‚ΠΎΡ€Ρ‹

          ΠŸΡƒΡΡ‚ΡŒ \(C\) — трСхмСрная гладкая кривая, прСдставлСнная \(\vecs r\) Π½Π° ΠΎΡ‚ΠΊΡ€Ρ‹Ρ‚ΠΎΠΌ ΠΈΠ½Ρ‚Π΅Ρ€Π²Π°Π»Π΅ \(I\) . Если \(\vecs Tβ€²(t)β‰ \vecs 0\), Ρ‚ΠΎ Π³Π»Π°Π²Π½Ρ‹ΠΉ Π΅Π΄ΠΈΠ½ΠΈΡ‡Π½Ρ‹ΠΉ Π²Π΅ΠΊΡ‚ΠΎΡ€ Π½ΠΎΡ€ΠΌΠ°Π»ΠΈ Π² Ρ‚ΠΎΡ‡ΠΊΠ΅ \(t\) опрСдСляСтся ΠΊΠ°ΠΊ

          \[\vecs N(t)=\dfrac{\vecs Tβ€²(t)}{β€–\vecs Tβ€²(t)β€–}. \label{EqNormal} \]

          Π’Π΅ΠΊΡ‚ΠΎΡ€ Π±ΠΈΠ½ΠΎΡ€ΠΌΠ°Π»Π΅ΠΉ Π² Ρ‚ΠΎΡ‡ΠΊΠ΅ \(t\) опрСдСляСтся ΠΊΠ°ΠΊ

          \[\vecs B(t)=\vecs T(t)Γ—\vecs N(t),\label{EqBinormal } \]

          Π³Π΄Π΅ \(\vecs T(t)\) β€” Π΅Π΄ΠΈΠ½ΠΈΡ‡Π½Ρ‹ΠΉ ΠΊΠ°ΡΠ°Ρ‚Π΅Π»ΡŒΠ½Ρ‹ΠΉ Π²Π΅ΠΊΡ‚ΠΎΡ€.

          ΠžΠ±Ρ€Π°Ρ‚ΠΈΡ‚Π΅ Π²Π½ΠΈΠΌΠ°Π½ΠΈΠ΅, Ρ‡Ρ‚ΠΎ Π±ΠΈΠ½ΠΎΡ€ΠΌΠ°Π»ΠΈ ΠΏΠΎ ΠΎΠΏΡ€Π΅Π΄Π΅Π»Π΅Π½ΠΈΡŽ ΠΎΡ€Ρ‚ΠΎΠ³ΠΎΠ½Π°Π»ΡŒΠ½Ρ‹ ΠΊΠ°ΠΊ Π΅Π΄ΠΈΠ½ΠΈΡ‡Π½ΠΎΠΌΡƒ ΠΊΠ°ΡΠ°Ρ‚Π΅Π»ΡŒΠ½ΠΎΠΌΡƒ Π²Π΅ΠΊΡ‚ΠΎΡ€Ρƒ, Ρ‚Π°ΠΊ ΠΈ Π²Π΅ΠΊΡ‚ΠΎΡ€Ρƒ Π½ΠΎΡ€ΠΌΠ°Π»ΠΈ. ΠšΡ€ΠΎΠΌΠ΅ Ρ‚ΠΎΠ³ΠΎ, \(\vecs B(t)\) всСгда являСтся Π΅Π΄ΠΈΠ½ΠΈΡ‡Π½Ρ‹ΠΌ Π²Π΅ΠΊΡ‚ΠΎΡ€ΠΎΠΌ. Π­Ρ‚ΠΎ ΠΌΠΎΠΆΠ½ΠΎ ΠΏΠΎΠΊΠ°Π·Π°Ρ‚ΡŒ, ΠΈΡΠΏΠΎΠ»ΡŒΠ·ΡƒΡ Ρ„ΠΎΡ€ΠΌΡƒΠ»Ρƒ для Π²Π΅Π»ΠΈΡ‡ΠΈΠ½Ρ‹ Π²Π΅ΠΊΡ‚ΠΎΡ€Π½ΠΎΠ³ΠΎ произвСдСния.

          \[β€–\vecs B(t)β€–=β€–\vecs T(t)Γ—\vecs N(t)β€–=β€–\vecs T(t)β€–β€–\vecs N(t)β€– \sin \theta, \nonumber \]

          , Π³Π΄Π΅ \(\theta\) β€” ΡƒΠ³ΠΎΠ» ΠΌΠ΅ΠΆΠ΄Ρƒ \(\vecs T(t)\) ΠΈ \(\vecs N(t)\). ΠŸΠΎΡΠΊΠΎΠ»ΡŒΠΊΡƒ \(\vecs N(t)\) являСтся ΠΏΡ€ΠΎΠΈΠ·Π²ΠΎΠ΄Π½ΠΎΠΉ Π΅Π΄ΠΈΠ½ΠΈΡ‡Π½ΠΎΠ³ΠΎ Π²Π΅ΠΊΡ‚ΠΎΡ€Π°, свойство (vii) ΠΏΡ€ΠΎΠΈΠ·Π²ΠΎΠ΄Π½ΠΎΠΉ Π²Π΅ΠΊΡ‚ΠΎΡ€-Ρ„ΡƒΠ½ΠΊΡ†ΠΈΠΈ Π³ΠΎΠ²ΠΎΡ€ΠΈΡ‚ Π½Π°ΠΌ, Ρ‡Ρ‚ΠΎ \(\vecs T(t)\) ΠΈ \(\vecs N(t)\) ΠΎΡ€Ρ‚ΠΎΠ³ΠΎΠ½Π°Π»ΡŒΠ½Ρ‹ Π΄Ρ€ΡƒΠ³ Π΄Ρ€ΡƒΠ³Ρƒ, поэтому \(\theta=Ο€/2\). ΠšΡ€ΠΎΠΌΠ΅ Ρ‚ΠΎΠ³ΠΎ, ΠΎΠ½ΠΈ ΠΎΠ±Π° ΡΠ²Π»ΡΡŽΡ‚ΡΡ Π΅Π΄ΠΈΠ½ΠΈΡ‡Π½Ρ‹ΠΌΠΈ Π²Π΅ΠΊΡ‚ΠΎΡ€Π°ΠΌΠΈ, поэтому ΠΈΡ… Π²Π΅Π»ΠΈΡ‡ΠΈΠ½Π° Ρ€Π°Π²Π½Π° 1. Π‘Π»Π΅Π΄ΠΎΠ²Π°Ρ‚Π΅Π»ΡŒΠ½ΠΎ, \(β€–\vecs T(t)β€–β€–\vecs N(t)β€– \sin \theta=(1)(1) \sin (Ο€/ 2)=1\) ΠΈ \(\vecs B(t)\) — Π΅Π΄ΠΈΠ½ΠΈΡ‡Π½Ρ‹ΠΉ Π²Π΅ΠΊΡ‚ΠΎΡ€.

          Π“Π»Π°Π²Π½Ρ‹ΠΉ Π΅Π΄ΠΈΠ½ΠΈΡ‡Π½Ρ‹ΠΉ Π²Π΅ΠΊΡ‚ΠΎΡ€ Π½ΠΎΡ€ΠΌΠ°Π»ΠΈ ΠΌΠΎΠΆΠ΅Ρ‚ Π±Ρ‹Ρ‚ΡŒ слоТным для вычислСния, ΠΏΠΎΡΠΊΠΎΠ»ΡŒΠΊΡƒ Π΅Π΄ΠΈΠ½ΠΈΡ‡Π½Ρ‹ΠΉ Π²Π΅ΠΊΡ‚ΠΎΡ€ ΠΊΠ°ΡΠ°Ρ‚Π΅Π»ΡŒΠ½ΠΎΠΉ Π²ΠΊΠ»ΡŽΡ‡Π°Π΅Ρ‚ частноС, Π° это частноС часто ΠΈΠΌΠ΅Π΅Ρ‚ ΠΊΠ²Π°Π΄Ρ€Π°Ρ‚Π½Ρ‹ΠΉ ΠΊΠΎΡ€Π΅Π½ΡŒ Π² Π·Π½Π°ΠΌΠ΅Π½Π°Ρ‚Π΅Π»Π΅. Π’ Ρ‚Ρ€Π΅Ρ…ΠΌΠ΅Ρ€Π½ΠΎΠΌ случаС Π½Π°Ρ…ΠΎΠΆΠ΄Π΅Π½ΠΈΠ΅ Π²Π΅ΠΊΡ‚ΠΎΡ€Π½ΠΎΠ³ΠΎ произвСдСния Π΅Π΄ΠΈΠ½ΠΈΡ‡Π½ΠΎΠ³ΠΎ ΠΊΠ°ΡΠ°Ρ‚Π΅Π»ΡŒΠ½ΠΎΠ³ΠΎ Π²Π΅ΠΊΡ‚ΠΎΡ€Π° ΠΈ Π΅Π΄ΠΈΠ½ΠΈΡ‡Π½ΠΎΠ³ΠΎ Π²Π΅ΠΊΡ‚ΠΎΡ€Π° Π½ΠΎΡ€ΠΌΠ°Π»ΠΈ ΠΌΠΎΠΆΠ΅Ρ‚ Π±Ρ‹Ρ‚ΡŒ Π΅Ρ‰Π΅ Π±ΠΎΠ»Π΅Π΅ Π³Ρ€ΠΎΠΌΠΎΠ·Π΄ΠΊΠΈΠΌ. К ΡΡ‡Π°ΡΡ‚ΡŒΡŽ, Ρƒ нас Π΅ΡΡ‚ΡŒ Π°Π»ΡŒΡ‚Π΅Ρ€Π½Π°Ρ‚ΠΈΠ²Π½Ρ‹Π΅ Ρ„ΠΎΡ€ΠΌΡƒΠ»Ρ‹ для нахоТдСния этих Π΄Π²ΡƒΡ… Π²Π΅ΠΊΡ‚ΠΎΡ€ΠΎΠ², ΠΈ ΠΎΠ½ΠΈ прСдставлСны Π² Ρ€Π°Π·Π΄Π΅Π»Π΅ Β«Π”Π²ΠΈΠΆΠ΅Π½ΠΈΠ΅ Π² пространствС».

          ΠŸΡ€ΠΈΠΌΠ΅Ρ€ \(\PageIndex{4}\): Π½Π°Ρ…ΠΎΠΆΠ΄Π΅Π½ΠΈΠ΅ Π²Π΅ΠΊΡ‚ΠΎΡ€Π° Π½ΠΎΡ€ΠΌΠ°Π»ΠΈ ΠΊ Π³Π»Π°Π²Π½ΠΎΠΉ Π΅Π΄ΠΈΠ½ΠΈΡ†Π΅ ΠΈ Π²Π΅ΠΊΡ‚ΠΎΡ€Π° Π±ΠΈΠ½ΠΎΡ€ΠΌΠ°Π»ΠΈ 92 t}} \\[4pt]

          &=βˆ’ \cos t\,\hat{\mathbf{i}}+ \sin t\,\hat{\mathbf{j}}. \end{align*}\]

          ΠžΠ±Ρ€Π°Ρ‚ΠΈΡ‚Π΅ Π²Π½ΠΈΠΌΠ°Π½ΠΈΠ΅, Ρ‡Ρ‚ΠΎ Π΅Π΄ΠΈΠ½ΠΈΡ‡Π½Ρ‹ΠΉ Π²Π΅ΠΊΡ‚ΠΎΡ€ ΠΊΠ°ΡΠ°Ρ‚Π΅Π»ΡŒΠ½ΠΎΠΉ ΠΈ Π³Π»Π°Π²Π½Ρ‹ΠΉ Π΅Π΄ΠΈΠ½ΠΈΡ‡Π½Ρ‹ΠΉ Π²Π΅ΠΊΡ‚ΠΎΡ€ Π½ΠΎΡ€ΠΌΠ°Π»ΠΈ ΠΎΡ€Ρ‚ΠΎΠ³ΠΎΠ½Π°Π»ΡŒΠ½Ρ‹ Π΄Ρ€ΡƒΠ³ Π΄Ρ€ΡƒΠ³Ρƒ для всСх Π·Π½Π°Ρ‡Π΅Π½ΠΈΠΉ \(t\):

          \[\begin{align*} \vecs T (t)Β·\vecs N(t) &=βŸ¨βˆ’ \sin t,βˆ’ \cos tβŸ©Β·βŸ¨βˆ’ \cos t, \sin t⟩ \\[4pt] &= \sin t \cos tβˆ’\cos Ρ‚ \sin Ρ‚ \\[4pt] &=0. \end{align*}\]

          ΠšΡ€ΠΎΠΌΠ΅ Ρ‚ΠΎΠ³ΠΎ, Π³Π»Π°Π²Π½Ρ‹ΠΉ Π΅Π΄ΠΈΠ½ΠΈΡ‡Π½Ρ‹ΠΉ Π²Π΅ΠΊΡ‚ΠΎΡ€ Π½ΠΎΡ€ΠΌΠ°Π»ΠΈ ΡƒΠΊΠ°Π·Ρ‹Π²Π°Π΅Ρ‚ ΠΊ Ρ†Π΅Π½Ρ‚Ρ€Ρƒ окруТности ΠΈΠ· ΠΊΠ°ΠΆΠ΄ΠΎΠΉ Ρ‚ΠΎΡ‡ΠΊΠΈ окруТности. ΠŸΠΎΡΠΊΠΎΠ»ΡŒΠΊΡƒ \(\vecs r(t)\) опрСдСляСт ΠΊΡ€ΠΈΠ²ΡƒΡŽ Π² Π΄Π²ΡƒΡ… измСрСниях, ΠΌΡ‹ Π½Π΅ ΠΌΠΎΠΆΠ΅ΠΌ Π²Ρ‹Ρ‡ΠΈΡΠ»ΠΈΡ‚ΡŒ Π²Π΅ΠΊΡ‚ΠΎΡ€ Π±ΠΈΠ½ΠΎΡ€ΠΌΠ°Π»Π΅ΠΉ. 92βˆ’3t)\,\hat{\mathbf{i}}+(4t+1)\,\hat{\mathbf{j}}\) ΠΈ ΠΎΡ†Π΅Π½ΠΈΡ‚ΡŒ Π΅Π³ΠΎ ΠΏΡ€ΠΈ \(t=2\).

          Подсказка

          Π‘Π½Π°Ρ‡Π°Π»Π° Π½Π°ΠΉΠ΄ΠΈΡ‚Π΅ \(\vecs T(t)\), Π·Π°Ρ‚Π΅ΠΌ ΠΈΡΠΏΠΎΠ»ΡŒΠ·ΡƒΠΉΡ‚Π΅ \(\ref{EqNormal}\).

          ΠžΡ‚Π²Π΅Ρ‚ΠΈΡ‚ΡŒ

          \(\vecs N(2)=\dfrac{\sqrt{2}}{2}(\,\hat{\mathbf{i}}βˆ’\,\hat{\mathbf{j}})\)

          Для любой Π³Π»Π°Π΄ΠΊΠΎΠΉ ΠΊΡ€ΠΈΠ²ΠΎΠΉ Π² Ρ‚Ρ€Π΅Ρ… измСрСниях, Π·Π°Π΄Π°Π½Π½ΠΎΠΉ Π²Π΅ΠΊΡ‚ΠΎΡ€-Ρ„ΡƒΠ½ΠΊΡ†ΠΈΠ΅ΠΉ, Ρ‚Π΅ΠΏΠ΅Ρ€ΡŒ Ρƒ нас Π΅ΡΡ‚ΡŒ Ρ„ΠΎΡ€ΠΌΡƒΠ»Ρ‹ для Π΅Π΄ΠΈΠ½ΠΈΡ‡Π½ΠΎΠ³ΠΎ ΠΊΠ°ΡΠ°Ρ‚Π΅Π»ΡŒΠ½ΠΎΠ³ΠΎ Π²Π΅ΠΊΡ‚ΠΎΡ€Π° \(\vecs T\), Π΅Π΄ΠΈΠ½ΠΈΡ‡Π½ΠΎΠ³ΠΎ Π²Π΅ΠΊΡ‚ΠΎΡ€Π° Π½ΠΎΡ€ΠΌΠ°Π»ΠΈ \(\vecs N\) ΠΈ Π±ΠΈΠ½ΠΎΡ€ΠΌΠ°Π»ΡŒΠ½Ρ‹ΠΉ Π²Π΅ΠΊΡ‚ΠΎΡ€ \(\vecs B\). Π•Π΄ΠΈΠ½ΠΈΡ‡Π½Ρ‹ΠΉ Π²Π΅ΠΊΡ‚ΠΎΡ€ Π½ΠΎΡ€ΠΌΠ°Π»ΠΈ ΠΈ Π²Π΅ΠΊΡ‚ΠΎΡ€ Π±ΠΈΠ½ΠΎΡ€ΠΌΠ°Π»ΠΈ ΠΎΠ±Ρ€Π°Π·ΡƒΡŽΡ‚ ΠΏΠ»ΠΎΡΠΊΠΎΡΡ‚ΡŒ, ΠΏΠ΅Ρ€ΠΏΠ΅Π½Π΄ΠΈΠΊΡƒΠ»ΡΡ€Π½ΡƒΡŽ ΠΊΡ€ΠΈΠ²ΠΎΠΉ Π² любой Ρ‚ΠΎΡ‡ΠΊΠ΅ ΠΊΡ€ΠΈΠ²ΠΎΠΉ, Π½Π°Π·Ρ‹Π²Π°Π΅ΠΌΡƒΡŽ ΠΏΠ»ΠΎΡΠΊΠΎΡΡ‚ΡŒΡŽ Π½ΠΎΡ€ΠΌΠ°Π»ΠΈ. ΠšΡ€ΠΎΠΌΠ΅ Ρ‚ΠΎΠ³ΠΎ, эти Ρ‚Ρ€ΠΈ Π²Π΅ΠΊΡ‚ΠΎΡ€Π° ΠΎΠ±Ρ€Π°Π·ΡƒΡŽΡ‚ систСму отсчСта Π² Ρ‚Ρ€Π΅Ρ…ΠΌΠ΅Ρ€Π½ΠΎΠΌ пространствС, Π½Π°Π·Ρ‹Π²Π°Π΅ΠΌΡƒΡŽ 9.0008 БистСма ΠΊΠΎΠΎΡ€Π΄ΠΈΠ½Π°Ρ‚ Π€Ρ€Π΅Π½Π΅ (Ρ‚Π°ΠΊΠΆΠ΅ называСмая систСмой ΠΊΠΎΠΎΡ€Π΄ΠΈΠ½Π°Ρ‚ TNB ) (рис. \(\PageIndex{2}\)). НаконСц, ΠΏΠ»ΠΎΡΠΊΠΎΡΡ‚ΡŒ, опрСдСляСмая Π²Π΅ΠΊΡ‚ΠΎΡ€Π°ΠΌΠΈ \(\vecs T\) ΠΈ \(\vecs N\), ΠΎΠ±Ρ€Π°Π·ΡƒΠ΅Ρ‚ ΡΠΎΠΏΡ€ΠΈΠΊΠ°ΡΠ°ΡŽΡ‰ΡƒΡŽΡΡ ΠΏΠ»ΠΎΡΠΊΠΎΡΡ‚ΡŒ \(C\) Π² любой Ρ‚ΠΎΡ‡ΠΊΠ΅ \(P\) Π½Π° ΠΊΡ€ΠΈΠ²ΠΎΠΉ.

          Рисунок \(\PageIndex{2}\): На этом рисункС ΠΏΠΎΠΊΠ°Π·Π°Π½Π° систСма отсчСта Π€Ρ€Π΅Π½Π΅. Π’ ΠΊΠ°ΠΆΠ΄ΠΎΠΉ Ρ‚ΠΎΡ‡ΠΊΠ΅ \(P\) Π½Π° Ρ‚Ρ€Π΅Ρ…ΠΌΠ΅Ρ€Π½ΠΎΠΉ ΠΊΡ€ΠΈΠ²ΠΎΠΉ Π²Π΅ΠΊΡ‚ΠΎΡ€Π° Π΅Π΄ΠΈΠ½ΠΈΡ‡Π½ΠΎΠΉ ΠΊΠ°ΡΠ°Ρ‚Π΅Π»ΡŒΠ½ΠΎΠΉ, Π΅Π΄ΠΈΠ½ΠΈΡ‡Π½ΠΎΠΉ Π½ΠΎΡ€ΠΌΠ°Π»ΠΈ ΠΈ Π±ΠΈΠ½ΠΎΡ€ΠΌΠ°Π»ΠΈ ΠΎΠ±Ρ€Π°Π·ΡƒΡŽΡ‚ Ρ‚Ρ€Π΅Ρ…ΠΌΠ΅Ρ€Π½ΡƒΡŽ систСму отсчСта.

          ΠŸΡ€Π΅Π΄ΠΏΠΎΠ»ΠΎΠΆΠΈΠΌ, ΠΌΡ‹ ΠΎΠ±Ρ€Π°Π·ΡƒΠ΅ΠΌ ΠΎΠΊΡ€ΡƒΠΆΠ½ΠΎΡΡ‚ΡŒ Π² ΡΠΎΠΏΡ€ΠΈΠΊΠ°ΡΠ°ΡŽΡ‰Π΅ΠΉΡΡ плоскости \(Π‘\) Π² Ρ‚ΠΎΡ‡ΠΊΠ΅ \(Π \) Π½Π° ΠΊΡ€ΠΈΠ²ΠΎΠΉ. ΠŸΡ€Π΅Π΄ΠΏΠΎΠ»ΠΎΠΆΠΈΠΌ, Ρ‡Ρ‚ΠΎ ΠΎΠΊΡ€ΡƒΠΆΠ½ΠΎΡΡ‚ΡŒ ΠΈΠΌΠ΅Π΅Ρ‚ Ρ‚Ρƒ ΠΆΠ΅ ΠΊΡ€ΠΈΠ²ΠΈΠ·Π½Ρƒ, Ρ‡Ρ‚ΠΎ ΠΈ кривая Π² Ρ‚ΠΎΡ‡ΠΊΠ΅ \(P\), ΠΈ ΠΏΡƒΡΡ‚ΡŒ ΠΎΠΊΡ€ΡƒΠΆΠ½ΠΎΡΡ‚ΡŒ ΠΈΠΌΠ΅Π΅Ρ‚ радиус \(r\). Π’ΠΎΠ³Π΄Π° ΠΊΡ€ΠΈΠ²ΠΈΠ·Π½Π° ΠΊΡ€ΡƒΠ³Π° опрСдСляСтся ΠΊΠ°ΠΊ \(\frac{1}{r}\). ΠœΡ‹ Π½Π°Π·Ρ‹Π²Π°Π΅ΠΌ \(r\) радиусом ΠΊΡ€ΠΈΠ²ΠΈΠ·Π½Ρ‹ ΠΊΡ€ΠΈΠ²ΠΎΠΉ, ΠΈ ΠΎΠ½ Ρ€Π°Π²Π΅Π½ ΠΎΠ±Ρ€Π°Ρ‚Π½ΠΎΠΉ Π²Π΅Π»ΠΈΡ‡ΠΈΠ½Π΅ ΠΊΡ€ΠΈΠ²ΠΈΠ·Π½Ρ‹. Если эта ΠΎΠΊΡ€ΡƒΠΆΠ½ΠΎΡΡ‚ΡŒ Π»Π΅ΠΆΠΈΡ‚ Π½Π° Π²ΠΎΠ³Π½ΡƒΡ‚ΠΎΠΉ сторонС ΠΊΡ€ΠΈΠ²ΠΎΠΉ ΠΈ касаСтся ΠΊΡ€ΠΈΠ²ΠΎΠΉ Π² Ρ‚ΠΎΡ‡ΠΊΠ΅ \(P\), Ρ‚ΠΎ эта ΠΎΠΊΡ€ΡƒΠΆΠ½ΠΎΡΡ‚ΡŒ называСтся ΡΠΎΠΏΡ€ΠΈΠΊΠ°ΡΠ°ΡŽΡ‰ΠΈΠΉΡΡ ΠΊΡ€ΡƒΠ³ ΠΈΠ· \(C\) Π² Ρ‚ΠΎΡ‡ΠΊΠ΅ \(P\), ΠΊΠ°ΠΊ ΠΏΠΎΠΊΠ°Π·Π°Π½ΠΎ Π½Π° рисункС \(\PageIndex{3}\).

          Рисунок \(\PageIndex{3}\): Π’ этой ΡΠΎΠΏΡ€ΠΈΠΊΠ°ΡΠ°ΡŽΡ‰Π΅ΠΉΡΡ окруТности ΠΎΠΊΡ€ΡƒΠΆΠ½ΠΎΡΡ‚ΡŒ касаСтся ΠΊΡ€ΠΈΠ²ΠΎΠΉ \(C\) Π² Ρ‚ΠΎΡ‡ΠΊΠ΅ \(P\) ΠΈ ΠΈΠΌΠ΅Π΅Ρ‚ Ρ‚Ρƒ ΠΆΠ΅ ΠΊΡ€ΠΈΠ²ΠΈΠ·Π½Ρƒ.

          Для получСния Π΄ΠΎΠΏΠΎΠ»Π½ΠΈΡ‚Π΅Π»ΡŒΠ½ΠΎΠΉ ΠΈΠ½Ρ„ΠΎΡ€ΠΌΠ°Ρ†ΠΈΠΈ ΠΎ ΡΠΎΠΏΡ€ΠΈΠΊΠ°ΡΠ°ΡŽΡ‰ΠΈΡ…ΡΡ окруТностях см. эту Π΄Π΅ΠΌΠΎΠ½ΡΡ‚Ρ€Π°Ρ†ΠΈΡŽ ΠΊΡ€ΠΈΠ²ΠΈΠ·Π½Ρ‹ ΠΈ кручСния, эту ΡΡ‚Π°Ρ‚ΡŒΡŽ ΠΎ ΡΠΎΠΏΡ€ΠΈΠΊΠ°ΡΠ°ΡŽΡ‰ΠΈΡ…ΡΡ окруТностях ΠΈ это обсуТдСниС Ρ„ΠΎΡ€ΠΌΡƒΠ» Π‘Π΅Ρ€Ρ€Π΅.

          Π§Ρ‚ΠΎΠ±Ρ‹ Π½Π°ΠΉΡ‚ΠΈ ΡƒΡ€Π°Π²Π½Π΅Π½ΠΈΠ΅ ΡΠΎΠΏΡ€ΠΈΠΊΠ°ΡΠ°ΡŽΡ‰Π΅ΠΉΡΡ окруТности Π² Π΄Π²ΡƒΡ… измСрСниях, Π½Π°ΠΌ Π½ΡƒΠΆΠ½ΠΎ Π½Π°ΠΉΡ‚ΠΈ Ρ‚ΠΎΠ»ΡŒΠΊΠΎ Ρ†Π΅Π½Ρ‚Ρ€ ΠΈ радиус окруТности. 9{3/2}}. \nonumber \]

          Π­Ρ‚ΠΎ Π΄Π°Π΅Ρ‚ \(ΞΊ=6\). Π‘Π»Π΅Π΄ΠΎΠ²Π°Ρ‚Π΅Π»ΡŒΠ½ΠΎ, радиус ΡΠΎΠΏΡ€ΠΈΠΊΠ°ΡΠ°ΡŽΡ‰Π΅ΠΉΡΡ окруТности опрСдСляСтся Π²Ρ‹Ρ€Π°ΠΆΠ΅Π½ΠΈΠ΅ΠΌ \(R=\frac{1}{ΞΊ}=\dfrac{1}{6}\). Π”Π°Π»Π΅Π΅ ΠΌΡ‹ вычисляСм ΠΊΠΎΠΎΡ€Π΄ΠΈΠ½Π°Ρ‚Ρ‹ Ρ†Π΅Π½Ρ‚Ρ€Π° ΠΊΡ€ΡƒΠ³Π°. Когда \(x=1\), Π½Π°ΠΊΠ»ΠΎΠ½ ΠΊΠ°ΡΠ°Ρ‚Π΅Π»ΡŒΠ½ΠΎΠΉ Ρ€Π°Π²Π΅Π½ Π½ΡƒΠ»ΡŽ. 2=\frac{1}{36}\). Π“Ρ€Π°Ρ„ΠΈΠΊ ΠΈ Π΅Π³ΠΎ ΡΠΎΠΏΡ€ΠΈΠΊΠ°ΡΠ°ΡŽΡ‰ΠΈΠΉΡΡ ΠΊΡ€ΡƒΠ³ ΠΏΠΎΠΊΠ°Π·Π°Π½Ρ‹ Π½Π° ΡΠ»Π΅Π΄ΡƒΡŽΡ‰Π΅ΠΌ Π³Ρ€Π°Ρ„ΠΈΠΊΠ΅. 9{3/2}}\)

          Π’ Ρ‚ΠΎΡ‡ΠΊΠ΅ \(x=1\) ΠΊΡ€ΠΈΠ²ΠΈΠ·Π½Π° Ρ€Π°Π²Π½Π° \(4\). Π‘Π»Π΅Π΄ΠΎΠ²Π°Ρ‚Π΅Π»ΡŒΠ½ΠΎ, радиус ΡΠΎΠΏΡ€ΠΈΠΊΠ°ΡΠ°ΡŽΡ‰Π΅ΠΉΡΡ окруТности Ρ€Π°Π²Π΅Π½ \(\frac{1}{4}\).

          Π”Π°Π»Π΅Π΅ появится Π³Ρ€Π°Ρ„ΠΈΠΊ этой Ρ„ΡƒΠ½ΠΊΡ†ΠΈΠΈ:

          Π’Π΅Ρ€ΡˆΠΈΠ½Π° этой ΠΏΠ°Ρ€Π°Π±ΠΎΠ»Ρ‹ находится Π² Ρ‚ΠΎΡ‡ΠΊΠ΅ \((1,3)\). ΠšΡ€ΠΎΠΌΠ΅ Ρ‚ΠΎΠ³ΠΎ, Ρ†Π΅Π½Ρ‚Ρ€ ΡΠΎΠΏΡ€ΠΈΠΊΠ°ΡΠ°ΡŽΡ‰Π΅ΠΉΡΡ окруТности находится прямо Π½Π°Π΄ Π²Π΅Ρ€ΡˆΠΈΠ½ΠΎΠΉ. Π‘Π»Π΅Π΄ΠΎΠ²Π°Ρ‚Π΅Π»ΡŒΠ½ΠΎ, ΠΊΠΎΠΎΡ€Π΄ΠΈΠ½Π°Ρ‚Ρ‹ Ρ†Π΅Π½Ρ‚Ρ€Π° Ρ€Π°Π²Π½Ρ‹ \((1,\frac{13}{4})\). Π£Ρ€Π°Π²Π½Π΅Π½ΠΈΠ΅ ΡΠΎΠΏΡ€ΠΈΠΊΠ°ΡΠ°ΡŽΡ‰Π΅ΠΉΡΡ окруТности

          9{3/2}}\)
        1. Π’Π΅ΠΊΡ‚ΠΎΡ€ Π½ΠΎΡ€ΠΌΠ°Π»ΠΈ ΠΊ Π³Π»Π°Π²Π½ΠΎΠΉ Π΅Π΄ΠΈΠ½ΠΈΡ†Π΅
          \(\vecs N(t)=\frac{\vecs Tβ€²(t)}{β€–\vecs Tβ€²(t)β€–}\)
        2. Π’Π΅ΠΊΡ‚ΠΎΡ€ Π±ΠΈΠ½ΠΎΡ€ΠΌΠ°Π»Π΅ΠΉ
          \(\vecs B(t)=\vecs T(t)Γ—\vecs N(t)\)

    Глоссарий

    функция Π΄Π»ΠΈΠ½Ρ‹ Π΄ΡƒΠ³ΠΈ
    функция \(s(t)\), которая описываСт Π΄Π»ΠΈΠ½Ρƒ Π΄ΡƒΠ³ΠΈ ΠΊΡ€ΠΈΠ²ΠΎΠΉ \(C\) ΠΊΠ°ΠΊ Ρ„ΡƒΠ½ΠΊΡ†ΠΈΡŽ \(t\)
    парамСтризация Π΄Π»ΠΈΠ½Ρ‹ Π΄ΡƒΠ³ΠΈ
    рСпарамСтризация Π²Π΅ΠΊΡ‚ΠΎΡ€-Ρ„ΡƒΠ½ΠΊΡ†ΠΈΠΈ, Π² ΠΊΠΎΡ‚ΠΎΡ€ΠΎΠΉ ΠΏΠ°Ρ€Π°ΠΌΠ΅Ρ‚Ρ€ Ρ€Π°Π²Π΅Π½ Π΄Π»ΠΈΠ½Π΅ Π΄ΡƒΠ³ΠΈ
    Π±ΠΈΠ½ΠΎΡ€ΠΌΠ°Π»ΡŒΠ½Ρ‹ΠΉ Π²Π΅ΠΊΡ‚ΠΎΡ€
    Π΅Π΄ΠΈΠ½ΠΈΡ‡Π½Ρ‹ΠΉ Π²Π΅ΠΊΡ‚ΠΎΡ€, ΠΎΡ€Ρ‚ΠΎΠ³ΠΎΠ½Π°Π»ΡŒΠ½Ρ‹ΠΉ Π΅Π΄ΠΈΠ½ΠΈΡ‡Π½ΠΎΠΌΡƒ ΠΊΠ°ΡΠ°Ρ‚Π΅Π»ΡŒΠ½ΠΎΠΌΡƒ Π²Π΅ΠΊΡ‚ΠΎΡ€Ρƒ ΠΈ Π΅Π΄ΠΈΠ½ΠΈΡ‡Π½ΠΎΠΌΡƒ Π²Π΅ΠΊΡ‚ΠΎΡ€Ρƒ Π½ΠΎΡ€ΠΌΠ°Π»ΠΈ
    ΠΊΡ€ΠΈΠ²ΠΈΠ·Π½Π°
    производная Π΅Π΄ΠΈΠ½ΠΈΡ‡Π½ΠΎΠ³ΠΎ ΠΊΠ°ΡΠ°Ρ‚Π΅Π»ΡŒΠ½ΠΎΠ³ΠΎ Π²Π΅ΠΊΡ‚ΠΎΡ€Π° ΠΏΠΎ ΠΏΠ°Ρ€Π°ΠΌΠ΅Ρ‚Ρ€Ρƒ Π΄Π»ΠΈΠ½Ρ‹ Π΄ΡƒΠ³ΠΈ
    БистСма ΠΊΠΎΠΎΡ€Π΄ΠΈΠ½Π°Ρ‚ Π€Ρ€Π΅Π½Π΅
    (ΠΊΠ°Π΄Ρ€ TNB) систСма отсчСта Π² Ρ‚Ρ€Π΅Ρ…ΠΌΠ΅Ρ€Π½ΠΎΠΌ пространствС, образованная Π΅Π΄ΠΈΠ½ΠΈΡ‡Π½Ρ‹ΠΌ ΠΊΠ°ΡΠ°Ρ‚Π΅Π»ΡŒΠ½Ρ‹ΠΌ Π²Π΅ΠΊΡ‚ΠΎΡ€ΠΎΠΌ, Π΅Π΄ΠΈΠ½ΠΈΡ‡Π½Ρ‹ΠΌ Π²Π΅ΠΊΡ‚ΠΎΡ€ΠΎΠΌ Π½ΠΎΡ€ΠΌΠ°Π»ΠΈ ΠΈ Π±ΠΈΠ½ΠΎΡ€ΠΌΠ°Π»ΡŒΠ½Ρ‹ΠΌ Π²Π΅ΠΊΡ‚ΠΎΡ€ΠΎΠΌ
    ΠΎΠ±Ρ‹Ρ‡Π½Ρ‹ΠΉ самолСт
    ΠΏΠ»ΠΎΡΠΊΠΎΡΡ‚ΡŒ, пСрпСндикулярная ΠΊΡ€ΠΈΠ²ΠΎΠΉ Π² любой Ρ‚ΠΎΡ‡ΠΊΠ΅ ΠΊΡ€ΠΈΠ²ΠΎΠΉ
    ΡΠΎΠΏΡ€ΠΈΠΊΠ°ΡΠ°ΡŽΡ‰ΠΈΠΉΡΡ ΠΊΡ€ΡƒΠ³
    ΠΎΠΊΡ€ΡƒΠΆΠ½ΠΎΡΡ‚ΡŒ, ΠΊΠ°ΡΠ°ΡŽΡ‰Π°ΡΡΡ ΠΊΡ€ΠΈΠ²ΠΎΠΉ \(C\) Π² Ρ‚ΠΎΡ‡ΠΊΠ΅ \(P\) ΠΈ ΠΈΠΌΠ΅ΡŽΡ‰Π°Ρ Ρ‚Ρƒ ΠΆΠ΅ ΠΊΡ€ΠΈΠ²ΠΈΠ·Π½Ρƒ
    ΡΠΎΠΏΡ€ΠΈΠΊΠ°ΡΠ°ΡŽΡ‰Π°ΡΡΡ ΠΏΠ»ΠΎΡΠΊΠΎΡΡ‚ΡŒ
    ΠΏΠ»ΠΎΡΠΊΠΎΡΡ‚ΡŒ, опрСдСляСмая Π΅Π΄ΠΈΠ½ΠΈΡ‡Π½ΠΎΠΉ ΠΊΠ°ΡΠ°Ρ‚Π΅Π»ΡŒΠ½ΠΎΠΉ ΠΈ Π΅Π΄ΠΈΠ½ΠΈΡ‡Π½Ρ‹ΠΌ Π²Π΅ΠΊΡ‚ΠΎΡ€ΠΎΠΌ Π½ΠΎΡ€ΠΌΠ°Π»ΠΈ
    Π²Π΅ΠΊΡ‚ΠΎΡ€ Π½ΠΎΡ€ΠΌΠ°Π»ΠΈ ΠΊ Π³Π»Π°Π²Π½ΠΎΠΉ Π΅Π΄ΠΈΠ½ΠΈΡ†Π΅
    Π²Π΅ΠΊΡ‚ΠΎΡ€, ΠΎΡ€Ρ‚ΠΎΠ³ΠΎΠ½Π°Π»ΡŒΠ½Ρ‹ΠΉ Π΅Π΄ΠΈΠ½ΠΈΡ‡Π½ΠΎΠΌΡƒ ΠΊΠ°ΡΠ°Ρ‚Π΅Π»ΡŒΠ½ΠΎΠΌΡƒ Π²Π΅ΠΊΡ‚ΠΎΡ€Ρƒ, опрСдСляСмый Ρ„ΠΎΡ€ΠΌΡƒΠ»ΠΎΠΉ \(\frac{\vecs T'(t)}{β€–\vecs T'(t)β€–}\)
    радиус ΠΊΡ€ΠΈΠ²ΠΈΠ·Π½Ρ‹
    обратная ΠΊΡ€ΠΈΠ²ΠΈΠ·Π½Π°
    гладкая
    ΠΊΡ€ΠΈΠ²Ρ‹Ρ…, Π³Π΄Π΅ Π²Π΅ΠΊΡ‚ΠΎΡ€-функция \(\vecs r(t)\) Π΄ΠΈΡ„Ρ„Π΅Ρ€Π΅Π½Ρ†ΠΈΡ€ΡƒΠ΅ΠΌΠ° с Π½Π΅Π½ΡƒΠ»Π΅Π²ΠΎΠΉ ΠΏΡ€ΠΎΠΈΠ·Π²ΠΎΠ΄Π½ΠΎΠΉ

    Π­Ρ‚Π° страница ΠΏΠΎΠ΄ Π½Π°Π·Π²Π°Π½ΠΈΠ΅ΠΌ 13. 3: Π”Π»ΠΈΠ½Π° Π΄ΡƒΠ³ΠΈ ΠΈ ΠΊΡ€ΠΈΠ²ΠΈΠ·Π½Π° распространяСтся ΠΏΠΎΠ΄ Π»ΠΈΡ†Π΅Π½Π·ΠΈΠ΅ΠΉ CC BY-NC-SA 4.0 ΠΈ Π±Ρ‹Π»Π° создана, ΠΈΠ·ΠΌΠ΅Π½Π΅Π½Π° ΠΈ/ΠΈΠ»ΠΈ ΠΊΡƒΡ€ΠΈΡ€ΠΎΠ²Π°Π½Π° Π“ΠΈΠ»Π±Π΅Ρ€Ρ‚ΠΎΠΌ Бтрэнгом ΠΈ Π­Π΄Π²ΠΈΠ½ΠΎΠΌ Β«Π”ΠΆΠ΅Π΄ΠΎΠΌΒ» Π₯Π΅Ρ€ΠΌΠ°Π½ΠΎΠΌ (OpenStax) Ρ‡Π΅Ρ€Π΅Π· исходный ΠΊΠΎΠ½Ρ‚Π΅Π½Ρ‚, ΠΊΠΎΡ‚ΠΎΡ€Ρ‹ΠΉ Π±Ρ‹Π» ΠΎΡ‚Ρ€Π΅Π΄Π°ΠΊΡ‚ΠΈΡ€ΠΎΠ²Π°Π½ΠΎ Π² соотвСтствии со стилСм ΠΈ стандартами ΠΏΠ»Π°Ρ‚Ρ„ΠΎΡ€ΠΌΡ‹ LibreTexts; подробная история рСдактирования доступна ΠΏΠΎ запросу.

    1. НавСрх
      • Π‘Ρ‹Π»Π° Π»ΠΈ эта ΡΡ‚Π°Ρ‚ΡŒΡ ΠΏΠΎΠ»Π΅Π·Π½ΠΎΠΉ?
      1. Вип издСлия
        Π Π°Π·Π΄Π΅Π» ΠΈΠ»ΠΈ страница
        Автор
        ΠžΠΏΠ΅Π½Π‘Ρ‚Π°ΠΊΡ
        ЛицСнзия
        Π‘Π‘ BY-NC-SA
        ВСрсия Π»ΠΈΡ†Π΅Π½Π·ΠΈΠΈ
        4,0
        ΠŸΡ€ΠΎΠ³Ρ€Π°ΠΌΠΌΠ° OER ΠΈΠ»ΠΈ Publisher
        ΠžΠΏΠ΅Π½Π‘Ρ‚Π°ΠΊΡ
        ΠŸΠΎΠΊΠ°Π·Π°Ρ‚ΡŒ страницу Π‘ΠΎΠ΄Π΅Ρ€ΠΆΠ°Π½ΠΈΠ΅
        Π½Π΅Ρ‚
      2. Π’Π΅Π³ΠΈ
        1. Π”Π»ΠΈΠ½Π° Π΄ΡƒΠ³ΠΈ
        2. функция Π΄Π»ΠΈΠ½Ρ‹ Π΄ΡƒΠ³ΠΈ
        3. парамСтризация Π΄Π»ΠΈΠ½Ρ‹ Π΄ΡƒΠ³ΠΈ
        4. Π°Π²Ρ‚ΠΎΡ€ @ Π­Π΄Π²ΠΈΠ½ Β«Π”ΠΆΠ΅Π΄Β» Π“Π΅Ρ€ΠΌΠ°Π½
        5. Π°Π²Ρ‚ΠΎΡ€@Π“ΠΈΠ»Π±Π΅Ρ€Ρ‚ Π‘Ρ‚Ρ€Π°Π½Π³
        6. Π±ΠΈΠ½ΠΎΡ€ΠΌΠ°Π»ΡŒΠ½Ρ‹ΠΉ Π±ΠΈΠ½ΠΎΡ€ΠΌΠ°Π»ΡŒΠ½Ρ‹ΠΉ Π²Π΅ΠΊΡ‚ΠΎΡ€
        7. ΠΊΡ€ΠΈΠ²ΠΈΠ·Π½Π°
        8. БистСма ΠΊΠΎΠΎΡ€Π΄ΠΈΠ½Π°Ρ‚ Π€Ρ€Π΅Π½Π΅
        9. ΠΎΠ±Ρ‹Ρ‡Π½Ρ‹ΠΉ самолСт
        10. ΡΠΎΠΏΡ€ΠΈΠΊΠ°ΡΠ°ΡŽΡ‰ΠΈΠΉΡΡ ΠΊΡ€ΡƒΠ³
        11. ΡΠΎΠΏΡ€ΠΈΠΊΠ°ΡΠ°ΡŽΡ‰Π°ΡΡΡ ΠΏΠ»ΠΎΡΠΊΠΎΡΡ‚ΡŒ
        12. Π²Π΅ΠΊΡ‚ΠΎΡ€ Π½ΠΎΡ€ΠΌΠ°Π»ΠΈ ΠΊ Π³Π»Π°Π²Π½ΠΎΠΉ Π΅Π΄ΠΈΠ½ΠΈΡ†Π΅
        13. радиус ΠΊΡ€ΠΈΠ²ΠΈΠ·Π½Ρ‹
        14. гладкая
        15. источник@https://openstax. org/details/books/calculus-volume-1

      MathScene — Π’Π΅ΠΊΡ‚ΠΎΡ€Ρ‹ — Π£Ρ€ΠΎΠΊ 3

      MathScene — Π’Π΅ΠΊΡ‚ΠΎΡ€Ρ‹ — Π£Ρ€ΠΎΠΊ 3

      2008 Расмус Π­Ρ„ ΠΈ Π”ΠΆΠ°Π½Π½ Π‘Π°ΠΊ

      Π£Ρ€ΠΎΠΊ 3

      Π’Π΅ΠΊΡ‚ΠΎΡ€Ρ‹ Π² систСмС ΠΊΠΎΠΎΡ€Π΄ΠΈΠ½Π°Ρ‚

      Β 


      ΠŸΡ€ΠΈΠΌΠ΅Ρ€ 1

      Ρ‚ΠΎΡ‡ΠΊΠ° А ΠΈΠΌΠ΅Π΅Ρ‚ ΠΊΠΎΠΎΡ€Π΄ΠΈΠ½Π°Ρ‚Ρ‹ (2, 2), Π° Ρ‚ΠΎΡ‡ΠΊΠ° Π’ β€” ΠΊΠΎΠΎΡ€Π΄ΠΈΠ½Π°Ρ‚Ρ‹ (6, 5) (см. схСму). ΠšΠΎΠΎΡ€Π΄ΠΈΠ½Π°Ρ‚Ρ‹ Π²Π΅ΠΊΡ‚ΠΎΡ€Π°

      ΠœΡ‹ ΠΌΠΎΠΆΠ½ΠΎ ΠΈΡΠΏΠΎΠ»ΡŒΠ·ΠΎΠ²Π°Ρ‚ΡŒ Ρ„ΠΎΡ€ΠΌΡƒΠ»Ρƒ расстояния ΠΌΠ΅ΠΆΠ΄Ρƒ двумя Ρ‚ΠΎΡ‡ΠΊΠ°ΠΌΠΈ, Ρ‡Ρ‚ΠΎΠ±Ρ‹ Π½Π°ΠΉΡ‚ΠΈ расстояниС ΠΌΠ΅ΠΆΠ΄Ρƒ A ΠΈ B, Ρ‚ΠΎ Π΅ΡΡ‚ΡŒ Π΄Π»ΠΈΠ½Π° Π²Π΅ΠΊΡ‚ΠΎΡ€Π°
      (см. ΠŸΡ€Π°Π²ΠΈΠ»ΠΎ ΠŸΠΈΡ„Π°Π³ΠΎΡ€Π° Π² ΡƒΡ€ΠΎΠΊΠ΅ 2). Π€ΠΎΡ€ΠΌΡƒΠ»Π° выглядит ΡΠ»Π΅Π΄ΡƒΡŽΡ‰ΠΈΠΌ ΠΎΠ±Ρ€Π°Π·ΠΎΠΌ:

      ΠŸΠΎΠ΄ΡΡ‚Π°Π²Π»ΡΡ Π·Π°Π΄Π°Π½Π½Ρ‹Π΅ ΠΊΠΎΠΎΡ€Π΄ΠΈΠ½Π°Ρ‚Ρ‹ Π² Ρ„ΠΎΡ€ΠΌΡƒΠ»Ρƒ ΠΏΠΎΠ»ΡƒΡ‡Π°Π΅ΠΌ:

      ΠœΡ‹ Π²ΠΈΠ΄ΠΈΠΌ, Ρ‡Ρ‚ΠΎ числа ΠΏΠΎΠ΄ ΠΊΠ²Π°Π΄Ρ€Π°Ρ‚Π½Ρ‹ΠΌ ΠΊΠΎΡ€Π½Π΅ΠΌ β€” это просто ΠΊΠΎΠΎΡ€Π΄ΠΈΠ½Π°Ρ‚Ρ‹ Π²Π΅ΠΊΡ‚ΠΎΡ€. Π­Ρ‚ΠΎ, ΠΊΠΎΠ½Π΅Ρ‡Π½ΠΎ, ΠΏΠΎΡ‚ΠΎΠΌΡƒ, Ρ‡Ρ‚ΠΎ Π΄Π»ΠΈΠ½Π° Π²Π΅ΠΊΡ‚ΠΎΡ€Π° β€” это просто Π³ΠΈΠΏΠΎΡ‚Π΅Π½ΡƒΠ·Π° Π² ΠΏΡ€ΡΠΌΠΎΡƒΠ³ΠΎΠ»ΡŒΠ½ΠΎΠΌ Ρ‚Ρ€Π΅ΡƒΠ³ΠΎΠ»ΡŒΠ½ΠΈΠΊΠ΅ с Π±ΠΎΠ»Π΅Π΅ ΠΊΠΎΡ€ΠΎΡ‚ΠΊΠΈΠΌΠΈ сторонами 3 ΠΈ 4.

      Π€ΠΎΡ€ΠΌΡƒΠ»Π° Π΄Π»ΠΈΠ½Ρ‹ Π²Π΅ΠΊΡ‚ΠΎΡ€Π°, Π½Π°Ρ‡ΠΈΠ½Π°ΡŽΡ‰Π΅Π³ΠΎΡΡ Π² Ρ‚ΠΎΡ‡ΠΊΠ΅
      А = (Ρ… 1 , y 1 ) ΠΈ заканчиваСтся Π½Π° B = (x 2 , Ρƒ 2 ) это:

      Если ΠΊΠΎΠΎΡ€Π΄ΠΈΠ½Π°Ρ‚Ρ‹ Π²Π΅ΠΊΡ‚ΠΎΡ€Π° Ρ‚ΠΎ ΠΈΠΌΠ΅Π΅ΠΌ ΡΠ»Π΅Π΄ΡƒΡŽΡ‰Π΅Π΅ ΠΏΡ€Π°Π²ΠΈΠ»ΠΎ:



      ΠŸΡ€ΠΈΠΌΠ΅Ρ€ 2

      НайдитС Π²Π΅ΠΊΡ‚ΠΎΡ€ Ρ‡Ρ‚ΠΎ ΠΏΠ°Ρ€Π°Π»Π»Π΅Π»ΡŒΠ½ΠΎ ΠΈ ΠΊΠΎΡ‚ΠΎΡ€Ρ‹ΠΉ ΠΈΠΌΠ΅Π΅Ρ‚ Π΄Π»ΠΈΠ½Ρƒ 2 Π΅Π΄ΠΈΠ½ΠΈΡ†Ρ‹ (Π²ΠΈΠ΄Π΅Ρ‚ΡŒ Π΄ΠΈΠ°Π³Ρ€Π°ΠΌΠΌΡƒ).

      Π”Π²Π° Ρ‚Ρ€Π΅ΡƒΠ³ΠΎΠ»ΡŒΠ½ΠΈΠΊΠ° Π½Π° Π΄ΠΈΠ°Π³Ρ€Π°ΠΌΠΌΠ΅ ΠΏΠΎΠ΄ΠΎΠ±Π½Ρ‹, поэтому ΡΠΎΠΎΡ‚Π²Π΅Ρ‚ΡΡ‚Π²ΡƒΡŽΡ‰ΠΈΠ΅ стороны находятся Π² ΠΎΠ΄ΠΈΠ½Π°ΠΊΠΎΠ²ΠΎΠΌ ΡΠΎΠΎΡ‚Π½ΠΎΡˆΠ΅Π½ΠΈΠΈ.
      || = Ρ‚βˆ™||. Число t Π΅ΡΡ‚ΡŒ ΠΎΡ‚Π½ΠΎΡˆΠ΅Π½ΠΈΠ΅ ΠΌΠ΅ΠΆΠ΄Ρƒ ΡΠΎΠΎΡ‚Π²Π΅Ρ‚ΡΡ‚Π²ΡƒΡŽΡ‰ΠΈΠΌΠΈ сторонами. ΠžΡ‚Π½ΠΎΡˆΠ΅Π½ΠΈΠ΅ Ρ‚Π°ΠΊΠΎΠ΅.
      ΠœΡ‹ ΠΌΠΎΠΆΠ΅ΠΌ Π½Π°ΠΉΡ‚ΠΈ ΠΊΠΎΠΎΡ€Π΄ΠΈΠ½Π°Ρ‚Ρ‹ Β ΠΊΠ°ΠΊ слСдуСт:

      Если Π²Π΅ΠΊΡ‚ΠΎΡ€Ρ‹ Π° Ρ‚Π°ΠΊΠΆΠ΅ находятся ΠΏΠ°Ρ€Π°Π»Π»Π΅Π»ΡŒΠ½ΠΎ, Ρ‚ΠΎ сущСствуСт число t Ρ‚Π°ΠΊΠΎΠ΅, Ρ‡Ρ‚ΠΎ:

      = Ρ‚βˆ™


      ΠŸΡ€ΠΈΠΌΠ΅Ρ€ 3

      КакиС ΠΈΠ· ΡΠ»Π΅Π΄ΡƒΡŽΡ‰ΠΈΡ… Π²Π΅ΠΊΡ‚ΠΎΡ€ΠΎΠ² ΠΏΠ°Ρ€Π°Π»Π»Π΅Π»ΡŒΠ½Ρ‹ Π° Ρ‚Π°ΠΊΠΆΠ΅ .

      Если Π²Π΅ΠΊΡ‚ΠΎΡ€Ρ‹ Π° Ρ‚Π°ΠΊΠΆΠ΅ находятся ΠΏΠ°Ρ€Π°Π»Π»Π΅Π»ΡŒΠ½ΠΎ, Ρ‚ΠΎ сущСствуСт число t Ρ‚Π°ΠΊΠΎΠ΅, Ρ‡Ρ‚ΠΎ Β Β Β = Ρ‚βˆ™. Если Π²Π΅ΠΊΡ‚ΠΎΡ€Ρ‹ Π° Ρ‚Π°ΠΊΠΆΠ΅ находятся ΠΏΠ°Ρ€Π°Π»Π»Π΅Π»ΡŒΠ½ΠΎ сущСствуСт число r Ρ‚Π°ΠΊΠΎΠ΅, Ρ‡Ρ‚ΠΎ Π·Π½Π°ΠΊ Ρ€Π°Π²Π½ΠΎ Ρ€βˆ™.

      ΠœΡ‹ ΠΌΠΎΠΆΠ½ΠΎ Π½Π°ΠΉΡ‚ΠΈ числа t ΠΈ r, ΠΈΡΠΏΠΎΠ»ΡŒΠ·ΡƒΡ ΠΊΠΎΠΎΡ€Π΄ΠΈΠ½Π°Ρ‚Ρ‹ x, Π° Π·Π°Ρ‚Π΅ΠΌ ΠΏΡ€ΠΎΠ²Π΅Ρ€ΠΈΡ‚ΡŒ, Ρ‡Ρ‚ΠΎΠ±Ρ‹ ΡƒΠ²ΠΈΠ΄Π΅Ρ‚ΡŒ Π½Π°ΠΉΠ΄Π΅Π½Ρ‹ Π»ΠΈ Ρ‚Π΅ ΠΆΠ΅ значСния, ΠΊΠΎΠ³Π΄Π° ΠΌΡ‹ ΠΈΡΠΏΠΎΠ»ΡŒΠ·ΡƒΠ΅ΠΌ ΠΊΠΎΠΎΡ€Π΄ΠΈΠ½Π°Ρ‚Ρ‹ y.

      = Ρ‚βˆ™

      3 = tβˆ™13 Π΄Π°Π΅Ρ‚ t = 3/13 = 2/9

      4 = tβˆ™18 Ρ‚Π°ΠΊΠΆΠ΅ Π΄Π°Π΅Ρ‚ t = 4/18 = 2/9

      Π²Π΅ΠΊΡ‚ΠΎΡ€Ρ‹ Β ΠΈ Β Π΅ΡΡ‚ΡŒ ΠΏΠ°Ρ€Π°Π»Π»Π΅Π»ΡŒ .

      = Ρ€βˆ™

      3 = rβˆ™6 Π΄Π°Π΅Ρ‚ r =

      4 = rβˆ™9 Π΄Π°Π΅Ρ‚ r = 4/9

      Π²Π΅ΠΊΡ‚ΠΎΡ€Ρ‹ Β ΠΈ Β Π΅ΡΡ‚ΡŒ Π½Π΅ ΠΏΠ°Ρ€Π°Π»Π»Π΅Π»ΡŒΠ½ΠΎ (Π­Ρ‚ΠΎ ΠΎΠ·Π½Π°Ρ‡Π°Π΅Ρ‚, Ρ‡Ρ‚ΠΎ Π° Ρ‚Π°ΠΊΠΆΠ΅ находятся Ρ‚ΠΎΠΆΠ΅ Π½Π΅ ΠΏΠ°Ρ€Π°Π»Π»Π΅Π»ΡŒΠ½ΠΎ).

      Β 

      Π’Π΅ΠΊΡ‚ΠΎΡ€ Π½Π° Π΄ΠΈΠ°Π³Ρ€Π°ΠΌΠΌΠ΅ ΠΈΠΌΠ΅Π΅Ρ‚ ΠΊΠΎΠΎΡ€Π΄ΠΈΠ½Π°Ρ‚Ρ‹ . Π²Π΅ΠΊΡ‚ΠΎΡ€ начинаСтся Π² Ρ‚ΠΎΡ‡ΠΊΠ΅ (0, 0) ΠΈ заканчиваСтся Π² (3, 2), поэтому ΠΊΠΎΠΎΡ€Π΄ΠΈΠ½Π°Ρ‚Ρ‹ конСчная Ρ‚ΠΎΡ‡ΠΊΠ° совпадаСт с ΠΊΠΎΠΎΡ€Π΄ΠΈΠ½Π°Ρ‚Π°ΠΌΠΈ самого Π²Π΅ΠΊΡ‚ΠΎΡ€Π°. Π­Ρ‚ΠΎ относится ΠΊ всС Π²Π΅ΠΊΡ‚ΠΎΡ€Ρ‹, ΠΊΠΎΡ‚ΠΎΡ€Ρ‹Π΅ Π½Π°Ρ‡ΠΈΠ½Π°ΡŽΡ‚ΡΡ Π² Π½Π°Ρ‡Π°Π»Π΅ систСмы ΠΊΠΎΠΎΡ€Π΄ΠΈΠ½Π°Ρ‚, Ρ‚ΠΎ Π΅ΡΡ‚ΡŒ Π² Ρ‚ΠΎΡ‡ΠΊΠ° (0, 0).

      Π’Π΅ΠΊΡ‚ΠΎΡ€, ΠΊΠΎΡ‚ΠΎΡ€Ρ‹ΠΉ начинаСтся Π² Ρ‚ΠΎΡ‡ΠΊΠ΅ (0, 0), ΠΈΠΌΠ΅Π΅Ρ‚ Ρ‚Π΅ ΠΆΠ΅ ΠΊΠΎΠΎΡ€Π΄ΠΈΠ½Π°Ρ‚Ρ‹, Ρ‡Ρ‚ΠΎ ΠΈ Π΅Π³ΠΎ конСчная Ρ‚ΠΎΡ‡ΠΊΠ°. Π­Ρ‚ΠΎΡ‚ Π²Π΅ΠΊΡ‚ΠΎΡ€ называСтся Π²Π΅ΠΊΡ‚ΠΎΡ€ΠΎΠΌ полоТСния для A.

      КаТдая Ρ‚ΠΎΡ‡ΠΊΠ° Π² систСмС ΠΊΠΎΠΎΡ€Π΄ΠΈΠ½Π°Ρ‚ ΠΌΠΎΠΆΠ΅Ρ‚ Π±Ρ‹Ρ‚ΡŒ прСдставлСна ​​своим Π²Π΅ΠΊΡ‚ΠΎΡ€ΠΎΠΌ полоТСния. ΠšΠΎΠΎΡ€Π΄ΠΈΠ½Π°Ρ‚Ρ‹ Ρ‚ΠΎΡ‡ΠΊΠΈ ΠΈ Π²Π΅ΠΊΡ‚ΠΎΡ€ Π΅Π΅ полоТСния ΡΠΎΠ²ΠΏΠ°Π΄Π°ΡŽΡ‚. Π­Ρ‚ΠΎ ΠΌΠΎΠΆΠ΅Ρ‚ Π±Ρ‹Ρ‚ΡŒ ΠΎΡ‡Π΅Π½ΡŒ ΠΏΠΎΠ»Π΅Π·Π½ΠΎ ΠΏΡ€ΠΈ просмотрС ΠΏΠ΅Ρ€Π΅Π²ΠΎΠ΄ΠΎΠ² Π² систСмС ΠΊΠΎΠΎΡ€Π΄ΠΈΠ½Π°Ρ‚.


      ΠŸΡ€ΠΈΠΌΠ΅Ρ€ 4

      Π’Ρ€Π΅ΡƒΠ³ΠΎΠ»ΡŒΠ½ΠΈΠΊ, ΠΏΠΎΠΊΠ°Π·Π°Π½Π½Ρ‹ΠΉ Π½Π° Π΄ΠΈΠ°Π³Ρ€Π°ΠΌΠΌΠ΅, Π΄ΠΎΠ»ΠΆΠ΅Π½ Π±Ρ‹Ρ‚ΡŒ ΠΏΠ΅Ρ€Π΅Π²Π΅Π΄Π΅Π½ Π²Π΅ΠΊΡ‚ΠΎΡ€ΠΎΠΌ .

      ΠœΡ‹ ΠΈΡΠΏΠΎΠ»ΡŒΠ·ΡƒΠ΅ΠΌ Π²Π΅ΠΊΡ‚ΠΎΡ€Ρ‹ полоТСния Π²Π΅Ρ€ΡˆΠΈΠ½Π½Ρ‹Ρ… Ρ‚ΠΎΡ‡Π΅ΠΊ (βˆ’3, 0),
      (2, βˆ’2) ΠΈ (3, 1) ΠΈ добавляСм Π²Π΅ΠΊΡ‚ΠΎΡ€ ΠΊΠ°ΠΆΠ΄ΠΎΠΌΡƒ ΠΈΠ· Π½ΠΈΡ….

      Π­Ρ‚ΠΎ Π΄Π°Π΅Ρ‚ Π½Π°ΠΌ Π½ΠΎΠ²Ρ‹ΠΉ Π²Π΅ΠΊΡ‚ΠΎΡ€ полоТСния ΠΊΠ°ΠΆΠ΄ΠΎΠΉ Π²Π΅Ρ€ΡˆΠΈΠ½Ρ‹. Π”ΠΈΠ°Π³Ρ€Π°ΠΌΠΌΠ° Π½ΠΈΠΆΠ΅ ΠΏΠΎΠΊΠ°Π·Ρ‹Π²Π°Π΅Ρ‚ ΠΏΠ΅Ρ€Π΅Π²ΠΎΠ΄.


      ΠŸΡ€ΠΈΠΌΠ΅Ρ€ 5

      Π’Π΅ΠΏΠ΅Ρ€ΡŒ ΠΌΡ‹ Π±ΡƒΠ΄Π΅ΠΌ ΠΈΡΠΏΠΎΠ»ΡŒΠ·ΠΎΠ²Π°Ρ‚ΡŒ Π²Π΅ΠΊΡ‚ΠΎΡ€Ρ‹ полоТСния, Ρ‡Ρ‚ΠΎΠ±Ρ‹ Π½Π°ΠΉΡ‚ΠΈ сСрСдину ΠΎΡ‚Ρ€Π΅Π·ΠΊΠ° AB, Ссли А = (1, 2) ΠΈ Π’ = (4, 3).

      Как ΠΎΠ±Ρ‹Ρ‡Π½ΠΎ, Ρ‚ΠΎΡ‡ΠΊΠ° O являСтся Π½Π°Ρ‡Π°Π»ΠΎΠΌ систСмы ΠΊΠΎΠΎΡ€Π΄ΠΈΠ½Π°Ρ‚. Если M сСрСдина AB Ρ‚ΠΎΠ³Π΄Π°:

      Π·Π½Π°ΠΊ Ρ€Π°Π²Π½ΠΎ + βˆ™

      Π’Π΅ΠΊΡ‚ΠΎΡ€ являСтся Π²Π΅ΠΊΡ‚ΠΎΡ€ полоТСния Ρ‚ΠΎΡ‡ΠΊΠΈ M ΠΈ, ΡΠ»Π΅Π΄ΠΎΠ²Π°Ρ‚Π΅Π»ΡŒΠ½ΠΎ, ΠΈΠΌΠ΅Π΅Ρ‚ Ρ‚Π΅ ΠΆΠ΅ ΠΊΠΎΠΎΡ€Π΄ΠΈΠ½Π°Ρ‚Ρ‹, Ρ‡Ρ‚ΠΎ ΠΈ Ρ‚ΠΎΡ‡ΠΊΡƒ М, ΠΊΠΎΡ‚ΠΎΡ€ΡƒΡŽ ΠΌΡ‹ Ρ…ΠΎΡ‚ΠΈΠΌ Π²Ρ‹Ρ‡ΠΈΡΠ»ΠΈΡ‚ΡŒ. ВСктор – это Π²Π΅ΠΊΡ‚ΠΎΡ€ полоТСния A. Π§Ρ‚ΠΎΠ±Ρ‹ Π΄ΠΎΡΡ‚ΠΈΡ‡ΡŒ сСрСдины M, Π½Π°ΠΌ Π½ΡƒΠΆΠ½ΠΎ Π΄ΠΎΠ±Π°Π²ΠΈΡ‚ΡŒ ΠΏΠΎΠ»ΠΎΠ²ΠΈΠ½Ρƒ Π²Π΅ΠΊΡ‚ΠΎΡ€ . НарисуйтС схСму, Ρ‡Ρ‚ΠΎΠ±Ρ‹ ΡƒΠ²ΠΈΠ΄Π΅Ρ‚ΡŒ это.

      Π‘Π½Π°Ρ‡Π°Π»Π° Π½Π°ΠΌ Π½ΡƒΠΆΠ½ΠΎ Π½Π°ΠΉΡ‚ΠΈ Π²Π΅ΠΊΡ‚ΠΎΡ€ .

      Π’Π΅ΠΏΠ΅Ρ€ΡŒ ΠΌΡ‹ ΠΌΠΎΠΆΠ΅ΠΌ Π½Π°ΠΉΡ‚ΠΈ .

      Π·Π½Π°ΠΊ Ρ€Π°Π²Π½ΠΎ + βˆ™

      ΠšΠΎΠΎΡ€Π΄ΠΈΠ½Π°Ρ‚Ρ‹ M Ρ‚Π°ΠΊΠΈΠ΅ ΠΆΠ΅, ΠΊΠ°ΠΊ Ρƒ Π²Π΅ΠΊΡ‚ΠΎΡ€Π° полоТСния ΠΈΠ»ΠΈ (2, 2) .


      Π›Π΅Π³ΠΊΠΎ Π½Π°ΠΉΡ‚ΠΈ Ρ„ΠΎΡ€ΠΌΡƒΠ»Ρƒ, ΠΏΠΎ ΠΊΠΎΡ‚ΠΎΡ€ΠΎΠΉ ΠΌΠΎΠΆΠ½ΠΎ Π½Π°ΠΉΡ‚ΠΈ ΠΊΠΎΠΎΡ€Π΄ΠΈΠ½Π°Ρ‚Ρ‹ Ρ‚ΠΎΡ‡ΠΊΠΈ. сСрСдина ΠΎΡ‚Ρ€Π΅Π·ΠΊΠ° АВ.

      Из Π΄ΠΈΠ°Π³Ρ€Π°ΠΌΠΌΡ‹ Π²ΠΈΠ΄Π½ΠΎ, Ρ‡Ρ‚ΠΎ Π² сСрСдину М ΠΌΠΎΠΆΠ½ΠΎ ΠΏΠΎΠΏΠ°ΡΡ‚ΡŒ ΠΈΠ· Π΄Π²ΡƒΡ… направлСниях, ΠΎΡ‚ O Ρ‡Π΅Ρ€Π΅Π· A Π΄ΠΎ M ΠΈ ΠΎΡ‚ O Ρ‡Π΅Ρ€Π΅Π· B Π΄ΠΎ M.

      Π’Π°ΠΊΠΈΠΌ ΠΎΠ±Ρ€Π°Π·ΠΎΠΌ, ΠΌΡ‹ ΠΌΠΎΠΆΠ΅ΠΌ Π½Π°ΠΏΠΈΡΠ°Ρ‚ΡŒ Π΄Π²Π° Π²Π΅ΠΊΡ‚ΠΎΡ€Π½Ρ‹Ρ… уравнСния для .

      Π·Π½Π°ΠΊ Ρ€Π°Π²Π½ΠΎ + βˆ™

      Π·Π½Π°ΠΊ Ρ€Π°Π²Π½ΠΎ — βˆ™

      Бкладывая эти Π΄Π²Π° уравнСния вмСстС, ΠΌΡ‹ ΠΏΠΎΠ»ΡƒΡ‡Π°Π΅ΠΌ

      2 = + βˆ™ + — βˆ™

      ΠœΡ‹ Π²ΠΈΠ΄ΠΈΠΌ, Ρ‡Ρ‚ΠΎ Π²Π΅ΠΊΡ‚ΠΎΡ€ полоТСния сСрСдины ΠΎΡ‚Ρ€Π΅Π·ΠΊΠ° прСдставляСт собой своСго Ρ€ΠΎΠ΄Π° срСднСС Π·Π½Π°Ρ‡Π΅Π½ΠΈΠ΅ Π²Π΅ΠΊΡ‚ΠΎΡ€ΠΎΠ² полоТСния ΠΊΠΎΠ½Π΅Ρ‡Π½Ρ‹Ρ… Ρ‚ΠΎΡ‡Π΅ΠΊ. ΠŸΠΎΡΡ‚ΠΎΠΌΡƒ ΠΌΡ‹ ΠΌΠΎΠΆΠ΅ΠΌ Π½Π°ΠΉΡ‚ΠΈ ΠΊΠΎΠΎΡ€Π΄ΠΈΠ½Π°Ρ‚Ρ‹ срСднСй Ρ‚ΠΎΡ‡ΠΊΠΈ, найдя срСднСС Π·Π½Π°Ρ‡Π΅Π½ΠΈΠ΅ ΠΊΠΎΠΎΡ€Π΄ΠΈΠ½Π°Ρ‚ x ΠΈ y ΠΊΠΎΠΎΡ€Π΄ΠΈΠ½Π°Ρ‚Ρ‹ соотвСтствСнно.
      Π­Ρ‚ΠΎ ΠΏΡ€ΠΈΠ²ΠΎΠ΄ΠΈΡ‚ нас ΠΊ ΠΏΡ€Π°Π²ΠΈΠ»Ρƒ, ΠΊΠΎΡ‚ΠΎΡ€ΠΎΠ΅ ΠΌΡ‹ Π½Π°Π·Ρ‹Π²Π°Π΅ΠΌ ΠΏΡ€Π°Π²ΠΈΠ»ΠΎΠΌ срСднСй Ρ‚ΠΎΡ‡ΠΊΠΈ.

      Π‘Π΅Ρ€Π΅Π΄ΠΈΠ½Π° М ΠΎΡ‚Ρ€Π΅Π·ΠΊΠ° АВ опрСдСляСтся ΠΏΠΎ ΠΏΡ€Π°Π²ΠΈΠ»Ρƒ:

      ΠŸΡ€ΠΈ использовании ΠΊΠΎΠΎΡ€Π΄ΠΈΠ½Π°Ρ‚ ΠΏΡ€Π°Π²ΠΈΠ»ΠΎ:


      ΠŸΡ€ΠΈΠΌΠ΅Ρ€ 6

      Π’Π΅Ρ€ΡˆΠΈΠ½Π°ΠΌΠΈ Ρ‚Ρ€Π΅ΡƒΠ³ΠΎΠ»ΡŒΠ½ΠΈΠΊΠ° ABC ΡΠ²Π»ΡΡŽΡ‚ΡΡ A = (1, 2), B = (4, 3) ΠΈ C = (3, 0).

      НайдитС Π΄Π»ΠΈΠ½Ρƒ прямой, ΠΏΡ€ΠΎΠ²Π΅Π΄Π΅Π½Π½ΠΎΠΉ ΠΎΡ‚ А Π΄ΠΎ сСрСдины стороны Π’Π‘ (ΠΌΠ΅Π΄ΠΈΠ°Π½Ρƒ стороны Π’Π‘). Ρ‚Ρ€Π΅ΡƒΠ³ΠΎΠ»ΡŒΠ½ΠΈΠΊ АВБ).

      ΠœΡ‹ Π½Π°Ρ‡Π½Π΅ΠΌ с нахоТдСния сСрСдины BC, ΠΈΡΠΏΠΎΠ»ΡŒΠ·ΡƒΡ ΠΏΡ€ΠΈΠ²Π΅Π΄Π΅Π½Π½ΠΎΠ΅ Π²Ρ‹ΡˆΠ΅ ΠΏΡ€Π°Π²ΠΈΠ»ΠΎ.

      НазовСм сСрСдину M ΠΈ Π½Π°ΠΉΠ΄Π΅ΠΌ Π΅Π΅ Π²Π΅ΠΊΡ‚ΠΎΡ€ полоТСния (Π²ΠΈΠ΄Π΅Ρ‚ΡŒ схСму).

      = βˆ™ + βˆ™

      Π‘Π»Π΅Π΄ΠΎΠ²Π°Ρ‚Π΅Π»ΡŒΠ½ΠΎ, M, сСрСдина Π’Π‘, ΠΈΠΌΠ΅Π΅Ρ‚ ΠΊΠΎΠΎΡ€Π΄ΠΈΠ½Π°Ρ‚Ρ‹
      . М = (3, 1).

      Π”Π°Π»Π΅Π΅ Π½Π°Ρ…ΠΎΠ΄ΠΈΠΌ ΠΊΠΎΠΎΡ€Π΄ΠΈΠ½Π°Ρ‚Ρ‹ Π²Π΅ΠΊΡ‚ΠΎΡ€Π° .

      НаконСц, ΠΌΡ‹ ΠΌΠΎΠΆΠ΅ΠΌ Π½Π°ΠΉΡ‚ΠΈ Π΄Π»ΠΈΠ½Ρƒ Π²Π΅ΠΊΡ‚ΠΎΡ€Π° ΠΊΠ°ΠΊ трСбуСтся.

      Β Β Β Β Β Β Β Β Β Β  β‰ˆ 2,55

      Π’Ρ€ΠΈ ΠΌΠ΅Π΄ΠΈΠ°Π½Ρ‹ Ρ‚Ρ€Π΅ΡƒΠ³ΠΎΠ»ΡŒΠ½ΠΈΠΊΠ° ΠΏΠ΅Ρ€Π΅ΡΠ΅ΠΊΠ°ΡŽΡ‚ΡΡ Π² ΠΎΠ΄Π½ΠΎΠΉ Ρ‚ΠΎΡ‡ΠΊΠ΅, Π½Π°Π·Ρ‹Π²Π°Π΅ΠΌΠΎΠΉ Ρ†Π΅Π½Ρ‚Ρ€ Ρ‚Ρ€Π΅ΡƒΠ³ΠΎΠ»ΡŒΠ½ΠΈΠΊΠ° (ΠΎΠ±ΠΎΠ·Π½Π°Ρ‡Π΅Π½ Π’ Π½Π° Π΄ΠΈΠ°Π³Ρ€Π°ΠΌΠΌΠ΅). Если ΠΌΡ‹ Π·Π½Π°Π΅ΠΌ ΠΊΠΎΠΎΡ€Π΄ΠΈΠ½Π°Ρ‚Ρ‹ Π²Π΅Ρ€ΡˆΠΈΠ½ Ρ‚Ρ€Π΅ΡƒΠ³ΠΎΠ»ΡŒΠ½ΠΈΠΊΠ° ΠΌΠΎΠΆΠ½ΠΎ Π½Π°ΠΉΡ‚ΠΈ ΠΊΠΎΠΎΡ€Π΄ΠΈΠ½Π°Ρ‚Ρ‹ T ΠΏΠΎ простой Ρ„ΠΎΡ€ΠΌΡƒΠ»Π΅. Π­Ρ‚Π° Ρ„ΠΎΡ€ΠΌΡƒΠ»Π° находится Π°Π½Π°Π»ΠΎΠ³ΠΈΡ‡Π½ΠΎ ΠŸΡ€Π°Π²ΠΈΠ»ΠΎ срСднСй Ρ‚ΠΎΡ‡ΠΊΠΈ.

      ΠœΡ‹ ΠΌΠΎΠΆΠ΅ΠΌ Π΄ΠΎΡΡ‚ΠΈΡ‡ΡŒ T Ρ‡Π΅Ρ€Π΅Π· всС Ρ‚Ρ€ΠΈ Π²Π΅Ρ€ΡˆΠΈΠ½Ρ‹ Ρ‚Ρ€Π΅ΡƒΠ³ΠΎΠ»ΡŒΠ½ΠΈΠΊΠ°, Ρ‚ΠΎΠ³Π΄Π° ΠΌΡ‹ добавляСм Ρ‚Ρ€ΠΈ Π²Π΅ΠΊΡ‚ΠΎΡ€Π½Ρ‹Ρ… выраТСния вмСстС.

      Π’ ΡƒΡ€ΠΎΠΊΠ΅ 2 ΠΎ Ρ‚Ρ€Π΅ΡƒΠ³ΠΎΠ»ΡŒΠ½ΠΈΠΊΠ°Ρ… ΠΌΡ‹ Π²ΠΈΠ΄Π΅Π»ΠΈ, Ρ‡Ρ‚ΠΎ всС ΠΌΠ΅Π΄ΠΈΠ°Π½Ρ‹ ΠΏΠ΅Ρ€Π΅ΡΠ΅ΠΊΠ°ΡŽΡ‚ΡΡ Π² ΠΎΠ΄Π½ΠΎΠΉ Ρ‚ΠΎΡ‡ΠΊΠ΅. Ρ‚ΠΎΡ‡ΠΊΠΈ, дСлящиС Π΄Ρ€ΡƒΠ³ Π΄Ρ€ΡƒΠ³Π° Π² ΡΠΎΠΎΡ‚Π½ΠΎΡˆΠ΅Π½ΠΈΠΈ 2:1 ΠΈΠ»ΠΈ 2/1. ΠžΡ‚ΡΡŽΠ΄Π° ΠΌΡ‹ Π·Π½Π°Π΅ΠΌ, Ρ‡Ρ‚ΠΎ Π΄Π»ΠΈΠ½Π° Π²Π΅ΠΊΡ‚ΠΎΡ€Π° Π² Π΄Π²Π° Ρ€Π°Π·Π° большС, Ρ‡Π΅ΠΌ ΠΈ поэтому

      Π·Π½Π°ΠΊ Ρ€Π°Π²Π½ΠΎ βˆ™ Π° Ρ‚Π°ΠΊΠΆΠ΅ Π·Π½Π°ΠΊ Ρ€Π°Π²Π½ΠΎ βˆ’βˆ™. Π˜ΡΠΏΠΎΠ»ΡŒΠ·ΡƒΡ это, ΠΌΡ‹ ΠΌΠΎΠΆΠ΅ΠΌ Π½Π°ΠΏΠΈΡΠ°Ρ‚ΡŒ Ρ‚Ρ€ΠΈ уравнСния:

      = + βˆ™Β 

      = + βˆ™ — βˆ™

      = — βˆ™ — βˆ™

      Когда ΠΌΡ‹ слоТим ΠΈΡ… вмСстС, Π²Ρ‹Ρ…ΠΎΠ΄ΠΈΡ‚ ΠΈ ΠΏΠΎΠ»ΡƒΡ‡Π°Π΅ΠΌ:

      3= + +

      Π§Ρ‚ΠΎΠ±Ρ‹ Π½Π°ΠΉΡ‚ΠΈ ΠΊΠΎΠΎΡ€Π΄ΠΈΠ½Π°Ρ‚Ρ‹ T, ΠΌΡ‹ Π±Π΅Ρ€Π΅ΠΌ срСднСС Π·Π½Π°Ρ‡Π΅Π½ΠΈΠ΅ x ΠΈ y ΠΊΠΎΠΎΡ€Π΄ΠΈΠ½Π°Ρ‚Ρ‹ Π²Π΅Ρ€ΡˆΠΈΠ½ соотвСтствСнно.

      Π’Π°ΠΊΠΈΠΌ ΠΎΠ±Ρ€Π°Π·ΠΎΠΌ, ΠΌΡ‹ Π½Π°Ρ…ΠΎΠ΄ΠΈΠΌ Ρ‚ΠΎΡ‡ΠΊΡƒ пСрСсСчСния T ΠΌΠ΅Π΄ΠΈΠ°Π½ Ρ‚Ρ€Π΅ΡƒΠ³ΠΎΠ»ΡŒΠ½ΠΈΠΊΠ° ΠΏΡƒΡ‚Π΅ΠΌ нахоТдСния своСго Ρ€ΠΎΠ΄Π° срСднСго Π²Π΅ΠΊΡ‚ΠΎΡ€ΠΎΠ² полоТСния Π²Π΅Ρ€ΡˆΠΈΠ½Ρ‹. Π’Π°ΠΊΠΈΠΌ ΠΎΠ±Ρ€Π°Π·ΠΎΠΌ, это ΠΏΡ€Π°Π²ΠΈΠ»ΠΎ являСтся Ρ€Π°ΡΡˆΠΈΡ€Π΅Π½ΠΈΠ΅ΠΌ ΠΏΡ€Π°Π²ΠΈΠ»Π° срСднСй Ρ‚ΠΎΡ‡ΠΊΠΈ.


      ΠŸΡ€ΠΈΠΌΠ΅Ρ€ 7

      НайдитС Ρ‚ΠΎΡ‡ΠΊΡƒ пСрСсСчСния Π’ ΠΌΠ΅Π΄ΠΈΠ°Π½ Ρ‚Ρ€Π΅ΡƒΠ³ΠΎΠ»ΡŒΠ½ΠΈΠΊΠ° АВБ ( Ρ†Π΅Π½Ρ‚Ρ€ ) ΠΏΡ€ΠΈ условии, Ρ‡Ρ‚ΠΎ A = (1, 2), B = (4, 3) ΠΈ C = (3, 0) (см. схСму).

      Π¦Π΅Π½Ρ‚Ρ€ Π’ = (2, 1) .


      ΠŸΠΎΠΏΡ€ΠΎΠ±ΡƒΠΉΡ‚Π΅ Π’ΠΈΠΊΡ‚ΠΎΡ€ΠΈΠ½Π° 3 Π½Π° Π’Π΅ΠΊΡ‚ΠΎΡ€Ρ‹.
      НС Π·Π°Π±Ρ‹Π²Π°ΠΉΡ‚Π΅ ΠΈΡΠΏΠΎΠ»ΡŒΠ·ΠΎΠ²Π°Ρ‚ΡŒ ΠΊΠΎΠ½Ρ‚Ρ€ΠΎΠ»ΡŒΠ½Ρ‹ΠΉ список, Ρ‡Ρ‚ΠΎΠ±Ρ‹ ΠΎΡ‚ΡΠ»Π΅ΠΆΠΈΠ²Π°Ρ‚ΡŒ свою Ρ€Π°Π±ΠΎΡ‚Ρƒ.

      Β 

      Β 

      Β 

      Β 

      Β 

      Β 

      Β 

      Β 

      Β 

      Π˜ΡΡ‡ΠΈΡΠ»Π΅Π½ΠΈΠ΅ III. Π”Π»ΠΈΠ½Π° Π΄ΡƒΠ³ΠΈ с Π²Π΅ΠΊΡ‚ΠΎΡ€Π½Ρ‹ΠΌΠΈ функциями

      Онлайн-Π·Π°ΠΌΠ΅Ρ‚ΠΊΠΈ Пола
      Главная / Π˜ΡΡ‡ΠΈΡΠ»Π΅Π½ΠΈΠ΅ III / Π’Ρ€Π΅Ρ…ΠΌΠ΅Ρ€Π½ΠΎΠ΅ пространство / Π”Π»ΠΈΠ½Π° Π΄ΡƒΠ³ΠΈ с Π²Π΅ΠΊΡ‚ΠΎΡ€Π½Ρ‹ΠΌΠΈ функциями

      ΠŸΠΎΠΊΠ°Π·Π°Ρ‚ΡŒ мобильноС ΡƒΠ²Π΅Π΄ΠΎΠΌΠ»Π΅Π½ΠΈΠ΅ ΠŸΠΎΠΊΠ°Π·Π°Ρ‚ΡŒ всС ΠΏΡ€ΠΈΠΌΠ΅Ρ‡Π°Π½ΠΈΡΒ Π‘ΠΊΡ€Ρ‹Ρ‚ΡŒ всС примСчания

      МобильноС ΡƒΠ²Π΅Π΄ΠΎΠΌΠ»Π΅Π½ΠΈΠ΅

      ΠŸΠΎΡ…ΠΎΠΆΠ΅, Π²Ρ‹ ΠΈΡΠΏΠΎΠ»ΡŒΠ·ΡƒΠ΅Ρ‚Π΅ устройство с Β«ΡƒΠ·ΠΊΠΎΠΉΒ» ΡˆΠΈΡ€ΠΈΠ½ΠΎΠΉ экрана ( Ρ‚. Π΅. Π²Ρ‹, вСроятно, ΠΈΡΠΏΠΎΠ»ΡŒΠ·ΡƒΠ΅Ρ‚Π΅ ΠΌΠΎΠ±ΠΈΠ»ΡŒΠ½Ρ‹ΠΉ Ρ‚Π΅Π»Π΅Ρ„ΠΎΠ½). Из-Π·Π° Ρ…Π°Ρ€Π°ΠΊΡ‚Π΅Ρ€Π° ΠΌΠ°Ρ‚Π΅ΠΌΠ°Ρ‚ΠΈΠΊΠΈ Π½Π° этом сайтС Π»ΡƒΡ‡ΡˆΠ΅ всСго ΠΏΡ€ΠΎΡΠΌΠ°Ρ‚Ρ€ΠΈΠ²Π°Ρ‚ΡŒ Π² Π»Π°Π½Π΄ΡˆΠ°Ρ„Ρ‚Π½ΠΎΠΌ Ρ€Π΅ΠΆΠΈΠΌΠ΅. Если вашС устройство Π½Π΅ находится Π² Π»Π°Π½Π΄ΡˆΠ°Ρ„Ρ‚Π½ΠΎΠΌ Ρ€Π΅ΠΆΠΈΠΌΠ΅, ΠΌΠ½ΠΎΠ³ΠΈΠ΅ уравнСния Π±ΡƒΠ΄ΡƒΡ‚ ΠΎΡ‚ΠΎΠ±Ρ€Π°ΠΆΠ°Ρ‚ΡŒΡΡ сбоку вашСго устройства (Π΄ΠΎΠ»ΠΆΠ½Π° Π±Ρ‹Ρ‚ΡŒ Π²ΠΎΠ·ΠΌΠΎΠΆΠ½ΠΎΡΡ‚ΡŒ ΠΏΡ€ΠΎΠΊΡ€ΡƒΡ‚ΠΊΠΈ, Ρ‡Ρ‚ΠΎΠ±Ρ‹ ΡƒΠ²ΠΈΠ΄Π΅Ρ‚ΡŒ ΠΈΡ…), Π° Π½Π΅ΠΊΠΎΡ‚ΠΎΡ€Ρ‹Π΅ ΠΏΡƒΠ½ΠΊΡ‚Ρ‹ мСню Π±ΡƒΠ΄ΡƒΡ‚ ΠΎΠ±Ρ€Π΅Π·Π°Π½Ρ‹ ΠΈΠ·-Π·Π° ΡƒΠ·ΠΊΠΎΠΉ ΡˆΠΈΡ€ΠΈΠ½Ρ‹ экрана.

      Π Π°Π·Π΄Π΅Π» 1-9: Π”Π»ΠΈΠ½Π° Π΄ΡƒΠ³ΠΈ с Π²Π΅ΠΊΡ‚ΠΎΡ€Π½Ρ‹ΠΌΠΈ функциями

      Π’ этом Ρ€Π°Π·Π΄Π΅Π»Π΅ ΠΌΡ‹ ΠΏΠ΅Ρ€Π΅Π²Π΅Π΄Π΅ΠΌ ΡΡ‚Π°Ρ€ΡƒΡŽ Ρ„ΠΎΡ€ΠΌΡƒΠ»Ρƒ Π² Ρ‚Π΅Ρ€ΠΌΠΈΠ½Ρ‹ Π²Π΅ΠΊΡ‚ΠΎΡ€Π½Ρ‹Ρ… Ρ„ΡƒΠ½ΠΊΡ†ΠΈΠΉ. ΠœΡ‹ Ρ…ΠΎΡ‚ΠΈΠΌ ΠΎΠΏΡ€Π΅Π΄Π΅Π»ΠΈΡ‚ΡŒ Π΄Π»ΠΈΠ½Ρƒ Π²Π΅ΠΊΡ‚ΠΎΡ€Π½ΠΎΠΉ Ρ„ΡƒΠ½ΠΊΡ†ΠΈΠΈ,

      \[\vec r\left( t \right) = \left\langle {f\left( t \right),g\left( t \right),h\left( t \right)} \right\rangle \ ]

      Π½Π° ΠΈΠ½Ρ‚Π΅Ρ€Π²Π°Π»Π΅ \(a \le t \le b\).

      ΠœΡ‹ ΡƒΠΆΠ΅ Π·Π½Π°Π΅ΠΌ, ΠΊΠ°ΠΊ это ΡΠ΄Π΅Π»Π°Ρ‚ΡŒ. Напомним, Ρ‡Ρ‚ΠΎ ΠΌΡ‹ ΠΌΠΎΠΆΠ΅ΠΌ Π·Π°ΠΏΠΈΡΠ°Ρ‚ΡŒ Π²Π΅ΠΊΡ‚ΠΎΡ€Π½ΡƒΡŽ Ρ„ΡƒΠ½ΠΊΡ†ΠΈΡŽ Π² ΠΏΠ°Ρ€Π°ΠΌΠ΅Ρ‚Ρ€ΠΈΡ‡Π΅ΡΠΊΡƒΡŽ Ρ„ΠΎΡ€ΠΌΡƒ,

      \[x = f\left( t \right)\hspace{0.25in}y = g\left( t \right)\hspace{0. 25in}z = h\left( t \right)\] 9{{\,b}}{{\Π²Π»Π΅Π²ΠΎ\| {\ vec r ‘\ Π²Π»Π΅Π²ΠΎ ( Ρ‚ \ Π²ΠΏΡ€Π°Π²ΠΎ)} \ Π²ΠΏΡ€Π°Π²ΠΎ \ | \, dt}} \]

      Π”Π°Π²Π°ΠΉΡ‚Π΅ рассмотрим быстрый ΠΏΡ€ΠΈΠΌΠ΅Ρ€ этого.

      ΠŸΡ€ΠΈΠΌΠ΅Ρ€ 1 ΠžΠΏΡ€Π΅Π΄Π΅Π»ΠΈΡ‚ΡŒ Π΄Π»ΠΈΠ½Ρƒ ΠΊΡ€ΠΈΠ²ΠΎΠΉ \(\vec r\left( t \right) = \left\langle {2t,3\sin \left( {2t} \right),3\cos \left( {2t} \right)} \right\rangle \) Π½Π° ΠΎΡ‚Ρ€Π΅Π·ΠΊΠ΅ \(0 \le t \le 2\pi \).

      ΠŸΠΎΠΊΠ°Π·Π°Ρ‚ΡŒ Ρ€Π΅ΡˆΠ΅Π½ΠΈΠ΅

      Π‘Π½Π°Ρ‡Π°Π»Π° Π½Π°ΠΌ понадобится ΠΊΠ°ΡΠ°Ρ‚Π΅Π»ΡŒΠ½Ρ‹ΠΉ Π²Π΅ΠΊΡ‚ΠΎΡ€ ΠΈ Π΅Π³ΠΎ Π²Π΅Π»ΠΈΡ‡ΠΈΠ½Π°.

      9{{\, Ρ‚}} {{\ Π²Π»Π΅Π²ΠΎ \| {\ vec r ‘\ Π²Π»Π΅Π²ΠΎ ( ΠΈ \ Π²ΠΏΡ€Π°Π²ΠΎ)} \ Π²ΠΏΡ€Π°Π²ΠΎ \ | \, du}} \]

      ΠŸΡ€Π΅ΠΆΠ΄Π΅ Ρ‡Π΅ΠΌ ΠΌΡ‹ рассмотрим, ΠΏΠΎΡ‡Π΅ΠΌΡƒ это ΠΌΠΎΠΆΠ΅Ρ‚ Π±Ρ‹Ρ‚ΡŒ Π²Π°ΠΆΠ½ΠΎ, Π΄Π°Π²Π°ΠΉΡ‚Π΅ рассмотрим нСбольшой ΠΏΡ€ΠΈΠΌΠ΅Ρ€.

      ΠŸΡ€ΠΈΠΌΠ΅Ρ€ 2. ΠžΠΏΡ€Π΅Π΄Π΅Π»ΠΈΡ‚ΡŒ Ρ„ΡƒΠ½ΠΊΡ†ΠΈΡŽ Π΄Π»ΠΈΠ½Ρ‹ Π΄ΡƒΠ³ΠΈ для \(\vec r\left( t \right) = \left\langle {2t,3\sin \left( {2t} \right),3\cos \left( {2t} \ΠΏΡ€Π°Π²ΠΎ)} \ΠΏΡ€Π°Π²ΠΎ\ΡƒΠ³ΠΎΠ» \).

      ΠŸΠΎΠΊΠ°Π·Π°Ρ‚ΡŒ Ρ€Π΅ΡˆΠ΅Π½ΠΈΠ΅

      Из ΠΏΡ€Π΅Π΄Ρ‹Π΄ΡƒΡ‰Π΅Π³ΠΎ ΠΏΡ€ΠΈΠΌΠ΅Ρ€Π° ΠΌΡ‹ Π·Π½Π°Π΅ΠΌ, Ρ‡Ρ‚ΠΎ

      \[\слСва\| {\ vec r ‘\ Π²Π»Π΅Π²ΠΎ ( Ρ‚ \ Π²ΠΏΡ€Π°Π²ΠΎ)} \ Π²ΠΏΡ€Π°Π²ΠΎ \ | = 2\ΠΊΠ²Π°Π΄Ρ€Π°Ρ‚ {10} \] 9Ρ‚ = 2\sqrt {10} \,Ρ‚\]

      Π›Π°Π΄Π½ΠΎ, Ρ‚ΠΎΠ»ΡŒΠΊΠΎ Π·Π°Ρ‡Π΅ΠΌ Π½Π°ΠΌ это? Π§Ρ‚ΠΎ ΠΆ, Π΄Π°Π²Π°ΠΉΡ‚Π΅ возьмСм Ρ€Π΅Π·ΡƒΠ»ΡŒΡ‚Π°Ρ‚ ΠΏΡ€ΠΈΠΌΠ΅Ρ€Π° Π²Ρ‹ΡˆΠ΅ ΠΈ Ρ€Π΅ΡˆΠΈΠΌ Π΅Π³ΠΎ для \(t\).

      \[t = \frac{s}{{2\sqrt {10} }}\]

      Π’Π΅ΠΏΠ΅Ρ€ΡŒ, взяв это ΠΈ подставив Π² ΠΈΡΡ…ΠΎΠ΄Π½ΡƒΡŽ Π²Π΅ΠΊΡ‚ΠΎΡ€Π½ΡƒΡŽ Ρ„ΡƒΠ½ΠΊΡ†ΠΈΡŽ, ΠΌΡ‹ ΠΌΠΎΠΆΠ΅ΠΌ Ρ€Π΅ΠΏΠ°Ρ€Π°ΠΌΠ΅Ρ‚Ρ€ΠΈΠ·ΠΎΠ²Π°Ρ‚ΡŒ Ρ„ΡƒΠ½ΠΊΡ†ΠΈΡŽ Π² Π²ΠΈΠ΄Π΅ \(\vec r\left( {t\left( s \right)} \right)\). Для нашСй Ρ„ΡƒΠ½ΠΊΡ†ΠΈΠΈ это

      . \[\ vec r\left( {t\left(s\right)} \right) = \left\langle {\ frac {s}{{\ sqrt {10}}},3\sin \left( {\ frac {s} {{\ sqrt {10} }}} \right), 3 \ cos \ left ( {\ frac {s} {{\ sqrt {10} }}} \ right)} \ right \ rangle \]

      Π˜Ρ‚Π°ΠΊ, Π·Π°Ρ‡Π΅ΠΌ Π½Π°ΠΌ это? Π§Ρ‚ΠΎ ΠΆ, с Ρ€Π΅ΠΏΠ°Ρ€Π°ΠΌΠ΅Ρ‚Ρ€ΠΈΠ·Π°Ρ†ΠΈΠ΅ΠΉ ΠΌΡ‹ Ρ‚Π΅ΠΏΠ΅Ρ€ΡŒ ΠΌΠΎΠΆΠ΅ΠΌ ΡΠΊΠ°Π·Π°Ρ‚ΡŒ, Π³Π΄Π΅ ΠΌΡ‹ находимся Π½Π° ΠΊΡ€ΠΈΠ²ΠΎΠΉ послС Ρ‚ΠΎΠ³ΠΎ, ΠΊΠ°ΠΊ ΠΌΡ‹ ΠΏΡ€ΠΎΡˆΠ»ΠΈ расстояниС \(s\) вдоль ΠΊΡ€ΠΈΠ²ΠΎΠΉ. Π’Π°ΠΊΠΆΠ΅ ΠΎΠ±Ρ€Π°Ρ‚ΠΈΡ‚Π΅ Π²Π½ΠΈΠΌΠ°Π½ΠΈΠ΅, Ρ‡Ρ‚ΠΎ ΠΌΡ‹ Π½Π°Ρ‡Π½Π΅ΠΌ ΠΈΠ·ΠΌΠ΅Ρ€Π΅Π½ΠΈΠ΅ расстояния с Ρ‚ΠΎΠ³ΠΎ мСста, Π³Π΄Π΅ ΠΌΡ‹ находимся Π² Ρ‚ΠΎΡ‡ΠΊΠ΅ \(t = 0\).

      ΠŸΡ€ΠΈΠΌΠ΅Ρ€ 3. Π“Π΄Π΅ Π½Π° ΠΊΡ€ΠΈΠ²ΠΎΠΉ \(\vec r\left( t \right) = \left\langle {2t,3\sin \left( {2t} \right),3\cos \left( {2t} \right )} \right\rangle \) послС ΠΏΡƒΡ‚Π΅ΡˆΠ΅ΡΡ‚Π²ΠΈΡ Π½Π° расстояниС \(\displaystyle \frac{{\pi \sqrt {10} }}{3}\)?

      ΠŸΠΎΠΊΠ°Π·Π°Ρ‚ΡŒ Ρ€Π΅ΡˆΠ΅Π½ΠΈΠ΅

      Π§Ρ‚ΠΎΠ±Ρ‹ ΠΎΠΏΡ€Π΅Π΄Π΅Π»ΠΈΡ‚ΡŒ это, Π½Π°ΠΌ Π½ΡƒΠΆΠ½Π° рСпарамСтризация, ΠΊΠΎΡ‚ΠΎΡ€ΡƒΡŽ ΠΌΡ‹ ΠΏΠΎΠ»ΡƒΡ‡ΠΈΠ»ΠΈ свСрху.

      \[\ vec r\left( {t\left(s\right)} \right) = \left\langle {\ frac {s}{{\ sqrt {10}}},3\sin \left( {\ frac {s} {{\ sqrt {10} }}} \right), 3 \ cos \ left ( {\ frac {s} {{\ sqrt {10} }}} \ right)} \ right \ rangle \]

      Π—Π°Ρ‚Π΅ΠΌ, Ρ‡Ρ‚ΠΎΠ±Ρ‹ ΠΎΠΏΡ€Π΅Π΄Π΅Π»ΠΈΡ‚ΡŒ, Π³Π΄Π΅ ΠΌΡ‹ находимся, всС, Ρ‡Ρ‚ΠΎ Π½Π°ΠΌ Π½ΡƒΠΆΠ½ΠΎ ΡΠ΄Π΅Π»Π°Ρ‚ΡŒ, это ΠΏΠΎΠ΄ΡΡ‚Π°Π²ΠΈΡ‚ΡŒ сюда \(s = \frac{{\pi \sqrt {10} }}{3}\), ΠΈ ΠΌΡ‹ ΠΏΠΎΠ»ΡƒΡ‡ΠΈΠΌ нашС мСстополоТСниС.

      \[\ vec r \ left ( {t \ left ( {\ frac {{\ pi \ sqrt {10}}} {3}} \ right)} \ right) = \ left \ langle {\ frac {\ pi} {3},3\sin\left({\frac{\pi}{3}}\right),3\cos\left({\frac{\pi}{3}}\right)} \right\rangle = \ left \ langle {\ frac {\ pi} {3}, \ frac {{3 \ sqrt 3}} {2}, \ frac {3} {2}} \ right \ rangle \]

      Π˜Ρ‚Π°ΠΊ, пройдя расстояниС \(\frac{{\pi \sqrt {10}}}{3}\) ΠΏΠΎ ΠΊΡ€ΠΈΠ²ΠΎΠΉ, ΠΌΡ‹ находимся Π² Ρ‚ΠΎΡ‡ΠΊΠ΅ \(\left( {\frac{\pi }{ 3},\frac{{3\sqrt 3 }}{2},\frac{3}{2}} \right)\).

      Π’ΠΈΠ΄Π΅ΠΎ-вопрос: вычислСниС Π΄Π»ΠΈΠ½ Π΄Π²ΡƒΡ… Π²Π΅ΠΊΡ‚ΠΎΡ€ΠΎΠ² ΠΏΠΎ ΠΈΡ… Π²Π΅ΠΊΡ‚ΠΎΡ€Π½ΠΎΠΌΡƒ ΠΏΡ€ΠΎΠΈΠ·Π²Π΅Π΄Π΅Π½ΠΈΡŽ ΠΏΡ€ΠΈ условии, Ρ‡Ρ‚ΠΎ Π²Π΅ΠΊΡ‚ΠΎΡ€Ρ‹ пСрпСндикулярны Π΄Ρ€ΡƒΠ³ Π΄Ρ€ΡƒΠ³Ρƒ

      Π‘Ρ‚Π΅Π½ΠΎΠ³Ρ€Π°ΠΌΠΌΠ° Π²ΠΈΠ΄Π΅ΠΎ

      Π”Π²Π° Π²Π΅ΠΊΡ‚ΠΎΡ€Π° 𝐚 ΠΈ 𝐛 ΠΈΠΌΠ΅ΡŽΡ‚ ΠΎΠ΄ΠΈΠ½Π°ΠΊΠΎΠ²ΡƒΡŽ Π΄Π»ΠΈΠ½Ρƒ ΠΈ пСрпСндикулярны Π΄Ρ€ΡƒΠ³ Π΄Ρ€ΡƒΠ³Ρƒ. Π’Π΅ΠΊΡ‚ΠΎΡ€Π½ΠΎΠ΅ ΠΏΡ€ΠΎΠΈΠ·Π²Π΅Π΄Π΅Π½ΠΈΠ΅ 𝐚 ΠΈ 𝐛 ΠΈΠΌΠ΅Π΅Ρ‚ Π²Π΅Π»ΠΈΡ‡ΠΈΠ½Ρƒ Π΄Π΅Π²ΡΡ‚ΡŒ. Какова Π΄Π»ΠΈΠ½Π° ΠΊΠ°ΠΆΠ΄ΠΎΠ³ΠΎ Π²Π΅ΠΊΡ‚ΠΎΡ€Π°?

      Π˜Ρ‚Π°ΠΊ, это вопрос ΠΎ Π²Π΅ΠΊΡ‚ΠΎΡ€Π½Ρ‹Ρ… произвСдСниях. Π’ этом вопросС Π½Π°ΠΌ ΡΠΎΠΎΠ±Ρ‰Π°ΡŽΡ‚ Π²Π΅Π»ΠΈΡ‡ΠΈΠ½Ρƒ Π²Π΅ΠΊΡ‚ΠΎΡ€Π½ΠΎΠ³ΠΎ произвСдСния Π΄Π²ΡƒΡ… Π²Π΅ΠΊΡ‚ΠΎΡ€ΠΎΠ² 𝐚 ΠΈ 𝐛. И Π½Π°ΠΌ говорят, Ρ‡Ρ‚ΠΎ эти Π²Π΅ΠΊΡ‚ΠΎΡ€Ρ‹ ΠΈΠΌΠ΅ΡŽΡ‚ ΠΎΠ΄ΠΈΠ½Π°ΠΊΠΎΠ²ΡƒΡŽ Π΄Π»ΠΈΠ½Ρƒ ΠΈ Ρ‡Ρ‚ΠΎ ΠΎΠ½ΠΈ пСрпСндикулярны Π΄Ρ€ΡƒΠ³ Π΄Ρ€ΡƒΠ³Ρƒ. Нас просят Π½Π°ΠΉΡ‚ΠΈ Π΄Π»ΠΈΠ½Ρƒ ΠΊΠ°ΠΆΠ΄ΠΎΠ³ΠΎ Π²Π΅ΠΊΡ‚ΠΎΡ€Π°. Π”Π°Π²Π°ΠΉΡ‚Π΅ Π½Π°Ρ‡Π½Π΅ΠΌ с Ρ‚ΠΎΠ³ΠΎ, Ρ‡Ρ‚ΠΎ возьмСм ΠΈΠ½Ρ„ΠΎΡ€ΠΌΠ°Ρ†ΠΈΡŽ, ΠΊΠΎΡ‚ΠΎΡ€ΡƒΡŽ Π΄Π°Π΅Ρ‚ Π½Π°ΠΌ вопрос ΠΎ Π²Π΅ΠΊΡ‚ΠΎΡ€Π°Ρ… 𝐚 ΠΈ 𝐛, ΠΈ ΠΏΠΎΠΏΡ€ΠΎΠ±ΡƒΠ΅ΠΌ ΠΈΡΠΏΠΎΠ»ΡŒΠ·ΠΎΠ²Π°Ρ‚ΡŒ Π΅Π΅ для записи Π²Ρ‹Ρ€Π°ΠΆΠ΅Π½ΠΈΠΉ для 𝐚 ΠΈ 𝐛 Π² ΠΊΠΎΠΌΠΏΠΎΠ½Π΅Π½Ρ‚Π½ΠΎΠΉ Ρ„ΠΎΡ€ΠΌΠ΅.

      Нам Π½ΠΈΡ‡Π΅Π³ΠΎ Π½Π΅ говорят ΠΎΠ± Π°Π±ΡΠΎΠ»ΡŽΡ‚Π½ΠΎΠΉ ΠΎΡ€ΠΈΠ΅Π½Ρ‚Π°Ρ†ΠΈΠΈ ΠΎΠ±ΠΎΠΈΡ… Π²Π΅ΠΊΡ‚ΠΎΡ€ΠΎΠ², Ρ‚ΠΎΠ»ΡŒΠΊΠΎ Ρ‚ΠΎ, Ρ‡Ρ‚ΠΎ ΠΎΠ½ΠΈ пСрпСндикулярны Π΄Ρ€ΡƒΠ³ Π΄Ρ€ΡƒΠ³Ρƒ. Π’Π°ΠΊΠΈΠΌ ΠΎΠ±Ρ€Π°Π·ΠΎΠΌ, ΠΌΡ‹ Π²ΠΎΠ»ΡŒΠ½Ρ‹ ΠΎΡ€ΠΈΠ΅Π½Ρ‚ΠΈΡ€ΠΎΠ²Π°Ρ‚ΡŒ ΠΈΡ… ΠΏΠΎ своСму ΡƒΡΠΌΠΎΡ‚Ρ€Π΅Π½ΠΈΡŽ. ΠΠ±ΡΠΎΠ»ΡŽΡ‚Π½ΠΎΠ΅ Π½Π°ΠΏΡ€Π°Π²Π»Π΅Π½ΠΈΠ΅ Π² пространствС Π½Π΅ повлияСт Π½Π° наш ΠΎΡ‚Π²Π΅Ρ‚. И Π²ΠΎΠΎΠ±Ρ‰Π΅, Π² этом случаС ΠΏΡ€ΠΎΡ‰Π΅ всСго Π²Ρ‹Π±Ρ€Π°Ρ‚ΡŒ ΠΎΡ€ΠΈΠ΅Π½Ρ‚Π°Ρ†ΠΈΡŽ Π²Π΅ΠΊΡ‚ΠΎΡ€ΠΎΠ² ΠΏΠΎ осям ΠΊΠΎΠΎΡ€Π΄ΠΈΠ½Π°Ρ‚ для выполнСния расчСта. Π˜Ρ‚Π°ΠΊ, Π΄Π°Π²Π°ΠΉΡ‚Π΅ прСдставим, Ρ‡Ρ‚ΠΎ наш Π²Π΅ΠΊΡ‚ΠΎΡ€ 𝐚 ΡƒΠΊΠ°Π·Ρ‹Π²Π°Π΅Ρ‚ вдоль оси π‘₯, Π° наш Π²Π΅ΠΊΡ‚ΠΎΡ€ 𝐛 ΡƒΠΊΠ°Π·Ρ‹Π²Π°Π΅Ρ‚ Π½Π° 𝑦-ось. Π­Ρ‚ΠΎ выполняСт Ρ‚Ρ€Π΅Π±ΠΎΠ²Π°Π½ΠΈΠ΅ вопроса ΠΎ Ρ‚ΠΎΠΌ, Ρ‡Ρ‚ΠΎ Π΄Π²Π° Π²Π΅ΠΊΡ‚ΠΎΡ€Π° пСрпСндикулярны Π΄Ρ€ΡƒΠ³ Π΄Ρ€ΡƒΠ³Ρƒ.

      Π—Π°Ρ‚Π΅ΠΌ ΠΌΡ‹ ΠΌΠΎΠΆΠ΅ΠΌ Π·Π°ΠΏΠΈΡΠ°Ρ‚ΡŒ наши Π²Π΅ΠΊΡ‚ΠΎΡ€Ρ‹ 𝐚 ΠΈ 𝐛 Π² ΠΊΠΎΠΌΠΏΠΎΠ½Π΅Π½Ρ‚Π½ΠΎΠΉ Ρ„ΠΎΡ€ΠΌΠ΅. Напомним, Ρ‡Ρ‚ΠΎ 𝐒 β€” это Π΅Π΄ΠΈΠ½ΠΈΡ‡Π½Ρ‹ΠΉ Π²Π΅ΠΊΡ‚ΠΎΡ€ Π² π‘₯-Π½Π°ΠΏΡ€Π°Π²Π»Π΅Π½ΠΈΠΈ, поэтому Π²Π΅ΠΊΡ‚ΠΎΡ€ 𝐚 ΠΌΠΎΠΆΠ½ΠΎ Π·Π°ΠΏΠΈΡΠ°Ρ‚ΡŒ ΠΊΠ°ΠΊ Π²Π΅Π»ΠΈΡ‡ΠΈΠ½Ρƒ 𝐚, ΡƒΠΌΠ½ΠΎΠΆΠ΅Π½Π½ΡƒΡŽ Π½Π° Π΅Π΄ΠΈΠ½ΠΈΡ‡Π½Ρ‹ΠΉ Π²Π΅ΠΊΡ‚ΠΎΡ€ 𝐒, Π° 𝐣 β€” Π΅Π΄ΠΈΠ½ΠΈΡ‡Π½Ρ‹ΠΉ Π²Π΅ΠΊΡ‚ΠΎΡ€ Π² 𝑦-Π½Π°ΠΏΡ€Π°Π²Π»Π΅Π½ΠΈΠΈ, поэтому Π²Π΅ΠΊΡ‚ΠΎΡ€ 𝐛 ΠΌΠΎΠΆΠ½ΠΎ записываСтся ΠΊΠ°ΠΊ Π²Π΅Π»ΠΈΡ‡ΠΈΠ½Π° 𝐛, умноТСнная Π½Π° Π΅Π΄ΠΈΠ½ΠΈΡ‡Π½Ρ‹ΠΉ Π²Π΅ΠΊΡ‚ΠΎΡ€ 𝐣. Но Π½Π° самом Π΄Π΅Π»Π΅ вопрос Π΄Π°Π» Π½Π°ΠΌ Π΅Ρ‰Π΅ ΠΎΠ΄Π½Ρƒ ΠΈΠ½Ρ„ΠΎΡ€ΠΌΠ°Ρ†ΠΈΡŽ ΠΎ Π²Π΅ΠΊΡ‚ΠΎΡ€Π°Ρ… 𝐚 ΠΈ 𝐛. Нам сказали, Ρ‡Ρ‚ΠΎ эти Π΄Π²Π° Π²Π΅ΠΊΡ‚ΠΎΡ€Π° ΠΈΠΌΠ΅ΡŽΡ‚ ΠΎΠ΄ΠΈΠ½Π°ΠΊΠΎΠ²ΡƒΡŽ Π΄Π»ΠΈΠ½Ρƒ.

      Π’Π΅ΠΏΠ΅Ρ€ΡŒ Π΄Π»ΠΈΠ½Π° ΠΈ Π²Π΅Π»ΠΈΡ‡ΠΈΠ½Π° Π²Π΅ΠΊΡ‚ΠΎΡ€Π° эквивалСнтны. Π˜Ρ‚Π°ΠΊ, это ΠΎΠ·Π½Π°Ρ‡Π°Π΅Ρ‚, Ρ‡Ρ‚ΠΎ Π²Π΅Π»ΠΈΡ‡ΠΈΠ½Π° Π²Π΅ΠΊΡ‚ΠΎΡ€Π° 𝐚 Π΄ΠΎΠ»ΠΆΠ½Π° Ρ€Π°Π²Π½ΡΡ‚ΡŒΡΡ Π²Π΅Π»ΠΈΡ‡ΠΈΠ½Π΅ Π²Π΅ΠΊΡ‚ΠΎΡ€Π° 𝐛. Π˜Ρ‚Π°ΠΊ, Π΄Π°Π²Π°ΠΉΡ‚Π΅ установим Π²Π΅Π»ΠΈΡ‡ΠΈΠ½Ρƒ 𝐚 ΠΈ Π²Π΅Π»ΠΈΡ‡ΠΈΠ½Ρƒ 𝐛 Ρ€Π°Π²Π½Ρ‹ΠΌΠΈ ΠΎΠ΄Π½ΠΎΠΌΡƒ ΠΈ Ρ‚ΠΎΠΌΡƒ ΠΆΠ΅ Π·Π½Π°Ρ‡Π΅Π½ΠΈΡŽ, ΠΊΠΎΡ‚ΠΎΡ€ΠΎΠ΅ ΠΌΡ‹ Π±ΡƒΠ΄Π΅ΠΌ Π½Π°Π·Ρ‹Π²Π°Ρ‚ΡŒ π‘š для Π²Π΅Π»ΠΈΡ‡ΠΈΠ½Ρ‹. Π’ΠΎΠ³Π΄Π° ΠΌΡ‹ ΠΈΠΌΠ΅Π΅ΠΌ, Ρ‡Ρ‚ΠΎ Π²Π΅ΠΊΡ‚ΠΎΡ€ 𝐚 Ρ€Π°Π²Π΅Π½ π‘š, ΡƒΠΌΠ½ΠΎΠΆΠ΅Π½Π½ΠΎΠΌΡƒ Π½Π° Π΅Π΄ΠΈΠ½ΠΈΡ‡Π½Ρ‹ΠΉ Π²Π΅ΠΊΡ‚ΠΎΡ€ 𝐒, ΠΈ Ρƒ нас Π΅ΡΡ‚ΡŒ, Ρ‡Ρ‚ΠΎ 𝐛 Ρ€Π°Π²Π΅Π½ π‘š, ΡƒΠΌΠ½ΠΎΠΆΠ΅Π½Π½ΠΎΠΌΡƒ Π½Π° Π΅Π΄ΠΈΠ½ΠΈΡ‡Π½Ρ‹ΠΉ Π²Π΅ΠΊΡ‚ΠΎΡ€ 𝐣. Π˜Ρ‚Π°ΠΊ, Ρ‚Π΅ΠΏΠ΅Ρ€ΡŒ Ρƒ нас Π΅ΡΡ‚ΡŒ выраТСния для Π²Π΅ΠΊΡ‚ΠΎΡ€Π° 𝐚 ΠΈ Π²Π΅ΠΊΡ‚ΠΎΡ€Π° 𝐛 Π² ΠΊΠΎΠΌΠΏΠΎΠ½Π΅Π½Ρ‚Π½ΠΎΠΉ Ρ„ΠΎΡ€ΠΌΠ΅. Π˜Ρ‚Π°ΠΊ, ΠΌΡ‹ достигли ΠΏΠ΅Ρ€Π²ΠΎΠ³ΠΎ шага.

      Π§Ρ‚ΠΎ дальшС? Π§Ρ‚ΠΎ ΠΆ, другая Ρ‡Π°ΡΡ‚ΡŒ ΠΈΠ½Ρ„ΠΎΡ€ΠΌΠ°Ρ†ΠΈΠΈ, ΠΊΠΎΡ‚ΠΎΡ€ΡƒΡŽ Π΄Π°Π» Π½Π°ΠΌ вопрос, касаСтся Π²Π΅ΠΊΡ‚ΠΎΡ€Π½ΠΎΠ³ΠΎ произвСдСния 𝐚 ΠΈ 𝐛. Нам говорят, Ρ‡Ρ‚ΠΎ это Π²Π΅ΠΊΡ‚ΠΎΡ€Π½ΠΎΠ΅ ΠΏΡ€ΠΎΠΈΠ·Π²Π΅Π΄Π΅Π½ΠΈΠ΅ ΠΈΠΌΠ΅Π΅Ρ‚ Π²Π΅Π»ΠΈΡ‡ΠΈΠ½Ρƒ Π΄Π΅Π²ΡΡ‚ΡŒ. Π˜Ρ‚Π°ΠΊ, Ρ€Π°Π·ΡƒΠΌΠ½Ρ‹ΠΌ ΡΠ»Π΅Π΄ΡƒΡŽΡ‰ΠΈΠΌ шагом Π±Ρ‹Π»ΠΎ Π±Ρ‹ вычислСниС Π²Π΅ΠΊΡ‚ΠΎΡ€Π½ΠΎΠ³ΠΎ произвСдСния 𝐚 ΠΈ 𝐛. И Π·Π°Ρ‚Π΅ΠΌ нашим послСдним шагом Π±ΡƒΠ΄Π΅Ρ‚ использованиС Π²Π΅ΠΊΡ‚ΠΎΡ€Π½ΠΎΠ³ΠΎ произвСдСния, Ρ‡Ρ‚ΠΎΠ±Ρ‹ Π½Π°ΠΉΡ‚ΠΈ Π΄Π»ΠΈΠ½Ρƒ π‘š ΠΊΠ°ΠΆΠ΄ΠΎΠ³ΠΎ ΠΈΠ· Π²Π΅ΠΊΡ‚ΠΎΡ€ΠΎΠ² 𝐚 ΠΈ 𝐛. Π˜Ρ‚Π°ΠΊ, Π΄Π°Π²Π°ΠΉΡ‚Π΅ вспомним ΠΎΠΏΡ€Π΅Π΄Π΅Π»Π΅Π½ΠΈΠ΅ Π²Π΅ΠΊΡ‚ΠΎΡ€Π½ΠΎΠ³ΠΎ произвСдСния Π΄Π²ΡƒΡ… Π²Π΅ΠΊΡ‚ΠΎΡ€ΠΎΠ².

      Π‘Π½Π°Ρ‡Π°Π»Π° ΠΌΡ‹ ΠΎΠΏΡ€Π΅Π΄Π΅Π»ΠΈΠΌ Π΄Π²Π° ΠΎΠ±Ρ‰ΠΈΡ… Π²Π΅ΠΊΡ‚ΠΎΡ€Π° Π² π‘₯𝑦-плоскости, Π·Π°Π³Π»Π°Π²Π½ΡƒΡŽ 𝐀 ΠΈ Π·Π°Π³Π»Π°Π²Π½ΡƒΡŽ 𝐁. ΠœΡ‹ ΠΌΠΎΠΆΠ΅ΠΌ Π·Π°ΠΏΠΈΡΠ°Ρ‚ΡŒ ΠΈΡ… Π² ΠΊΠΎΠΌΠΏΠΎΠ½Π΅Π½Ρ‚Π½ΠΎΠΉ Ρ„ΠΎΡ€ΠΌΠ΅ ΡΠ»Π΅Π΄ΡƒΡŽΡ‰ΠΈΠΌ ΠΎΠ±Ρ€Π°Π·ΠΎΠΌ. Заглавная 𝐀 Ρ€Π°Π²Π½Π° π‘₯-ΠΊΠΎΠΌΠΏΠΎΠ½Π΅Π½Ρ‚Π½ΠΎΠΉ Π·Π°Π³Π»Π°Π²Π½ΠΎΠΉ 𝐴 индСксу π‘₯, ΡƒΠΌΠ½ΠΎΠΆΠ΅Π½Π½ΠΎΠΉ Π½Π° Π΅Π΄ΠΈΠ½ΠΈΡ‡Π½Ρ‹ΠΉ Π²Π΅ΠΊΡ‚ΠΎΡ€ 𝐒 плюс 𝑦-компонСнтная заглавная 𝐴 ниТняя индСкса 𝑦, ΡƒΠΌΠ½ΠΎΠΆΠ΅Π½Π½ΠΎΠΉ Π½Π° Π΅Π΄ΠΈΠ½ΠΈΡ‡Π½Ρ‹ΠΉ Π²Π΅ΠΊΡ‚ΠΎΡ€ 𝐣, ΠΈ Π°Π½Π°Π»ΠΎΠ³ΠΈΡ‡Π½ΠΎ для Π·Π°Π³Π»Π°Π²Π½ΠΎΠΉ 𝐁 с π‘₯-ΠΊΠΎΠΌΠΏΠΎΠ½Π΅Π½Ρ‚Π½ΠΎΠΉ Π·Π°Π³Π»Π°Π²Π½ΠΎΠΉ 𝐡 Π½ΠΈΠΆΠ½ΠΈΠΌ индСксом π‘₯ ΠΈ 𝑦-ΠΊΠΎΠΌΠΏΠΎΠ½Π΅Π½Ρ‚Π½Ρ‹ΠΉ Π·Π°Π³Π»Π°Π²Π½Ρ‹ΠΉ 𝐡 Π½ΠΈΠΆΠ½ΠΈΠΉ индСкс 𝑦. Π’ΠΎΠ³Π΄Π° Π²Π΅ΠΊΡ‚ΠΎΡ€Π½ΠΎΠ΅ ΠΏΡ€ΠΎΠΈΠ·Π²Π΅Π΄Π΅Π½ΠΈΠ΅ 𝐀 пСрСсСчСния 𝐁 Ρ€Π°Π²Π½ΠΎ π‘₯-ΠΊΠΎΠΌΠΏΠΎΠ½Π΅Π½Ρ‚Π΅ 𝐴, ΡƒΠΌΠ½ΠΎΠΆΠ΅Π½Π½ΠΎΠΉ Π½Π° 𝑦-ΠΊΠΎΠΌΠΏΠΎΠ½Π΅Π½Ρ‚Ρƒ 𝐡 минус 𝑦-ΠΊΠΎΠΌΠΏΠΎΠ½Π΅Π½Ρ‚Π° 𝐴, ΡƒΠΌΠ½ΠΎΠΆΠ΅Π½Π½ΠΎΠΉ Π½Π° π‘₯-ΠΊΠΎΠΌΠΏΠΎΠ½Π΅Π½Ρ‚Ρƒ 𝐡, ΡƒΠΌΠ½ΠΎΠΆΠ΅Π½Π½ΠΎΠΉ Π½Π° 𝐀, ΠΊΠΎΡ‚ΠΎΡ€Ρ‹ΠΉ являСтся Π΅Π΄ΠΈΠ½ΠΈΡ‡Π½Ρ‹ΠΌ Π²Π΅ΠΊΡ‚ΠΎΡ€ΠΎΠΌ Π² 𝑧-Π½Π°ΠΏΡ€Π°Π²Π»Π΅Π½ΠΈΠΈ.

      Π˜Ρ‚Π°ΠΊ, Π΄Π°Π²Π°ΠΉΡ‚Π΅ ΠΏΡ€ΠΈΠΌΠ΅Π½ΠΈΠΌ это ΠΊ нашим Π²Π΅ΠΊΡ‚ΠΎΡ€Π°ΠΌ ΠΈΠ· вопроса, строчным 𝐚 ΠΈ 𝐛. ΠœΡ‹ ΠΌΠΎΠΆΠ΅ΠΌ Π½Π΅ΠΌΠ½ΠΎΠ³ΠΎ ΠΏΡ€ΠΎΡΡΠ½ΠΈΡ‚ΡŒ, Ρ‡Ρ‚ΠΎ происходит, Ссли явно Π²ΠΊΠ»ΡŽΡ‡ΠΈΠΌ ΠΊΠ°ΠΊ π‘₯-, Ρ‚Π°ΠΊ ΠΈ 𝑦-ΠΊΠΎΠΌΠΏΠΎΠ½Π΅Π½Ρ‚Ρ‹ для ΠΎΠ±ΠΎΠΈΡ… Π²Π΅ΠΊΡ‚ΠΎΡ€ΠΎΠ². Π’Π΅ΠΊΡ‚ΠΎΡ€ 𝐚 ΠΈΠΌΠ΅Π΅Ρ‚ Ρ‚ΠΎΠ»ΡŒΠΊΠΎ π‘₯-ΠΊΠΎΠΌΠΏΠΎΠ½Π΅Π½Ρ‚Ρƒ. Но ΠΌΡ‹ ΠΌΠΎΠΆΠ΅ΠΌ ΠΏΠ΅Ρ€Π΅ΠΏΠΈΡΠ°Ρ‚ΡŒ это ΠΊΠ°ΠΊ 𝐚 Ρ€Π°Π²Π½ΠΎ π‘š, ΡƒΠΌΠ½ΠΎΠΆΠ΅Π½Π½ΠΎΠ΅ Π½Π° 𝐒 плюс ноль, ΡƒΠΌΠ½ΠΎΠΆΠ΅Π½Π½ΠΎΠ΅ Π½Π° 𝐣, Ρ‡Ρ‚ΠΎΠ±Ρ‹ явно Π²ΠΊΠ»ΡŽΡ‡ΠΈΡ‚ΡŒ 𝑦-ΠΊΠΎΠΌΠΏΠΎΠ½Π΅Π½Ρ‚Ρƒ нуля. И ΠΌΡ‹ ΠΌΠΎΠΆΠ΅ΠΌ ΡΠ΄Π΅Π»Π°Ρ‚ΡŒ Ρ‚ΠΎ ΠΆΠ΅ самоС для Π²Π΅ΠΊΡ‚ΠΎΡ€Π° 𝐛, ΠΊΠΎΡ‚ΠΎΡ€Ρ‹ΠΉ ΠΈΠΌΠ΅Π΅Ρ‚ Ρ‚ΠΎΠ»ΡŒΠΊΠΎ 𝑦-ΠΊΠΎΠΌΠΏΠΎΠ½Π΅Π½Ρ‚Ρƒ. Но ΠΌΡ‹ ΠΌΠΎΠΆΠ΅ΠΌ явно Π²ΠΊΠ»ΡŽΡ‡ΠΈΡ‚ΡŒ π‘₯-ΠΊΠΎΠΌΠΏΠΎΠ½Π΅Π½Ρ‚Ρƒ нуля ΠΈ ΠΏΠ΅Ρ€Π΅ΠΏΠΈΡΠ°Ρ‚ΡŒ Π΅Π΅ ΠΊΠ°ΠΊ ноль, ΡƒΠΌΠ½ΠΎΠΆΠ΅Π½Π½Ρ‹ΠΉ Π½Π° 𝐒 плюс π‘š, ΡƒΠΌΠ½ΠΎΠΆΠ΅Π½Π½Ρ‹ΠΉ Π½Π° 𝐣.

      Π˜Ρ‚Π°ΠΊ, Ρ‚Π΅ΠΏΠ΅Ρ€ΡŒ ΠΌΡ‹ Ρ…ΠΎΡ‚ΠΈΠΌ Π²Ρ‹Ρ‡ΠΈΡΠ»ΠΈΡ‚ΡŒ Π²Π΅ΠΊΡ‚ΠΎΡ€Π½ΠΎΠ΅ ΠΏΡ€ΠΎΠΈΠ·Π²Π΅Π΄Π΅Π½ΠΈΠ΅ 𝐚 ΠΈ 𝐛. Но Π΄ΠΎΠ»ΠΆΠ½Ρ‹ Π»ΠΈ ΠΌΡ‹ Ρ€Π°ΡΡΡ‡ΠΈΡ‚Ρ‹Π²Π°Ρ‚ΡŒ 𝐚 крСст 𝐛 ΠΈΠ»ΠΈ 𝐛 крСст 𝐚? На самом Π΄Π΅Π»Π΅ оказываСтся, Ρ‡Ρ‚ΠΎ Π½Π΅ ΠΈΠΌΠ΅Π΅Ρ‚ значСния, ΠΊΠ°ΠΊΠΎΠΉ ΠΈΠ· Π½ΠΈΡ… ΠΌΡ‹ Π²Ρ‹Π±ΠΈΡ€Π°Π΅ΠΌ для расчСта. Π§Ρ‚ΠΎΠ±Ρ‹ ΠΏΠΎΠ½ΡΡ‚ΡŒ ΠΏΠΎΡ‡Π΅ΠΌΡƒ, Π΄Π°Π²Π°ΠΉΡ‚Π΅ вСрнСмся ΠΊ Π½Π°ΡˆΠ΅ΠΌΡƒ ΠΎΠ±Ρ‰Π΅ΠΌΡƒ Π²Ρ‹Ρ€Π°ΠΆΠ΅Π½ΠΈΡŽ для Π²Π΅ΠΊΡ‚ΠΎΡ€Π½ΠΎΠ³ΠΎ Π²Π΅ΠΊΡ‚ΠΎΡ€Π½ΠΎΠ³ΠΎ произвСдСния. Если Π±Ρ‹ вмСсто этого ΠΌΡ‹ написали 𝐁 Ρ‡Π΅Ρ€Π΅Π· 𝐀, наш ΠΏΠ΅Ρ€Π²Ρ‹ΠΉ Ρ‡Π»Π΅Π½ Π±Ρ‹Π» Π±Ρ‹ π‘₯-ΠΊΠΎΠΌΠΏΠΎΠ½Π΅Π½Ρ‚ΠΎΠΉ 𝐡, ΡƒΠΌΠ½ΠΎΠΆΠ΅Π½Π½ΠΎΠΉ Π½Π° 𝑦-ΠΊΠΎΠΌΠΏΠΎΠ½Π΅Π½Ρ‚Ρƒ 𝐴. И наш Π²Ρ‚ΠΎΡ€ΠΎΠΉ Ρ‡Π»Π΅Π½ Π±ΡƒΠ΄Π΅Ρ‚ 𝑦-ΠΊΠΎΠΌΠΏΠΎΠ½Π΅Π½Ρ‚ΠΎΠΉ 𝐡, ΡƒΠΌΠ½ΠΎΠΆΠ΅Π½Π½ΠΎΠΉ Π½Π° π‘₯-ΠΊΠΎΠΌΠΏΠΎΠ½Π΅Π½Ρ‚Ρƒ 𝐴.

      А ΠΏΠΎΡ‚ΠΎΠΌ всС это Π±ΡƒΠ΄Π΅Ρ‚ ΡƒΠΌΠ½ΠΎΠΆΠ΅Π½ΠΎ Π½Π° наш Π΅Π΄ΠΈΠ½ΠΈΡ‡Π½Ρ‹ΠΉ Π²Π΅ΠΊΡ‚ΠΎΡ€ 𝐀 Π² 𝑧-Π½Π°ΠΏΡ€Π°Π²Π»Π΅Π½ΠΈΠΈ. ВынСсСниС ΠΈΠ· скобок мноТитСля минус ΠΎΠ΄ΠΈΠ½ Π΄Π°Π΅Ρ‚ Π½Π°ΠΌ здСсь это Π²Ρ‹Ρ€Π°ΠΆΠ΅Π½ΠΈΠ΅. Π—Π°Ρ‚Π΅ΠΌ эти Π΄Π²Π° Π·Π½Π°ΠΊΠ° минус ΡΠΎΠΊΡ€Π°Ρ‰Π°ΡŽΡ‚ΡΡ, давая Π½Π°ΠΌ плюс. А Ссли ΠΏΠΎΠΌΠ΅Π½ΡΡ‚ΡŒ мСстами эти Π΄Π²Π° Ρ‡Π»Π΅Π½Π° Π² скобках, Ρ‚ΠΎ получится, Ρ‡Ρ‚ΠΎ Π²Π΅ΠΊΡ‚ΠΎΡ€Π½ΠΎΠ΅ ΠΏΡ€ΠΎΠΈΠ·Π²Π΅Π΄Π΅Π½ΠΈΠ΅ 𝐁 крСст 𝐀 Ρ€Π°Π²Π½ΠΎ минус Π΅Π΄ΠΈΠ½ΠΈΡ†Π΅ ΡƒΠΌΠ½ΠΎΠΆΠ΅Π½Π½ΠΎΠΌΡƒ Π½Π° 𝐡 Π½ΠΈΠΆΠ½Π΅Π³ΠΎ индСкса 𝑦 ΡƒΠΌΠ½ΠΎΠΆΠ΅Π½Π½ΠΎΠΌΡƒ Π½Π° 𝐴 Π½ΠΈΠΆΠ½Π΅ΠΌΡƒ индСксу π‘₯ минус 𝐡 Π½ΠΈΠΆΠ½Π΅ΠΌΡƒ индСксу π‘₯ ΡƒΠΌΠ½ΠΎΠΆΠ΅Π½Π½ΠΎΠΌΡƒ Π½Π° 𝐴 Π½ΠΈΠΆΠ½Π΅ΠΌΡƒ индСксу 𝑦 снова ΡƒΠΌΠ½ΠΎΠΆΠ΅Π½Π½ΠΎΠΌΡƒ Π½Π° Π΅Π΄ΠΈΠ½ΠΈΡ†Ρƒ Π²Π΅ΠΊΡ‚ΠΎΡ€ 𝐀 Π² 𝑧-Π½Π°ΠΏΡ€Π°Π²Π»Π΅Π½ΠΈΠΈ.

      Π”Π°Π²Π°ΠΉΡ‚Π΅ посмотрим Π½Π° наш ΠΏΠ΅Ρ€Π²Ρ‹ΠΉ Ρ‡Π»Π΅Π½ Π² скобках 𝐡 Π½ΠΈΠΆΠ½ΠΈΠΉ индСкс 𝑦, ΡƒΠΌΠ½ΠΎΠΆΠ΅Π½Π½Ρ‹ΠΉ Π½Π° 𝐴 Π½ΠΈΠΆΠ½ΠΈΠΉ индСкс π‘₯. ΠœΡ‹ Π΄ΠΎΠ»ΠΆΠ½Ρ‹ ΠΏΠΎΠΌΠ½ΠΈΡ‚ΡŒ, Ρ‡Ρ‚ΠΎ ΠΊΠΎΠ³Π΄Π° ΠΌΡ‹ ΡƒΠΌΠ½ΠΎΠΆΠ°Π΅ΠΌ Π΄Π²Π° числа, порядок этих Π΄Π²ΡƒΡ… чисСл Π² ΡƒΠΌΠ½ΠΎΠΆΠ΅Π½ΠΈΠΈ Π½Π΅ ΠΈΠΌΠ΅Π΅Ρ‚ значСния. Π’Π°ΠΊΠΈΠΌ ΠΎΠ±Ρ€Π°Π·ΠΎΠΌ, этот Ρ‚Π΅Ρ€ΠΌΠΈΠ½ эквивалСнтСн индСксу 𝐴 π‘₯, ΡƒΠΌΠ½ΠΎΠΆΠ΅Π½Π½ΠΎΠΌΡƒ Π½Π° индСкс 𝐡 𝑦. Π˜Ρ‚Π°ΠΊ, Π΄Π°Π²Π°ΠΉΡ‚Π΅ ΠΏΠ΅Ρ€Π΅ΠΏΠΈΡˆΠ΅ΠΌ Π΅Π³ΠΎ. И Π°Π½Π°Π»ΠΎΠ³ΠΈΡ‡Π½ΠΎ, для нашСго Π²Ρ‚ΠΎΡ€ΠΎΠ³ΠΎ Ρ‡Π»Π΅Π½Π° Π² скобках, 𝐡 Π½ΠΈΠΆΠ½Π΅Π³ΠΎ индСкса π‘₯, ΡƒΠΌΠ½ΠΎΠΆΠ΅Π½Π½ΠΎΠ³ΠΎ Π½Π° 𝐴 Π½ΠΈΠΆΠ½Π΅Π³ΠΎ индСкса 𝑦, ΠΌΡ‹ ΠΌΠΎΠΆΠ΅ΠΌ ΠΏΠ΅Ρ€Π΅ΠΏΠΈΡΠ°Ρ‚ΡŒ это ΠΊΠ°ΠΊ 𝐴 Π½ΠΈΠΆΠ½Π΅Π³ΠΎ индСкса 𝑦, ΡƒΠΌΠ½ΠΎΠΆΠ΅Π½Π½ΠΎΠ³ΠΎ Π½Π° 𝐡 Π½ΠΈΠΆΠ½Π΅Π³ΠΎ индСкса π‘₯.

      Π’ΠΎΠ³Π΄Π°, Ссли ΠΌΡ‹ сравним нашС Π²Ρ‹Ρ€Π°ΠΆΠ΅Π½ΠΈΠ΅ для 𝐁 cross 𝐀 с нашим ΠΎΠΏΡ€Π΅Π΄Π΅Π»Π΅Π½ΠΈΠ΅ΠΌ 𝐀 cross 𝐁, ΠΌΡ‹ ΡƒΠ²ΠΈΠ΄ΠΈΠΌ, Ρ‡Ρ‚ΠΎ эти Π±ΠΈΡ‚Ρ‹ Π² скобках Π°Π±ΡΠΎΠ»ΡŽΡ‚Π½ΠΎ ΠΎΠ΄ΠΈΠ½Π°ΠΊΠΎΠ²Ρ‹ Π² ΠΊΠ°ΠΆΠ΄ΠΎΠΌ случаС. И СдинствСнная Ρ€Π°Π·Π½ΠΈΡ†Π° Π² Ρ‚ΠΎΠΌ, Ρ‡Ρ‚ΠΎ Π² нашСм 𝐁 кроссС 𝐀 Ρƒ нас этот коэффициСнт минус ΠΎΠ΄ΠΈΠ½ Π²ΠΏΠ΅Ρ€Π΅Π΄ΠΈ. Π­Ρ‚ΠΎΡ‚ Ρ„Π°ΠΊΡ‚ΠΎΡ€ просто Π΄Π°Π΅Ρ‚ Π½Π°ΠΌ ΠΎΠ±Ρ‰ΠΈΠΉ Π·Π½Π°ΠΊ минус. Π˜Ρ‚Π°ΠΊ, ΠΌΡ‹ ΠΌΠΎΠΆΠ΅ΠΌ ΡΠΊΠ°Π·Π°Ρ‚ΡŒ, Ρ‡Ρ‚ΠΎ 𝐁 крСст 𝐀 Ρ€Π°Π²Π΅Π½ ΠΎΡ‚Ρ€ΠΈΡ†Π°Ρ‚Π΅Π»ΡŒΠ½ΠΎΠΌΡƒ 𝐀 крСсту 𝐁. А ΠΏΠΎΡΠΊΠΎΠ»ΡŒΠΊΡƒ Π² этом вопросС нас интСрСсуСт Ρ‚ΠΎΠ»ΡŒΠΊΠΎ Π²Π΅Π»ΠΈΡ‡ΠΈΠ½Π° Π²Π΅ΠΊΡ‚ΠΎΡ€Π½ΠΎΠ³ΠΎ произвСдСния, Ρ‚ΠΎ Π½Π΅ ΠΈΠΌΠ΅Π΅Ρ‚ значСния, вычисляСм Π»ΠΈ ΠΌΡ‹ 𝐀 крСст 𝐁 ΠΈΠ»ΠΈ 𝐁 крСст 𝐀, ΠΏΠΎΡ‚ΠΎΠΌΡƒ Ρ‡Ρ‚ΠΎ ΠΎΠ½ΠΈ ΠΎΠ±Π° ΠΈΠΌΠ΅ΡŽΡ‚ ΠΎΠ΄ΠΈΠ½Π°ΠΊΠΎΠ²Ρ‹Π΅ Π²Π΅Π»ΠΈΡ‡ΠΈΠ½Ρ‹. Они просто ΠΈΠΌΠ΅ΡŽΡ‚ ΠΏΡ€ΠΎΡ‚ΠΈΠ²ΠΎΠΏΠΎΠ»ΠΎΠΆΠ½Ρ‹Π΅ Π·Π½Π°ΠΊΠΈ, поэтому Π²Π΅ΠΊΡ‚ΠΎΡ€Ρ‹ ΡƒΠΊΠ°Π·Ρ‹Π²Π°ΡŽΡ‚ Π² ΠΏΡ€ΠΎΡ‚ΠΈΠ²ΠΎΠΏΠΎΠ»ΠΎΠΆΠ½Ρ‹Ρ… направлСниях.

      Π˜Ρ‚Π°ΠΊ, Π²ΠΎΠ·Π²Ρ€Π°Ρ‰Π°ΡΡΡŒ ΠΊ вопросу ΠΈ нашим Π΄Π²ΡƒΠΌ Π²Π΅ΠΊΡ‚ΠΎΡ€Π°ΠΌ Π² Π½ΠΈΠΆΠ½Π΅ΠΌ рСгистрС 𝐚 ΠΈ Π½ΠΈΠΆΠ½Π΅ΠΌ рСгистрС 𝐛, Π΄Π°Π²Π°ΠΉΡ‚Π΅ Ρ€Π΅ΡˆΠΈΠΌ Π²Ρ‹Ρ‡ΠΈΡΠ»ΠΈΡ‚ΡŒ Π²Π΅ΠΊΡ‚ΠΎΡ€Π½ΠΎΠ΅ ΠΏΡ€ΠΎΠΈΠ·Π²Π΅Π΄Π΅Π½ΠΈΠ΅ 𝐚 крСст 𝐛. Π­Ρ‚ΠΎ Π²Π΅ΠΊΡ‚ΠΎΡ€Π½ΠΎΠ΅ ΠΏΡ€ΠΎΠΈΠ·Π²Π΅Π΄Π΅Π½ΠΈΠ΅ задаСтся π‘₯-ΠΊΠΎΠΌΠΏΠΎΠ½Π΅Π½Ρ‚ΠΎΠΉ 𝐚, которая Ρ€Π°Π²Π½Π° π‘š, ΡƒΠΌΠ½ΠΎΠΆΠ΅Π½Π½ΠΎΠΉ Π½Π° 𝑦-ΠΊΠΎΠΌΠΏΠΎΠ½Π΅Π½Ρ‚Ρƒ 𝐛, которая Ρ‚Π°ΠΊΠΆΠ΅ являСтся π‘š, Π·Π° Π²Ρ‹Ρ‡Π΅Ρ‚ΠΎΠΌ 𝑦-ΠΊΠΎΠΌΠΏΠΎΠ½Π΅Π½Ρ‚Ρ‹ 𝐚, Ρ€Π°Π²Π½ΠΎΠΉ Π½ΡƒΠ»ΡŽ, ΡƒΠΌΠ½ΠΎΠΆΠ΅Π½Π½ΠΎΠΉ Π½Π° π‘₯-ΠΊΠΎΠΌΠΏΠΎΠ½Π΅Π½Ρ‚Ρƒ 𝐛, Ρ‡Ρ‚ΠΎ Ρ‚Π°ΠΊΠΆΠ΅ Ρ€Π°Π²Π½ΠΎ Π½ΡƒΠ»ΡŽ. А ΠΏΠΎΡ‚ΠΎΠΌ всС это умноТаСтся Π½Π° Π΅Π΄ΠΈΠ½ΠΈΡ‡Π½Ρ‹ΠΉ Π²Π΅ΠΊΡ‚ΠΎΡ€ 𝐀. ΠŸΠΎΡΠΊΠΎΠ»ΡŒΠΊΡƒ Π΅Π΄ΠΈΠ½ΠΈΡ‡Π½Ρ‹ΠΉ Π²Π΅ΠΊΡ‚ΠΎΡ€ 𝐀 ΠΏΠΎ ΠΎΠΏΡ€Π΅Π΄Π΅Π»Π΅Π½ΠΈΡŽ ΠΈΠΌΠ΅Π΅Ρ‚ Π²Π΅Π»ΠΈΡ‡ΠΈΠ½Ρƒ Π΅Π΄ΠΈΠ½ΠΈΡ†Ρƒ, Ρ‚ΠΎ Π²Π΅Π»ΠΈΡ‡ΠΈΠ½Π° Π²Π΅ΠΊΡ‚ΠΎΡ€Π½ΠΎΠ³ΠΎ произвСдСния 𝐚 крСст 𝐛 задаСтся этой Ρ‡Π°ΡΡ‚ΡŒΡŽ выраТСния здСсь. ΠœΡ‹ ΠΌΠΎΠΆΠ΅ΠΌ Π·Π°ΠΏΠΈΡΠ°Ρ‚ΡŒ это ΠΊΠ°ΠΊ π‘š Π² ΠΊΠ²Π°Π΄Ρ€Π°Ρ‚Π΅. Π˜Ρ‚Π°ΠΊ, ΠΌΡ‹ ΠΌΠΎΠΆΠ΅ΠΌ ΡΠΊΠ°Π·Π°Ρ‚ΡŒ, Ρ‡Ρ‚ΠΎ Π²Π΅Π»ΠΈΡ‡ΠΈΠ½Π° крСста 𝐛 Ρ€Π°Π²Π½Π° π‘š Π² ΠΊΠ²Π°Π΄Ρ€Π°Ρ‚Π΅.

      Π˜Ρ‚Π°ΠΊ, ΠΊ этому ΠΌΠΎΠΌΠ΅Π½Ρ‚Ρƒ ΠΌΡ‹ ΠΎΠΏΡ€Π΅Π΄Π΅Π»Π΅Π½Π½ΠΎ Π²Ρ‹ΠΏΠΎΠ»Π½ΠΈΠ»ΠΈ Π²Ρ‚ΠΎΡ€ΠΎΠΉ шаг ΠΈ вычислили Π²Π΅ΠΊΡ‚ΠΎΡ€Π½ΠΎΠ΅ ΠΏΡ€ΠΎΠΈΠ·Π²Π΅Π΄Π΅Π½ΠΈΠ΅ 𝐚 ΠΈ 𝐛. ВсС, Ρ‡Ρ‚ΠΎ ΠΎΡΡ‚Π°Π»ΠΎΡΡŒ, β€” это Ρ‚Ρ€Π΅Ρ‚ΠΈΠΉ шаг, ΠΊΠΎΡ‚ΠΎΡ€Ρ‹ΠΉ состоит Π² Ρ‚ΠΎΠΌ, Ρ‡Ρ‚ΠΎΠ±Ρ‹ ΠΈΡΠΏΠΎΠ»ΡŒΠ·ΠΎΠ²Π°Ρ‚ΡŒ эту Π²Π΅Π»ΠΈΡ‡ΠΈΠ½Ρƒ Π²Π΅ΠΊΡ‚ΠΎΡ€Π½ΠΎΠ³ΠΎ произвСдСния, Ρ‡Ρ‚ΠΎΠ±Ρ‹ Π½Π°ΠΉΡ‚ΠΈ Π΄Π»ΠΈΠ½Ρƒ π‘š ΠΊΠ°ΠΆΠ΄ΠΎΠ³ΠΎ Π²Π΅ΠΊΡ‚ΠΎΡ€Π°. Вопрос Π³ΠΎΠ²ΠΎΡ€ΠΈΡ‚ Π½Π°ΠΌ, Ρ‡Ρ‚ΠΎ Π²Π΅Π»ΠΈΡ‡ΠΈΠ½Π° Π²Π΅ΠΊΡ‚ΠΎΡ€Π½ΠΎΠ³ΠΎ произвСдСния Ρ€Π°Π²Π½Π° дСвяти. Π˜Ρ‚Π°ΠΊ, ΠΌΡ‹ ΠΌΠΎΠΆΠ΅ΠΌ ΡƒΡΡ‚Π°Π½ΠΎΠ²ΠΈΡ‚ΡŒ Π½Π°ΡˆΡƒ Π²Π΅Π»ΠΈΡ‡ΠΈΠ½Ρƒ 𝐚 крСст 𝐛 Ρ€Π°Π²Π½Ρ‹ΠΌ π‘š Π² ΠΊΠ²Π°Π΄Ρ€Π°Ρ‚Π΅ Ρ€Π°Π²Π½Ρ‹ΠΌ дСвяти.

      Π’Π΅ΠΏΠ΅Ρ€ΡŒ нас попросили Π½Π°ΠΉΡ‚ΠΈ Π΄Π»ΠΈΠ½Ρƒ ΠΈΠ»ΠΈ Π²Π΅Π»ΠΈΡ‡ΠΈΠ½Ρƒ ΠΊΠ°ΠΆΠ΄ΠΎΠ³ΠΎ ΠΈΠ· Π²Π΅ΠΊΡ‚ΠΎΡ€ΠΎΠ² 𝐚 ΠΈ 𝐛.

    Π”ΠΎΠ±Π°Π²ΠΈΡ‚ΡŒ ΠΊΠΎΠΌΠΌΠ΅Π½Ρ‚Π°Ρ€ΠΈΠΉ

    Π’Π°Ρˆ адрСс email Π½Π΅ Π±ΡƒΠ΄Π΅Ρ‚ ΠΎΠΏΡƒΠ±Π»ΠΈΠΊΠΎΠ²Π°Π½. ΠžΠ±ΡΠ·Π°Ρ‚Π΅Π»ΡŒΠ½Ρ‹Π΅ поля ΠΏΠΎΠΌΠ΅Ρ‡Π΅Π½Ρ‹ *